Sei sulla pagina 1di 505

5.16.

Airline Company
Length: Medium (30 Minutes)

Problem:
Our client is a major airline company and the CEO contacts you (a BCG partner) and said
that he wanted to put one more passenger on each of his company’s flights. We will go to
a meeting with him now and we need to prepare two things:
• The financial return of one more passenger per flight. Is it worthwhile to do so?
• The marketing strategy to attract these new passengers.

Solution:
First element of the solution: The financial return
To calculate the financial return, the candidate should look that the revenue and cost side
of bringing one more passenger per flight.
One the revenue side, we need:
• Average cost of a ticket;
• Number of new passengers.
The candidate should also briefly ask about if he can assume that there is enough capacity
to put one more passenger onboard.
Second element of the solution: The average cost of a ticket
The candidate should ask about the kind of routes, the market share, and then price of
each of them.
In this case, we were talking about the Brazilian market and this airline has only domestic
flights. The interviewer simplified the numbers a lot the make the calculations easier.
You should adapt the numbers to US domestic airline market or any other you would
like. For Brazil, the interviewer gave the following information:
• 60% of the flights are between two major cities (Sao Paulo and Rio de Janeiro),
which are 45 minutes apart and the ticket costs US$100.00;
• 20% between other major capitals which are 1 hour and a half apart from each
other and the ticket also costs US100.00;
• 20% between other cities which are 3 hours apart and cost US$ 300.00.
So the average ticket price is: (80% * 100 + 20% * 300) / 100 = $ 140.00
Third Element of Solution: Number of new passengers
There are several ways to come up with the number of new passengers. I proposed two:
1. Calculate the number of flights: estimate the number of airplanes that the
company owns and then estimate the number of flights each airplane does.
2. Calculate the number of cities (average) in which the company makes flights
and the average number of flights per city.
The interviewer said that I should use the first approach since he had looked at the
company’s website and he found out that they have 100 airplanes.
I asked if I could assume that 80% of the planes were used for the 1 hour flights and 20%
for 3 hour flights. He answered that this was not 100% true in practice because depending

Haas Consulting Club Case Book 52


on the city, we could have more frequent flights than others, but he agreed that I could do
this approximation. The interviewee can come up with other reasonable assumptions.

The candidate should also notice that normally the company spends a lot of money and
time performing maintenance on its airplanes. If the candidate forgets to mention this and
starts using all 110 airplanes, you should ask what assumptions he made to use the all the
planes at once. For this company, the interviewer mentioned that this company had a little
less than 10% of its airplane in maintenance.

So, assuming 100 airplanes available, we have:


• 100 × 80% = 80 airplanes doing 1 hour flights
• 100 × 20% = 20 airplanes doing 3 hours flights
To estimate the number of flights per airplane per day, we need the range of hours of
flights during a day and the interval of time in which an airplane stays on the ground.
What the interviewer wants to hear is not the right number, but what are your
assumptions to get to the numbers, for example:
Fourth Element of Solution: Flight time
• The first airplane departs at 6:00 am because the executives must arrive at its
client’s at 8:00 am, for example.
• There will be no flights after 10:00 pm, so we will assume that the last flight will
arrive at midnight.
So, there is no flight between midnight and 6:00 am.
Fifth Element of Solution: On the ground time:
• Taxing – 10 minutes
• Passengers going out – 15 minutes
• Cleaning / food supply – 10 minutes
• Passenger going in – 15 minutes
• Taxing – 10 minutes
Total: 60 minutes  1 hour.
Therefore, based on the above we have:
• 18h / 2h = 9 short flights per day
• 18/h / 4h ~ 4 long flights per day
• 80 × 9 short flights per day = 720 flights
• 20 × 4 long flights per day = 80 flight
• Total = 800 flights per day
So, the extra revenue in one month is 800 flights / day * 30 days * $140 = $ 3,360,000
On the cost side, the candidate should realize that the cost of one extra passenger is only
the marginal cost, which can be ignored. Therefore, the extra profit is the extra revenue.
The interviewer asked why was it important to have this row estimation about the extra
profit before going to meet the client. I said that it was important to have a sense of the
size of the clients’ issue and the interviewer agreed.

Haas Consulting Club Case Book 53


Case 11: R & D Productivity

BACKGROUND

Firm: BCG
Round: 2008 Summer, Second
Content: Qualitative

CASE QUESTION

Our client is the head of the R & D department of a pharmaceutical company. He wants
us to advise him on how to improve the productivity in the R & D department

INTERVIEWER BRIEFING

Recommended approach: Explain a good structure to answering the case. Suggest a


possible framework and a solution.

Key facts:
Facts that are disclosed during the course of the case:
 There are basically several steps leading from the discovery of the molecule to the
launch.
 It normally takes about 10 years to discover a drug and get it approved.
 The drugs are usually patent protected

EXAMPLE DIALOGUE

Interviewer: There are basically several steps leading from the discovery of the
molecule to the launch. It normally takes about 10 years to discover a drug and get it
approved.

Discovery Preclinical Phase 1 Phase 2 Phase 3 Approval


(discovery (testing on (testing (testing on (testing on &
of the Animals) on humans) humans) Launch
molecule) humans)

How would you measure the productivity of the R & D department?

Interviewee: I would look at the success of the R & D department in terms of the drugs
discovered.

Interviewer: If you had to measure the productivity in terms of a formula that you could
present to the shareholders, what would that be?
Management Consulting Association Case Book 2008

- 48 -
Interviewee: The formula would look like this:

NPV of future cashflows - R & D cost of


of successful drugs all drugs
R&D cost of all drugs

Interviewer: What are the big drivers to increase the productivity ?

Interviewe: Let’s look at each aspect of the formula that we have written

NPV of future cashflows .


The big drivers of the NPV of the future cashflows are as follows:
 Quality of the compound
Quality of the compound is the most important factor that influences the cash
flows. Say Lipitor for instance – it’s the highest quality compound that’s out
there in the market and its capturing record revenues year after year. Its
important to ensure that sufficient tests have been done during the test phase
to ensure that there are no side effects to the drugs.
 Length of research
As it takes around 10 years to discover the drug, reducing the length of time
taken will increase the patent time thus maximizing our return
 Marketing & Sales promotion efforts
While this aspect is beyond the hands of our client, its an important element of
the future cashflows. The sales and marketing team are responsible to build
awareness and demand for the product.

R & D cost of all drugs


The 2nd part of the formula is the cost of the producing drugs. Its important to minimize
the cost of research to have the maximum productivity. That can be done in the following
manner:
 Strict controls and parameters at each stage of the development
No drug which has a likelihood of failing the stage should be passed to the
next stage. This ensures that costs of failure are kept to the minimum. If the
controls are not strict, then a lot more money would be spent on patient testing
and the drugs end up getting rejected in the later stages. This not only wastes
money but also valuable time which could have been spent on another drug.
 Quality scientists
In order to control the costs of R & D and make it more productive, we need
high quality scientists.
 Acquire compounds developed successfully upto certain stages
Due to the length of time a complete in-house discovery takes, it might be
worthwhile to explore outside and either acquire or obtain partnerships with

Management Consulting Association Case Book 2008

- 49 -
other companies for promising drugs. This diversifies the risk as well as may
yield a good return

Interviewer: Any other ways to increase productivity? What about the R & D
organization?

Interviewee: We need to keep the scientists sufficiently motivated as it’s a very long
process before their drug is successful. We need to ensure that they do not move
organizations quickly leading to loss of critical talent.

Interviewer: What is the biggest challenge that the R & D head faces today?

Interviewee: The biggest challenge the R & D head faces today is on evaluation of
scientist teams. What kind of measurable metrics to set and how to evaluate the scientist
against it.

Interviewer: What is the biggest mistake that an R & D head could make?

Interviewee: The biggest mistake an R & D head could make is to measure the scientists
based on the number of drugs they pass from one stage to the next. This will motivate
them to pass certain drugs which maybe borderline or even not fit to be passed to the next
stage. This increases the R & D costs significantly and as mentioned before wastes time
that could have been spent on other products.

Their incentive structure should be based on compliance, how strictly they have adhered
to the guidelines. Any product that fails at a later stage and where the cause can be
directly attributed to a prior stage should impact their performance.

Interviewer: Excellent! Lets summarize and recommend

Interviewee: We have seen the different stages of drug discovery and know that it takes
10 years to discover a drug. We have also looked at the drivers of productivity which are
mainly the future cashflows from successful drugs and R & D investment in all drugs. In
order to increase the productivity I would recommend that we take steps to
 Shorten the length of research so that we get maximum patent protection time to
maximize our returns.
 Establish strict guidelines at each stage of the process to ensure that failures are
discovered more quickly
 Change the incentive structure of the scientists so that the metrics for evaluating
them is compliance driven and not based on the number of drugs passed to the
next stage.

Management Consulting Association Case Book 2008

- 50 -
Case 13: Ice Cream Manufacturer

BACKGROUND

Firm: BCG
Round: 2008 Summer, Second Round
Content: Qualitative and Quantitative

CASE QUESTION

The client is an ice cream manufacturer. They are the European market leader with a
share of 31% of the ice cream market. The client is concerned because its sales are more
seasonal than those of competitors, even though overall profits are excellent. Note that
90% of his costs are fixed.

INTERVIEWER BRIEFING

Recommended approach: A good possible framework is to look at:

 Company
o Product mix (characteristics, seasonality, how perishable it is, etc.)
o Operations (is the manufacturing capacity used continuously or seasonally
throughout the year, can we use the capacity to produce less seasonal products
than ice creams)
 Customer
o Segments, trends, characteristics in terms of seasonality
 Competitors
o How do they differ from the client with regards to the two previous points
(Company and Customer)

It is important that the candidate recognizes that there are two ways to reduce seasonality
here:
o Modify the seasonality of sales themselves
o Modify the production or the products to come to a less seasonal use of
capacity.

Key facts:
Facts that are disclosed during the course of the case

 Products
o Ice creams, sorbets

Management Consulting Association Case Book 2008

- 55 -
o Very perishable products (by nature and regulation)
o Very extensive to store
 Operations
o Production follows seasonal sales pattern (too expensive to store)
o The production line could be used to produce other types of items.
 3 main types of Customers
o Ice cream stores (highly seasonal)
o Retail / supermarkets (moderately seasonal)
o Restaurants (almost not seasonal)

 Market situation

Channel Market split Client market share on Contribution to its


this channel overall market share
Retail / Supermarket 40% 40% 16%
Restaurants 30% 10% 3%
Ice cream stores 30% 40% 12%
Total 100% 31%

EXAMPLE DIALOGUE

Interviewer: How would you go about approaching this problem?

Interviewee: [Interviewee should come up with a framework. See briefing above.]

Interviewer: Good points. Where do you want to start?

Interviewee: Let’s talk about the operations first. The demand for ice cream is by nature
seasonal. Does the client currently produce ice cream continuously or seasonally
throughout the year?

Interviewer: There are peaks in production to match demand. Why do you ask?

Interviewee: Considering that 90% of the current costs are fixed, it might be less
expensive to smooth production over the year. The capacity would not need to be as high
since there would not be peaks anymore, therefore the fixed costs would be reduced.
However, this would involve storage of ice cream during the months of low demand.

Interviewer: This is an interesting thought. However, our ice cream is too perishable to
be stored for more than 2 weeks.

Management Consulting Association Case Book 2008

- 56 -
Interviewee: Maybe we could modify our products by adding preservatives and
stabilizers.

Interviewer: Not a bad idea. However, regulations prevent us from doing this.

Interviewee: Ok. It seems that production of our ice cream will remain seasonal then. To
maximize the use of capacity and reduce our fixed costs burden, could the plants be used
to manufacture other types of products during non-peaks periods?

Interviewer: This would be a possibility, but we have not explored it in details. What else
would you need to know?

Interviewee: We discussed the possibility to reduce the seasonality of operations. Now, I


would like to talk about reducing the seasonality of sales. Who are our current
customers?

Interviewer: Who do you think they might be?

Interviewee: As a customer, I can buy ice cream at supermarkets, ice cream


stores/chains, restaurants. Do we sell to all of these channels?

Interviewer: Yes, they are all our customers. Which do you think has the most seasonal
demand?

Interviewee: Ice cream stores would definitely be the most seasonal. Supermarkets
would be less seasonal because people still eat ice cream at home during winter watching
TV, even though they buy less than during summer. Finally, I would say that restaurants
are the least seasonal because they always have ice creams on their menu and add it with
apple pies, chocolat tarts, etc. Does it seem reasonable?

Interviewer: It makes sense.

Interviewee: How do we split our sales to these channels? And how does it compare to
our competitors?

Interviewer: Let me give you the following data. We miss some client information here.
Could you please calculate what’s left for me?

Channel Market split Client market share of Contribution to its


this channel overall market share
Retail / Supermarket 40% 40% 16%
Restaurants 30% ? ?
Ice cream stores 30% 40% 12%
Management Consulting Association Case Book 2008

- 57 -
Interviewee: You mentioned initially that the total market share of the client is 31%.
Since retail/supermarket and ice cream stores contribute to 16% and 12%, restaurants
must contribute to (31-(16+12)=) 3%. Therfore, the client’s market share on restaurants
must be 10%. (30%/3%=10%)

Interviewer: Based on everything we discussed, what would you recommend the client do
to address its seasonality concern?

Interviewee: Firstly, I would recommend targeting more strongly the restaurants, as their
demand is less seasonal and the client is relatively weakly positioned there compared to
competitors. Secondly, I would recommend looking at products we could manufacture in
the current plants during non-peak times of the year, so as to optimize the use of the fixed
cost capacities.

Interviewer: Thank you.

Management Consulting Association Case Book 2008

- 58 -
Case 18: Liquor Company
BACKGROUND

Firm: BCG
Round: 2007 Summer, Second
Content: Mainly qualitative

CASE QUESTION

Our client is a multi-billion dollar liquor distributor. It has hired a consultant to


determine why its whisky brand is not growing.

INTERVIEWER BRIEFING

The key to this case is to understand the customer preferences are shifting away from the
medium range into either the high end or the low end. The high end provides the 70%
male segment with the refined taste and distinguished image and taste that they are
attracted to. The low end provides the 30% “bar crowd” a cheap whisky to imbibe. The
rise in price has driven many of the customers away and the company needs to reconsider
its pricing strategy and branding strategy.

Competition and Market Share

Market Segment High Medium Low


Growth this year 14% -6% 8%
Competitiors 10 total (5 new) 2 total (including client) 4 total

Our client is in the medium segment

Customer segmentation and trends

Our client’s customers are as follows (give all this information at once):

70% 30%
Male Younger males and females
40-55 years 21-35
Like taste, brand Bar and club crowd
Usually enjoys a drink after work Drink mixed
Into the tradition and brand
Drink whisky straight or on the rocks

Management Consulting Association Case Book 2008

- 73 -
Product

 Our client has increased the price from $11 to $12


 Competitors in the medium range are priced at $11
 Distillers have changed the ingredients to save $0.25/bottle on COGS but have
not changed the taste.

EXAMPLE DIALOGUE

Interviewer: How would you approach this case?

Interviewee: Well, I’d start externally: What has happened with the competition and the
customers? In terms of the competition, I would look at changes in market share, any
new competition, the segmentation, and market growth. Regarding segmentation, I
would also look at the customers, what do they want? Finally, I would look internally
and look at our product, marketing, and pricing.

Interviewer: Great. Where do you want to start?

Interviewee: Well, what do we know about the market? How is it broken up?

Interviewer: Well there is a high, medium and low end of the market? The high end has
10 competitors, the medium end has 2 including the client, and the low end has 4.

Interviewee: Okay, and how is each market growing?

Interviewer: The high end is growing at 14%, the low end is growing at 8%, and the
medium range is contracting at 6%.

Interviewee: Well, I can see a problem already. We’re in a segment of the market that’s
contracting. Has share changed or have there been any new products introduced?

Interviewer: No.

Interviewee: Then, let’s look at customers. What do we know about our whiskey
drinkers?

Interviewer: Our client basically has two types of drinkers:

Management Consulting Association Case Book 2008

- 74 -
70% 30%
Male Younger males and females
40-55 years 21-35
Like taste, brand Bar and club crowd
Usually enjoys a drink after work Drink mixed
Into the tradition and brand
Drink whisky straight or on the rocks

Interviewee: That’s interesting. We must be rather traditional brand if we have such an


older following. Let’s turn internally. What do we know about our product? Have we
changed price, marketing, or the product recently?

Interviewer: Our marketing has been consistent. The distillers recently changed
ingredients to save 0.25$ per bottle, but that hasn’t effected the taste. Also, we have
recently risen in price from $11 to $12.

Interviewee: That’s interesting. What is our competition priced at?

Interviewer: The competition at the mid-range part of the market is at $11.

Interviewee: Well, that’s interesting. Do we know if this price change affected the
younger crowd?

Interviewer: What do you think?

Interviewee: I would assume this would affect them because they drink our product
mixed, and there’s probably little discernable difference to them. They probably have
less brand loyalty as well.

Interviewee: Good. Now what would you recommend to our client?

Interviewee: Well, it seems that we’ve learned a few things:

 Younger, bar crowd customers are price sensitive


 But 70% of our customers have great brand loyalty and would like migrate to the
higher end market
 The 30% bar crowd are moving to the lower end market because they drink
whiskey mixed
 The client should try to enter the higher premium market to recapture their
customers and grow market share
 Client could also consider entering the low end market with a brand extension

Management Consulting Association Case Book 2008

- 75 -
Goizueta Consulting Association
Goizueta Business School, Emory University, 1300 Clifton Road, Atlanta GA 30322

Increase Profitability

Increase Profitability
- Revenues
- Price
- Elasticity
- Substitutes
- Volume
- Promotion
- Place
- New marketing campaign
- Economies of scope
- Cost of goods sold
- Material
- Scale
- Supply
- Inventory management
- Labor
- Consolidation of operations

Sample Case: Paper Towel Profits (BCG)


Interviewer
Your client is a large consumer goods manufacturer and is concerned about the paper towel
business. Profits for the business are okay, but the client wants to increase them. The client has
already done some research and following are the three findings:
• 75% of the business is done when towels are put on promotions.
• There is great variability in event performance.
• Retailer profitability is variable

Your objective is to help the client


• Increase profits
• Decease variability

How will you go about analyzing the problem?

Candidate
I need a quick clarification before I start structuring my thoughts. You mentioned that 75% sales
happen during promotions. Are these promotions to the trade or to the consumers?

Interviewer
These are trade promotions. We expect the retailers to pass-on at least a part of the benefit to the
consumers. However, consumers don’t necessarily get the benefit.

Candidate
Okay.
Consulting Interview Book
Compiled by Vivek Pundir, Goizueta MBA’06
Page 73 of 666
Goizueta Consulting Association
Goizueta Business School, Emory University, 1300 Clifton Road, Atlanta GA 30322

Industry Firm Revenues Costs Product Sales


(Var)
Profitability Promotions Number of towels Fixed PLC/ Buying
Variability Share-of-voice Number of Variable Cycle
Competitors Scanner-based consumers - Manufacturing Frequency of
Frontloading? - Distribution promotions
Increase through Anticipation of
price reductions? promotions
New products? Distribution
I’d want to explore four broad areas: industry, firm, revenues and costs. Starting with the
industry, how is the profitability in the industry in general? And variability?

Interviewer
Good questions. But those are not the issues here.

Candidate
How many competitors are there?

Interviewer
There are three big competitors. All players own close to 25% market share each.

Candidate
Seems like industry factors are not important here….moving on to the firm…I’d be particularly
interested in finding out about the promotions. What is the share of voice? How are these
promotions run – are they scanner based, or just based on order quantities?

Interviewer
The promotions are run quite simply. During the promotions, the retailers get volume discounts
on their orders over and above their regular trade margins.

Candidate
Hmm….interesting. I’m jumping ahead of myself here, but could the way the promotions are run
be a reason? If the promotions are run based on the quantities ordered by the retailer, the retailer
will frontload his inventory by ordering during promotion periods all the quantity that he expects
to sell till the next promotion. Thus demand would surge during promotions while it will sag
before and after them. Before – in anticipation of the promotion, and after because the guy has
already overstocked. The fact that 75% sales are made during promotions seems to go along well
with this hypothesis.

Interviewer
Okay, let us suppose that the problem is exactly what you suggested. What will you do about it?

Candidate
A good solution could be to have scanner-based promotions. The items sold by retailers are
scanned at the cash-counter. The discounts will be based on these items. In essence, we are
telling the retailer, that they will get discounts on the items they sell during the promotion period,

Consulting Interview Book


Compiled by Vivek Pundir, Goizueta MBA’06
Page 74 of 666
Goizueta Consulting Association
Goizueta Business School, Emory University, 1300 Clifton Road, Atlanta GA 30322

not on the items they buy. This will prevent frontloading, and will encourage the retailers to
promote sales by passing on some benefit to the consumer or providing us some display space
etc. Finally, I think we can reduce fluctuations by strengthening the brand. That will make the
retailers feel more secure.

Interviewer
That sounds impressive. But how will you convince the retailers to agree to this?

Candidate
I think the best way to convince the retailers is to make sure that they will earn more, and to
make them aware of the fact. For instance, buying in advance necessitates that they warehouse
the product. By making the system more “on demand” and less dependent on warehousing, we
are saving them money in inventory costs. Also, we may help the retailers put the technology in
place for the operation. This can be used for other products as well, saving them even more
money. Besides, if the promotions are scanner-bases, they will focus on selling more, giving the
product display etc. which will increase their revenues.

Interviewer
Cool. This is going well. Let us assume that scanner promotions will actually save money. Let’s
consider 5,000 units
• Scan Costs are those of 5,000 units
• Bill-back (post facto educated guess) Costs for those 5,000 units are equivalent to the costs
for 7,000 units under the scanner-based method.
• Off-invoice (before fact educated guess) Costs are equivalent to those of 10,000 units.
Given that the profit per unit is $15 and the cost per unit is $10, how do the return on
investments for these methods compare?

Candidate
Profit – Cost
ROI = --------------- x 100
Cost

Scan
75000 – 50000
------------------ x 100 = 50%
50000

Bill-back
75000 – 70000
------------------ x 100 = 100 / 14 = 7.14%
70000

Off-invoice
75000 – 100000
-------------------- x 100 = - 25%
100000

Consulting Interview Book


Compiled by Vivek Pundir, Goizueta MBA’06
Page 75 of 666
Goizueta Consulting Association
Goizueta Business School, Emory University, 1300 Clifton Road, Atlanta GA 30322

BCG - Trevor's Toys


You have been assigned to work on a product-offering strategy case for Trevor's Toys, Inc. Your
team has been asked to deliver a recommendation quickly: you have only 12 weeks (or 60
working days) to complete the project. You must use your time wisely if you’re to deliver
maximum value for Trevor' s Toys.

Trevor's Toys is a regional toy retailer focused on the high-tech toy market in the United States.
It generated $600 million in revenue last year from 30 stores located primarily in urban areas
throughout the Northeast and the Midwest. Management has considered expanding its product
offering to include low-tech toys a number of times, but has never been convinced that it was
making a fully informed decision. Revenue growth for the company has historically been strong,
although it has slowed in the last few years.

Trevor' s Toys'competitors include traditional toy stores and online Web sites that sell both high-
tech and low-tech toys, some of which are quite large and utilize both sales channels.
Management is concerned that the continued growth of the large traditional stores and online
toys sites will erode its customer base over time unless Trevor's offers a full selection of toys.

Your task is to analyze the decision facing management at Trevor' s Toys. Your first step is to
frame your analysis, decide which aspects of it are most important, and then ask questions about
the most relevant topics. Afterward, you' ll develop a recommendation and a rationale to support
it.

Doing the analysis begins with framing the issues. This allows you to establish the scope. By
doing so, you'
ll ensure that the questions you select have real impact, and that your attention is
applied where it matters most.

The list below identifies many common issues. You must select four to investigate. As in most
consulting engagements, your time and the client' s money are limited, which means you won' t be
able to investigate every potential issue. Choose the four that you believe are most important to
this assignment.

Potential acquisitions
Industry threats
Market opportunity
Customers
Competitors
Projected Profitability
Pricing
International expansion
Promotion
Store growth
Potential entrants
Cost structure
Toy suppliers

Consulting Interview Book


Compiled by Vivek Pundir, Goizueta MBA’06
Page 127 of 666
Goizueta Consulting Association
Goizueta Business School, Emory University, 1300 Clifton Road, Atlanta GA 30322

Process re-engineering
Implementation
Organizational effectiveness

Assignments can be framed in any number of ways. The key is to use a logical structure that
enables you to analyze and respond to the issues facing the client. Although the four issues
below may not be the ones you selected, we recommend that to achieve the greatest impact for
the Trevor's Toys management team, you thoroughly investigate them during the time you have
remaining.

Customers
What are the needs of the client'
s customers, and how would a broader selection of toys address
those needs? Will a move to expand the product offering add value to the customers'experience?

Competitors
How will the current competitors in the traditional toy space react to the decision by Trevor'
s
Toys to offer low-tech toys?

Market opportunity
Is the market large enough to support a profitable Trevor's Toys venture into low-tech toys?
Would Trevor' s capture enough share for this to be successful?

Implementation
Does Trevor's Toys have the resources and capabilities to increase its product offering? What
would be required to make that happen?

As with almost all project work, you won' t have the time to do as much analysis as you would
like. The clock will continue to run as you ask each question. Keep this in mind as you select the
data you would like to have.

With each question, you will be scored according to both the significance of the question and
time. The more significant the questions you select, the higher your score in the topic area will
be. But each question takes time to answer because of the need to find and interpret the data.
Consequently, each answer deducts a predetermined number of days from the project calendar.

You can make a recommendation whenever you feel you have learned enough to make a
persuasive argument to support your conclusions. Each answer you receive is recorded in the
notebook, which opens in a separate browser window. You can refer to it at any time.

Questions: Consumer Research


• What kind of customer overlap is there between Trevor's and the larger low-tech stores? 2
days
• What do customers like about shopping at Trevor's? 3 days
• Why do customers shop at larger toy stores? What do they like/dislike? 2 days
• How much money do customers spend in traditional stores (Trevor' s Toys, competitors')? 2
days

Consulting Interview Book


Compiled by Vivek Pundir, Goizueta MBA’06
Page 128 of 666
Goizueta Consulting Association
Goizueta Business School, Emory University, 1300 Clifton Road, Atlanta GA 30322

• What are the attributes of Trevor'


s Toys'customers? 3 days
• What are customers looking for in an ideal toy store? 4 days
• Would customers do all of their toy shopping at Trevor's if the company were to offer both
low- and high-tech toys? 3 days
• How loyal are Trevor' s customers? Are many sales repeat sales to the same customers? 3
days
• How much time is required for an average toy shopping trip in a traditional store (ours,
competitors')? 2 days
• Who are Trevor' s Toys'primary customers shopping for? 1 day
• Do customers shop online for toys? Why? For what kinds of toys? 1 day
• How do they decide if they are going to purchase online? 2 days

Questions: Competitors
• Who are Trevor' s Toys'traditional competitors? 2 days
• Who are the market share leaders in the total toy market? 1 day
• What are the different product offerings among online competitors? 2 days
• What are the competitors'growth rates? 1 day
• What do competitors offer that we can' t? 3 days
• Are the competitors making any profit? 1 day
• What are the competitors'pricing policies? 2 days
• How are the competitors promoting and marketing products? 2 days
• What is the selection of low-tech toys available at the competition? 2 days
• What is the selection of high-tech toys available at the competition? 2 days
• Other policies affecting customers (returns, customer service, etc.)? 2 days
• What sites for online toys attract the most traffic? 1 day
• Who are the market share leaders in the online market? 1 day

Questions: Economics
• What is Trevor's Toys'historical financial performance? 1 day
• On average, what are the margins on low and high-tech toys? 2 days
• How much additional retail space would be needed? Is it available? How much would it cost?
3 days
• Would additional staff be needed? 2 days
• Would the company need additional warehouse space? If so, how much would this space
cost? 3 days
• How does distribution currently take place? 1 day
• What will be the average high-tech toy purchase amount per visit in 2007? 1 day
• What is the additional revenue per customer projected if Trevor's expands its product
offering? 2 days
• What additional costs are incurred by processing orders for online retailers? 2 days

Questions: Market Sizing


• What is the overall size of the toy market? 2 days
• What is the size of the high tech toy market? 2 days

Consulting Interview Book


Compiled by Vivek Pundir, Goizueta MBA’06
Page 129 of 666
Goizueta Consulting Association
Goizueta Business School, Emory University, 1300 Clifton Road, Atlanta GA 30322

• How fast has the toy market grown over the past five years? 1 day
• How fast has the high tech toy market grown over the past five years? 1 day
• Why is the toy market growing so fast? 2 days
• What is the average amount spent per child? 2 days
• What is the size of the online toy market today? 1 day
• What is the anticipated growth for the online toy market? 2 days
• How much of online spending is expected to be for high tech toys? 2 days

Questions: Implementation
• Can the current facilities be used for the sale of low-tech toys? 2 days
• How does management at Trevor' s Toys view the low-tech business as a competitive threat?
3 days
• Are the suppliers of low- and high-tech toys different? 2 days
• Where would Trevor' s Toys source the low-tech toys? 2 days
• What inventory system does Trevor' s currently use? 1 day
• What is the organizational structure at Trevor' s Toys? 3 days
• Would the current internal culture fit the venture into low-tech toys? 3 days
• How will Trevor' s Toys advertise the change? 2 days
• Are there any international trade issues? 1 day

Would customers do all of their toy shopping at Trevor' s if the company were to offer both low-
and high-tech toys?
Although many customers would purchase both types of toys at Trevor' s if they were available, it
is unlikely that customers would stop going to traditional toy stores. Given their wider selection
of toys, larger size, and lower prices, those stores will continue to be a major draw for many of
Trevor' s customers.

What do customers like about shopping at Trevor' s?


Most customers surveyed cited the knowledgeable and friendly sales staff, good selection of
high-tech toys, ease of store navigation and relatively short lines, and lack of any "feeding
frenzy" atmosphere as their primary reasons for shopping at Trevor' s. Also, Trevor's recently put
in a rating system for its toys and games to help parents better understand the violence level of
the toys they are buying. Interestingly, many customers said they had a relatively clear idea of
what they hoped to buy when they came to the store.

Why do customers shop at larger toy stores? What do they like/dislike?


The most common response to why customers shop at the large stores was that these stores had
the widest selection of high- and low-tech toys. In addition, many people said they often went to
the larger stores with less idea about what they were going to purchase and would go with their
kids and spend more time browsing the aisles to make their selection. However, a number of
respondents mentioned that they disliked these stores because of the large crowds and long lines
that are almost always present.

How loyal are Trevor'


s customers? Are many sales repeat sales to the same customers?

Consulting Interview Book


Compiled by Vivek Pundir, Goizueta MBA’06
Page 130 of 666
Goizueta Consulting Association
Goizueta Business School, Emory University, 1300 Clifton Road, Atlanta GA 30322

Most of Trevor'
s customers are fairly loyal to the company and visit the store, on average, three
times a year.

Do customers shop online for toys? Why? For what kinds of toys?
Yes, a number of customers shop online. For the most part, people purchase online for the
convenience of not having to visit the store. The amounts of high-tech and low-tech toys sold
online are roughly equal and are expected to remain so for the next few years. An exit survey at a
number of Trevor' s stores revealed that only 20 percent of Trevor'
s current customers shop
online.

Who are Trevor' s Toys'traditional competitors?


The biggest traditional store competitors are Toys R Us and the large discount stores, like Wal-
Mart, J.C. Penney, and Target. They each have hundreds of stores nationwide. Each of these
stores sells a small selection of very competitively priced SKUs, which Trevor's Toys also
carries. Trevor's Toys differs by offering a wider product range and narrower geographic
coverage. Trevor' s Toys is located only in the U.S. Northeast and Midwest. No one else offers
the same variety and depth of product.

What do competitors offer that we can' t?


The large competitors offer a great variety of toys in all product categories. The competitors also
have national coverage. Currently, Trevor' s Toys does not serve customers outside its store
regions.

What are the competitors'growth rates?


The large stores (Toys R Us and Wal-Mart) have been seeing 3 percent and 7 percent growth in
this category over the past few years. The pure-play online toy retailers have been growing at
very high rates of between 75 percent and 150 percent per year since they began operation in
1996. BrainSmart and SmartKids are both private companies (revenue numbers aren' t publicly
available), but growth estimates for both are in the range of 50 percent to 100 percent since
startup, though from a much smaller starting base.

What are the competitors'pricing policies?


Toys R Us and Wal-Mart have been in a prolonged battle for low prices, particularly for low-
tech toys. Historically, both have been known to make certain popular toys loss leaders to drive
traffic into the store and encourage people to buy higher-margin products. The online sites such
as Amazon.com and eToys price their product in a similar way.

Who are the market share leaders in the total toy market?
Toys R Us, the historical market share leader, has 25% of the total toy market, but has recently
been surpassed by Wal-Mart, which has 30%. None of the other retailers have more than 8%.
Trevor's Toys currently has 3% of the market overall and 10% of the high tech toy market.

What is the additional revenue per customer projected if Trevor' s expands its product offering?
This can be broken down into two components: 1) The 1.6 million existing Trevor' s customers
are projected to spend approximately $135 per visit (up from $125 now). These customers
generally visit the store three times per year, on average, and will continue to do so. 2) In

Consulting Interview Book


Compiled by Vivek Pundir, Goizueta MBA’06
Page 131 of 666
Goizueta Consulting Association
Goizueta Business School, Emory University, 1300 Clifton Road, Atlanta GA 30322

addition, Trevor'
s hopes to attract about 500,000 new customers who will visit the twice a year
and spend $40 per visit.

How much additional retail space would be needed? Is it available? How much would it cost?
Each store is approximately 3,000 square feet and would need to be tripled to accommodate the
increased inventory. The current store base is mainly in traditional and strip malls, where space
is relatively limited, and rent averages approximately $50 per square foot.

Would additional staff be needed?


Yes. Though the warehouses might need to add a few people, the majority of additional staff
would be needed in retail operations. Trevor' s estimates that each of its 30 stores would probably
require five new staff. The standard cost for a retail employee, including training and health care,
is $40,000 per year.

Would the company need additional warehouse space? If so, how much would this space cost?
The company currently leases four warehouses (two each in the Northeast and the Midwest).
Each warehouse is approximately 75,000 square feet, and management estimates that it would
need to triple the size of each one to make room for the low-tech toys. The current warehouses
are in relatively cheap locations; additional space is available at $2 per square foot.

On average, what are the margins on low and high-tech toys?


On average, low-tech toys have a gross margin per item of about 15 percent and high-tech toys
about 20 percent. Both of these numbers vary widely from item to item, however, given the
pricing policies of most retailers.

What is the overall size of the toy market?


Approximately $30 billion is spent annually in all toy categories in the U.S.

What is the size of the high tech toy market?


Approximately $10 billion is spent annually on high tech toys (including video games) in the U.S.

How fast has the toy market grown over the past five years?
The overall toy category has been growing at 6 percent annually over the last five years.

How fast has the high tech toy market grown over the past five years?
The high tech toy market has been growing at 20% annually over the past five years. Projections
are that the number of high tech toys bought online will grow faster than the number of
traditional toys.

Why is the toy market growing so fast?


The likely reason is that the amount of disposable income for parents and teenagers has been
increasing consistently in recent years. There is no conclusive proof of this rationale.

Where would Trevor' s Toys source the low-tech toys?


The new venture into low-tech toys would require Trevor'
s to form relationships with a large
number of toy manufacturers from which it does not now receive inventory. There are a number

Consulting Interview Book


Compiled by Vivek Pundir, Goizueta MBA’06
Page 132 of 666
Goizueta Consulting Association
Goizueta Business School, Emory University, 1300 Clifton Road, Atlanta GA 30322

of large low-tech toy companies such as Mattel and Hasbro that Trevor's does not deal with now,
but would need to purchase from, if it moved into the low-tech toy market.

Are the suppliers of low- and high-tech toys different?


Yes, many of the high-tech toy companies are large firms that produce video games as well as a
host of other products. This is in contrast to the low-tech toy companies, which are typically
focused mainly on the toy market.

Can the current facilities be used for the sale of low-tech toys?
Yes, but only with additional space. The current inventory system could be expanded with little
effort or expenditure to include the new toys to be stocked, but the warehouse and retail spaces
would need significant additions.

How does management at Trevor' s Toys view the low-tech business as a competitive threat?
Most of the senior managers view the low-tech business as something that could be an additional
revenue stream in the future. They realize that most of their key competitors currently sell both
low- and high-tech toys, but don' t see the low-tech market as a threat to the high-tech market.
They believe that this is reflected in the different growth rates of the sectors.

Would the current internal culture fit the venture into low-tech toys?
Most of the front-line personnel are young, tech-savvy individuals. And although there is no
doubt that Trevor's will need to hire a number of front-line people to deal with the increased
number of products, there is some concern that this will dilute the overall quality of the sales
force.

Days Spent = 50

Should Trevor's Toys go ahead with plans to offer low-tech toys?


No, avoid expanding into low-tech toys.

Recommending that Trevor'


s Toys avoid expanding into low-tech toys is correct.

A recommendation that Trevor' s Toys avoid an entry into the low-tech toy market is likely to
meet with strong agreement and support from Trevor' s Toys'senior management. An expansion
of the current offering to include low-tech toys is unlikely to generate enough revenue to offset
the additional costs associated with the change. In addition, Trevor' s may lose some customers
who no longer feel that Trevor' s offers the same benefits as before, such as knowledgeable and
friendly customer service and a less hectic shopping experience than in the large stores.

Your overall direction against an increased toy offering seems sound, but it is important to assure
senior management that the decision is defensible and the rationale is robust. There are five areas
that senior management and the directors would like to understand before they present the
findings to the board. Now you must develop recommendations for each of those areas based on
the information and data you gathered. Good luck.

• What value would be created or destroyed for customers with a full toy offering at Trevor'
s?

Consulting Interview Book


Compiled by Vivek Pundir, Goizueta MBA’06
Page 133 of 666
Goizueta Consulting Association
Goizueta Business School, Emory University, 1300 Clifton Road, Atlanta GA 30322

• How would competitors respond?


• How attractive is the low-tech toy market?
• Would Trevor' s Toys make money?
• What are the implementation issues?

Now that you've performed your analysis, you need to integrate your information into a cohesive
whole. Often overlooked, this is a key aspect of every assignment. It'
s important to highlight the
main findings and show how they relate to each other. The way to begin is to decide whether
Trevor's Toys should offer low-tech toys. To make this recommendation, you should revisit the
questions you asked and the analysis you performed to help you support your conclusions.

What value will the customers get from an online offering?


Trevor's Toys is unlikely to meet customers'needs by expanding its offering. First and foremost,
doing so might detract from the traditional shopping experience at Trevor' s. Trevor' s Toys is
widely recognized for its knowledgeable and friendly sales staff. If it increases the product
offering, the company will have to hire more retail staff and may have problems finding the same
quality of people. In addition, the increased customer traffic is likely to result in heavier crowds,
longer lines, and more stockouts. Management should be very wary of alienating its core
customers given their loyalty to the stores, illustrated by their frequent visits (three times a year).

From the questions you asked, it appears that you have a good grasp of the Trevor's Toys
customer and how an expansion to low-tech toys might or might not appeal to them. You have
answered enough of the questions to sit with the senior management team and convince them
that they have enough informaiton to make a decision on whether or not to offer the low-tech
toys. You delved into the important customer needs and desires to better understand how a low-
tech toys offering would address those needs.

How are the competitors positioned?

By expanding its offering, Trevor' s would be going head-to-head with the two largest players,
Toys R Us and Wal-Mart. One or both of them might engage in a pricing war if they felt
threatened by the change. In addition, Trevor' s would be competing with the online retailers that
sell both low- and high-tech toys. Although online product selection is a key to success,
customer service remains a significant differentiator, so there is less risk for the company in that
area. The customer service levels at the big competitors range from low to medium and Trevor' s
would have to be extremely careful not to let this area drop.

The two additional areas where Trevor' s Toys is able to distinguish itself from the competitors
are in offering a toy rating system and employing a knowledgeable sales staff. These factors are
less relevant for low-tech toys and would not provide an edge over the competition

You have a reasonable understanding of the competition in the low-tech toy retailing industry.
There are a number of key competitive points you discovered that will help Trevor' s Toys decide
whether or not to expand into low-tech toys. When asked by Trevor' s Toys, you will be able to
put together a compelling story of the competitive landscape and justify the move to go online.

Consulting Interview Book


Compiled by Vivek Pundir, Goizueta MBA’06
Page 134 of 666
Goizueta Consulting Association
Goizueta Business School, Emory University, 1300 Clifton Road, Atlanta GA 30322

How attractive is the market?


The low-tech toy market is somewhat attractive from a revenue standpoint, but it is lower margin
and growing significantly slower than the high-tech side of the market. And although the market
size for low-tech toys is very large at $20 billion, Trevor'
s forecasts estimate that the company
would pick up less than 1 percent of this segment, a substantial risk given the potential alienation
of some of its core customers.

Market size is one of the most important questions senior management must answer when
considering any venture into low-tech toys. From the questions you asked, you appear to have an
excellent understanding of the size of the overall toy market and how the high and low-tech
markets are different. Before senior management is willing to put time and resources against
such a significant venture, it wants to be sure that the market potential is substantial. During the
final presentation to the board of directors, you will be able to solidly back up the estimations of
the total retail toy market and convince everyone that it may not be the best strategy for Trevor' s
to follow.

How will Trevor' s Toys make money?


In short, it is difficult to see how Trevor'
s Toys would make money if it began offering low-tech
toys. To come to this conclusion we need to look at the basic revenue and cost streams associated
with the proposed endeavor

Firstly, looking at the projected revenue stream we can see that Trevor'
s would expect to receive
an incremental $88 million from selling low-tech toys. This has two components:

1) $48 million from existing customers: (three visits per year) * ($10 more per visit) * (1.6
million customers)

2) $40 million from new customers: ($40 per visit) * (two visits per year) * (500,000
customers)

However, this does not take into account any loss of existing customers that might occur if
service levels change.

Second, the costs associated with the increased product line should be examined. The three
largest components of this are cost of goods sold, additional rent for new space, and the costs of
hiring additional staff.

(A) Cost of goods sold (cost of purchasing the toys from the manufacturers) is about 85 percent
of revenue and amounts to about $75 million: ($88 million revenue) ($88 million revenue) *
(15 percent margin)

(B) Trevor's will have an additional $10 million of rent per year, which, roughly speaking, can
be broken down into the following:

1) $1.2 million for warehouse space: (four warehouses) * (150,000 sq.ft. needed per
warehouse) * ($2 per sq.ft.)

Consulting Interview Book


Compiled by Vivek Pundir, Goizueta MBA’06
Page 135 of 666
Goizueta Consulting Association
Goizueta Business School, Emory University, 1300 Clifton Road, Atlanta GA 30322

2) $9 million for retail space: (30 stores) * (6,000 sq.ft. needed per store) * ($50 per sft)

(C) Finally, the additional cost of retail personnel is approximately $6 million per year, based
on the following: (30 stores) * (5 new staff per store) * ($40,000 cost per person)

The sum of these costs is $91 million, more than $3 million more than the aggressive revenue
forecast. Therefore, it is clear that the venture into low-tech toys is unlikely to be profitable for
Trevor's in the near term.

The question looming on every executive' s mind during a new business venture is, "Can we
make money?" To answer this question, it is important to investigate and develop a set of
assumptions about the potential of the business. From the questions you asked, the revenue
potential and predicted costs are clear. When determining the viability of ecommerce ventures, it
is imperative that you gather as much data as possible to solidify your estimates of future growth
and profit potential. The board of directors should feel assured othat your decision is the right
one based on the data you collected and your estimations. However, you may have gathered too
much data in this area and risked missing information on other important topics. Take care to use
your time wisely, balancing the need to gather enough data to support solid conclusions against
the danger of spending time reconfirming established facts.

What are the implementation issues?


The key to implementation lies in Trevor' s ability to source new toys. This could present a
problem, as the company does not currently have strong relationships with many of the major
low-tech toy makers and will probably have to exert significant effort to build them. In addition,
because Trevor' s is so small (30 stores) in comparison with the large toy retailers, it probably
will not be able to source the products at the same low prices as the bigger companies.

Many companies fall short on implementation when investigating a new business venture. To
ensure success, it is very important to investigate all of the implementation issues and potential
barriers. It appears that you asked enough questions to adequately understand whether there were
any serious challenges to expandig Trevor' s product offering. You uncovered the key points that
may be bottlenecks or hurdles during implementation stages. Trevor' s Toys executives will be
quick to drill into the implementation issues, and you will be able to show them that the company
can execute the changes required to go into low-tech toys.

The management team from Trevor' s Toys will now review your work and decide what to do.
The team is grateful for your help in understanding the issues and challenges it faces in pursuing
a low-tech toy strategy, as well as for the solutions you'
ve proposed.

Consulting Interview Book


Compiled by Vivek Pundir, Goizueta MBA’06
Page 136 of 666
Goizueta Consulting Association
Goizueta Business School, Emory University, 1300 Clifton Road, Atlanta GA 30322

BCG - GenCo
Interviewer
Your client is GenCo, a large, international, diversified company with a health care division that
produces a wide variety of medical instruments and related services. Five years ago, it expanded
into the health care software industry by purchasing MedCount, which markets administrative
systems to large U.S. hospitals. These systems are designed primarily for back-office functions;
they are not designed for managing patients or providing other physician and technical support.
Since it was purchased, the software division has failed to deliver the growth needed to justify
the multiple GenCo paid for it. GenCo feels it has already squeezed margins as much as possible,
and now is looking for new sales opportunities. MedCount turned to BCG to help identify
potential ways to increase revenues. How would you approach this problem?

Candidate
First, let me make sure I understand the problem. The parent company produces medical devices
and services, but before the acquisition was not involved in health care software. The company it
purchased, MedCount, sells only administrative systems software to large hospitals. It is now
looking for opportunities to increase revenues.

Interviewer
That is correct.

Candidate
Could I take a moment to jot down a few thoughts?

Interviewer
Sure, that would be fine.

Candidate
I would suggest using the following framework: First, I' d want to understand the market size and
growth rates for MedCount' s market and related software markets. Next, I would like to explore
the competition and their market shares. Third, I would like to examine customer requirements
and then, given those external conditions, look at the division'
s capabilities to understand how
well prepared it is to meet the needs of the marketplace.

Interviewer
That sounds fine. So what do you want to know about the market?

Candidate
Well, the first hurdle would be to identify the markets the company would be interested in.
Besides administration systems, what other types of medical software systems do large hospitals
purchase?

Interviewer
There are many software systems, but for the sake of time, the team focused on three primary
markets: administration systems, patient administration, and physician support systems.

Consulting Interview Book


Compiled by Vivek Pundir, Goizueta MBA’06
Page 137 of 666
Goizueta Consulting Association
Goizueta Business School, Emory University, 1300 Clifton Road, Atlanta GA 30322

Candidate
What do those systems do?

Interviewer
Patient administration includes systems like admissions and tracking. Physician support systems
are more specialized, for individual physician procedures.

Candidate
I would like to know how large each market is and how fast each is growing. I would use
secondary sources such as press releases, analyst reports, and published market studies, to obtain
this information.

Interviewer
Great! That is what we did during the market study. Our information revealed the following
market sizes and growth rates.
Administration Patient administration Physician support
Market size ($M) 1,500 1,000 1,200
Growth rate 5% 5% 12%

Candidate
From a size and growth perspective, physician support systems looks like a very attractive
market. I'
d like to know a little about the customers themselves. The client is currently targeting
large hospitals. Approximately what percentage of the market do they represent?

Interviewer
We were unable to get an exact breakdown, but we know that these hospitals make up the vast
majority of the total medical software market.

Candidate
That would make sense, since the more sophisticated procedures at a hospital might necessitate
more advanced software solutions. I know that there have been a lot of changes in the industry as
a result of managed care. I don'
t know much about the industry, so I would want to look at
market studies and press clippings to get a better sense of the hospital market in general and any
technology or software trends more specifically.

Interviewer
Okay. Let' s say that you did that and were presented with this summary of market trends:
• Consolidation in the industry, with three to four large hospital networks dominating 45
percent of the market
• Cost controls instituted, particularly as these large hospital networks acquire smaller hospitals
(centralization of functions being a key cost issue)
• Many hospitals seeking to consolidate their vendor base
• With regard to technology, many hospitals upgrading their older systems

Candidate

Consulting Interview Book


Compiled by Vivek Pundir, Goizueta MBA’06
Page 138 of 666
Goizueta Consulting Association
Goizueta Business School, Emory University, 1300 Clifton Road, Atlanta GA 30322

If hospitals are consolidating vendors, perhaps our client has an advantage in being part of a
larger medical company. Maybe the client could also gain some advantages by expanding into
other software segments. Are the people responsible for purchasing software at the hospital the
same for all three segments?

Interviewer
Like all things, it differs by hospital, but the larger hospital networks, have tried to consolidate
their purchasing not only within but also across hospitals.

Candidate
Is the decision maker for medical software the same as for medical instrumentation and devices?

Interviewer
In some cases, the head of purchasing influences both decisions, but the person who makes the
final choice is different. Software decisions are usually made by the hospital IT function, and
those for instrumentation by the medical staff.

Candidate
I think I have a pretty good understanding of the market for now. Let'
s look at competition next.
We could identify all the competitors and build up the market shares using a combination of
public data and estimates.

Interviewer
Well, let'
s assume that you don' t have an infinite amount of time to look at all the competitors.
You can only look at the top five competitors in each market. You are given the following data:
Administration Systems Sales ($M) Growth (%)
MedCount 700 4%
HCS Software Systems 100 7%
Morningside Software 80 3%
Admin Systems Solutions 70 2%
HTI Software 50 15%
Patient Administration Sales ($M) Growth (%)
HTI 300 5%
Registration Software Solutions 240 4%
Signup Software 60 3%
HCS Software Systems 30 16%
Patient Software 20 -1%
Physician Support Sales ($M) Growth (%)
HCS Software Systems 150 16%
Physician Support Systems 100 11%
Medical Technology Inc 25 18%
HTI 20 32%
MedSys 5 15%

Candidate
Very interesting. The first thing I would note from the data is that the market concentrations are
very different. In administrative systems, the top five competitors control 66 percent of the
market and in patient administration, they control 65 percent. But in the physician support
market, they control only 25 percent.

Consulting Interview Book


Compiled by Vivek Pundir, Goizueta MBA’06
Page 139 of 666
Goizueta Consulting Association
Goizueta Business School, Emory University, 1300 Clifton Road, Atlanta GA 30322

I would want to know what gross margins look like in each of these markets as well. I might turn
to analyst reports and look at competitors'financial statements to deduce whether they are
making money in each market.

Interviewer
Gross margins vary, of course, but the analyst reports have margins of 25 to 30 percent for
administrative systems and for patient administration. For physician support, the margins tend to
be higher, more like 45 to 50 percent.

Candidate
I see that two competitors, HTI and HCS Software Systems, have very large revenue growth in
all three sectors, although they each dominate one. I would want to look at their financials,
annual reports, and press releases to find out a bit more about their strategy in each of these areas.

Interviewer
You' d find that they recently entered these noncore markets. Why might they have done that?

Candidate
Perhaps, like our client, each had a strong position in its own segment, HTI in patient
administration and HCS Software Systems in physician support. Maybe they too decided to
branch out into the other segments to find additional growth.

Interviewer
That is a very good hypothesis. Let'
s say there is evidence in the sources you consult that
supports your assertion.

Candidate
Well, if that were true, these two companies could be a threat not only in the other two segments,
but also in our client'
s segment, administrative systems. It looks as if the client is slowly losing
market share in its segment, since it is growing more slowly than its market.

Interviewer
Good observation.

Candidate
The market and competitor trends could also suggest that the client may want to enter these other
markets. In particular, the physician support market looks attractive, given it has high growth and
lack of a dominant competitor. The higher gross margins may provide attractive returns on the
necessary investment in software development. However, the patient administration market may
also be attractive. Although it is more concentrated and offers lower margins than physician
support, the client may be able to enter this segment with a smaller up-front investment. Given
the trend toward upgrading existing computer systems, it may be important for MedCount to
have a product offering in each of the three market segments. That should not be too difficult,
since the company is already in the software industry.

Interviewer

Consulting Interview Book


Compiled by Vivek Pundir, Goizueta MBA’06
Page 140 of 666
Goizueta Consulting Association
Goizueta Business School, Emory University, 1300 Clifton Road, Atlanta GA 30322

Perhaps, but you should think a little more closely about these types of software. Are all software
systems alike?

Candidate
Well, let me think about that for a moment. I suspect patient administration would have
relatively low entry barriers. From your earlier description, these systems appear to be pretty
basic, dealing primarily with admissions and patient tracking However, the entry barriers in
physician support might be higher, since these systems are more complex and there are probably
multiple systems for the various physician procedures. I guess it would be harder to get into
those types of systems.

Interviewer
That would make sense.

Candidate
Since the company might want to go into only some of the segments, I would want to know how
important it is to have products in all three segments. Do we know if the competitors are
marketing their products as a bundle?

Interviewer
How might you find that out?

Candidate
Since it would be difficult to talk to a competitor directly, I would probably target a competitor'
s
customer, particularly one that just converted from our client' s software.

Interviewer
Let's say you get an interview with a customer that recently switched to HTI. You discover that
the competitor was offering it a better pricing deal and service for software products in all three
segments.

Candidate
How were MedCount'
s software and service perceived in relation to those of competitors?

Interviewer
The customer thought that its administrative systems were adequate, "the old standby," but not
stellar.

Candidate
Were there any other key reasons it switched from MedCount'
s system?

Interviewer
When it decided to upgrade its systems, it tried to contact MedCount, but could never get a
representative to describe its options.

Candidate

Consulting Interview Book


Compiled by Vivek Pundir, Goizueta MBA’06
Page 141 of 666
Goizueta Consulting Association
Goizueta Business School, Emory University, 1300 Clifton Road, Atlanta GA 30322

Interesting. How did HTI perform?

Interviewer
The HTI representative had heard that the company was considering switching software vendors
and provided a sales representative to pitch HTI'
s administrative product the next day.

Candidate
It definitely sounds as if there was a problem with the sales function and that customer relations
need to be improved, particularly for the larger hospital chains. There also seems to be an
advantage from both a marketing and sales perspective in having multiple software products. I
would want to confirm those views by doing further interviews.

Interviewer
Let's say further interviews support those assumptions.

Candidate
Since we have already looked at the external conditions, I would like to move on to the client
itself. I'
d like to know more about its marketing and selling organization as well as its software
development skills.

So far, we know that our client offers administrative software and that there may be a problem
with sales and marketing. Could you tell me a little about the marketing department?

Interviewer
The marketing department is organized regionally. Teams are assigned to hospitals within each
state or geographic region, such as New England.

Candidate
That could explain some of the problems with MedCount' s marketing and sales. If hospital
purchasing is centralized, the marketing organization may be outdated. Does the company have
any teams dedicated to the four or five biggest hospital networks?

Interviewer
No, there are no dedicated teams. They talked about doing that for a while, but it conflicted with
the regional structure it had in place.

Candidate
With regard to software, does the company feel it has any strengths or weaknesses?

Interviewer
It feels that their administrative product is very strong ("best of breed") and is the dominant
technology. Also, the product is modular in design, which allows for easier upgrades. Although
the company has never branched out into other market segments, the software developers believe
that certain modules could be used to build the foundation for other administrative software
programs. The company feels customer support is also an area in which it excels.

Consulting Interview Book


Compiled by Vivek Pundir, Goizueta MBA’06
Page 142 of 666
Goizueta Consulting Association
Goizueta Business School, Emory University, 1300 Clifton Road, Atlanta GA 30322

Candidate
Let's start with our client'
s market. The client dominates the administrative software market,
which is fairly large but growing slowly, and the company appears to be slowly losing market
share. Patient administration is also growing relatively slowly. Both markets are relatively
concentrated and appear to offer lower margins than physician support. The physician support
market is large and less concentrated, and could potentially provide higher margins, but would
require a larger investment. The hospital market itself is becoming more concentrated and is
pushing to consolidate vendors. The purchasing agent is often the same for the three types of
software.

Looking at our client's competitors, two, HTI and HCS Software Systems, appear to be
particularly threatening. Each has a dominant position in one segment and is branching out into
other areas. They appear to be marketing their products and services as a bundle and are using
service as a key point of differentiation.

The client offers only one type of system and appears to have some weaknesses in its marketing
organization, particularly in marketing to the larger hospital networks, which offer the most
promising market opportunities.

Interviewer
How would you recommend proceeding?

Candidate
The first priority should be to fix the marketing organization, particularly for the large hospital
networks. MedCount will have trouble expanding into new markets if it can' t defend its current
position and shore up its existing customer relationships. There should be a team dedicated to
each of the major chains. The client should also look at improving customer tracking so that it is
clear when its customers are going to upgrade. There should also be clear contacts so that the
customer can easily keep in touch with MedCount.

Next, I would recommend that the client explore entering the other market segments by
leveraging its dominant position in administrative systems. At first glance, patient administration
does not appear to be very attractive, with slow growth, low margins, and large, dominant
competitors. There appears to be some advantage, however, in having products across the
product range. I would recommend that we interview some of MedCount' s existing customers to
better understand their needs and future IT requirements. If the customer base is interested in one
software provider for both back-office administration and patient administration functions, this
segment looks promising.

If the client does decide to enter this market, it should look at the lowest-cost method of entry,
either developing a product internally or acquiring a competitor. The modular design of its
existing administrative software suggests internal development of the patient administration
product may be the way to go, but we would need a more thorough comparison of the internal
development and acquisition options, including both cost and time to market. I think that
physician support offers our client an exciting growth opportunity, given its high margins, high
growth, and fragmented competition. I would definitely think about an acquisition strategy, since

Consulting Interview Book


Compiled by Vivek Pundir, Goizueta MBA’06
Page 143 of 666
Goizueta Consulting Association
Goizueta Business School, Emory University, 1300 Clifton Road, Atlanta GA 30322

the client may lack the technical capabilities to enter this specialized market. I would recommend
going for one of the larger companies, as that would give the client a stronger position. Smaller
companies would probably not offer an important enough position in the market. More research
would be needed, however, for us to better understand the intricacies of the market and each
potential acquisition.

Interviewer
Those are very interesting conclusions. Thank you.

Consulting Interview Book


Compiled by Vivek Pundir, Goizueta MBA’06
Page 144 of 666
Goizueta Consulting Association
Goizueta Business School, Emory University, 1300 Clifton Road, Atlanta GA 30322

BCG - Fighter Jet


Interviewer
Your client is a U.S. defense contractor that manufactures the Mohawk Light Fighter Jet for the
British Royal Air Force. The company has produced the $20 million fighter jet for the past 12
years. The British government has decided to put the contract out to bid, however, and to win the
program, the client' s purchasing agents have estimated, the company will need to cut its costs by
5 percent. It has asked BCG to help it reduce costs.

Candidate
Let me first clarify the question. The client manufactures a $20 million jet and, because of
competitive forces, has to reduce its cost by 5 percent. Is BCG's role also to verify the purchasing
department' s estimate?

Interviewer
No, you can assume that the purchasing estimate is correct. BCG'
s role is to find the cost savings
to meet that estimate.

Candidate
Could I take a few minutes to think about the case?

Interviewer
Sure, please do so.

Candidate
First, I would like to understand the cost structure of the jet to see what we should look at first.
Next, I would like to look at major factors driving the costs we are targeting. Finally, I would
like to explore potential ideas to reduce cost.

Interviewer
That sounds like a very logical approach. Let'
s proceed.

Candidate
Because the time for the interview is limited, I think we should try to identify those areas most
responsible for the cost of the jet.

Interviewer
Time is limited on real projects as well, so I think that would be a good idea! You have the
following cost information for the jet. How would you interpret it?

Consulting Interview Book


Compiled by Vivek Pundir, Goizueta MBA’06
Page 145 of 666
Goizueta Consulting Association
Goizueta Business School, Emory University, 1300 Clifton Road, Atlanta GA 30322

Candidate
The major cost driver for the jet appears to be purchased materials. Within manufacturing, direct
labor is a fairly large component of cost, as are program management and corporate overhead
within overhead. I think we would want to concentrate most on materials, however, since that' s
where most of the costs can be found.

Interviewer
That sounds like a good place to start. Where would you look within materials?

Candidate
I see that materials are broken down into purchased subassemblies, components, and raw
materials. I understand what raw materials would be, but what would be the difference between
components and subassemblies?

Interviewer
A subassembly functions on its own. An example is the pilot night vision system. A component
is a smaller part, such as a part of the engine.

Candidate
I know that governmental agencies often have very strict guidelines about purchasing that could
affect the cost of materials.

Interviewer
For the sake of this case, you can assume that the British Ministry of Defense, MOD, allows
"commercial off-the-shelf" purchases, which means that the client is free to purchase from
whomever it wants, as long as it can ensure that the parts meet MOD quality guidelines.

Consulting Interview Book


Compiled by Vivek Pundir, Goizueta MBA’06
Page 146 of 666
Goizueta Consulting Association
Goizueta Business School, Emory University, 1300 Clifton Road, Atlanta GA 30322

Candidate
I see that purchased subassemblies comprise more than 70 percent of materials. How many
suppliers are there for these subassemblies?

Interviewer
There are seven suppliers of major subassemblies that go into the fighter jet.

Candidate
That seems like a relatively small number. Are there more suppliers that are qualified to do this
type of work?

Interviewer
The manufacture of these parts requires a substantial investment in R&D, engineering, and
infrastructure. It would be very costly for new suppliers to make the required investment,
particularly if the client is trying to reduce the price it pays to the subassembly manufacturers.

Candidate
Since there are only a few subassembly suppliers, and the investment hurdle would preclude
bringing in competing manufacturers, it would be difficult to reduce the price paid. Perhaps we
should look elsewhere for savings.

Interviewer
But remember, if your client loses the contract, it will lose its customer unless it is teamed with
the competing bidder. Even then, if the competitor is underbidding your client, there will be even
less room for it to profit.

Candidate
Perhaps it would have an incentive to reduce its costs in order to maintain the contract. Are the
majority of its costs in materials as well?

Interviewer
How could you find that out?

Candidate
I would want to interview the purchasing and engineering personnel of the different
subcontractors in order to understand their cost structures. If we had a better understanding of
their economics, our client might be able to reduce cost across the board, allowing it to compete
more effectively for the contract without killing everyone' s margins.

Interviewer
Let's say that purchased materials average approximately 70 percent of the price paid to most of
the manufacturers.

Candidate
If the cost of subassemblies represents 40 percent of the jet cost and 70 percent of that is
purchased materials, total purchased materials would be approximately 28 percent of the cost for

Consulting Interview Book


Compiled by Vivek Pundir, Goizueta MBA’06
Page 147 of 666
Goizueta Consulting Association
Goizueta Business School, Emory University, 1300 Clifton Road, Atlanta GA 30322

subassemblies. Purchases of raw materials and components represent another 15 percent, for a
total of around 43 percent of the cost of the jet. If our client could reduce the cost of raw
materials by 20 percent, it could reduce the cost of the jet by more than 8 percent, more than
enough to offset the 5 percent reduction it would need to win the contract.

Interviewer
That sounds reasonable, but 20 percent is a very lofty goal. How would you go about doing that?

Candidate
First, I would look at the number of suppliers. Are there a large number of suppliers to the
subassembly manufacturers?

Interviewer
The client estimates that there are approximately 125 suppliers of raw materials and components
among the manufacturers of the subassemblies and itself.

Candidate
Well, that sounds like a large number of suppliers. Of course, they could be providing very
specialized materials to the subassembly manufacturers. Are these suppliers providing
customized or more commodity products?

Interviewer
About 80 percent of these products are commodities, such as sheet metal and wire harnesses.
Even some of the electronics, such as printed wire boards and circuitry, are fairly generic.

Candidate
That sounds promising, but I would need to know whether these commodities are
interchangeable, so that our client could concentrate spending with fewer suppliers. Are there
many commonalities among the parts used by the different subassembly manufacturers? We
could talk to their engineers and look at the designs and bills of material to determine how much
overlap there is.

Interviewer
Let's say that you did this and discovered that approximately 30 percent of the cost of raw
materials is from similar materials used across the subassembly manufacturers.

Candidate
It seems safe to assume that the client would need more commonality to be successful in
concentrating its purchasing and reducing costs. Do the engineers believe that the percentage of
overlap could be increased if the designs were modified?

Interviewer
They believe they could increase that percentage substantially, particularly with basic materials
such as screws and sheet metal, but also in other more customized areas.

Candidate

Consulting Interview Book


Compiled by Vivek Pundir, Goizueta MBA’06
Page 148 of 666
Goizueta Consulting Association
Goizueta Business School, Emory University, 1300 Clifton Road, Atlanta GA 30322

That'
s great news, but we would still need to know whether the subcontractors are using the same
suppliers. We could analyze the number of suppliers for each of the areas of overlap.

Interviewer
Good suggestion. Although there are some common suppliers, the analysis indicates that the
subassembly manufacturers tend to use different suppliers.

Candidate
Our client needs to reduce costs by 5 percent. The largest area of opportunity appears to be in
purchased materials, the majority of which comprise subassemblies manufactured by seven
subcontractors. By looking at its purchases in total, the client can target approximately 40
percent of costs. To achieve the 5 percent cost reduction, it would need to reduce costs by 15 to
20 percent. It could try to do that by increasing commonality in the design of the subassemblies
and components and by shifting volume to a smaller number of suppliers.

Interviewer
Considering that the majority of the raw materials and components are purchased commodities,
do you think the 15-20 percent cost reduction is achievable?

Candidate
Well, I know that typically have lower margins than more customized products. I suspect it may
be challenging to hit the client'
s savings target by focusing only on these purchases. But since
raw materials and components represent about 40 percent of costs and there is an opportunity to
concentrate purchasing, I think we should start here.

Interviewer
Where else could you look for savings?

Candidate
If I look back at the cost data on the jet, direct labor is another large cost component. As a
contingency, we could look into that area as well. I' ve read that other companies use outsourcing
to lower their manufacturing costs-perhaps our client could do the same. For example, it might
want to increase its use of purchased subassemblies and reduce the amount of direct
manufacturing it does. Of course this would work only if it could drive direct labor costs below
the offsetting cost of these subassemblies. The client will be working closely with the
subassembly suppliers to implement its purchasing initiative. This may give it an opportunity to
explore the suppliers'capabilities at the same time.

Interviewer
That' s an interesting suggestion. How would you recommend the company pursue both of the
initiatives you have discussed?

Candidate
I would look first to combine purchases across the subassembly suppliers with our client' s
purchases. I suspect that the client and the subassembly suppliers will need to share a great deal
of information, including engineering drawings and specifications, with potential suppliers of the

Consulting Interview Book


Compiled by Vivek Pundir, Goizueta MBA’06
Page 149 of 666
Goizueta Consulting Association
Goizueta Business School, Emory University, 1300 Clifton Road, Atlanta GA 30322

raw materials and components. The Internet could prove to be a very effective medium for
forming a single "virtual" purchasing department to consolidate both the flow of information and
purchase orders across the companies. Our client might also want to use a bidding system for
those materials that are true commodities.

Next, I would turn to the engineering departments and form cross-company teams to look for
areas in which to increase commonality of design. At the same time, those teams could explore
opportunities to use more purchased subassemblies and decrease the client's direct labor costs.

Interviewer
That sounds great, and is very similar to a project we did. I would caution you, however, to
examine the upfront costs involved in your recommendations, both for the redesign and for the
implementation of the purchasing system, before going ahead.

Consulting Interview Book


Compiled by Vivek Pundir, Goizueta MBA’06
Page 150 of 666
Goizueta Consulting Association
Goizueta Business School, Emory University, 1300 Clifton Road, Atlanta GA 30322

BCG - Foods, Inc.


Interviewer
Your client is the sugar cereal division of Foods Inc., a U.S.-based distributor and manufacturer
of packaged foods. According to the division president, Foods Inc.' s traditional strength has been
with grocery stores, which still account for the majority of its $1.1 billion in sugar cereal sales.
But Big M Mart, a discount chain, has been growing at a healthy rate of almost 15 percent per
year and has now become Food Inc' s largest customer. Your client is not sure how to react, and
has asked BCG for assistance with its distribution strategy.

Candidate
First, let me make sure I understand the problem. Our client specializes in sugar cereals, and has
traditionally distributed through grocery stores. Sales to Big M Mart, a discount chain, have been
growing at 15 percent per year, and the chain has recently become the largest distributor of the
client's product nationwide. We are here to help evaluate the distribution strategy in light of Big
M Mart' s growth.

Interviewer
That is correct.

Candidate
Could you explain to me how grocery stores differ from discount stores?

Interviewer
Sure. Grocery stores generally specialize in food, as well as selling some household goods and
over-the-counter pharmaceuticals. Discount stores, on the other hand, offer food alongside a
wide variety of merchandise, including clothing, home electronics, and housewares.

Candidate
Does Big M Mart market its food products differently than do grocery stores?

Interviewer
Discount stores advertise lower prices for a wide variety of foods, particularly staple,
nonperishable foods.

Candidate
Could I take a moment to write a few notes to myself?

Interviewer
Please feel free.

Candidate
Before making recommendations, I think we would need to evaluate whether sales growth at Big
M Mart is good or bad for Foods, Inc. To do that, I would first look at how its sugar cereal
performance at Big M Mart compares with that in other distribution channels. Second, I would

Consulting Interview Book


Compiled by Vivek Pundir, Goizueta MBA’06
Page 151 of 666
Goizueta Consulting Association
Goizueta Business School, Emory University, 1300 Clifton Road, Atlanta GA 30322

look at its performance at Big M Mart in relation to competitors'performance. Next, I would


determine what drives customer purchases. Finally, I would want to understand the supply chain.

Interviewer
That certainly sounds like a reasonable approach. Let'
s proceed.

Candidate
First, I would like to get a better sense of where Big M Mart stands relative to our client'
s other
distribution channels by examining the client' s sales data and margins, by distributor.

Interviewer
The marketing department does not have margins by channel, but tracks the sales and volume for
its top five distributors.
Sales ($M) 1997 1999 2001 5-Yr CAGR
Big M Mart 142 162 246 14.7%
R.J.'
s 157 185 200 6.2%
Bozo Mart 143 175 189 7.3%
Ace Grocery 101 109 153 11.0%
Shoppers Mart 57 62 67 4.0%
Total Top 5 600 693 856 9.3%
Total All Distributors 1,000 1,079 1,150 3.6%

Volume (M boxes) 1997 1999 2001 5-Yr CAGR


Big M Mart 65 74 113 14.7%
R.J.'
s 72 81 85 4.2%
Bozo Mart 65 77 80 5.2%
Ace Grocery 46 47 64 8.8%
Shoppers Mart 26 27 28 2.0%
Total Top 5 274 307 370 7.8%
Total All Distributors 450 468 487 2.0%
What does this imply about Big M Mart as a distribution outlet?

Candidate
It looks as if the top distributors have been growing more important, but particularly Big M Mart,
which is growing faster than all the others. This is particularly true when we look at volume,
where Big M Mart' s growth is much higher than that of the other four channels.

Interviewer
And how could you interpret what this data says about margins?

Candidate
While the client'
s sales through other distribution channels are growing faster than volume, Big
M Mart volume and sales growth are the same, so the average price paid by Big M Mart has
Consulting Interview Book
Compiled by Vivek Pundir, Goizueta MBA’06
Page 152 of 666
Goizueta Consulting Association
Goizueta Business School, Emory University, 1300 Clifton Road, Atlanta GA 30322

remained constant. That implies that sales growth at Big M Mart could have negative
implications for our client'
s margins. Next, I would like to look at how our client is doing in
relation to the competition within Big M Mart. Have they been gaining or losing market share?

Interviewer
How might you find that out?

Candidate
I would try to interview Big M Mart'
s purchasing personnel, since they would probably track
those data for their own purposes.

Interviewer
Why would they want to talk to you? How might you approach such an interview?

Candidate
I would approach the purchasing personnel and suggest that our client and Big M Mart work
together to identify best practices to reduce costs and increase sales of sugar cereals at Big M
Mart.

Interviewer
Let's say in a perfect world you could get a breakdown of Big M Mart sales for the four largest
competitors (see market shares below).

Interviewer
What can we infer about our client'
s competitors within this channel? Who should they be
worried about?

Candidate
It looks like our client is losing market share, as is Tasty Breakfast, while Cereal Co. and Private
Label are gaining share. Private label, however, looks to be growing from a very small base.

I would like to explore why our client is losing market share to Cereal Co. at Big M Marts. Are
their prices better than those of our client'
s?

Interviewer

Consulting Interview Book


Compiled by Vivek Pundir, Goizueta MBA’06
Page 153 of 666
Goizueta Consulting Association
Goizueta Business School, Emory University, 1300 Clifton Road, Atlanta GA 30322

After a period of price wars six to seven years ago that lowered industry margins, the cereal
companies have refrained from price competition within the same channel.

Candidate
If prices are not driving the difference, I would look at other factors such as brand selection,
percentage of shelf space, product placement, and in-store promotions.

Interviewer
Visits to Big M Marts indicate that each name brand company holds 30 percent of the shelf space,
while private label has ten percent. However, Cereal Co. brands tend to be placed lower on the
shelf than your client'
s products.

Candidate
Well, I suspect that children are a large target market for the sugar cereal manufacturers. The
lower shelf placement could be especially important to children who are looking at the different
types of cereals. Are there any other promotions?

Interviewer
Some Cereal Co. brands have sales promotion tags, and the team notes that store flyers advertise
specials on Cereal Co. brands for Big M Mart customer cardholders.

Candidate
So, even if all the companies are maintaining product prices, maybe Cereal Co. is strategically
discounting prices in order to gain market share.

It seems like there is evidence of cooperation between Cereal Co. and Big M Mart. Do we know
anything about the relationship between Big M Mart and Cereal Co.?

Interviewer
During earlier discussions with Big M Mart, you discovered that your client's competitors have
50 sales representatives dedicated to the Big M Mart account. Your client has seven.

Candidate
Cereal Co. appears to be dedicating more resources to its relationship with Big M Mart than is
our client. I think I have a good sense of distribution and competition. I would like to look at the
customers and why they choose to select the products that they do.

I think I have a good sense of distribution and competition. I would now like to look at the
customers and understand why they select the products they do. One hypothesis I have is that
shifting brand loyalties are hurting our client'
s market share at Big M Mart.

Interviewer
That's interesting. What do you think might motivate purchases of sugar cereals?

Candidate

Consulting Interview Book


Compiled by Vivek Pundir, Goizueta MBA’06
Page 154 of 666
Goizueta Consulting Association
Goizueta Business School, Emory University, 1300 Clifton Road, Atlanta GA 30322

There are lots of factors, such as the games in the boxes, the price of the cereal itself, how it
tastes. To better understand consumer behavior, we should conduct market research, possibly
through focus groups, customer observation, and price sensitivity studies.

Interviewer
BCG teams often do such research. Let' s assume your team conducts some analysis. Your
research concludes that most buyers tend to fall into two categories. Approximately 60 percent of
buyers go straight to one cereal and grab it. We can call this group the "brand-loyal" shoppers.
Another 40 percent of shoppers look at all the cereals and then select one that interests them.
Let's call this group the "impulse" buyers.

Candidate
For the brand-loyal shopper, the priority would be product availability, while product placement
would be important for consumers who like to shop around.

Interviewer
In general, your research indicates that consumers are not price sensitive and are extremely loyal
to their preferred brand. But when the preferred cereal is unavailable, the brand-loyal customers
will purchase discounted cereals approximately 35 percent of the time.

Candidate
Well, from that information, it appears that price is not a major driver of purchases unless the
preferred cereal is out of stock. In these stock-out situations, you said, brand-loyal customers will
purchase discounted cereals 35 percent of the time. What happens when the customer does not
purchase a discounted cereal?

Interviewer
In approximately 25 percent of cases, the customer walks away without purchasing any cereal at
all. In the remaining 40 percent of cases, the brand-loyal customer will act like an impulse
shopper and select another brand.

Candidate
Interesting. It seems as if product availability could be a major driver of total cereal volume for
Big M Mart. Of course, we would need to know how often stock-outs that cause consumers to
walk away without purchasing cereal occur.

Since I have a pretty good understanding of customer motivation, I' d now like to ask a few
questions about the client'
s supply chain. I would want to talk to our client'
s distribution
personnel to understand the distribution process and to determine how often stock-outs occur.
Can you describe how our client' s cereal is distributed at Big M Mart?

Interviewer
Cereals are distributed from the factory to the distributor'
s warehouse twice monthly. The retailer
then stocks the shelves itself.

Candidate

Consulting Interview Book


Compiled by Vivek Pundir, Goizueta MBA’06
Page 155 of 666
Goizueta Consulting Association
Goizueta Business School, Emory University, 1300 Clifton Road, Atlanta GA 30322

Do we have any knowledge about when the individual stores are out of stock?

Interviewer
No, we do not, since our client delivers only to the warehouses and has no direct access to in-
store inventory information.

Candidate
Since we identified product availability as a key success factor earlier on, I would want to make
sure that the stores were stocking the product correctly.

Interviewer
Let's say that in your earlier in-store investigations, you found out that Big M Mart stores
averaged 15 percent of sugar cereal brands out-of-stock, across all brands.

Candidate
Stock-outs would be a major problem for our client, since 60 percent of customers look for a
specific brand of cereal and 35 percent of them would buy a discounted brand in a stock-out
situation. Big M Mart would also have an incentive to reduce out-of-stock incidents, since 25
percent of the time, a brand-loyal customer will walk away without buying anything.

At this point, I would like to summarize what I know.

Big M Mart is our client' s leading customer, accounting for more than 20 percent of our client' s
sugar cereal revenue. Although sales to Big M Mart are increasing on an absolute basis, our
client's margins there are lower than in its other channels and its competitive position is eroding
in that channel.

At Big M Mart, our client faces competition from both private label and Cereal Co., although the
latter appears to be the greater threat. There appears to be a relationship between Big M Mart and
Cereal Co. as evidenced by their joint promotions, the superior placement of the Cereal Co.
product, and the substantial resources that Cereal Co. has dedicated to the Big M Mart account.

We learned that 60 percent of customers are brand-loyal, implying product availability is most
important. However, 40 percent like to try different kinds of cereal, indicating product placement
is also important. Purchasers do not appear to be price conscious, unless the type of cereal they
are looking for is out of stock, in which case there is a stronger tendency to base purchases on
price promotions.

In terms of distribution, our client is making deliveries twice a month to Big M Mart' s
warehouses. Big M Mart, in turn, is responsible for stocking the shelves. We currently have no
direct knowledge of when our client' s items are out of stock at the individual stores, but there is
evidence that stock-outs do occur with some frequency.

Interviewer
Well, it sounds as if you understand the situation. What would you recommend the client do?

Consulting Interview Book


Compiled by Vivek Pundir, Goizueta MBA’06
Page 156 of 666
Goizueta Consulting Association
Goizueta Business School, Emory University, 1300 Clifton Road, Atlanta GA 30322

Candidate
The sales through Big M Mart appear to have a negative impact on the bottom line, as they have
lower margins than sales through grocery stores. The client could work with grocery stores to
ensure that they are able to compete effectively with Big M Mart in the sugar cereal market. This
strategy could be risky, however, since Big M Mart is a large and important customer. Therefore,
I would recommend that our client work more collaboratively with Big M Mart.

To defend its current position at Big M Mart stores, the client should move toward a partnership
with Big M Mart and dedicate more resources to the relationship. The customer and competitor
data indicate that our client'
s first priority should be to improve distribution to ensure better
product availability. In addition, it should push for product placement equal to, if not better than,
that of its competitors.

Interviewer
Why would Big M Mart be willing to enter into a partnership with Foods Inc?

Candidate
Foods Inc could offer to share its information about customer behavior to help increase revenues
for both itself and Big M Mart. Stock-outs hurt Big M Mart in two ways. First, some brand-loyal
customers simply walk away without purchasing cereal whenever their preferred brand is
unavailable. Second, we know that other brand-loyal customers purchase lower-priced cereal
whenever they encounter a stock-out of their preferred brand. Both of these instances lower Big
M Mart' s revenue.

By eliminating stock-outs, Big M Mart could increase its sales by simply ensuring that customers
don't walk away without making a purchase. Converting these purchase occasions to sales would
increase Big M Mart's sales of sugar cereals by more than 2 percent1.

Better availability also helps Big M Mart and our client increase their revenue by deterring the
brand-loyal shoppers from trading down to lower-priced cereals. Recall that 35 percent of the
brand-loyal shoppers purchase a discounted cereal if their preferred brand is not available. If
improved distribution now makes the preferred brands more consistently available, the customers
will pay a higher price for these products.

Finally, we could use the information about consumer purchase behavior to help persuade Big M
Mart to share information about product availability in its individual stores. We could work with
our client and Big M Mart to improve the current distribution system to allow for more
economical deliveries, while at the same time ensuring that our client' s product is consistently
available in the store.

Interviewer
Thank you. Those sound like solid recommendations, but I would suggest that you fully
understand the root cause of the stock-out situations and the cost to eliminate them before
moving ahead.

Consulting Interview Book


Compiled by Vivek Pundir, Goizueta MBA’06
Page 157 of 666
Goizueta Consulting Association
Goizueta Business School, Emory University, 1300 Clifton Road, Atlanta GA 30322

BCG - Canada Co.


Interviewer
Your client is the largest discount retailer in Canada, with 500 stores spread throughout the
country. Let' s call it CanadaCo. For several years running, CanadaCo has surpassed the second-
largest Canadian retailer (300 stores) in both relative market share and profitability. However,
the largest discount retailer in the United States, USCo, has just bought out CanadaCo' s
competition and is planning to convert all 300 stores to USCo stores. The CEO of CanadaCo is
quite perturbed by this turn of events, and asks you the following questions: Should I be
worried? How should I react? How would you advise the CEO?

Candidate
So, the client, CanadaCo, is facing competition in Canada from a U.S. competitor. Our task is to
evaluate the extent of the threat and advise the client on a strategy. Before I can advise the CEO I
need some more information about the situation. First of all, I' m not sure I understand what a
discount retailer is!

Interviewer
A discount retailer sells a large variety of consumer goods at discounted prices, generally
carrying everything from housewares and appliances to clothing. Kmart, Woolworth, and Wal-
Mart are prime examples in the U.S.

Candidate
Oh, I see. Then I think it makes sense to structure the problem this way: First, let's understand
the competition in the Canadian market and how CanadaCo has become the market leader. Then
let'
s look at the U.S. to understand how USCo has achieved its position. At the end, we can
merge the two discussions to understand whether USCo' s strength in the U.S. is transferable to
the Canadian market.

Interviewer
That sounds fine. Let'
s start, then, with the Canadian discount retail market. What would you like
to know?

Candidate
Are CanadaCo' s 500 stores close to the competition'
s 300 stores, or do they serve different
geographic areas?

Interviewer
The stores are located in similar geographic regions. In fact, you might even see a CanadaCo
store on one corner, and the competition on the very next corner.

Candidate
Do CanadaCo and the competition sell a similar product mix?

Interviewer

Consulting Interview Book


Compiled by Vivek Pundir, Goizueta MBA’06
Page 158 of 666
Goizueta Consulting Association
Goizueta Business School, Emory University, 1300 Clifton Road, Atlanta GA 30322

Yes. CanadaCo' s stores tend to have a wider variety of brand names, but by and large, the
product mix is similar.

Candidate
Are CanadaCo'
s prices significantly lower than the competition'
s?

Interviewer
No. For certain items CanadaCo is less expensive, and for others the competition is less
expensive, but the average price level is similar.

Candidate
Is CanadaCo more profitable just because it has more stores, or does it have higher profits per
store?

Interviewer
It actually has higher profits than the competition on a per-store basis.

Candidate
Well, higher profits could be the result of lower costs or higher revenues. Are the higher per-
store profits due to lower costs than the competition's or the result of higher per-store sales?

Interviewer
CanadaCo' s cost structure isn'
t any lower than the competition'
s. Its higher per-store profits are
due to higher per-store sales.

Candidate
Is that because it has bigger stores?

Interviewer
No. CanadaCo'
s average store size is approximately the same as that of the competition.

Candidate
If they'
re selling similar products at similar prices in similarly-sized stores in similar locations,
why are CanadaCo' s per-store sales higher than the competition' s?

Interviewer
It'
s your job to figure that out!

Candidate
Is CanadaCo better managed than the competition?

Interviewer
I don't know that CanadaCo as a company is necessarily better managed, but I can tell you that
its management model for individual stores is significantly different.

Candidate

Consulting Interview Book


Compiled by Vivek Pundir, Goizueta MBA’06
Page 159 of 666
Goizueta Consulting Association
Goizueta Business School, Emory University, 1300 Clifton Road, Atlanta GA 30322

How so?

Interviewer
The competitor' s stores are centrally owned by the company, while CanadaCo uses a franchise
model in which each individual store is owned and managed by a franchisee who has invested in
the store and retains part of the profit.

Candidate
In that case, I would guess that the CanadaCo stores are probably better managed, since the
individual storeowners have a greater incentive to maximize profit.

Interviewer
You are exactly right. It turns out that CanadaCo's higher sales are due primarily to a
significantly higher level of customer service. The stores are cleaner, more attractive, better
stocked, and so on. The company discovered this through a series of customer surveys last year. I
think you've sufficiently covered the Canadian market-let' s move now to a discussion of the U.S.
market.

Candidate
How many stores does USCo own in the U.S., and how many does the second-largest discount
retailer own?

Interviewer
USCo owns 4,000 stores and the second-largest competitor owns approximately 1,000 stores.

Candidate
Are USCo stores bigger than those of the typical discount retailer in the U.S.?

Interviewer
Yes. USCo stores average 200,000 square feet, whereas the typical discount retail store is
approximately 100,000 square feet.

Candidate
Those numbers suggest that USCo should be selling roughly eight times the volume of the
nearest U.S. competitor!

Interviewer
Close. USCo' s sales are approximately $5 billion, whereas the nearest competitor sells about $1
billion worth of merchandise.

Candidate
I would think that sales of that size give USCo significant clout with suppliers. Does it have a
lower cost of goods than the competition?

Interviewer
In fact, its cost of goods is approximately 15 percent less than that of the competition.

Consulting Interview Book


Compiled by Vivek Pundir, Goizueta MBA’06
Page 160 of 666
Goizueta Consulting Association
Goizueta Business School, Emory University, 1300 Clifton Road, Atlanta GA 30322

Candidate
So it probably has lower prices.

Interviewer
Right again. Its prices are on average about ten percent lower than those of the competition.

Candidate
So it seems that USCo has been so successful primarily because it has lower prices than its
competitors.

Interviewer
That's partly right. Its success probably also has something to do with a larger selection of
products, given the larger average store size.

Candidate
How did USCo get so much bigger than the competition?

Interviewer
It started by building superstores in rural markets served mainly by mom-and-pop stores and
small discount retailers. USCo bet that people would be willing to buy from it, and it was right.
As it grew and developed more clout with suppliers, it began to buy out other discount retailers
and convert their stores to the USCo format.

Candidate
So whenever USCo buys out a competing store, it also physically expands it?

Interviewer
Not necessarily. Sometimes it does, but when I said it converts it to the USCo format, I meant
that it carries the same brands at prices that are on average ten percent lower than the
competition' s.

Candidate
What criteria does USCo use in deciding whether it should physically expand a store it'
s just
bought out?

Interviewer
It depends on a lot of factors, such as the size of the existing store, local market competition,
local real estate costs, and so on, but I don'
t think we need to go into that here.

Candidate
Well, I thought it might be relevant in terms of predicting what it will do with the 300 stores that
it bought in Canada.

Interviewer
Let's just assume that it doesn'
t plan to expand the Canadian stores beyond their current size.

Consulting Interview Book


Compiled by Vivek Pundir, Goizueta MBA’06
Page 161 of 666
Goizueta Consulting Association
Goizueta Business School, Emory University, 1300 Clifton Road, Atlanta GA 30322

Candidate
OK. I think I've learned enough about USCo. I'
d like to ask a few questions about USCo'
s ability
to succeed in the Canadian market. Does USCo have a strong brand name in Canada?

Interviewer
No. Although members of the Canadian business community are certainly familiar with the
company because of its U.S. success, the Canadian consumer is basically unaware of USCo's
existence.

Candidate
Does CanadaCo carry products similar to USCo' s, or does the Canadian consumer expect
different products and brands than the U.S. discount retail consumer?

Interviewer
The two companies carry similar products, although the CanadaCo stores lean more heavily
toward Canadian suppliers.

Candidate
How much volume does CanadaCo actually sell?

Interviewer
About $750 million worth of goods annually.

Candidate
Is there any reason to think that the costs of doing business for USCo will be higher in the
Canadian market?

Interviewer
Can you be more specific?

Candidate
I mean, for example, are labor or leasing costs higher in Canada than in the U.S.?

Interviewer
Canada does have significantly higher labor costs, and I'
m not sure about the costs of leasing
space. What are you driving at?

Candidate
I was thinking that if there were a higher cost of doing business in Canada, perhaps USCo would
have to charge higher prices than it does in the U.S. to cover its costs.

Interviewer
That's probably true, but remember, CanadaCo must also cope with the same high labor costs.
Can you think of additional costs incurred by USCo'
s Canadian operations that would not be
incurred by CanadaCo?

Consulting Interview Book


Compiled by Vivek Pundir, Goizueta MBA’06
Page 162 of 666
Goizueta Consulting Association
Goizueta Business School, Emory University, 1300 Clifton Road, Atlanta GA 30322

Candidate
USCo might incur higher distribution costs than CanadaCo because it will have to ship product
from its U.S. warehouses up to Canada.

Interviewer
You are partially right. CanadaCo has the advantage in distribution costs, since its network spans
less geographic area and it gets more products from Canadian suppliers. However, since
CanadaCo continues to get a good deal of product from the U.S., the actual advantage to
CanadaCo is not great-only about two percent of overall costs.

Candidate
All this suggests that USCo will be able to retain a significant price advantage over CanadaCo'
s
stores: if not ten percent, then at least seven to eight percent.

Interviewer
I would agree with that conclusion.

Candidate
I would tell the CEO the following: In the near term, you might be safe. Your stores have a much
stronger brand name in Canada than USCo' s, and they seem to be well managed. However, as
consumers get used to seeing prices that are consistently seven to eight percent less at USCo,
they will realize that shopping at USCo means significant savings over the course of the year.
Although some consumers will remain loyal out of habit or because of your high level of service,
it is reasonable to expect the discount shopper to shop where prices are lowest. Moreover, over
time your brand-name advantage will erode as USCo becomes more familiar to Canadian
consumers. You certainly have to worry about losing significant share to USCo stores in the long
term. You should probably do something about it now, before it' s too late.

Interviewer
Can you suggest possible strategies for CanadaCo?

Candidate
Maybe it can find ways to cut costs and make the organization more efficient, so it can keep
prices low even if its cost of goods is higher.

Interviewer
Anything else?

Candidate
It might consider instituting something like a frequent shopper program, where consumers
accumulate points that entitle them to future discounts on merchandise.

Interviewer
What might be a potential problem with that?

Consulting Interview Book


Compiled by Vivek Pundir, Goizueta MBA’06
Page 163 of 666
Goizueta Consulting Association
Goizueta Business School, Emory University, 1300 Clifton Road, Atlanta GA 30322

Candidate
Well, it might not be that cost-effective, since it would be rewarding a significant number of
shoppers who would have continued to shop there anyway.

Interviewer
Any other suggestions?

Candidate
CanadaCo might want to prepare a marketing or advertising campaign that highlights its high
level of service. It might even institute a CanadaCo Service Guarantee that surpasses any
guarantees offered by USCo.

Interviewer
Assuming the only way to keep customers is through competitive pricing, is there anything
CanadaCo can do to appear competitive to the consumer?

Candidate
It might want to consider offering fewer product lines, so that it can consolidate its buying power
and negotiate prices with suppliers that are competitive with USCo' s. It might lose some
customers who want the variety of products that USCo has, but it may be able to retain the
customer who is buying a limited array of items and is just looking for the best price.

Interviewer
All of your suggestions are interesting, and you would want to analyze the advantages and
disadvantages of each in more detail before making any recommendations to the CEO.

Consulting Interview Book


Compiled by Vivek Pundir, Goizueta MBA’06
Page 164 of 666
Goizueta Consulting Association
Goizueta Business School, Emory University, 1300 Clifton Road, Atlanta GA 30322

BCG - Phones in Manhattan

Q: How many pay phones are there on the island of Manhattan?

A: A logical place to begin your analysis might be to ballpark the number of pay phones on
Manhattan street corners. If you think of New York City as a grid of streets, you might guess it is
about 300 streets long (north to south) by ten streets wide (east to west), so it has approximately
3,000 intersections. You might then assume there is one pay phone for every two intersections,
for a total of about 1,500 pay phones.

If you’re feeling really creative, you might subtract the number of intersections that are
“invalidated” because they fall in the area of Central Park. Say Central Park is ten blocks long by
two blocks wide, or 20 intersections. Using your one-pay-phone-for-every-two-intersections
assumption, you would want to subtract ten pay phones from the original 1,500.

You might then add to the 1,490 the number of pay phones that might be found in restaurants,
hotels, schools, hospitals, and office-building lobbies.

Consulting Interview Book


Compiled by Vivek Pundir, Goizueta MBA’06
Page 165 of 666
Goizueta Consulting Association
Goizueta Business School, Emory University, 1300 Clifton Road, Atlanta GA 30322

BCG - Hotel-sized Shampoo

Q: How many hotel-sized bottles of shampoo and conditioner are produced each year around the
world?

A: You might begin by assuming that hotel-sized bottles are produced for two purposes only:

1. To supply hotels and upscale motels


2. To provide samples for gift packs, salons, and so on

You would then want to start by estimating the number of hotels and motels around the world
that offer the products to their guests. One way of estimating the number of hotels is to assume
that hotels are found predominantly in major cities and resorts. Figure that there are 2,000 major
cities and resorts around the world, an average of ten for each of the world’s approximately 200
countries. Assume that each city averages 20 hotels that offer bottled hair products to their guests.
Multiplying 20 by 2,000 gives you 40,000 hotels around the world that require shampoo and/or
conditioner for their guests.

To understand how many bottles of shampoo and conditioner the 40,000 hotels require, you now
need to estimate the total number of uses each hotel on average represents. You can arrive at that
number through the following calculation: assume that there are 100 rooms in each hotel, and
that those rooms are occupied 50 percent of the time. Multiplying 40,000 by 100 by 0.5 by 365
(don’t forget the number of days in the year!) gives you approximately 750 million.

However, it is probably reasonable to assume that a guest staying for longer than a day will not
use a whole shampoo bottle every day. If you assume that an average of one shampoo bottle is
used for every two occupied days in a given room, you can now divide your 750 million estimate
in half to 375 million. To get to the number of bottles of conditioner, estimate a ratio between the
use of shampoo and the use of conditioner. Since many of us do not condition every time we
shampoo, you might assume that the ratio is 2:1. Dividing 375 million in half gives you
approximately 190 million. Your conclusion would then be that 375 million bottles of shampoo
and 190 million bottles of conditioner are required for hotel use every year.

To estimate the total market size, you can probably make things easy on yourself by assuming
that the number produced for sample purposes is a small percentage of the total, say ten percent.
Combining your two markets would give you approximately 400 million bottles of shampoo and
210 million bottles of conditioner.

Finally, you might want to “reality check” your total figure. Assuming 610 million bottles are
produced and sold each year at an average price of 25 cents each, the worldwide market for
miniature bottles of shampoo and conditioner is about $150 million. Does that sound reasonable?

Consulting Interview Book


Compiled by Vivek Pundir, Goizueta MBA’06
Page 166 of 666
Goizueta Consulting Association
Goizueta Business School, Emory University, 1300 Clifton Road, Atlanta GA 30322

BCG - Light Bulbs

Q: You are in a room with three light switches, each of which controls one of three light bulbs in
the next room. Your task is to determine which switch controls which bulb. All lights are off.
Your constraints are: you may flick only two switches and you may enter the room with the light
bulbs only once. How would you set about determining which switch controls which bulb?

A: To solve this riddle you must do some out-of-the-box thinking. The best way to determine
which light bulb is which is to flick one switch on, wait for five minutes and flick it off. Then
flick one of the remaining two switches on and leave the other off. When you enter the room
with the bulbs, you can determine which switch controls which of the two lights that are off by
feeling to see which of the bulbs is hot (from having burned for five minutes).

Other creative solutions involve pushing the constraints of the game. You might ask if the room
you’re in has a phone, so you could call somebody to help you. You might ask if the rooms have
a connecting window. You might assume you can leave the first room a number of times, and
therefore go out, buy a drill, and bore a hole through the wall so you can see which light bulb is
connected to which switch. Or, you might buy a mirror and place it strategically outside the door
to guide you.

Remember, you are limited only by your imagination.

Consulting Interview Book


Compiled by Vivek Pundir, Goizueta MBA’06
Page 167 of 666
Goizueta Consulting Association
Goizueta Business School, Emory University, 1300 Clifton Road, Atlanta GA 30322

Cure for Common Headaches (BCG)


Your firm just discovered a breakthrough formula for common headaches. What would you do
now?

Information to be given if asked:

• You are the CEO of this firm and your firm is a large MNC (multi national corporation).
• The product has passed the first round of in-company testing very successfully. We are highly
confident that it will be provide the masses instant relief from almost all types of headaches.
This is a unique discovery, and no existing product comes close to it in terms of effectiveness.
• Almost the same answers to all other questions – “Please make a reasonable assumption”
• No tables and no graphs

Solution:

• Test structured thoughts: What is critical – given the limited time, the candidate should first
outline a high level picture (set the scope) and then probe the details of each section making
reasonable assumptions and displaying their knowledge of frameworks and tools. One
example of setting such a high-level outline is to explore:
• Company’s current status
• Impact of new discovery
• Feasibility of product’s market success
• Next steps for the firm
• Having done something akin to the above the candidate should be able to proceed on the
detailed analysis by leveraging some frameworks like Customer/Competitor/Company
analysis, Internal/ External elements, SWOT, Cost revenue and profitability etc make
necessary assumptions like implications of FDA regulations, patent protection, clinical testing
success, competitor response etc

Summary:

Essentially, the candidate needs to provide a structure to the problem, flesh out issues with
probing analysis and produce a clear next-steps summary for the firm is the winning solution.

Consulting Interview Book


Compiled by Vivek Pundir, Goizueta MBA’06
Page 366 of 666
Goizueta Consulting Association
Goizueta Business School, Emory University, 1300 Clifton Road, Atlanta GA 30322

Amusement Park Expansion (BCG)

Our client is XYZ Corporation (“XYZ”), the owner of a single amusement park. XYZ has been
approached by the local government and offered 100 acres of land adjoining the current
amusement park for $10 million. XYZ has engaged us to help them assess whether or not they
should purchase the land and/or expand their existing park.

This is intended to be a two part question. Try to push the interviewee towards the qualitative
(“strategic”) aspects of the case first. Midway through the interview, focus on the quantitative
analysis of the case.

You:
The following facts are available only upon request for the strategic analysis:
• XYZ’s park is considered a regional park and does not get national attention;
• This is the only amusement park for a 250 mile radius;
• The average park visitor travels 30 miles to the park;
• Only 1% of park visitors travel more than 100 miles to visit the park;
• Other competing businesses in the area include: go carts, putt putt golf, video game arcades,
water skiing and other thrill sports;
• The park’s attendance has been growing at an annual rate of 10% over the past five years;
• The average park visitor is 17 years old;
• 30% of the park visitors are adults (over the age of 18);
• All of the parks vendors (food, video games, and shops) are wholly owned by the XYZ;
• XYZ has the financial wherewithal to acquire the land and develop all 100 acres;
• Annual population growth for the 250 mile radius is expected to remain flat at 3%;
• It can be assumed that XYZ has the internal management expertise to operate a larger park
and that enough local employees are available to run the new operations at existing wage rates.

The following is a potential framework that to organize qualitative aspects of your answer:
Market and Competition:
• After a careful analysis of the above facts, it appears that an expansion might be feasible, and
is worth further consideration. Growth at the amusement park seems to be strong and
although competitors exist for pieces of the amusement park’s business, no businesses directly
compete with the amusement park for a 250 mile radius. Additionally, it appears that
amusement park visitors of this park are generally unwilling to travel outside of the area to go
to the next closest amusement park.
• It is also safe to assume that amusement parks, in general, have high entry barriers due to the
initial capital investment, which might discourage potential new entrants. Along these same
lines, the expansion of XYZ’s existing park might actually serve to discourage new entrants,
since XYZ could end up having excess capacity which could give it a competitive advantage
due to pricing flexibility.

XYZ’s Corporations Capabilities/Limitations

Consulting Interview Book


Compiled by Vivek Pundir, Goizueta MBA’06
Page 462 of 666
Goizueta Consulting Association
Goizueta Business School, Emory University, 1300 Clifton Road, Atlanta GA 30322

XYZ appears to have the financial wherewithal to expand the park, as well as the internal
management to run the expanded operation. Also, given that XYZ owns all of the park’s
vendors, an expansion could bring in significant revenues in addition to entrance fees.

Interviewer:
About halfway through the interview, move on to the quantitative part of the interview.

You:
The following information is available upon request for the quantitative analysis:
• The amusement park averages 70% capacity, and is open year round;
• The capacity for the amusement park is currently 2,000 visitors;
• On 50 days a year, the park fills to capacity, resulting in long ride lines for visitors;
• It is estimated that the excess demand on these 50 days is approximately 600 visitors;
• The average ticket price is $23 ($30 for adults and $20 for kids);
• The average visitor spends $17 on food, games, and souvenirs;
• The land would cost $1 million per 20 acres to develop (i.e. add rides, attractions, shops,
restaurants, etc.);
• The expansion project would increase the amusement park’s capacity by 25%;
• The expected rate of return on the existing business is 12%;
• XYZ has access to funds at its existing weighted average cost of capital;
• The profit margin on XYZ’s operations is 20%.

An NPV analysis is very useful in the assessment of this potential project. All of the costs of the
project should be considered as well as the annual free cash flow from the project. The costs
include the cost of the land, the cost to develop the land, and the marginal costs of running the
new operations.

Costs:

Item Cost
Land $10 million (all year 0)
Development Costs $5 million (all year 0)
Marginal Costs of Running 80% of Revenues (on-going)
Operations

Revenues:
Assuming that historical growth trends continue, and that when the park fills to capacity visitors
stop coming, the following is a potential NPV scenario…

On the 50 days that the park experiences excess capacity, 500 additional visitors could be
admitted due to the park’s expansion. Each visitor is assumed to spend $23 to enter the park, and
$17 at the vendors, for a total of $40 per visitor.

Annual Revenue = 50 days * 500 additional visitors * $40 per visitor = $1 million

Consulting Interview Book


Compiled by Vivek Pundir, Goizueta MBA’06
Page 463 of 666
Goizueta Consulting Association
Goizueta Business School, Emory University, 1300 Clifton Road, Atlanta GA 30322

Assuming that XYZ’s incremental profit margin will remain the same for the expanded section
of the park, the 20% profit margin can be applied to these revenues to determine the net profit
margin.

Annual Profit Margin = $1 million * 20% = $200,000

Assuming that future capital expenditures match depreciation on the expanded section of the
park, annual free cash flows due to the expanded section of the park would equal the annual
profit margin of $200,000. Assuming that the growth is constant at the historical rate of 10%,
and that the appropriate discount rate is 12%, the free cash flows can be discounted as a
perpetuity.

NPV of Project = -Land and Development Costs + [Free Cash Flows / (Discount Rate – Growth
Rate)]
= -$15 million + [$200,000 / (12% - 10%)] = - $5 million

Based on this, it appears that XYZ should not expand its existing amusement park since the
project has a negative Net Present Value.

Consulting Interview Book


Compiled by Vivek Pundir, Goizueta MBA’06
Page 464 of 666
Goizueta Consulting Association
Goizueta Business School, Emory University, 1300 Clifton Road, Atlanta GA 30322

Amusement Park Expansion (BCG) – Alternative Solution

You are a large theme park, the government is selling the land next door. Do you want to buy it?

Possible Answer:
This is a new New-Market Question (disguised)
Framework: Three C with Revenue minus Costs in it

I discussed various uses of the land, but she led me to the question of building a second theme
park.
Based validity of my answers to the data she gave me:

Competitor:
One other local theme-park: focused on teenagers and rides, I focused on families. Also
compete with other leisure activities:

Customer:
• Thought about segmenting to another market: no lead there.
• Realize 30% local market, 70% national market. National market has small growth, my old
park has solid growth. Market is there.

Company:
Current theme park has 80% utilization. We also own nearby hotels to get extra revenue, at 70%
utilization.

New Venture:
Cost Fixed up-front 800,000,000

Need to estimate revenues:


Interviewer says old park:
• 35$ average ticket
• 15$ average concession
• 10$ merchandise average.
We need to realize that the new theme park may cannibalize or may add value to old theme park
through price.

Other key info gleaned during interview:


• Land costs 200 million. To build up the area would cost 600 million.
• Variable cost per customer is currently $25. After an expansion, it would be $40.
• Average revenue per customer is $55. With park addition, average revenue would by $60.
• Six months out of the year, the park sells out. With additional capacity, the park could raise
attendance by 30% during this period.
• There are twelve million total customers during the year. Eight million during the peak
season.

Consulting Interview Book


Compiled by Vivek Pundir, Goizueta MBA’06
Page 465 of 666
Goizueta Consulting Association
Goizueta Business School, Emory University, 1300 Clifton Road, Atlanta GA 30322

Analysis:
I first tried to determine whether the current demand was sustainable. It was a basic marketing
analysis of trying to figure out who the customers were, the key drivers of demand, and whether
the theme park could continue to offer a product that met these demands.

Second, I did a an NPV of the project:

Revenues after expansion:


Customers: 12 million + (30% of 8 million) = 14.4 million
Rev/customer: $60
Revenues: $864 million per year

Costs:
Fixed costs: $800 million
Variable cost/customer: $40
Total variable costs: 576 million

Revenues Costs
Y1 $864 million 1.376 billion
Y2 $864 million 576 million
Y3 $864 million 576 million

I assumed that this addition would generate revenue for 20 years. Even after discounting, the
project has a positive NPV, so the amusement park should purchase the land.

Consulting Interview Book


Compiled by Vivek Pundir, Goizueta MBA’06
Page 466 of 666
Goizueta Consulting Association
Goizueta Business School, Emory University, 1300 Clifton Road, Atlanta GA 30322

Power Transformers Market-share (BCG)

A manufacturer of power transformers has been experiencing a decline in the market share. The
client has the biggest market share in the industry, closely followed by number 2 and 3 players.
Number 4 and 5 players have been relatively new competitors, been able to gain market share
quickly. Our client has been experiencing declining revenues and profits. What should the client
do?

Possible Solution 1:
(I used very little frameworks, I had to ask a lot of questions to find out all the info. The
interview was very interactive.)

It turns out that there has been no new technology in the industry and our client' s prices have
been relatively stable. The product is sold mostly to utilities by bidding, and lately, we have not
been winning enough bids. Therefore, our competitors must have been able to undercut us on
price. The hypothesis then is that they have a lower cost structure but we don' t have any info on
their cost structure. The product does not differ much in quality.

When our client receives the order, the engineers design the product based on the specifications
given. It turns out that engineers spent a lot of time designing the products because they enjoy
coming up with innovative ideas. However, they are not necessarily adding value by designing
cool power transformers.

The products can be classified into three categories (medium, large, and something - not
important), in each of these categories the manufacturing process is somewhat similar.

Therefore:
1. Break down each activity as a percentage of total cost.
2. You will find out that design is really a much bigger percentage of total costs than what it
should be.

Recommendations:
1. Engineers need to be explained how their work affects the cost of all subsequent activities,
such as assembly, purchasing, etc. They need to design with cost savings in mind, for example,
use the same components when necessary instead of carrying huge inventories of every possible
component.
2. Share knowledge from one project to another, they shouldn' t have to duplicate the design part
that applies to every transformer every time.

Possible Solution 2:
Other A: Revenues falling can mean one of three things:
prices are steady and quantity is falling, or
quantity is steady and price is falling, or
both price and quantity are falling.

Consulting Interview Book


Compiled by Vivek Pundir, Goizueta MBA’06
Page 471 of 666
Goizueta Consulting Association
Goizueta Business School, Emory University, 1300 Clifton Road, Atlanta GA 30322

After a series of questions, I discover that the firm has been losing bids with their customers,
utility companies. To discover why, I backed up and considered the market that the firm was
operating and found out that they had 4 other competitors bidding on transformer sales. The #2
and #3 competitors were also losing revenues (and market share) while the #4 and #5
competitors were gaining share. The #4 and #5 companies were starting to win more bids from
the government because they always came in at the lowest price. Their ability to do this (as I
later discovered) was due to an effective integration of their bid, design and manufacturing
processes, which lowered their operating expenses and allowed them to come in with lower bid
prices yet maintain their margins.

An alternative approach would entail:


Taking inventory of all real estate holdings
Determining redundancies by location.
Combining those facilities where there will still be utilization is under capacity.
For those facilities where utilization would exceed capacity, determine NPV of staying put,
expanding existing facilities, or renting new space.

Consulting Interview Book


Compiled by Vivek Pundir, Goizueta MBA’06
Page 472 of 666
Goizueta Consulting Association
Goizueta Business School, Emory University, 1300 Clifton Road, Atlanta GA 30322

Financial-services Share-price (BCG)


All of the data in this case is public domain. Conseco is a company at the financial services
industry and more specifically at the business of life and health insurance. During the years 83-
98 Conseco was a great performer and lead the S&P 500. Conseco’s main growth engine was its
successful acquisitions. On average, the company acquired a target every 6 months. During 98,
Conseco acquired Green Tree Financials. Surprisingly, the day after the deal was announced
Conseco share price dropped 20% and a year after the share was down 50% from its price the
day before the announcement. You were hired by the CEO to explain this drop in the share price
and to suggest a course of action.

Additional data:
• Green Tree Financial is a provider of loans for homebuyers.
• Green Tree Financial is charging higher interest rates than Conseco.
• Green Tree deal was much larger than Conseco’s previous deals.
• Conseco share price before the acquisition was $57.7.
• Green Tree Financial share price before the deal was $29.
• The deal was a fixed equity exchange deal where 0.9165 shares of Conseco were awarded for
every share of Green Tree Financial.
• Conseco’s market cap before the deal was $7B.
• Green Tree owned approximately 50% of the company created by the M&A transaction.
• A year after Green Tree needed an additional investment of $1B.

Solution Structure:
• Identify the players attributes.
• Identify the exact deal structure.
• Identify misalignments in the deal that might cause the share price drop.
• Try to predict what will happen next and suggest course of action accordingly.

Solution Analysis:
• Problems with the deal structure:
• Misalignment in the companies business.
• The almost 1:1 stock exchange didn’t reflect the different market values of the two
companies.
• Conseco’s expertise was in smaller and more rapid acquisitions and this acquisition
wasn’t something they could handle.
• Problems with the acquisition target:
• From the last bullet in the additional data section it is obvious that Green Tree was at
a difficult situation before the acquisition and wasn’t a good target for acquisition.

The market adjusted Conseco’s share price to reflect these misalignments.


• What to do now (after a year)?
• Investigate the financial state of Green Tree after a year (it is evident it wasn’t good).
• If Green Tree continues to be in distress suggest dumping it.
• Conclusion

Consulting Interview Book


Compiled by Vivek Pundir, Goizueta MBA’06
Page 576 of 666
Goizueta Consulting Association
Goizueta Business School, Emory University, 1300 Clifton Road, Atlanta GA 30322

• Green Tree continued to suffer big loses and dragged Consico with it
• After several years Conseco was unlisted from the S&P.
• Additional questions
• What was Conseco’s management thinking?
• Where was Conseco’s board of directors?

Consulting Interview Book


Compiled by Vivek Pundir, Goizueta MBA’06
Page 577 of 666
Goizueta Consulting Association
Goizueta Business School, Emory University, 1300 Clifton Road, Atlanta GA 30322

Insurance Reporting (BCG)


Insure me is a Global Financial Services company at the insurance business. Recently, the CEO
of the company was fired and took with him all of the 10 employees of the company’s private
funding division, which was his pet project. No one that is left in the company knows what is
going on in that division, and there is no reporting system to rely on (the CEO took all of the data
with him). How would you go about managing this division?

Additional data:
• The company is operating in the US and Europe.
• The company provides car, life and other type of insurance.
• The company is one of the 4 leading players at its market with over $1B of annual revenues.
• The private funding division is type of a VC.
• We have a data sheet (see appendix) which list 4 of the division’s current investment.
• These 4 investments are only around 20% of the number of investments but form 80% of their
value.

Solution Structure:
• Identify the company’s business and core competency.
• Identify the assets under the division management.
• Identify any financial and strategic synergies between the division’s assets and the company.
• Analyze ways to leverage the division and its assets moving forward.

Solution Analysis:
• As mentioned the company’s core competency is in the insurance field.
• As could be observed from the appendix two assets are not complimentary to the company’s
business.
• From the remaining ones one is forecasted to lose money next year.
• As such there is one company it make sense to keep and the other are not a real asset to the
company.

Recommendations:
• Keep the company with the strategic fit that makes money and try to sell the others (for a
good deal).
• For the one that makes sense try to increase the company’s holding in it.
• The company with the fit will serve both to hedge the bets and in order to keep the finger on
the pulse of the new market needs.
• As for the division, try to find what would be needed (funds, time, efforts, HR etc.) in order to
bring it to an operational mode.
• Find what are the estimated operation costs.
• If it makes sense from the financial aspect you might want to keep this division as it hedge
your bets.
Name of company A B C D
Field High Value Stadiums Golf clubs Executive
commodities insurance renovation design insurance

Consulting Interview Book


Compiled by Vivek Pundir, Goizueta MBA’06
Page 578 of 666
Goizueta Consulting Association
Goizueta Business School, Emory University, 1300 Clifton Road, Atlanta GA 30322

This year’s revenue $150M $300M $100M $70M


This year’s expenses $100M $280M $150M $50M
Next year’s revenue growth (additional 300% 200% 100% 300%
on top of the current)
Next year’s expenses growth 500% 200% 150% 400%
(additional on top of the current)

Consulting Interview Book


Compiled by Vivek Pundir, Goizueta MBA’06
Page 579 of 666
Goizueta Consulting Association
Goizueta Business School, Emory University, 1300 Clifton Road, Atlanta GA 30322

Teflon Market-entry (BCG)


Your client is a Japanese manufacturer of a fluoroplastic material that has unique and valuable
characteristics in the manufacture of other metal goods. The material' s properties include: heat,
flame, and water retardant, chemically-resistant, non-conducting, bondable to other metallic
surfaces, and extremely slippery. The material is traditionally sold downstream to the
manufacturers of other metallic goods. Specifically, applications for the product, manufactured
in slightly different ways, fall into three general categories:

1) Molding - The material is used as a surface to protect the underbelly of cars and space
aircrafts for the automotive and aerospace industries.
2) Foaming - In a warmed state, the product "foams up" around the outside of copper wire and
cable. It then hardens around the wire or cable, forming a durable, protective, outer shell.
Customers of the foaming product are in the telecommunications industry.
3) Dispersing - The material is sprayed on the surface of pots and pans and other industrial
metallic products to give the products a slippery, non-stick surface. Customers make
metallic products of all varieties.

The Japanese firm is considering re-entry into the highly profitable U.S. teflon market. In fact, it
has already committed to the construction of a manufacturing facility in Decauter, Alabama.
What should the firm's basic market entry strategy be, and how should the firm best position its
product?

Response

What happened the first time the Japanese firm entered the U.S. market?
The firm was hit with a price-dumping suit by the market leader, and after a lengthy court
dispute, was forced to pay heavy penalties and withdraw from the U.S. market for 24 months.
Now that that time has expired, the firm is attempting re-entry.

What is the competitive landscape?


The industry leader is DuPont, the originator of teflon. They control 70% of the $600M U.S.
market. There are also three smaller niche players in the market, each controlling 10%.

Where are the market opportunities?


The interviewee should probe for how the product differs in each of the three application areas.
If he asks the right questions, here's what he will learn. The product has certain finishing and
raw material variations that make it unique to each of the major application areas: molding,
foaming and dispersing. Further within each major area, there are minor characteristics that
differentiate the product to different customers. Upon telling the candidate this, a picture of
competitor positions and market opportunities on an opportunity space map would be nice. See
below.

Consulting Interview Book


Compiled by Vivek Pundir, Goizueta MBA’06
Page 608 of 666
Goizueta Consulting Association
Goizueta Business School, Emory University, 1300 Clifton Road, Atlanta GA 30322

Competitive Map Opportunity Map

Comp #1 Cust #1
Characteristic 2

Characteristic 2
Comp #2 Cust #2

Op
po
rt u
ni
Comp #3 Cust #3 ty
Cust #4 !

Characteristic 1 Characteristic 1

Market potential?
Upon probing, the candidate learns that the market is subdivided as follows:

Revenue Size Contribution Margin


Molding 45% 30%
Foaming 30% 50%
Dispersing 25% 30%

Where do the competitors reside in this picture? or, Why is the contribution higher for foaming?
DuPont competes in all three markets. The three niche players compete in molding or dispersing
(but not foaming). The reason is that only DuPont has perfected the science of foaming up the
product, except for our client who can also do so. Therefore DuPont has a monopoly player
status in this market. Here is where our client should position itself. Also, the candidate might
question what the strengths and weaknesses of DuPont vs. his client are. DuPont has established
relationships and brand recognition. His client has a superior manufacturing process, and
product, with performance advantages. The market perceives these advantages from the last
time the client was in the U.S.. The candidate would thus want to position his product as a
performance leader.

Discussion of cost structures?


The candidate should attempt to think about how much the client can charge, and how much he
will make. If he discusses cost structures, ask him to hypothesize about who might possess a
cost advantage (DuPont or the client). Probably DuPont has a fixed cost edge given its
established, partially depreciated plants. But might the Japanese make this product better,
smarter, cheaper? Candidate could discuss variable costs:

Labor (about equal between our client in Alabama and DuPont in Connecticut)
Raw materials (about equal since they are commodities)
Manufacturing costs (about equal since the minimum efficient scale is achieved at 10% total
market share)

Consulting Interview Book


Compiled by Vivek Pundir, Goizueta MBA’06
Page 609 of 666
Goizueta Consulting Association
Goizueta Business School, Emory University, 1300 Clifton Road, Atlanta GA 30322

Distribution costs
(no size advantages for DuPont).

Pricing?
Press the candidate for how the product should be priced. Should they discount price (more
share gain, but chance of getting hit with another price dumping suit, and also chance to get into
a losing price war which DuPont is better able to handle), mirror price (less share gain, but less
risks), or premium price (and chase the high end niche). He should arrive that a mirror strategy
is probably best since you will tell them that there is no real justification for a premium price.
One final note, if the candidate asks (and he should), he will learn that price discounts are always
met by DuPont in its teflon markets, so the likelihood of a price war is high if they discount price.
Note..... if candidate selects discount pricing strategy, he probably missed the boat!

Consulting Interview Book


Compiled by Vivek Pundir, Goizueta MBA’06
Page 610 of 666
Goizueta Consulting Association
Goizueta Business School, Emory University, 1300 Clifton Road, Atlanta GA 30322

Telephone Manufacturing Competition (BCG)


Your client is a telecommunications equipment manufacturer in Chicago. His product is
business telephone sets, for use with private branch exchange (PBX) switches and centrex
service. The client has a corporate growth hurdle rate of 6%, but your client’s division is only
growing at 3-4% per year, and the industry is only growing at 2% per year. Why is the industry
growing so slowly, and what should our client do about it?

Response

After struggling with the question of why the industry is only growing 2% per year, the
candidate will deduce (or be told) that older generation products are being bought up and resold
by telephone resellers. The products are advertised and sold directly through a
telecommunications trade magazine (called Telecom Gear), which has 60 pages and is published
monthly. Thus small businesses are deferring the purchase of new phones by using very reliable,
feature-rich used telephone sets instead.

Why the growth of used equipment resale?


Candidate can arrive at some of these on his own, or be given some of the rest. Try to let him
struggle with these for a little while. Basically, Price, delivery speed, easy small quantity
ordering, the slower evolution of telephone technology (as opposed to the evolution of PC
technology), and quality/reliability of the product are responsible for growth in the used market.

What’s the competitive environment for new equipment sale?


AT&T, Rolm, and Nortel are the big three gorillas --- 33% share each. Assume our client is one
of the three. For the used equipment, it’s mostly small outfits.

Should I enter this new market?


Candidate should probe for and use all of this data. The new equipment market for us is $100M
in revenue. The used equipment market is $6M in revenue. The used market is highly
fragmented with no player owning more than 3% market share. So, even if we were able to grab
large share (like 25%) of this market, that $1.5M in additional revenue would not be large
enough to move us from 3-4% per year (on the $100M) to the 6% corporate hurdle rate. Also,
entering this market may cheapen or damage our brand identity. Probable answer then, is no.
But the used market is growing at 30% per year. So, will entering this market now help us
compete in it later. Probably.

Target markets for used Telecom Equipment


Two targets. First is small businesses who are price sensitive. Second is small quantity
purchasers in larger businesses who buy used because it’s quick and easy, and it’s priced right.
Also, both segments appreciate quick delivery times.

How do I fend off growth in this used market?


Licensing -- our lawyers tried and failed to prevent these players from selling our used products.

Consulting Interview Book


Compiled by Vivek Pundir, Goizueta MBA’06
Page 611 of 666
Goizueta Consulting Association
Goizueta Business School, Emory University, 1300 Clifton Road, Atlanta GA 30322

Change the standards – alter the standards on our new phones and switches every couple of years
to make old phones incompatible with them. This strategy would hurt the used vendors, but
might tick off our customers as well.

Buy back our own used equipment – this is a possibility, but it’s also very expensive. A variant
on this approach might be offering trade-in for a customer’s older, used phones.
Spin off a lower line new product – this will make our new product more price competitive with
the used product. The key here is trying to find what it is that customers like about the used
product, and trying to match this with our new products (i.e., price, delivery speed and frequency,
customer service, etc.).

Consulting Interview Book


Compiled by Vivek Pundir, Goizueta MBA’06
Page 612 of 666
Goizueta Consulting Association
Goizueta Business School, Emory University, 1300 Clifton Road, Atlanta GA 30322

Automobile Market-entry (BCG)


Your client is a joint venture between two European (German) companies (later learned this to
be Mercedes-Benz and Swatch) who have developed a new automobile. The car was designed
for the European market, but your client would like to know the viability of introducing the car
in the U.S. market. The car itself is 98 inches long (2.5 meters), has two seats, two doors, a 55
horsepower engine, and gets excellent gas mileage. It is very small, low cost, and its design was
driven by new technology. For example, this car is the first of its kind to offer detachable body
parts to facilitate easy changing of car colors, and its interior micro-electronics technology is
state-of-the-art. The car comes in two varieties: a regular hard-top version that would come
fully loaded for about $11,000 US dollars, and a convertible rag-top that comes similarly
equipped for $14,000. For purposes of this discussion, consider only the introduction of the
hard-top version. Your client would like three questions answered:
1) How would you segment the U.S. automobile market?
2) Which segments would this car most likely address?
3) How would you handle distribution?

Response

Question 1
Car customers can be segmented in a number of ways. For example: income level, sex, state of
life (i.e., married without children, married with children, single, retired, etc.), age, geographic
location (i.e., live in a city, a neighborhood, in a rural area), according to the criteria the
consumer uses to make his car purchase decision (i.e., sex appeal, styling, features, price,
practicality, etc.), or simply by type of car purchased (i.e., sport utility, sports car, 2-door sedan,
4-door sedan, wagon, etc.). Probably does not matter which angle the candidate takes, but he
should offer some level of detail in how he would pursue his chosen approach. Candidate could
also make a stab at market size for the segment he is targeting based upon his segmentation. The
interviewer offered that BCG segmented the car market along three parameters: income, stage of
life, and car buying criteria used. BCG loves little pictures, and showing these three segmenting
parameters in a small 3-D matrix was well received by the interviewer, and he referred to the
drawing again later in the interview.

Question 2
This car will address the middle to lower income segment (rag-top might be different), males or
females, probably commuter stage of lifers without kids, young folks, urban dwellers (great for
parallel parking), and consumers who buy based on practicality and maybe styling. Candidate
could discuss potential competitors: Geo, Saturn, Chevy Cavalier, etc.. Also could make some
hypothesis about projected market share capture. This would make it more complicated, and I
didn't do this in the interview, but if you want to make it more fun, consider how many
competitors are out there, how evenly distributed the market is, and what it takes to succeed.
This leads to the more interesting....

Question 3
Candidate will need to investigate how the client currently markets cars in the U.S.. Well, they
have a series of high-end dealerships all over the country, and an established reputation for

Consulting Interview Book


Compiled by Vivek Pundir, Goizueta MBA’06
Page 613 of 666
Goizueta Consulting Association
Goizueta Business School, Emory University, 1300 Clifton Road, Atlanta GA 30322

quality of engineering, styling, performance and reliability. The brand is top-notch in the states
(let the candidate dig for this, because this is the most important part). The dealerships would be
a nice distribution medium for the client. But what about the translation of the brand name.
Wouldn' t this low end car damage the existing high end reputation of the existing brand
(probably)? What about the price elasticity of demand of the two consumer segments (high end
versus low end)? High end buyers are more price inelastic (they purchase on performance, brand,
etc.), while the lower end buyers are more price elastic (they purchase on price). How will our
existing lattice of dealerships do in marketing such a different product to such a different target
market? Probably the only viable solution would be to create a new brand name and a new
network of dealerships, or at least let another type of dealer sell the cars for the client. So, brand
transferability is critical to the question of distribution. By the way, BCG advised the client not
to enter the U.S. market, primarily for the reasons discussed here.

Consulting Interview Book


Compiled by Vivek Pundir, Goizueta MBA’06
Page 614 of 666
Goizueta Consulting Association
Goizueta Business School, Emory University, 1300 Clifton Road, Atlanta GA 30322

U.S. Photographic Industry Re-entry (BCG)


Your client is a manufacturer and distributor of specialty photographic products in the US. The
products include all essential materials used to make the printing of a newspaper or magazine
happen, including but not limited to: imaging plates, film, specialty lights, and developing fluids.
The client is the #1 provider to newspapers nationwide. However, the client does not have a
presence in the commercial market. The commercial market includes magazines, brochures, and
any other type of glossy publication printed in the US. The client has already failed once in
penetrating this market, and would like help developing its re-entry strategy into this profitable
market. Why did the client fail the first time, and how should he proceed this time?

Response

Competitive Landscape
There are 5 large competitors. We lead the Newspaper Market with about 40% share, the other
four have about 15% each. The other 4 split the Commercial Market, with about 25% each. We
have none.

Company (Strengths and Weaknesses)/Customer (Market)


Start off trying to understand what makes the company tick. Since it is extremely successful in
the Newspaper Market, but has failed miserably in the Commercial Market, the candidate should
try to understand what has made the client successful in the former, and what are the differences
between the two markets. Basically the company has succeeded by manufacturing the highest
quality product in the Newspaper Market, and by developing a set of relationships with the 300
major clients. So, it's success has been based on quality of product, and distribution through
superior relationships. This should lead the candidate to further questions....

How is the client's quality in the Commercial Market?


The client has a quality problem here. He has received feedback that his product is not up to
snuff, and he says that the manufacturing people are blaming marketing for promising
specifications that operations cannot deliver, while marketing folks are blaming operations for
delivering a product not up to par. The client wants to know who is to blame? Pose this
question to the candidate not very long into the case.

This is a toughie. Probably neither is to blame. A discussion about the misalignment between
marketing strategy and manufacturing strategy would be excellent. Upon probing, the candidate
will learn that what it takes to succeed in the Commercial Market is different than what it takes
to succeed in the Newspaper Market. Newspapers are characterized by higher volumes (an
average order is about 9 times as large) and lower variety, while Commercial Markets by lower
volumes and higher variety, with more emphasis on meeting individual product specs for each of
the different commercial clients. So, in the Newspaper Markets, operations is making and
marketing selling a regular, high volume, standardized high quality product. They are aligned.
But in the Commercial Markets, operations is still only prepared to build high volume, low
variety, while marketing is off selling what the customer is asking for, unique specs, high variety,
and low volumes. They are not aligned.

Consulting Interview Book


Compiled by Vivek Pundir, Goizueta MBA’06
Page 615 of 666
Goizueta Consulting Association
Goizueta Business School, Emory University, 1300 Clifton Road, Atlanta GA 30322

In addition, the quality required to satisfy Commercial clients is different. The nature of the
printing process requires more exact print types, requiring more intricate photographic products.
Our client is not prepared to satisfy these requirements.

What is the cost of this quality?


Tell the candidate if he probes here that to achieve this quality he must lay out $6M in machinery
investment. Ask him whether this is a worthwhile investment. The candidate could talk about
the size of the market, and different competitor's share, and hypothesize what the client could
expect to capture given this investment. I didn' t get specific numbers, but just talked in terms of
using WACC to pay off these fixed assets with a projected stream of future earnings in this
market. This analysis lets the candidate show off some finance competence. Also, he could talk
operationally about different types of quality (performance vs. conformance), and the cost of 6
sigma (or some other level of) quality, and the ramifications that quality has on the process, labor,
the culture, and the bottom line.

What other differences prevent the client from succeeding in the Commercial Market?
Let the candidate struggle over this problem, but the primary problem is a tip you gave earlier.
The Commercial Market is characterized by many smaller orders, while the Newspaper Market
by fewer, long-standing high volume orders. There are only about 400 major newspapers in the
US, and our client' s sales force of 75 has great relationships with all of them. In the Commercial
Market, there are over 34,000 clients nationwide, touched by a lattice of about 1,200 dealers. So,
distribution is different. Our salespeople can' t touch everyone so easily in this new market. Also,
the Commercial Market is characterized by long standing orders with each of the clients.
Usually a client will select a photographic product vendor and stick with him for 5 to 7 years.

Is this an opportunity or a threat?


Both. It is an entry barrier, which prevents our client from accessing all new business in the first
year -- he can expect only 15-20% of clients to be in search of a vendor this year. But it is also
an opportunity, because once you win a client at a so-called "bake-off" or "beauty pageant" put
on by one of the distributors, you are locked into that business for 5-7 years. So your sales
people can focus on the next batch of "shopping" clients in years 2-5.

How do you reach these 34,000 clients with 75 salespeople?


This is the question I was forced to drill down on so hard. I don' t think there'
s a real answer.
Obviously you can' t visit every client on a regular basis, or even once. You generate too little
volume per client to justify personal touch. Hypothesizing making offers to the distributors is a
good thought, but it turns out every little special perk we try to offer the distributors is matched
by our competition, who have longer standing relationships with them. I finally proposed an
indirect sales model (similar to Dell' s), where you can only access our product over the phone (or
Internet), and then it is delivered by mail. But we' ll offer the best quality, whatever
customization clients could want, and at least a 10-15% price break against the competition
because our SG&A will be so low. And we' ll advertise and promote the heck out of ourselves to
grab initial share. I don' t know what the real answer is here, but let the candidate exhaust his
thinking and struggle a bit about the question of distribution.

Consulting Interview Book


Compiled by Vivek Pundir, Goizueta MBA’06
Page 616 of 666
10. Case examples

10.1 Discount retailer case (BCG)

Step 1: Actively listen to the case

Your client is the largest discount retailer in Canada, with 500 stores spread throughout the
country. Let's call it CanadaCo. For several years running, CanadaCo has surpassed the
second-largest Canadian retailer (300 stores) in both relative market share and profitability.
However, the largest discount retailer in the United States, USCo, has just bought out
CanadaCo's competition and is planning to convert all 300 stores to USCo stores. The CEO of
CanadaCo is quite perturbed by this turn of events, and asks you the following questions:
Should I be worried? How should I react? How would you advise the CEO?

Step 2: Establish understanding of the case

So, the client, CanadaCo, is facing competition in Canada from a U.S. competitor. Our task is
to evaluate the extent of the threat and advise the client on a strategy. Before I can advise
the CEO I need some more information about the situation. First of all, I'm not sure I
understand what a discount retailer is!

A discount retailer sells a large variety of consumer goods at discounted prices, generally
carrying everything from house wares and appliances to clothing. Kmart, Woolworth, and
Wal-Mart are prime examples in the U.S.

Step 3: Set up the framework

Oh, I see. Then I think it makes sense to structure the problem this way: First, let's
understand the competition in the Canadian market and how CanadaCo has become the
market leader. Then let's look at the U.S. to understand how USCo has achieved its position.
At the end, we can merge the two discussions to understand whether USCo's strength in the
U.S. is transferable to the Canadian market.

That sounds fine. Let's start, then, with the Canadian discount retail market. What would
you like to know?

Step 4: Evaluate the case using the framework

Are CanadaCo's 500 stores close to the competition's 300 stores, or do they serve different
geographic areas?

ICC Handbook 2011 Page 54


The stores are located in similar geographic regions. In fact, you might even see a CanadaCo
store on one corner, and the competition on the very next corner.

Do CanadaCo and the competition sell a similar product mix?

Yes. CanadaCo's stores tend to have a wider variety of brand names, but by and large, the
product mix is similar.

Are CanadaCo's prices significantly lower than the competition's?

No. For certain items CanadaCo is less expensive, and for others the competition is less
expensive, but the average price level is similar.

Is CanadaCo more profitable just because it has more stores, or does it have higher profits
per store?

It actually has higher profits than the competition on a per-store basis.

Well, higher profits could be the result of lower costs or higher revenues. Are the higher per-
store profits due to lower costs than the competition's or the result of higher per-store sales?

CanadaCo's cost structure isn't any lower than the competition's. Its higher per-store profits
are due to higher per-store sales.

Is that because it has bigger stores?

No. CanadaCo's average store size is approximately the same as that of the competition.

If they're selling similar products at similar prices in similarly-sized stores in similar locations,
why are CanadaCo's per-store sales higher than the competition's?

It's your job to figure that out!

Is CanadaCo better managed than the competition?

I don't know that CanadaCo as a company is necessarily better managed, but I can tell you
that its management model for individual stores is significantly different.

How so?

The competitor's stores are centrally owned by the company, while CanadaCo uses a
franchise model in which each individual store is owned and managed by a franchisee that
has invested in the store and retains part of the profit.

ICC Handbook 2011 Page 55


In that case, I would guess that the CanadaCo stores are probably better managed, since the
individual storeowners have a greater incentive to maximize profit.

You are exactly right. It turns out that CanadaCo's higher sales are due primarily to a
significantly higher level of customer service. The stores are cleaner, more attractive, better
stocked, and so on. The company discovered this through a series of customer surveys last
year. I think you've sufficiently covered the Canadian market-let's move now to a discussion
of the U.S. market.

How many stores does USCo own in the U.S., and how many does the second-largest
discount retailer own?

USCo owns 4,000 stores and the second-largest competitor owns approximately 1,000
stores.

Are USCo stores bigger than those of the typical discount retailer in the U.S.?

Yes. USCo stores average 200,000 square feet, whereas the typical discount retail store is
approximately 100,000 square feet.

Those numbers suggest that USCo should be selling roughly eight times the volume of the
nearest U.S. competitor!

Close. USCo's sales are approximately $5 billion, whereas the nearest competitor sells about
$1 billion worth of merchandise.

I would think that sales of that size give USCo significant clout with suppliers. Does it have a
lower cost of goods than the competition?

In fact, its cost of goods is approximately 15 percent less than that of the competition.

So it probably has lower prices.

Right again. Its prices are on average about ten percent lower than those of the
competition.

So it seems that USCo has been so successful primarily because it has lower prices than its
competitors.

That's partly right. Its success probably also has something to do with a larger selection of
products, given the larger average store size.

How did USCo get so much bigger than the competition?

It started by building superstores in rural markets served mainly by mom-and-pop stores


and small discount retailers. USCo bet that people would be willing to buy from it, and it was

ICC Handbook 2011 Page 56


right. As it grew and developed more clout with suppliers, it began to buy out other
discount retailers and convert their stores to the USCo format.

So whenever USCo buys out a competing store, it also physically expands it?

Not necessarily. Sometimes it does, but when I said it converts it to the USCo format, I
meant that it carries the same brands at prices that are on average ten percent lower than
the competition's.

What criteria does USCo use in deciding whether it should physically expand a store it's just
bought out?

It depends on a lot of factors, such as the size of the existing store, local market
competition, local real estate costs, and so on, but I don't think we need to go into that
here.

Well, I thought it might be relevant in terms of predicting what it will do with the 300 stores
that it bought in Canada.

Let's just assume that it doesn't plan to expand the Canadian stores beyond their current
size.
OK. I think I've learned enough about USCo. I'd like to ask a few questions about USCo's
ability to succeed in the Canadian market. Does USCo have a strong brand name in Canada?

No. Although members of the Canadian business community are certainly familiar with the
company because of its U.S. success, the Canadian consumer is basically unaware of USCo's
existence.

Does CanadaCo carry products similar to USCo's, or does the Canadian consumer expect
different products and brands than the U.S. discount retail consumer?

The two companies carry similar products, although the CanadaCo stores lean more heavily
toward Canadian suppliers.

How much volume does CanadaCo actually sell?

About $750 million worth of goods annually.

Is there any reason to think that the costs of doing business for USCo will be higher in the
Canadian market?

Can you be more specific?

I mean, for example, are labor or leasing costs higher in Canada than in the U.S.?

ICC Handbook 2011 Page 57


Canada does have significantly higher labor costs, and I'm not sure about the costs of leasing
space. What are you driving at?

I was thinking that if there were a higher cost of doing business in Canada, perhaps USCo
would have to charge higher prices than it does in the U.S. to cover its costs.

That's probably true, but remember, CanadaCo must also cope with the same high labor
costs. Can you think of additional costs incurred by USCo's Canadian operations that would
not be incurred by CanadaCo?

USCo might incur higher distribution costs than CanadaCo because it will have to ship
product from its U.S. warehouses up to Canada.

You are partially right. CanadaCo has the advantage in distribution costs, since its network
spans less geographic area and it gets more products from Canadian suppliers. However,
since CanadaCo continues to get a good deal of products from the U.S., the actual
advantage to CanadaCo is not great-only about two percent of overall costs.

All this suggests that USCo will be able to retain a significant price advantage over
CanadaCo's stores: if not ten percent, then at least seven to eight percent.

I would agree with that conclusion.

Step 5: Summarize and make recommendations

I would tell the CEO the following: In the near term, you might be safe. Your stores have a
much stronger brand name in Canada than USCo's, and they seem to be well managed.
However, as consumers get used to seeing prices that are consistently seven to eight percent
less at USCo, they will realize that shopping at USCo means significant savings over the
course of the year. Although some consumers will remain loyal out of habit or because of
your high level of service, it is reasonable to expect the discount shopper to shop where
prices are lowest. Moreover, over time your brand-name advantage will erode as USCo
becomes more familiar to Canadian consumers. You certainly have to worry about losing
significant share to USCo stores in the long term. You should probably do something about it
now, before it's too late.

Can you suggest possible strategies for CanadaCo?

Maybe it can find ways to cut costs and make the organization more efficient, so it can keep
prices low even if its cost of goods is higher.

Anything else?

It might consider instituting something like a frequent shopper program, where consumers
accumulate points that entitle them to future discounts on merchandise.

ICC Handbook 2011 Page 58


What might be a potential problem with that?

Well, it might not be that cost-effective, since it would be rewarding a significant number of
shoppers who would have continued to shop there anyway.

Any other suggestions?

CanadaCo might want to prepare a marketing or advertising campaign that highlights its
high level of service. It might even institute a CanadaCo Service Guarantee that surpasses
any guarantees offered by USCo.

Assuming the only way to keep customers is through competitive pricing, is there anything
CanadaCo can do to appear competitive to the consumer?

It might want to consider offering fewer product lines, so that it can consolidate its buying
power and negotiate prices with suppliers that are competitive with USCo's. It might lose
some customers who want the variety of products that USCo has, but it may be able to
retain the customer who is buying a limited array of items and is just looking for the best
price.

All of your suggestions are interesting, and you would want to analyze the advantages and
disadvantages of each in more detail before making any recommendations to the CEO.

Additional Case Questions

By the time you've gone through the case interview example and the interactive case you
probably will have developed a good idea of what a case interview is all about. The best way
to prepare for a case interview is to practice a few. Ask a friend or career counselor to give
you a case using the sample business problems below.
1. A German luxury car manufacturer is interested in entering the sport-utility vehicle
market (for example, Jeep Cherokee) after noticing that the market has grown dramatically
worldwide in the past two years. How would you advise the manufacturer? What does it
need to know before making an entry decision? If it chooses to enter, what might a viable
strategy be?

2. A North American manufacturer/retailer of high-end glassware experienced a dramatic


decline in same-store sales at its retail outlets last year. How would you begin to assess the
reasons for the decline? Using your analysis as a basis, what strategy would you recommend
for the manufacturer?
3. A large public utility formerly had a monopoly in the British electricity market. Now that
the market has been deregulated, small power-generation companies have already
captured a five percent share from the utility by offering to provide large businesses in the
U.K. with their own in-house power-generation capabilities. The CEO of the utility wants to
understand whether this trend will continue and how she can prevent further loss of share.
How would you answer her question?

ICC Handbook 2011 Page 59


4. A U.S.-based pharmaceutical company that focuses on discovering, developing, and
selling drugs for the treatment of cancer has been experiencing flat growth and is interested
in expanding into new businesses. In view of the growth and profitability of stand-alone
cancer treatment centers in the U.S., the company is considering establishing and operating
similar centers in China. This would be the company's first foray into the cancer treatment
center business. How would you evaluate the attractiveness of the opportunity?

5. The Swiss Ski Association has been petitioned by an international snowboarding club to
permit snowboarding on the ski slopes within its jurisdiction. (Assume that the association
currently forbids snowboarding on all Swiss ski slopes.) If the association is interested in
maximizing profits, how should it respond to the petition? What factors would the answer
depend upon?

10.2 Medical software industry case (BCG)

Step 1: Actively listen to the case

Your client is GenCo, a large, international, diversified company with a health care division
that produces a wide variety of medical instruments and related services. Five years ago, it
expanded into the health care software industry by purchasing MedCount, which markets
administrative systems to large U.S. hospitals. These systems are designed primarily for
back-office functions; they are not designed for managing patients or providing other
physician and technical support. Since it was purchased, the software division has failed to
deliver the growth needed to justify the multiple GenCo paid for it. GenCo feels it has
already squeezed margins as much as possible, and now is looking for new sales
opportunities. MedCount turned to BCG to help identify potential ways to increase
revenues. How would you approach this problem?

Step 2: Establish your understanding of the case

First, let me make sure I understand the problem. The parent company produces medical
devices and services, but before the acquisition was not involved in health care software. The
company it purchased, MedCount, sells only administrative systems software to large
hospitals. It is now looking for opportunities to increase revenues.

That is correct.
Could I take a moment to jot down a few thoughts?

Sure, that would be fine.

Step 3: Set up the framework

I would suggest using the following framework: First, I'd want to understand the market size
and growth rates for MedCount's market and related software markets. Next, I would like to
explore the competition and their market shares. Third, I would like to examine customer

ICC Handbook 2011 Page 60


requirements and then, given those external conditions, look at the division's capabilities to
understand how well prepared it is to meet the needs of the marketplace.

That sounds fine. So what do you want to know about the market?

Step 4: Evaluate the case using the framework

Well, the first hurdle would be to identify the markets the company would be interested in.
Besides administration systems, what other types of medical software systems do large
hospitals purchase?

There are many software systems, but for the sake of time, the team focused on three
primary markets: administration systems, patient administration, and physician support
systems.

What do those systems do?

Patient administration includes systems like admissions and tracking. Physician support
systems are more specialized, for individual physician procedures.

I would like to know how large each market is and how fast each is growing. I would use
secondary sources such as press releases, analyst reports, and published market studies, to
obtain this information.

Great! That is what we did during the market study. Our information revealed the following
market sizes and growth rates.

Administrati Patient Physician


on administration support
Market
1,500 1,000 1,200
size ($M)
Growth
5% 5% 12%
rate

From a size and growth perspective, physician support systems look like a very attractive
market. I'd like to know a little about the customers themselves. The client is currently
targeting large hospitals. Approximately what percentage of the market do they represent?
We were unable to get an exact breakdown, but we know that these hospitals make up the
vast majority of the total medical software market.
That would make sense, since the more sophisticated procedures at a hospital might
necessitate more advanced software solutions. I know that there have been a lot of changes
in the industry as a result of managed care. I don't know much about the industry, so I would
want to look at market studies and press clippings to get a better sense of the hospital
market in general and any technology or software trends more specifically.

ICC Handbook 2011 Page 61


Okay. Let's say that you did that and were presented with this summary of market trends:
Consolidation in the industry, with three to four large hospital networks dominating 45
percent of the market
Cost controls instituted, particularly as these large hospital networks acquire smaller
hospitals (centralization of functions being a key cost issue)
Many hospitals seeking to consolidate their vendor base
With regard to technology, many hospitals upgrading their older systems

If hospitals are consolidating vendors, perhaps our client has an advantage in being part of a
larger medical company. Maybe the client could also gain some advantages by expanding
into other software segments. Are the people responsible for purchasing software at the
hospital the same for all three segments?

Like all things, it differs by hospital, but the larger hospital networks have tried to
consolidate their purchasing not only within but also across hospitals.

Is the decision maker for medical software the same as for medical instrumentation and
devices?

In some cases, the head of purchasing influences both decisions, but the person who makes
the final choice is different. Software decisions are usually made by the hospital IT function
and those for instrumentation by the medical staff.

I think I have a pretty good understanding of the market for now. Let's look at competition
next. We could identify all the competitors and build up the market shares using a
combination of public data and estimates.

Well, let's assume that you don't have an infinite amount of time to look at all the
competitors. You can only look at the top five competitors in each market. You are given the
following data:

ICC Handbook 2011 Page 62


Administration
Sales ($M) Growth (%)
Systems

MedCount 700 4%
HCS Software Systems 100 7%
Morningside Software 80 3%
Admin Systems
70 2%
Solutions
HTI Software 50 15%
Patient Administration Sales ($M) Growth (%)

HTI 300 5%
Registration Software
240 4%
Solutions
Signup Software 60 3%
HCS Software Systems 30 16%
Patient Software 20 -1%

Physician Support Sales ($M) Growth (%)

HCS Software Systems 150 16%


Physician Support
100 11%
Systems
Medical Technology
25 18%
Inc
HTI 20 32%
MedSys 5 15%

Very interesting. The first thing I would note from the data is that the market concentrations
are very different. In administrative systems, the top five competitors control 66 percent of
the market and in patient administration, they control 65 percent. But in the physician
support market, they control only 25 percent.

I would want to know what gross margins look like in each of these markets as well. I might
turn to analyst reports and look at competitors' financial statements to deduce whether they
are making money in each market.

ICC Handbook 2011 Page 63


Gross margins vary, of course, but the analyst reports have margins of 25 to 30 percent for
administrative systems and for patient administration. For physician support, the margins
tend to be higher, more like 45 to 50 percent.

I see that two competitors, HTI and HCS Software Systems, have very large revenue growth
in all three sectors, although they each dominate one. I would want to look at their
financials, annual reports, and press releases to find out a bit more about their strategy in
each of these areas.

You'd find that they recently entered these non-core markets. Why might they have done
that?

Perhaps, like our client, each had a strong position in its own segment, HTI in patient
administration and HCS Software Systems in physician support. Maybe they too decided to
branch out into the other segments to find additional growth.

That is a very good hypothesis. Let's say there is evidence in the sources you consult that
supports your assertion.

Well, if that were true, these two companies could be a threat not only in the other two
segments, but also in our client's segment, administrative systems. It looks as if the client is
slowly losing market share in its segment, since it is growing more slowly than its market.

Good observation.

The market and competitor trends could also suggest that the client may want to enter these
other markets. In particular, the physician support market looks attractive; given it has high
growth and lack of a dominant competitor. The higher gross margins may provide attractive
returns on the necessary investment in software development. However, the patient
administration market may also be attractive. Although it is more concentrated and offers
lower margins than physician support, the client may be able to enter this segment with a
smaller up-front investment. Given the trend toward upgrading existing computer systems, it
may be important for MedCount to have a product offering in each of the three market
segments. That should not be too difficult, since the company is already in the software
industry.
Perhaps, but you should think a little more closely about these types of software. Are all
software systems alike?

Well, let me think about that for a moment. I suspect patient administration would have
relatively low entry barriers. From your earlier description, these systems appear to be pretty
basic, dealing primarily with admissions and patient tracking. However, the entry barriers in
physician support might be higher, since these systems are more complex and there are
probably multiple systems for the various physician procedures. I guess it would be harder to
get into those types of systems.

ICC Handbook 2011 Page 64


That would make sense.

Since the company might want to go into only some of the segments, I would want to know
how important it is to have products in all three segments. Do we know if the competitors
are marketing their products as a bundle?

How might you find that out?

Since it would be difficult to talk to a competitor directly, I would probably target a


competitor's customer, particularly one that just converted from our client's software.

Let's say you get an interview with a customer that recently switched to HTI. You discover
that the competitor was offering it a better pricing deal and service for software products in
all three segments.

How were MedCount's software and service perceived in relation to those of competitors?

The customer thought that its administrative systems were adequate, "the old standby," but
not stellar.

Were there any other key reasons it switched from MedCount's system?

When it decided to upgrade its systems, it tried to contact MedCount, but could never get a
representative to describe its options.

Interesting. How did HTI perform?

The HTI representative had heard that the company was considering switching software
vendors and provided a sales representative to pitch HTI's administrative product the next
day.
It definitely sounds as if there was a problem with the sales function and that customer
relations need to be improved, particularly for the larger hospital chains. There also seems to
be an advantage from both a marketing and sales perspective in having multiple software
products. I would want to confirm those views by doing further interviews.
Let's say further interviews support those assumptions.

Since we have already looked at the external conditions, I would like to move on to the client
itself. I'd like to know more about its marketing and selling organization as well as its
software development skills.

So far, we know that our client offers administrative software and that there may be a
problem with sales and marketing. Could you tell me a little about the marketing
department?

The marketing department is organized regionally. Teams are assigned to hospitals within
each state or geographic region, such as New England.

ICC Handbook 2011 Page 65


That could explain some of the problems with MedCount's marketing and sales. If hospital
purchasing is centralized, the marketing organization may be outdated. Does the company
have any teams dedicated to the four or five biggest hospital networks?

No, there are no dedicated teams. They talked about doing that for a while, but it conflicted
with the regional structure it had in place.

With regard to software, does the company feel it has any strengths or weaknesses?

It feels that their administrative product is very strong ("best of breed") and is the dominant
technology. Also, the product is modular in design, which allows for easier upgrades.
Although the company has never branched out into other market segments, the software
developers believe that certain modules could be used to build the foundation for other
administrative software programs. The company feels customer support is also an area in
which it excels.

Step 5: Summarize and make recommendations

Let's start with our client's market. The client dominates the administrative software market,
which is fairly large but growing slowly, and the company appears to be slowly losing market
share. Patient administration is also growing relatively slowly. Both markets are relatively
concentrated and appear to offer lower margins than physician support. The physician
support market is large and less concentrated, and could potentially provide higher margins,
but would require a larger investment. The hospital market itself is becoming more
concentrated and is pushing to consolidate vendors. The purchasing agent is often the same
for the three types of software.

Looking at our client's competitors, two, HTI and HCS Software Systems, appear to be
particularly threatening. Each has a dominant position in one segment and is branching out
into other areas. They appear to be marketing their products and services as a bundle and
are using service as a key point of differentiation.

The client offers only one type of system and appears to have some weaknesses in its
marketing organization, particularly in marketing to the larger hospital networks, which
offer the most promising market opportunities.

How would you recommend proceeding?


The first priority should be to fix the marketing organization, particularly for the large
hospital networks. MedCount will have trouble expanding into new markets if it can't defend
its current position and shore up its existing customer relationships. There should be a team
dedicated to each of the major chains. The client should also look at improving customer
tracking so that it is clear when its customers are going to upgrade. There should also be
clear contacts so that the customer can easily keep in touch with MedCount.

ICC Handbook 2011 Page 66


Next, I would recommend that the client explore entering the other market segments by
leveraging its dominant position in administrative systems. At first glance, patient
administration does not appear to be very attractive, with slow growth, low margins, and
large, dominant competitors. There appears to be some advantage, however, in having
products across the product range. I would recommend that we interview some of
MedCount's existing customers to better understand their needs and future IT requirements.
If the customer base is interested in one software provider for both back-office
administration and patient administration functions, this segment looks promising.

If the client does decide to enter this market, it should look at the lowest-cost method of
entry, either developing a product internally or acquiring a competitor. The modular design
of its existing administrative software suggests internal development of the patient
administration product may be the way to go, but we would need a more thorough
comparison of the internal development and acquisition options, including both cost and
time to market. I think that physician support offers our client an exciting growth
opportunity, given its high margins, high growth, and fragmented competition. I would
definitely think about an acquisition strategy, since the client may lack the technical
capabilities to enter this specialized market. I would recommend going for one of the larger
companies, as that would give the client a stronger position. Smaller companies would
probably not offer an important enough position in the market. More research would be
needed, however, for us to better understand the intricacies of the market and each
potential acquisition.

Those are very interesting conclusions. Thank you.

ICC Handbook 2011 Page 67


10.3 Jet fighter manufacturing case (BCG)

Step 1: Actively listen to the case

Your client is a U.S. defense contractor that manufactures the Mohawk Light Fighter Jet for
the British Royal Air Force. The company has produced the $20 million fighter jet for the
past 12 years. The British government has decided to put the contract out to bid, however,
and to win the program, the client's purchasing agents have estimated, the company will
need to cut its costs by 5 percent. It has asked BCG to help it reduce costs.

Step 1: Establish understanding of the case

Let me first clarify the question. The client manufactures a $20 million jet and, because of
competitive forces, has to reduce its cost by 5 percent. Is BCG's role also to verify the
purchasing department's estimate?

No, you can assume that the purchasing estimate is correct. BCG's role is to find the cost
savings to meet that estimate.

Could I take a few minutes to think about the case?


Sure, please do so.

Step 2: Set up the framework

First, I would like to understand the cost structure of the jet to see what we should look at
first. Next, I would like to look at major factors driving the costs we are targeting. Finally, I
would like to explore potential ideas to reduce cost.

That sounds like a very logical approach. Let's proceed.

Step 3: Evaluate the case using the framework

Because the time for the interview is limited, I think we should try to identify those areas
most responsible for the cost of the jet.

Time is limited on real projects as well, so I think that would be a good idea! You have the
following cost information for the jet. How would you interpret it?

ICC Handbook 2011 Page 68


The major cost driver for the jet appears to be purchased materials. Within manufacturing,
direct labor is a fairly large component of cost, as are program management and corporate
overhead within overhead. I think we would want to concentrate most on materials,
however, since that's where most of the costs can be found.

That sounds like a good place to start. Where would you look within materials?

I see that materials are broken down into purchased subassemblies, components, and raw
materials. I understand what raw materials would be, but what would be the difference
between components and subassemblies?

A subassembly functions on its own. An example is the pilot night vision system. A
component is a smaller part, such as a part of the engine.

I know that governmental agencies often have very strict guidelines about purchasing that
could affect the cost of materials.

For the sake of this case, you can assume that the British Ministry of Defense, MOD, allows
"commercial off-the-shelf" purchases, which means that the client is free to purchase from
whomever it wants, as long as it can ensure that the parts meet MOD quality guidelines.

I see that purchased subassemblies comprise more than 70 percent of materials. How many
suppliers are there for these subassemblies?

There are seven suppliers of major subassemblies that go into the fighter jet.

That seems like a relatively small number. Are there more suppliers that are qualified to do
this type of work?

ICC Handbook 2011 Page 69


The manufacture of these parts requires a substantial investment in R&D, engineering, and
infrastructure. It would be very costly for new suppliers to make the required investment,
particularly if the client is trying to reduce the price it pays to the subassembly
manufacturers.

Since there are only a few subassembly suppliers, and the investment hurdle would preclude
bringing in competing manufacturers, it would be difficult to reduce the price paid. Perhaps
we should look elsewhere for savings.

But remember, if your client loses the contract, it will lose its customer unless it is teamed
with the competing bidder. Even then, if the competitor is underbidding your client, there
will be even less room for it to profit.

Perhaps it would have an incentive to reduce its costs in order to maintain the contract. Are
the majority of its costs in materials as well?

How could you find that out?

I would want to interview the purchasing and engineering personnel of the different
subcontractors in order to understand their cost structures. If we had a better understanding
of their economics, our client might be able to reduce cost across the board, allowing it to
compete more effectively for the contract without killing everyone's margins.

Let's say that purchased materials average approximately 70 percent of the price paid to
most of the manufacturers.

If the cost of subassemblies represents 40 percent of the jet cost and 70 percent of that is
purchased materials, total purchased materials would be approximately 28 percent of the
cost for subassemblies. Purchases of raw materials and components represent another 15
percent, for a total of around 43 percent of the cost of the jet. If our client could reduce the
cost of raw materials by 20 percent, it could reduce the cost of the jet by more than 8
percent, more than enough to offset the 5 percent reduction it would need to win the
contract.

That sounds reasonable, but 20 percent is a very lofty goal. How would you go about doing
that?

First, I would look at the number of suppliers. Are there a large number of suppliers to the
subassembly manufacturers?

The client estimates that there are approximately 125 suppliers of raw materials and
components among the manufacturers of the subassemblies and itself.

Well, that sounds like a large number of suppliers. Of course, they could be providing very
specialized materials to the subassembly manufacturers. Are these suppliers providing
customized or more commodity products?

ICC Handbook 2011 Page 70


About 80 percent of these products are commodities, such as sheet metal and wire
harnesses. Even some of the electronics, such as printed wire boards and circuitry, are fairly
generic.

That sounds promising, but I would need to know whether these commodities are
interchangeable, so that our client could concentrate spending with fewer suppliers. Are
there many commonalities among the parts used by the different subassembly
manufacturers? We could talk to their engineers and look at the designs and bills of material
to determine how much overlap there is.

Let's say that you did this and discovered that approximately 30 percent of the cost of raw
materials is from similar materials used across the subassembly manufacturers.

It seems safe to assume that the client would need more commonality to be successful in
concentrating its purchasing and reducing costs. Do the engineers believe that the
percentage of overlap could be increased if the designs were modified?

They believe they could increase that percentage substantially, particularly with basic
materials such as screws and sheet metal, but also in other more customized areas.

That's great news, but we would still need to know whether the subcontractors are using the
same suppliers. We could analyze the number of suppliers for each of the areas of overlap.
Good suggestion. Although there are some common suppliers, the analysis indicates that
the subassembly manufacturers tend to use different suppliers.

Step 5: Summarize and make recommendations

Our client needs to reduce costs by 5 percent. The largest area of opportunity appears to be
in purchased materials, the majority of which comprise subassemblies manufactured by
seven subcontractors. By looking at its purchases in total, the client can target
approximately 40 percent of costs. To achieve the 5 percent cost reduction, it would need to
reduce costs by 15 to 20 percent. It could try to do that by increasing commonality in the
design of the subassemblies and components and by shifting volume to a smaller number of
suppliers.

Considering that the majority of the raw materials and components are purchased
commodities, do you think the 15-20 percent cost reduction is achievable?

Well, I know that commodities typically have lower margins than more customized products.
I suspect it may be challenging to hit the client's savings target by focusing only on these
purchases. But since raw materials and components represent about 40 percent of costs and
there is an opportunity to concentrate purchasing, I think we should start here.

ICC Handbook 2011 Page 71


Where else could you look for savings?

If I look back at the cost data on the jet, direct labor is another large cost component. As a
contingency, we could look into that area as well. I've read that other companies use
outsourcing to lower their manufacturing costs-perhaps our client could do the same. For
example, it might want to increase its use of purchased subassemblies and reduce the
amount of direct manufacturing it does. Of course this would work only if it could drive direct
labor costs below the offsetting cost of these subassemblies. The client will be working
closely with the subassembly suppliers to implement its purchasing initiative. This may give it
an opportunity to explore the suppliers' capabilities at the same time.

That's an interesting suggestion. How would you recommend the company pursue both of
the initiatives you have discussed?

I would look first to combine purchases across the subassembly suppliers with our client's
purchases. I suspect that the client and the subassembly suppliers will need to share a great
deal of information, including engineering drawings and specifications, with potential
suppliers of the raw materials and components. The Internet could prove to be a very
effective medium for forming a single "virtual" purchasing department to consolidate both
the flow of information and purchase orders across the companies. Our client might also
want to use a bidding system for those materials that are true commodities.

Next, I would turn to the engineering departments and form cross-company teams to look
for areas in which to increase commonality of design. At the same time, those teams could
explore opportunities to use more purchased subassemblies and decrease the client's direct
labor costs.

That sounds great, and is very similar to a project we did. I would caution you, however, to
examine the upfront costs involved in your recommendations, both for the redesign and for
the implementation of the purchasing system, before going ahead.

ICC Handbook 2011 Page 72


10.4 Gas retail case (BCG)

Client

Your client is the major operator (monopolist) in one of the largest European gas market.
His business includes two major activities:
Gas sales to households and firms (gas bought from large producers in Russia, Norway,
Algeria…)
Gas transportation from the national border, where it is delivered by the producer, to
the end consumers. This implies the existence of a large ensemble of infrastructures:
transportation network, distribution network, storage equipment, methane
terminals…

Let's discuss the challenges on the natural gas market after market liberalization in Europe.

Situation

Concretely, the market's deregulation means


The end of the monopoly for the gas sales; the arrival of new competitors
The preservation of the monopoly on transportation, but under the surveillance of an
independent authority that guarantees equal access to all competitors

Your client is at the head of the purchases/sales department. He is in the following situation:
Today, company market share is 100%
At a certain point in the next years the market will at once be opened to competition
(which is a simplified way of putting it since in reality there will be stages)

Client's question

About the gas sale activity that will be opened to competition


What will be the level of competitive intensity at opening?
What actors are likely to become my competitors?

Evaluate the case

According to you, how many and what types of competitors are likely to enter the market?
(Structure)

I believe I would need to evaluate the market attractiveness (market growth,


profitability/margin, risks) and the entry barriers (gas availability, brand). I would need to
ask the following questions:
What are the rules of the game/key success factors (access to suppliers, customer
intimacy, cost advantages, branding …)?
How are other players positioned to enter the market?
What are their competitive advantages thanks to synergies with other activities
(electricity, services …)?

ICC Handbook 2011 Page 73


Let us focus on the gas retail sale activity's attractiveness. There are three dimensions you
should consider: the natural gas market's growth potential, the profitability of this activity
and the risks associated with it.

Let us start with the market's growth potential. What are the market's growth levers?
(Structure)

I would differentiate between firms and households. The key levers by client type would be:
Households: network penetration, share of gas vs. other energies; consumption of
gas/household (climate, isolation …)
Firms: same as households, plus industry growth, productivity, competitiveness with
other energy forms
Given the market's main growth levers for the firms' segment and for the households'
segment, do you think that the market will strongly grow, stagnate, or decrease?
(Judgment)

For the households, I would forecast???? the rise of penetration (network extension) but,
overall, I think the consumption will decrease due to global warming and to better built
houses. For the firms, I think it will decrease, especially in industries that consume a lot of
gas (general price and risk issues).

So what is your conclusion? (Synthesis, So what)

I think there will be weak or inexistent growth. A new entrant will have to take clients from
the major player.

Can you imagine what a gas retailer's cost structure is? (turnover = 100)? (Synthesis,
Structure)

I believe it would include the energy itself (cost of goods – gas), the infrastructure cost and
sales and marketing costs (commercial).

Here is a simplified cost structure: gas – 50%, infrastructures – 40%, commercial costs – 7%
and the margin is around 3%. What cost advantage can a new entrant expect to build for
each one of these costs? (Judgment)

Most probably, there is a small opportunity of differentiation through costs:


Gas is sourced at comparable prices
Infrastructure prices are identical for all competitors
New entrants have to invest rather more in marketing
New entrants are not expected to have a productivity lever and only have a small pricing
lever.

I would have to check these assumptions.

ICC Handbook 2011 Page 74


Let us put ourselves in the shoes of a household client whose yearly gas invoice amounts to
€ 500. What is the price reduction potential for a new entrant? Can you give a rough
estimate? (Judgment, Rigor)

If I assume I can reduce commercial/marketing costs by 33% (500 x 7% x 33% = 11.55) and I
allow a 50% lower margin (500 x 3% x 50% = 7.5), then a new competitor can reduce the gas
price around € 15–20/year (11.55+7.5=19). This might allow it to compete with the
established client. Marketing costs can be reduced if the new entrant is already established
in other energy markets and benefits from scale and known brand name.

What can we conclude on a new entrant's margin level? (Synthesis)

Margin will necessarily have to be weak or inexistent to attract clients and draw away from
established player.

Let us now consider the risks borne by our retailer. In order to simplify, let us focus on what
is called the climatic risk. The sales volumes will vary a lot depending on the year, whether
the winter is cold or not. During a "warm" year, let's suppose that the heating volumes
decrease by 10%, that the cost of supply/gas are totally variable, that the commercial costs
are totally fixed, that the infrastructure costs are partly flexible, at 70%. What will be our gas
retailer's margin? (Structure, Rigor)
I am basing my analysis on the sales and cost structure of a normal year (turnover = 100).
Then I calculate the value of each cost block for a warm year, also the margin and compare
with the margin in a normal year.

Cold vs. warm


Sales: 100 vs. 90 (-10%)
_____________________
Gas: 50 vs. 45 (-10%)
Infrastructure: 40 vs. 38.8 (30% of 40 is variable, makes 12, 10% reduction makes 1.2)
Commercial: 7 stay 7

Total cost: 97 vs. 90.8

Margin: 3 vs. -0.8

In a warm year, it is more expensive to sell gas, so it is a high risk business.

What can we deduce from this risk calculation? (Judgment)

The climatic risk is too high to justify the small margin in a normal year.

Your first meeting with your client is tomorrow morning. What can you tell him/her to
answer his/her question based on the analyses that we have just done together? (Synthesis)

ICC Handbook 2011 Page 75


Well, the market is not that attractive and new entrants are a weak threat.

Finally, it looks like our major player does not have to worry; the gas retailer activity's
attractiveness is so weak that one would have to be stupid to venture in it at its opening!
But why would it be a big mistake to tell our client not to worry? (Creativity)

We are not working on the right strategic segment: the gas retail sale segment in not
independent of the electricity sale and services, as soon as the monopoly disappears. We
have been influenced by the client's historical view.

In fact there is a bias in our reasoning from the start. What is it? (Creativity)

We have looked at the gas market on a stand-alone basis. But we need to take into account
that the rules of the game might change and that other energy providers might enter the
market. Those providers might offer additional products to the gas client: electricity, oil,
services or other products.

Are there other levers that would enable a player to enter the gas market in a profitable
way? (Creativity)

By offering other energy products or services and products, there can be synergies with the
gas supply:
Channel diffusion/delivery costs
Margins from other services can cover production risk

On the other hand, there could be cost synergies on the commercialization:


Client back-offices could combine gas and electricity sales
Brand and client acquisition

Who could the other new players in the gas market be? (Judgment)

Potential new players that bring additional value to the client could be major electricity
firms, major oil producers and/or major retailers. For the electricity firms, synergies would be
mainly based on the commercialization cost synergies, also for retailers. For the oil
producers, there are synergies on the supply side.

What can we finally say to our client? (Judgment)

The threat is real; the firm's traditional strategic vision must be questioned due to the
emergence of the new market conditions and rules of the game. Examples of dangerous
players are large power firms, oil producers if they don't have more profitable investments to
make and a partnership between a large European energy player and a large retailer.

ICC Handbook 2011 Page 76


10.5 Consumer ADSL services cases (BCG)

The situation is as follows:

ADSL is a technology that enables the implementation of broadband Internet services via
the existing telephony infrastructure. This telephony infrastructure is owned by the
incumbent telecom operator in Norway. As in most European countries, the regulator has
ruled that new entrants may offer ADSL services, using the existing incumbent-owned
telephony infrastructure via MDF access. This means, the new entrant can hire the copper
wire from the incumbent operator but has to purchase its own ADSL equipment.

The technical set-up that a new entrant would need in order to establish an ADSL
connection basically consists of three elements: MDF access (copper wire), ADSL equipment,
Internet uplink capacity (fiber access connecting the ADSL equipment to the Internet).

The costs involved in establishing the technical set-up are given (amounts have been
converted to Euros):
MDF access tariff is EUR 12 per line per month, set by the regulator
Required investments for the ADSL equipment is EUR 120,000 per location, depreciation
period of 5 years. For simplicity reasons we assume linear depreciation with no
interest costs. In total, there are 250 locations where ADSL equipment could be
installed, covering all households in Norway
Internet uplink capacity is commercially available. Depending on the required end-user
speed, costs are on average EUR 2 per end-user per month
For simplicity reasons, we assume that the consumer ADSL modem is client-owned.

The case interview starts with an open question to test the candidate’s ability to apply
structure to a problem while a lot of information is still unknown.

Interviewer: Suppose you wanted to assess if a new entrant can run a profitable business in
offering consumer ADSL services, how would you approach this?

Candidate: Let’s see, since we are considering a new entrant, who has initially no customers,
the company will at first generate loss, and gradually will become more profitable as the
number of customers increases. I could make some assumptions on the pace at which the
customer base will grow, but rather than making this too complex at the beginning, I would
start by calculating some sort of break-even point. I mean, if we never reach break-even, this
idea will certainly not fly.

The structure is the basis for the entire case. A good structure should enable the candidate
to systematically solve the case, to set up some easy calculations to verify initial hypotheses,
and navigate back and forth through the case. Now, the interviewer asks the candidate to
do some easy calculations to see if he/she can apply the structure. This way the interviewer
tests if the candidate can combine the elements to a useful outcome (Synthesis), how
comfortable the candidate is in doing some basic calculations and if he/she applies sanity
checks to the outcome (Rigor).

ICC Handbook 2011 Page 77


I: Sounds good, how would you calculate the break-even point?

C: Hmmm, we know all the costs, but we don’t know the revenue, which would be price
times number of customers. So I could assume a price and then calculate the number of
customers we need. Subsequently, I would need to check if the required number of
customers would be a reasonable number. Do we know anything about the price?

I: We know that the incumbent charges on average EUR 20 per month.

C: OK, let’s assume we could also ask EUR 20 to start with. Later we may need to reconsider
this and see if we need to lower the price in order to become more competitive. So let’s see if
we can calculate this… hmmm, wait, I see we have a lot of information on technical costs,
but I don’t see anything on personnel costs, is this correct?

I: Well spotted, we also need an organization and a small marketing budget. The new
entrant has done some pre-work and has calculated that an annual budget of EUR 4.8
million would be sufficient to cover the entire country.

C: OK, then I can calculate the break-even point. Per customer, the new entrant earns EUR
20, of which it uses EUR 14 to cover the expenses for MDF access and the Internet uplink.
That means he has EUR 6 per customer per month to cover all other costs. We write off the
ADSL equipment in 5 years, so that’s EUR 2,000 per month per location. For 250 locations
that’s EUR 500,000 per month. In addition, we have to cover the organizational costs, which
is EUR 400,000 per month. So for break-even we need EUR 900,000/EUR 6=150,000
customers.

After the basic set-up of the case, the interviewer assesses the candidate's ability to make
some basic assumptions in order to get to the next level (Judgment) and play around a little
bit with that to see if the candidate can really see through the drivers of the case
(Synthesis).

I: OK, so what do you think, is that a realistic number to achieve? Let me add that we have
roughly 2 million households in Norway.

C: Hmmm, let's see...at first sight, 150,000 out of 2 million does not seem impossible, but I
would like to know a bit more about the market. For instance: is there a lot of growth in the
market? How many competitors are there? Can you tell me a bit more about that?

I: Yes, at the time of this situation, the incumbent operator was still the only provider of
ADSL services in Norway, but others have considered entering the market, just as we are
doing now. The distribution was as follows: 20% of the households had ADSL services, 20%
used Broadband via cable, 50% still used dial-up and 10% had no Internet. We know that
these percentages were established in 5 years time. That is: 5 years before no broadband
Internet existed. We have no data from in-between years.

ICC Handbook 2011 Page 78


C: OK… so that’s a fast growing market. I would say we start focusing on acquiring new
customers from the group that is currently using dial-up, since this is the largest group and
since it is probably easier to acquire new customers than to persuade customers from our
competitors to switch. Now, if we would take a situation for instance 5 years later and if we
assume that by then the market has grown again with roughly the same amount, the ADSL
market increases from 20% to 40%, which means 400,000 new households will subscribe to
ADSL. We would be competing for these clients, against the incumbent operator, and maybe
some other new entrants as you mentioned. So… let’s say the incumbent operator gets at
least a 50% share of that, since it has a strong presence in the market already. Let’s assume
we would be competing against 1 or 2 other competitors, which means we could get 17–25%
share of the new customers, or ~70,000–100,000 in total. So it seems that it won't be
possible to reach break-even from new customers only. We could also have a look at the
existing customers, both ADSL and cable customers. But then I would need to know how
likely it is that they would switch?

I: OK, that is something you may want to investigate. But what would be your upfront
judgment on this?”

C: I can imagine that customers would want to switch if they are very unsatisfied with the
current service, or if the new entrant would offer a much lower price. Do we know anything
about customer satisfaction?

I: Most customers are very satisfied, the service is very reliable.

C: OK, so let’s look at lowering the price. For instance a 10% discount…. Hmm… but that
would mean we also would need more customers to reach break-even. A 10% discount
means we drop from EUR 20 to EUR 18 contributing EUR 4 per customer instead of EUR 6 to
the fixed costs. So we would need 225,000 customers in this case. Even if we could get a
greater share from new customers, I still think it would be unrealistic to assume we could
persuade a large number of customers to switch for a EUR 2 discount, particularly if they are
satisfied with the current service. In addition, there’s the risk that our competitors would
lower their prices too. Concluding, I would say this is a risky business to step into.

When the basics of the case have been cracked, the interviewer would finally test if the
candidate is able to assess the problem from different angles and if he/she can come up
with alternative solutions (Creativity). If the case runs smoothly and there’s time left, the
candidate can even quantify part of these alternative solutions.

I: I agree. The way we have been looking at it, the business case looks very challenging
indeed. But let’s spend a minute to see if we can come up with some creative ideas here.
Could you think of some ideas that would make this business case more attractive?
C: I was struck by the negative effect a slight price decrease has. So, I was thinking: could we
somehow increase the price? Of course, this can only be justified by offering better service.
Would it be possible to increase the speed, for instance?

ICC Handbook 2011 Page 79


I: Yes, technically this would be possible. You would need to make sure that you buy enough
capacity on the Internet uplink. Let’s assume the speed you can offer is directly proportional
to the capacity you buy and the costs associated with it.

C: OK, so doubling the speed would increase the cost per subscriber by EUR 2 per month,
while we could probably charge a much higher tariff, maybe EUR 30 or so. If we could
diversify our offering, and if part of the customers subscribe to the premium packages, this
would certainly improve the case. Furthermore, I am considering offering additional services,
like telephony or even video services if that’s possible. Of course we would need to make a
separate business case for that, but since EUR 12 out of the 20 we charge is spent on MDF
access, any other service we can offer, which re-uses this asset, would greatly improve the
case. I mean, we would spread our largest cost component across multiple services.

I: OK, sounds like these ideas are worth investigating. Anything else you can think of?

C: What about starting in a limited number of regions, instead of covering the entire country
from the start? Is the number of customers per location equal across the country?

I: Good point! The area of each region is more or less the same, which means…

C: You have a huge variation in the number of households per location. If we pick the high
density areas, for instance the Oslo area, plus some other larger cities, we can perhaps reach
50% of the households through, I don’t know, maybe only 20% of the locations and thus only
20% of the ADSL equipment cost. That would make a difference.

I: You’re absolutely right, and you know, looking back this is exactly what new entrants have
done when entering the ADSL market: start in high density areas, offer higher speeds at
premium prices, in particular to small and medium-sized enterprises, and – at a later stage –
expand the service by offering additional services.

ICC Handbook 2011 Page 80


10.6 Call center case (BCG)

The case interview

There is a company, let's say a travel agency, that sets up a centralized call center where all
incoming phone calls from all branches are diverted to. Shortly after setting up the call
center, the travel agency is virtually unreachable because of the large amount of phone calls
received.

Before answering any questions, the candidate should check if he/she completely
understands the problem stated. The candidate can ask additional questions if the problem
is not completely clear.

The first question tests the candidate's creativity and ability to come up with more than one
idea, at the same time it tests the use of a structure, which helps coming up with more
ideas. First, the candidate should think of main causes, and then specify these main causes
in more detailed possibilities.

Interviewer: What could cause the overload of the call center?

Candidate: I believe there could be different types of reasons:

Caused by supply
Lack of call center agents (by sickness, by poor planning)
Poor organization of the call center (unstructured call-diverting)

Caused by technical difficulties


Calls are not transferred to the centralized call center

Caused by demand
Peak in client demand (more calls than expected due to international affairs, high-
season, bad weather…)

I: Let us assume that the overload in the call center is not caused by technical problems or
by peaks in demand, but is solely caused by the fact that the amount of required agents
does not match the number of agents in the center. How would you calculate the required
number of call center agents needed to fix the problem?

For this question, it is important to set up a good structure to tackle it. Not a general
structure, but one tailored to this specific situation. This is one of the main things the
interviewer tests with this question: can the candidate set up a good structure, tailored to
the problem. Furthermore, the interviewer tests if the candidate understands the problem
and has some (business) judgment (e.g. in this case: a call center agent is not productive 8
hours per day).

ICC Handbook 2011 Page 81


A possible answer could be as shown in the next figure:

Number of diverted
customers
x
Number of calls per
Call volume
customer
x
x Seasonal pattern (%)

Required number of
agents Call time (sec)
Average settlement time +
Wrap-up time (sec)

: Working hours per agent


(sec)
x
Capacity per agent Gross/net ratio (%)
x
Working Time Duration
(%)

This answer first divides the required number of agents in volume of calls, average time
spent on a call and capacity per agent. The next step would be to divide these three main
parts into smaller parts. For the volume of calls, the first important driver is the amount of
diverted customers. Furthermore, customers can call up more than once; this is called the
average number of calls per customer. The third driver of the volume of calls is the seasonal
pattern. The average time spent on a call is simply the duration of the call and the time
needed by the agent to wrap up the call (e.g. enter information in a computer).

The capacity per agent starts of course with the working hours per agent, but an agent is not
100% productive. There are several ways to define the productivity, in this example the
productivity is split up in two parts. The first is gross versus net working hours due to lunch
breaks etc. The second is "working time duration", this is the percentage of the time the
agent actually is busy with a phone call. An agent can be busier during prime time and less
during other hours of the day.

There is now a clear structure to show what drives the amount of agents needed, and
therewith a way to calculate the required amount of agents.

I: How many agents are required in week 20?

ICC Handbook 2011 Page 82


Most case interviews have some computation in them. To solve it, the candidate will have to
figure out the numbers for all parts of the "driver-tree" to compute the required number of
agents. There are several ways to figure out the numbers. First of all, an interviewer might
give the candidate some data to start with, and he/she will have to pick out the numbers to
use in the calculation. Secondly, the candidate can ask the interviewer for data, and thirdly
the candidate can make assumptions using common sense. The candidate should always
clearly state when he/she is making an assumption and how he/she comes up with it.
In this case interview, the following data is given by the interviewer:

Cumulative number of customers who are


diverted to the centralized call center
Number of 1.200.000
customers

1.000.000

800.000

600.000

400.000

200.000

0
0 10 20 30 40 50 60

Week

Note: At the moment of set-up of the call center, the graph starts with 0.

ICC Handbook 2011 Page 83


From this graph, the candidate can deduct the cumulative number of diverted customers in
week 20: approximately 140.000 customers. So in week 20, there are 140.000 clients "in the
database of the call center". To calculate the required number of agents, other data in the
driver-tree is needed, and the candidate will have to figure it out. He/she can start off with
asking the interviewer for data, but it's always best to make assumptions and test these
with the interviewer. For example: "I assume that on average, a client has contact with an
agent of the call center twice per year, so the average number of calls per customer is 2".
And: "I wonder if there are many more calls in week 20 versus the other weeks of the year,
for now I assume the seasonal pattern to be 0%". The candidate then calculates the calls'
volume: 140.000 / 52 * 2 + 0% = approximately 5.400 calls in week 20.

To determine the average settlement time, the candidate can ask the interviewer for
information. The interviewer may either tell the candidate to make an assumption, or give
the numbers. In this case, these are given: "Monitoring talk- and wrap-up-time gave an
average call time of 240 seconds and an average wrap-up-time of 60 seconds per call". This
information gives the candidate the average settlement time: 240 + 60 seconds = 5 minutes
= 1/12 hour.

The last driver is the capacity per agent. The candidate can assume that the number of
hours an agent works is 40 per week. To determine the gross/net ratio, the candidate could
ask the interviewer, though he/she could also easily make an assumption on own
experience: "I assume that of the 8 hours worked in a day, approximately 1.5 hour is lost to
lunch and other breaks. This makes the gross/net ratio ~80% ((8-1.5)/8 = 81%). The last
number to figure out is the working time duration, this is a hard number to guess without
knowledge of call centers, but the candidate could still give it a try. The candidate should
then ask the interviewer if the assumption is more or less right. From experience, the
number is ~50%. This gives the capacity of an agent per week: 40 * 80% * 50% = 16 hours
per week.

Now, the candidate has all numbers to calculate the amount of agents needed in week 20.
The candidate should not forget to use the same units for all parts of the equation (all in
seconds, hours, or weeks) and not make any unnecessary mistakes. In this case, the number
of agents needed in week 20 is: 5.400 calls *1/12 hours per call / 16 hours per week = 28.1,
so either 28 or 29 agents are needed.

I: There are no additional trained call center agents available. What other ways can you
think of to help the travel agency with its telephone problem?

It is stated that the amount of required agents does not match the number of agents in the
center. However, since there are no additional trained call center agents available, the
candidate has to find ways to decrease the amount of agents needed.

ICC Handbook 2011 Page 84


This question will test if the candidate really grasps the problem and if he/she uses his/her
structure in the right way. Looking again at the structure, the answer is clear: There are
three levers to decrease the amount of agents needed:

1. Reduce the call volume


2. Reduce the average settlement time
3. Increase the capacity per agent

To determine specific solutions, the candidate needs creativity and understanding of the
problem.

On the first lever:


Stop diverting the calls from branches to call center (and divert back)
Insert a computer-based menu which answers the most frequently asked questions

A possible solution to the second lever is:


Reduce the wrap-up time by monitoring it, determine the most common wrap-up
actions and shorten these actions where possible. For example introduction of a
computer application that helps the agents with the standard wrap-up actions such
as "sending a brochure"

For the third lever, a possible solution would be:


Align planning of agents with actual demand for agents by mobilizing more agents
during peaks in client demand and less during a low. This solution can be for example
achieved by stimulating the agents to work part-time.

10.7 Supermarket deli turnaround case (BCG)

Questions and Facts

1. Client’s deli financials

See Exhibit 1

2. Overall industry/ customers

Deli meat category has been flat to slightly declining recently. Prepared foods category has
been growing at roughly 10% per year as people have less time to cook at home.

3. Competitors

Increasing competition from deli departments of other supermarkets, discounters, etc. –


e.g., expanding product lines, increasing advertising. Also competes with fast food
restaurants in prepared foods category.

ICC Handbook 2011 Page 85


4. Client’s product mix and recent events

Mix has remained constant, with the exception of two products introduced a couple of years
ago – BBQ chicken wings and “made to order” sandwiches. Both products have been a
major boost to prepared foods revenue.

5. Info on new products

BBQ wings are similar to the chicken wings the company already sells, although they take a
little longer to fry and are tossed in BBQ sauce after frying. “Made to order” sandwiches is
client’s response to Subway, etc. – for two hours during lunchtime and two hours during
dinnertime, one employee’s sole task is to make sandwiches to order for customers.

6. Financials of new products

Revenues for each product are $40M annually. Costs are not broken down at the product
level.
See Exhibit 2.

SUPERMARKET DELI TURNAROUND


Exhibit 1

$M
2002 2003 2004
Deli meats Revenues 260 255 260

COGS 160 155 160

Prepared foods Revenues 360 400 440

COGS 190 230 270

Overall Revenues 620 655 700

COGS 350 385 430

Gross margin 270 270 270

ICC Handbook 2011 Page 86


SUPERMARKET DELI TURNAROUND
Exhibit 2

BBQ wings “Made-to-order” sandwiches


Price $5 for 20 pieces Price $4 per sandwich

Total material cost $0.10 per piece Avg. sales/store 20 sandwiches


per day
Prep time 15 minutes per
batch of 200 Total material cost $2 per sandwich

Employee cost $20 per hour Employee cost $20 per hour
(fully loaded) (fully loaded)

Total COGS $2.50 per 20 pieces Dedicated hours 4 hours per day

Margin $2.50 per 20 pieces Revenue $80 per store per day

Total COGS $120 per store per day

Margin $(40) per store per day

Note: Boxes indicate figures that should be calculated by the interviewee

SUPERMARKET DELI TURNAROUND


Sample Approach

Main question What should supermarket do to turn around deli?

Key areas to Revenue and profit breakdown within deli External factors influencing the overall deli
explore market

• Deli meat revenue and profits flat - consistent • People have less time to cook at home –
with overall category prepared foods category growing, deli meats
• Prepared foods showing revenue growth (10% category flat
consistent with category) but no profit growth • Increasing competition from other deli
Analysis – therefore declining margins – why? departments – starting to expand product lines,
- made-to-order (MTO) sandwiches losses increase advertising, etc.
offsetting profit growth from BBQ wings

• Eliminate made-to-order sandwiches (at least in low-traffic stores or during non-peak hours)
Recommend- • Raise or lower prices on MTO sandwiches (depending on demand elasticity)
ation • Boost demand for MTO sandwiches (e.g., advertising, promotions, merchandising)

• Eliminating MTO sandwiches or boosting demand can impact overall traffic in store and deli
Other factors

ICC Handbook 2011 Page 87


Framework and Analysis

There are three main questions asked to the candidate:


Which part of the business is responsible for the lack of profit growth – deli meats,
prepared foods, or both?
Is the lack of profit growth caused by flat revenues, increasing costs, or both?
What is causing the flat revenues or increasing costs (and what should the client do)?

Based on Exhibit 1, the candidate will see that gross margins for both business lines are flat.
Furthermore, deli meat sales have been basically flat while prepared foods sales have been
growing at 10%.

The candidate should recognize that the client’s deli meat and prepared food sales have
been growing at about the category averages; therefore, revenues are not the main issue
here. Deli meat COGS have been more or less flat, mirroring sales. However, despite robust
growth in prepared food sales, prepared food profits have been flat, implying deteriorating
margins.

At this point, the candidate is asked for some potential reasons for deteriorating margins
(e.g., change in product/sales mix, rising material costs, rising labor costs).

If the candidate asks about changes in product mix, the interviewer informs him/her about
the BBQ chicken wings and the “made to order” sandwiches. The candidate should be
suspicious at this point and ask to learn more about these products.

By doing a back-of-the-envelope analysis of product profitability (based on data in Exhibit 2),


the candidate can find that BBQ wings have a 50% margin, indicating that they are not a
problem. On the other hand, he/she will find that the client is losing a lot of money on the
“made to order” sandwich concept.

The candidate is then asked for recommendations, which could include:

1) Eliminating the “made to order” sandwich


2) Restricting the “made to order” sandwich to busier stores or during busier times of the
day (e.g., lunch hours only)
3) Raising or lowering prices (to either increase profit per sale or units sold – will depend on
demand elasticity)
4) Boost demand (through increased advertising, promotions, better merchandising, etc.).

The candidate can also consider the second-order effects of eliminating the product or
boosting sales (the effect on traffic in the deli and the overall store).

ICC Handbook 2011 Page 88


10.8 China outsourcing case (BCG)

Problem set-up

The client is a national manufacturer of plastic consumer products that are sold in a variety
of retail formats, including supermarkets, discounters, club stores, and dollar stores. The
company has three main product lines: 1) freezer bags, 2) plastic plates and utensils, and 3)
specialty plates and utensils.

The CEO has been reading for some time about American companies outsourcing their
production overseas to low-cost countries such as China. She wonders whether this makes
sense for her company as well. It worries her that none of her main competitors have
established foreign production capabilities; on the other hand, this could be a tremendous
opportunity to gain a competitive advantage.

We have been asked to help the client understand the benefits and risks of moving its
production capabilities to China and to provide a recommendation.

Question and Facts

All three product lines have similar cost structures and savings (see Exhibit 1)

1. Freezer bags

Plastic bags are used mainly to store food items in freezers. According to customers, top
purchase criterion is quality, since low quality bags will result in food spoilage. Client is #3 in
category, with 200 million lbs. sold. The category leader has a strong brand and strong
innovation.

2. Plastic plates and utensils

Disposable plates and utensils; intended for single/limited use. According to customers, top
purchase criterion is price. Client is #2 in category, with 300 million lbs. sold. Client is at cost
parity with category leader but has a weaker brand.

3. Specialty plates and utensils

Plastic plates and utensils produced for specific retailers, customized to their design specs.
According to customers, top purchase criterion is style/design. Because many products are
new and untested, demand is highly variable. Client is #1 in category, with 100 million lbs.
sold. No strong competitors.

ICC Handbook 2011 Page 89


4. Current client production capabilities

All products are made in a single factory in Ohio. The factory is at capacity and the company
is considering building or acquiring a nearby facility.

5. Chinese production options

Client has no previous experience in building and managing a factory overseas. Client has
met with several Chinese manufacturing partners and has done initial product testing.

All three product lines have similar cost structures and savings (Exhibit 1).

Quality: lower quality on freezer bags, equal quality on plastic plates and utensils (both
regular and specialty).

Lead time: need 3-4 weeks of additional lead time for each product line for transportation
from China to U.S. distribution center.

6. Chinese market, current client presence

All three categories are relatively underdeveloped but growing, dominated by local
manufacturers. Client does not currently have any sales in China, although a few of their
U.S. customers (e.g., Wal-Mart) do have presence there.

ICC Handbook 2011 Page 90


CHINA OUTSOURCING OPPORTUNITY
Exhibit 1

Costs in Costs in China Costs in


Costs U.S. ($/lb.) relative to U.S. China ($/lb.)
Labor 0.30 8% of wage rate
80% of productivity
Material
• Plastic resin 0.30 80%
• Other material (incl. packaging) 0.20 75%

Variable overhead 0.05 140%

Fixed overhead 0.10 60%

Transportation
• China to U.S. distribution center N/A $6K to ship 40K lbs.
• U.S. distribution center to customer 0.05 Same

Total 1.00

CHINA OUTSOURCING OPPORTUNITY


Sample Approach

Main question Should plastics manufacturer move production to China?

Key areas to Cost savings Consumer behavior and Effect on current


explore purchase criterion production capabilities

• Would save $0.25/lb. (25% of • Quality is top purchase • Current plant is at capacity
current costs) criterion for freezer bags - outsourcing would
• At current production levels, - lower quality from China eliminate need to build
would save: • Price is top criterion for plastic additional capacity
Analysis - $50M in freezer bags plates and utensils • Plastic plates and utensils are
- $75M in plastic plates and • Style is top criterion for 50% of total production
utensils specialty plates and utensils - outsourcing may create too
- $25M in specialty plates and - highly variable demand much extra capacity
utensils requires short lead times

• Outsource plastic plates and utensils to China


Recommend- • Do not outsource specialty plates and utensils
ation • Do not outsource freezer bags (although further analysis may be warranted)

• To compensate for extra capacity that would be created in current plant, could produce new product
Other factors
line, rent out spare capacity, or move to smaller facility

ICC Handbook 2011 Page 91


Framework and Analysis

The candidate should start with a brief overview of the potential benefits and risks of
outsourcing to China. The main benefit is lower costs, mostly driven by inexpensive labor. A
secondary benefit is a possible springboard into the emerging Chinese (and other Asian)
market. Risks include lower labor productivity, possible quality issues, longer lead times,
additional transportation costs, and potential communication/coordination issues. The
candidate can be asked about the ramifications of longer lead times – they include greater
carrying costs, higher cycle and safety stock, greater forecast error, and less responsiveness
to demand.

There are three main questions asked to the candidate:


How much cheaper is producing in China?
What do consumers value and how would outsourcing affect those criteria?
What are the client’s current production capabilities and how would outsourcing part/all
of their production affect the remainder?

First, the candidate should size the opportunity – is this a $5 million or a $500 million
opportunity? By solving for the last column in Exhibit 1, the candidate will find that the
client would save $0.25/lb. (25% of current costs) by outsourcing to China. Given current
production levels, the client would save $50 million by outsourcing freezer bags, $75 million
by outsourcing plastic plates and utensils, and $25 million by outsourcing specialty plates
and utensils. Two notes: 1) costs may increase if the Chinese Yuan rises versus the dollar and
2) these estimates do not include a profit margin for the Chinese outsourcing partner.

The candidate should also recognize that cost savings alone are not sufficient to make a
decision. It is important to understand how an outsourced product will affect sales. The
candidate should suggest market research to understand consumer behavior.

Freezer bags: since customers’ top purchase criterion is quality and outsourcing would
produce lower quality bags, the candidate should raise a red flag here. A more sophisticated
recommendation would be to conduct market research to see the impact on sales of the
lower quality bag at lower prices – even though quality is more important than price, the
magnitude of a price change may override the drop in product quality.

Plastic plates and utensils: the top purchase criterion here is price, which makes this product
line an attractive outsourcing opportunity. The candidate can be asked what the client
should do with the cost savings – potential recommendations include dropping price to steal
share, investing to defend its position in case competitors begin outsourcing (e.g., brand,
innovation, customer service), and milking the product line as a cash cow.

Specialty plates and utensils: the highly variable and unpredictable demand for these
products means that shorter lead times are critical in order to adjust production quickly.
Longer lead times will result in greater forecast errors, higher safety/cycle stock, and more
unsold inventory and/or out-of-stocks. Therefore, specialty plates and utensils should not be
outsourced.

ICC Handbook 2011 Page 92


An analysis of customer purchase behavior indicates that plastic plates and utensils should
be outsourced, specialty plates and utensils should not be, and freezer bags probably should
not be. The final step is to understand the impact of outsourcing on the client’s current
production capabilities. For example, will it lead to plant closings (resulting in closing costs
and possible negative publicity)? Will it lead to underutilization of current facilities?

Since the current plant is already near capacity, moving plastic plates and utensils offshore
would actually save the client from investing in new facilities. However, since that product
line makes up 50% of total production (in terms of lbs.), removing it may create too much
extra capacity in the current plant for the two remaining lines. To compensate, the client
could produce a new product line, rent out the extra capacity, or move to a smaller plant.

10.9 Specialty paper sales case (BCG)

Problem set-up

Your client is a leading manufacturer of specialty papers sold to commercial printers. The
client produces self-adhesive sheeted papers that are ultimately used in a variety of labeling
applications – including the labeling of consumer goods and the printing of self-adhesive
signs.

Your client’s operations are profitable, but the business has failed to grow over the past few
years. The client would like to invest in the business and you have been asked to identify
opportunities for growth.

ICC Handbook 2011 Page 93


GROWING SPECIALTY PAPER SALES
Questions and Facts (I)

Topic Inform ation


Supply chain
Raw mats Layering Sheeting Packaging Distrib.
• Rolls of paper • Adhesive and • Rolls of layered
• Adhesive liner applied paper cut into
• Non-stick liner to rolls sheets

• The client is not capacity constrained in its manufacturing processes


• The client’s manufacturing and packaging operations are currently configured to package
specialty papers in boxes

Customers • There are approximately 24,000 commercial printers in the United States
• Printers are roughly categorized into three groups: small, medium, and large
• Differences among the groups are driven by the type of printing technology employed and the
size of print jobs that the printers are able to serve
• Printers prefer to receive product from the specialty paper manufacturers in different forms,
primarily driven by the type of printing technology employed
- Small printers prefer to receive their specialty paper in boxes
- Medium printers prefer cartons of specialty paper
- Large printers prefer to receive palletized shipments of specialty paper

Market share • The client has approximately 30% market share with small printers and only 10% share with
medium and large printers

Client financials • Margins are currently acceptable but management is against cutting price to gain market share,
knowing that competitors can match price cuts
• Price and cost to serve per equivalent box are different for each customer type

GROWING SPECIALTY PAPER SALES


Questions and Facts (II)

All figures are per equivalent box


Small (boxes) Medium (carton) Large (pallets)

Price to printer 20.00 18.00 15.00

Materials 5.50 5.50 5.50

Coating 1.00 1.00 1.00

Sheeting 0.50 0.50 0.50

Packaging (direct costs) 3.00 2.00 1.00

Gross profit/($ per equivalent box) 10.00 9.00 7.00

Number of printers 20,000 3,000 1,000

Annual usage (number of equivalent 100 500 3,000


boxes)

Total potential profit pool ($) 20,000,000 13,500,000 21,000,000

Framework and Analysis

The problem set-up indicates that the client wants to invest in this business. Investment can
take many forms including expansion of manufacturing operations and capacity, expansion
of customer-facing activities, and acquisition of competitors. It is interesting to note that
this is currently a profitable, no-growth business for the client. Investment decisions cannot

ICC Handbook 2011 Page 94


be made unless the management team (and the candidate) understands the market
conditions as well as the client’s internal capabilities

This case does not lend itself well to traditional “case solution structures”. A strong initial
response is to list a set of internal and external factors that must be understood and
evaluated. Ultimately, the candidate should decide whether investment is warranted, and if
so, where.

Strong hypotheses might include:


Assuming the client is not capacity constrained, there are likely groups of customers that
represent opportunities for profitable growth
Depending upon the current go-to-market strategy, the client may need to re-evaluate
the way that it is configured to serve existing and potential customers

The client can expand its packaging operations to better serve medium or large customers,
but not both. Client economics and cost to serve each customer group are shown on Exhibit
1. For simplicity, taxes and depreciation are ignored and SG&A is assumed to be fixed.

The candidate should recognize that a comprehensive solution evaluates the required
investment to serve a particular market segment (packaging line, manufacturing operations,
additional SG&A) against the expected return.

The candidate should evaluate the profit pool from serving medium and large customers.
This should be based upon an assumption about the size of the market that the client can
capture. Assuming the client can match its small printer market share, the client could
capture an additional 20% of the medium or the large printer customer segment

The following is given to the candidate:


Investment and operation of carton packaging line would cost $675,000 per year
Investment and operation of the palletizing line would cost $2,300,000 per year

A logical conclusion would be that an investment in a carton packaging line would be a


superior investment compared to the palletizing line

Potential conclusions:
The carton packaging line is a less-risky investment (requires less up-front capital)
The solution assumes a static environment. If large printers are growing in number and
or usage of specialty paper, this may change the answer
The investment in a new carton packaging line would need to be evaluated against other
potential investments to understand if it is the optimal use of the client’s capital

ICC Handbook 2011 Page 95


This Practice Case was sponsored by:

Jet Fighter manufacturing case

STEP 1: ACTIVELY LISTEN TO THE CASE

Your client is a U.S. defense contractor that manufactures the Mohawk Light Fighter Jet for the British Royal
Air Force. The company has produced the $20 million fighter jet for the past 12 years. The British
government has decided to put the contract out to bid, however, and to win the program, the client's
purchasing agents have estimated, the company will need to cut its costs by 5 percent. It has asked BCG to
help it reduce costs.

STEP 2: ESTABLISH UNDERSTANDING OF THE CASE

Let me first clarify the question. The client manufactures a $20 million jet and, because of competitive forces, has to reduce its cost
by 5 percent. Is BCG's role also to verify the purchasing department's estimate?

No, you can assume that the purchasing estimate is correct. BCG's role is to find the cost savings to meet that
estimate.

Could I take a few minutes to think about the case?

Sure, please do so.

STEP 3: SET UP THE FRAMEWORK

First, I would like to understand the cost structure of the jet to see what we should look at first. Next, I would like to look at
major factors driving the costs we are targeting. Finally, I would like to explore potential ideas to reduce cost.

That sounds like a very logical approach. Let's proceed.

STEP 4: EVALUATE THE CASE USING THE FRAMEWORK

Because the time for the interview is limited, I think we should try to identify those areas most responsible for the cost of the jet.

Time is limited on real projects as well, so I think that would be a good idea! You have the following cost
information for the jet. How would you interpret it?

INSEAD Consulting Book – 2006

- 76 -
This Practice Case was sponsored by:

The major cost driver for the jet appears to be purchased materials. Within manufacturing, direct labor is a fairly large component
of cost, as are program management and corporate overhead within overhead. I think we would want to concentrate most on
materials, however, since that's where most of the costs can be found.

That sounds like a good place to start. Where would you look within materials?

I see that materials are broken down into purchased subassemblies, components, and raw materials. I understand what raw
materials would be, but what would be the difference between components and subassemblies?

A subassembly functions on its own. An example is the pilot night vision system. A component is a smaller
part, such as a part of the engine.

I know that governmental agencies often have very strict guidelines about purchasing that could affect the cost of materials.

For the sake of this case, you can assume that the British Ministry of Defense, MOD, allows "commercial off-
the-shelf" purchases, which means that the client is free to purchase from whomever it wants, as long as it can
ensure that the parts meet MOD quality guidelines.

I see that purchased subassemblies comprise more than 70 percent of materials. How many suppliers are there for these
subassemblies?

There are seven suppliers of major subassemblies that go into the fighter jet.

That seems like a relatively small number. Are there more suppliers that are qualified to do this type of work?

The manufacture of these parts requires a substantial investment in R&D, engineering, and infrastructure. It
would be very costly for new suppliers to make the required investment, particularly if the client is trying to
reduce the price it pays to the subassembly manufacturers.

Since there are only a few subassembly suppliers, and the investment hurdle would preclude bringing in competing manufacturers, it
would be difficult to reduce the price paid. Perhaps we should look elsewhere for savings.

INSEAD Consulting Book – 2006

- 77 -
This Practice Case was sponsored by:

But remember, if your client loses the contract, it will lose its customer unless it is teamed with the competing
bidder. Even then, if the competitor is underbidding your client, there will be even less room for it to profit.

Perhaps it would have an incentive to reduce its costs in order to maintain the contract. Are the majority of its costs in materials as
well?

How could you find that out?

I would want to interview the purchasing and engineering personnel of the different subcontractors in order to understand their cost
structures. If we had a better understanding of their economics, our client might be able to reduce cost across the board, allowing it
to compete more effectively for the contract without killing everyone's margins.

Let's say that purchased materials average approximately 70 percent of the price paid to most of the
manufacturers.

If the cost of subassemblies represents 40 percent of the jet cost and 70 percent of that is purchased materials, total purchased
materials would be approximately 28 percent of the cost for subassemblies. Purchases of raw materials and components represent
another 15 percent, for a total of around 43 percent of the cost of the jet. If our client could reduce the cost of raw materials by 20
percent, it could reduce the cost of the jet by more than 8 percent, more than enough to offset the 5 percent reduction it would need
to win the contract.

That sounds reasonable, but 20 percent is a very lofty goal. How would you go about doing that?

First, I would look at the number of suppliers. Are there a large number of suppliers to the subassembly manufacturers?

The client estimates that there are approximately 125 suppliers of raw materials and components among the
manufacturers of the subassemblies and itself.

Well, that sounds like a large number of suppliers. Of course, they could be providing very specialized materials to the subassembly
manufacturers. Are these suppliers providing customized or more commodity products?

About 80 percent of these products are commodities, such as sheet metal and wire harnesses. Even some of
the electronics, such as printed wire boards and circuitry, are fairly generic.

That sounds promising, but I would need to know whether these commodities are interchangeable, so that our client could
concentrate spending with fewer suppliers. Are there many commonalities among the parts used by the different subassembly
manufacturers? We could talk to their engineers and look at the designs and bills of material to determine how much overlap there
is.

Let's say that you did this and discovered that approximately 30 percent of the cost of raw materials is from
similar materials used across the subassembly manufacturers.

It seems safe to assume that the client would need more commonality to be successful in concentrating its purchasing and reducing
costs. Do the engineers believe that the percentage of overlap could be increased if the designs were modified?

They believe they could increase that percentage substantially, particularly with basic materials such as screws
and sheet metal, but also in other more customized areas.

That's great news, but we would still need to know whether the subcontractors are using the same suppliers. We could analyze the
number of suppliers for each of the areas of overlap.

Good suggestion. Although there are some common suppliers, the analysis indicates that the subassembly
manufacturers tend to use different suppliers.

INSEAD Consulting Book – 2006

- 78 -
This Practice Case was sponsored by:

STEP 5: SUMMARIZE AND MAKE RECOMMENDATIONS

Our client needs to reduce costs by 5 percent. The largest area of opportunity appears to be in purchased
materials, the majority of which comprise subassemblies manufactured by seven subcontractors. By looking at
its purchases in total, the client can target approximately 40 percent of costs.

To achieve the 5 percent cost reduction, it would need to reduce costs by 15 to 20 percent. It could try to do
that by increasing commonality in the design of the subassemblies and components and by shifting volume to
a smaller number of suppliers.

Considering that the majority of the raw materials and components are purchased commodities, do you think
the 15-20 percent cost reduction is achievable?

Well, I know that typically have lower margins than more customized products. I suspect it may be challenging to hit the client's
savings target by focusing only on these purchases. But since raw materials and components represent about 40 percent of costs and
there is an opportunity to concentrate purchasing, I think we should start here.

Where else could you look for savings?

If I look back at the cost data on the jet, direct labor is another large cost component. As a contingency, we could look into that
area as well. I've read that other companies use outsourcing to lower their manufacturing costs-perhaps our client could do the same.
For example, it might want to increase its use of purchased subassemblies and reduce the amount of direct manufacturing it does.
Of course this would work only if it could drive direct labor costs below the offsetting cost of these subassemblies. The client will be
working closely with the subassembly suppliers to implement its purchasing initiative. This may give it an opportunity to explore
the suppliers' capabilities at the same time.

That's an interesting suggestion. How would you recommend the company pursue both of the initiatives you
have discussed?

I would look first to combine purchases across the subassembly suppliers with our client's purchases. I suspect that the client and
the subassembly suppliers will need to share a great deal of information, including engineering drawings and specifications, with
potential suppliers of the raw materials and components.
The Internet could prove to be a very effective medium for forming a single "virtual" purchasing department to consolidate both the
flow of information and purchase orders across the companies. Our client might also want to use a bidding system for those
materials that are true commodities
.
Next, I would turn to the engineering departments and form cross-company teams to look for areas in which to increase
commonality of design. At the same time, those teams could explore opportunities to use more purchased subassemblies and
decrease the client's direct labor costs.

That sounds great, and is very similar to a project we did. I would caution you, however, to examine the
upfront costs involved in your recommendations, both for the redesign and for the implementation of the
purchasing system, before going ahead.

INSEAD Consulting Book – 2006

- 79 -
This Practice Case was sponsored by:

Discount retailer Case

STEP 1: ACTIVELY LISTEN TO THE CASE

Your client is the largest discount retailer in Canada, with 500 stores spread throughout the country. Let's call
it CanadaCo. For several years running, CanadaCo has surpassed the second-largest Canadian retailer (300
stores) in both relative market share and profitability. However, the largest discount retailer in the United
States, USCo, has just bought out CanadaCo's competition and is planning to convert all 300 stores to USCo
stores. The CEO of CanadaCo is quite perturbed by this turn of events, and asks you the following questions:
Should I be worried? How should I react? How would you advise the CEO?

STEP 2: ESTABLISH AN UNDERSTANDING OF THE CASE

So, the client, CanadaCo, is facing competition in Canada from a U.S. competitor. Our task is to evaluate the extent of the
threat and advise the client on a strategy. Before I can advise the CEO I need some more information about the situation. First of
all, I'm not sure I understand what a discount retailer is!

A discount retailer sells a large variety of consumer goods at discounted prices, generally carrying everything
from housewares and appliances to clothing. Kmart, Woolworth, and Wal-Mart are prime examples in the
U.S.

STEP 3: SET UP THE FRAMEWORK

Oh, I see. Then I think it makes sense to structure the problem this way: First, let's understand the competition in the Canadian
market and how CanadaCo has become the market leader. Then let's look at the U.S. to understand how USCo has achieved its
position. At the end, we can merge the two discussions to understand whether USCo's strength in the U.S. is transferable to the
Canadian market.

That sounds fine. Let's start, then, with the Canadian discount retail market. What would you like to know?

STEP 4: EVALUATE THE CASE USING THE FRAMEWORK

Are CanadaCo's 500 stores close to the competition's 300 stores, or do they serve different geographic areas?

The stores are located in similar geographic regions. In fact, you might even see a CanadaCo store on one
corner, and the competition on the very next corner.

Do CanadaCo and the competition sell a similar product mix?

Yes. CanadaCo's stores tend to have a wider variety of brand names, but by and large, the product mix is
similar.

Are CanadaCo's prices significantly lower than the competition's?

No. For certain items CanadaCo is less expensive, and for others the competition is less expensive, but the
average price level is similar.

Is CanadaCo more profitable just because it has more stores, or does it have higher profits per store?

It actually has higher profits than the competition on a per-store basis.

Well, higher profits could be the result of lower costs or higher revenues. Are the higher per-store profits due to lower costs than the
competition's or the result of higher per-store sales?

INSEAD Consulting Book – 2006

- 80 -
This Practice Case was sponsored by:

CanadaCo's cost structure isn't any lower than the competition's. Its higher per-store profits are due to higher
per-store sales.

Is that because it has bigger stores?

No. CanadaCo's average store size is approximately the same as that of the competition.

If they're selling similar products at similar prices in similarly-sized stores in similar locations, why are CanadaCo's per-store sales
higher than the competition's?

It's your job to figure that out!

Is CanadaCo better managed than the competition?

I don't know that CanadaCo as a company is necessarily better managed, but I can tell you that its
management model for individual stores is significantly different.

How so?

The competitor's stores are centrally owned by the company, while CanadaCo uses a franchise model in which
each individual store is owned and managed by a franchisee who has invested in the store and retains part of
the profit.

In that case, I would guess that the CanadaCo stores are probably better managed, since the individual storeowners have a greater
incentive to maximize profit.

You are exactly right. It turns out that CanadaCo's higher sales are due primarily to a significantly higher level
of customer service. The stores are cleaner, more attractive, better stocked, and so on. The company
discovered this through a series of customer surveys last year. I think you've sufficiently covered the Canadian
market-let's move now to a discussion of the U.S. market.

How many stores does USCo own in the U.S? How many does the 2nd largest discount retailer own?

USCo owns 4,000 stores and the second-largest competitor owns approximately 1,000 stores.

Are USCo stores bigger than those of the typical discount retailer in the U.S.?

Yes. USCo stores average 200,000 square feet, whereas the typical discount retail store is approximately
100,000 square feet.

Those numbers suggest that USCo should be selling roughly eight times the volume of the nearest U.S. competitor!

Close. USCo's sales are approximately $5 billion, whereas the nearest competitor sells about $1 billion worth
of merchandise.

I would think that sales of that size give USCo significant clout with suppliers. Does it have a lower cost of goods than the
competition?

In fact, its cost of goods is approximately 15 percent less than that of the competition.

So it probably has lower prices.

Right again. Its prices are on average about ten percent lower than those of the competition.

INSEAD Consulting Book – 2006

- 81 -
This Practice Case was sponsored by:

So it seems that USCo has been so successful primarily because it has lower prices than its competitors.

That's partly right. Its success probably also has something to do with a larger selection of products, given the
larger average store size.

How did USCo get so much bigger than the competition?

It started by building superstores in rural markets served mainly by mom-and-pop stores and small discount
retailers. USCo bet that people would be willing to buy from it, and it was right. As it grew and developed
more clout with suppliers, it began to buy out other discount retailers and convert their stores to the USCo
format.

So whenever USCo buys out a competing store, it also physically expands it?

Not necessarily. Sometimes it does, but when I said it converts it to the USCo format, I meant that it carries
the same brands at prices that are on average ten percent lower than the competition's.

What criteria does USCo use in deciding whether it should physically expand a store it's just bought out?

It depends on a lot of factors, such as the size of the existing store, local market competition, local real estate
costs, and so on, but I don't think we need to go into that here.

Well, I thought it might be relevant in terms of predicting what it will do with the 300 stores that it bought in Canada.

Let's just assume that it doesn't plan to expand the Canadian stores beyond their current size.

OK. I think I've learned enough about USCo. I'd like to ask a few questions about USCo's ability to succeed in the Canadian
market. Does USCo have a strong brand name in Canada?

No. Although members of the Canadian business community are certainly familiar with the company because
of its U.S. success, the Canadian consumer is basically unaware of USCo's existence.

Does CanadaCo carry products similar to USCo's, or does the Canadian consumer expect different products and brands than the
U.S. discount retail consumer?

The two companies carry similar products, although the CanadaCo stores lean more heavily toward Canadian
suppliers.

How much volume does CanadaCo actually sell?

About $750 million worth of goods annually.

Is there any reason to think that the costs of doing business for USCo will be higher in the Canadian market?

Can you be more specific?

I mean, for example, are labor or leasing costs higher in Canada than in the U.S.?

Canada does have significantly higher labor costs, and I'm not sure about the costs of leasing space. What are
you driving at?

I was thinking that if there were a higher cost of doing business in Canada, perhaps USCo would have to charge higher prices
than it does in the U.S. to cover its costs.

INSEAD Consulting Book – 2006

- 82 -
This Practice Case was sponsored by:

That's probably true, but remember, CanadaCo must also cope with the same high labor costs. Can you think
of additional costs incurred by USCo's Canadian operations that would not be incurred by CanadaCo?

USCo might incur higher distribution costs than CanadaCo because it will have to ship product from its U.S. warehouses up to
Canada.

You are partially right. CanadaCo has the advantage in distribution costs, since its network spans less
geographic area and it gets more products from Canadian suppliers. However, since CanadaCo continues to
get a good deal of product from the U.S., the actual advantage to CanadaCo is not great-only about two
percent of overall costs.

All this suggests that USCo will be able to retain a significant price advantage over CanadaCo's stores: if not ten percent, then at
least seven to eight percent.

I would agree with that conclusion.

STEP 5: SUMMARIZE AND MAKE RECOMMENDATIONS

I would tell the CEO the following: In the near term, you might be safe. Your stores have a much stronger brand name in
Canada than USCo's, and they seem to be well managed. However, as consumers get used to seeing prices that are consistently
seven to eight percent less at USCo, they will realize that shopping at USCo means significant savings over the course of the year.
Although some consumers will remain loyal out of habit or because of your high level of service, it is reasonable to expect the
discount shopper to shop where prices are lowest. Moreover, over time your brand-name advantage will erode as USCo becomes
more familiar to Canadian consumers. You certainly have to worry about losing significant share to USCo stores in the long term.
You should probably do something about it now, before it's too late.

Can you suggest possible strategies for CanadaCo?

Maybe it can find ways to cut costs and make the organization more efficient, so it can keep prices low even if its cost of goods is
higher.

Anything else?

It might consider instituting something like a frequent shopper program, where consumers accumulate points that entitle them to
future discounts on merchandise.

What might be a potential problem with that?

Well, it might not be that cost-effective, since it would be rewarding a significant number of shoppers who would have continued to
shop there anyway.

Any other suggestions?

CanadaCo might want to prepare a marketing or advertising campaign that highlights its high level of service. It might even
institute a CanadaCo Service Guarantee that surpasses any guarantees offered by USCo.

Assuming the only way to keep customers is through competitive pricing, is there anything CanadaCo can do
to appear competitive to the consumer?

It might want to consider offering fewer product lines, so that it can consolidate its buying power and negotiate prices with suppliers
that are competitive with USCo's. It might lose some customers who want the variety of products that USCo has, but it may be
able to retain the customer who is buying a limited array of items and is just looking for the best price.

All of your suggestions are interesting, and you would want to analyze the advantages and disadvantages of
each in more detail before making any recommendations to the CEO.

INSEAD Consulting Book – 2006

- 83 -
This Practice Case was sponsored by:

Medical Software industry Case


STEP 1: ACTIVELY LISTEN TO THE CASE

Your client is GenCo, a large, international, diversified company with a health care division that produces a
wide variety of medical instruments and related services. Five years ago, it expanded into the health care
software industry by purchasing MedCount, which markets administrative systems to large U.S. hospitals.
These systems are designed primarily for back-office functions; they are not designed for managing patients or
providing other physician and technical support. Since it was purchased, the software division has failed to
deliver the growth needed to justify the multiple GenCo paid for it. GenCo feels it has already squeezed
margins as much as possible, and now is looking for new sales opportunities. MedCount turned to BCG to
help identify potential ways to increase revenues. How would you approach this problem?

STEP 2: ESTABLISH YOUR UNDERSTANDING OF THE CASE

First, let me make sure I understand the problem. The parent company produces medical devices and services, but before the
acquisition was not involved in health care software. The company it purchased, MedCount, sells only administrative systems
software to large hospitals. It is now looking for opportunities to increase revenues.

That is correct.

Could I take a moment to jot down a few thoughts?

Sure, that would be fine.

STEP 3: SET UP THE FRAMEWORK

I would suggest using the following framework: First, I'd want to understand the market size and growth rates for MedCount's
market and related software markets. Next, I would like to explore the competition and their market shares. Third, I would like
to examine customer requirements and then, given those external conditions, look at the division's capabilities to understand how
well prepared it is to meet the needs of the marketplace.

That sounds fine. So what do you want to know about the market?

STEP 4: EVALUATE THE CASE USING THE FRAMEWORK

Well, the first hurdle would be to identify the markets the company would be interested in. Besides administration systems, what
other types of medical software systems do large hospitals purchase?

There are many software systems, but for the sake of time, the team focused on three primary markets:
administration systems, patient administration, and physician support systems.

What do those systems do?

Patient administration includes systems like admissions and tracking. Physician support systems are more
specialized, for individual physician procedures.

I would like to know how large each market is and how fast each is growing. I would use secondary sources such as press releases,
analyst reports, and published market studies, to obtain this information.

Great! That is what we did during the market study. Our information revealed the following market sizes and
growth rates.

INSEAD Consulting Book – 2006

- 84 -
This Practice Case was sponsored by:

Administration Patient administration Physician support


Market size ($M) 1,500 1,000 1,200
Growth rate 5% 5% 12%

From a size and growth perspective, physician support systems looks like a very attractive market. I'd like to know a little about
the customers themselves. The client is currently targeting large hospitals. Approximately what percentage of the market do they
represent?

We were unable to get an exact breakdown, but we know that these hospitals make up the vast majority of the
total medical software market.

That would make sense, since the more sophisticated procedures at a hospital might necessitate more advanced software solutions. I
know that there have been a lot of changes in the industry as a result of managed care. I don't know much about the industry, so I
would want to look at market studies and press clippings to get a better sense of the hospital market in general and any technology
or software trends more specifically.

Okay. Let's say that you did that and were presented with this summary of market trends:

• Consolidation in the industry, with three to four large hospital networks dominating 45 percent of the
market
• Cost controls instituted, particularly as these large hospital networks acquire smaller hospitals
(centralization of functions being a key cost issue)
• Many hospitals seeking to consolidate their vendor base
• With regard to technology, many hospitals upgrading their older systems

If hospitals are consolidating vendors, perhaps our client has an advantage in being part of a larger medical company. Maybe the
client could also gain some advantages by expanding into other software segments. Are the people responsible for purchasing
software at the hospital the same for all three segments?

Like all things, it differs by hospital, but the larger hospital networks, have tried to consolidate their
purchasing not only within but also across hospitals.

Is the decision maker for medical software the same as for medical instrumentation and devices?

In some cases, the head of purchasing influences both decisions, but the person who makes the final choice is
different. Software decisions are usually made by the hospital IT function, and those for instrumentation by
the medical staff.

I think I have a pretty good understanding of the market for now. Let's look at competition next. We could identify all the
competitors and build up the market shares using a combination of public data and estimates.

Well, let's assume that you don't have an infinite amount of time to look at all the competitors. You can only
look at the top five competitors in each market. You are given the following data:

Administration Systems Sales ($M) Growth (%)


MedCount 700 4%
HCS Software Systems 100 7%
Morningside Software 80 3%
Admin Systems Solutions 70 2%
HTI Software 50 15%

INSEAD Consulting Book – 2006

- 85 -
This Practice Case was sponsored by:

Patient Administration Sales ($M) Growth (%)


HTI 300 5%
Registration Software Solutions 240 4%
Signup Software 60 3%
HCS Software Systems 30 16%
Patient Software 20 -1%

Physician Support Sales ($M) Growth (%)


HCS Software Systems 150 16%
Physician Support Systems 100 11%
Medical Technology Inc 25 18%
HTI 20 32%
MedSys 5 15%

Very interesting. The first thing I would note from the data is that the market concentrations are very different. In administrative
systems, the top five competitors control 66 percent of the market and in patient administration, they control 65 percent. But in the
physician support market, they control only 25 percent.

I would want to know what gross margins look like in each of these markets as well. I might turn to analyst reports and look at
competitors' financial statements to deduce whether they are making money in each market

Gross margins vary, of course, but the analyst reports have margins of 25 to 30 percent for administrative
systems and for patient administration. For physician support, the margins tend to be higher, more like 45 to
50 percent.

I see that two competitors, HTI and HCS Software Systems, have very large revenue growth in all three sectors, although they
each dominate one. I would want to look at their financials, annual reports, and press releases to find out a bit more about their
strategy in each of these areas.

You'd find that they recently entered these noncore markets. Why might they have done that?

Perhaps, like our client, each had a strong position in its own segment, HTI in patient administration and HCS Software
Systems in physician support. Maybe they too decided to branch out into the other segments to find additional growth.

That is a very good hypothesis. Let's say there is evidence in the sources you consult that supports your
assertion.

Well, if that were true, these two companies could be a threat not only in the other two segments, but also in our client's segment,
administrative systems. It looks as if the client is slowly losing market share in its segment, since it is growing more slowly than its
market.

Good observation.

The market and competitor trends could also suggest that the client may want to enter these other markets. In particular, the
physician support market looks attractive, given it has high growth and lack of a dominant competitor. The higher gross margins
may provide attractive returns on the necessary investment in software development. However, the patient administration market
may also be attractive. Although it is more concentrated and offers lower margins than physician support, the client may be able to
enter this segment with a smaller up-front investment. Given the trend toward upgrading existing computer systems, it may be
INSEAD Consulting Book – 2006

- 86 -
This Practice Case was sponsored by:

important for MedCount to have a product offering in each of the three market segments. That should not be too difficult, since the
company is already in the software industry.

Perhaps, but you should think a little more closely about these types of software. Are all software systems
alike?

Well, let me think about that for a moment. I suspect patient administration would have relatively low entry barriers. From your
earlier description, these systems appear to be pretty basic, dealing primarily with admissions and patient tracking However, the
entry barriers in physician support might be higher, since these systems are more complex and there are probably multiple systems
for the various physician procedures. I guess it would be harder to get into those types of systems.

That would make sense.

Since the company might want to go into only some of the segments, I would want to know how important it is to have products in
all three segments. Do we know if the competitors are marketing their products as a bundle?

How might you find that out?

Since it would be difficult to talk to a competitor directly, I would probably target a competitor's customer, particularly one that
just converted from our client's software.

Let's say you get an interview with a customer that recently switched to HTI. You discover that the
competitor was offering it a better pricing deal and service for software products in all three segments.

How were MedCount's software and service perceived in relation to those of competitors?

The customer thought that its administrative systems were adequate, "the old standby," but not stellar.

Were there any other key reasons it switched from MedCount's system?

When it decided to upgrade its systems, it tried to contact MedCount, but could never get a representative to
describe its options.

Interesting. How did HTI perform?

The HTI representative had heard that the company was considering switching software vendors and
provided a sales representative to pitch HTI's administrative product the next day.

It definitely sounds as if there was a problem with the sales function and that customer relations need to be improved, particularly
for the larger hospital chains. There also seems to be an advantage from both a marketing and sales perspective in having multiple
software products. I would want to confirm those views by doing further interviews.

Let's say further interviews support those assumptions.

Since we have already looked at the external conditions, I would like to move on to the client itself. I'd like to know more about
its marketing and selling organization as well as its software development skills.

So far, we know that our client offers administrative software and that there may be a problem with sales and marketing. Could
you tell me a little about the marketing department?

The marketing department is organized regionally. Teams are assigned to hospitals within each state or
geographic region, such as New England.

INSEAD Consulting Book – 2006

- 87 -
This Practice Case was sponsored by:

That could explain some of the problems with MedCount's marketing and sales. If hospital purchasing is centralized, the
marketing organization may be outdated. Does the company have any teams dedicated to the four or five biggest hospital
networks?

No, there are no dedicated teams. They talked about doing that for a while, but it conflicted with the regional
structure it had in place.

With regard to software, does the company feel it has any strengths or weaknesses?

It feels that their administrative product is very strong ("best of breed") and is the dominant technology. Also,
the product is modular in design, which allows for easier upgrades. Although the company has never branched
out into other market segments, the software developers believe that certain modules could be used to build
the foundation for other administrative software programs. The company feels customer support is also an
area in which it excels.

STEP 5: SUMMARIZE AND MAKE RECOMMENDATIONS

Let's start with our client's market. The client dominates the administrative software market, which is fairly large but growing
slowly, and the company appears to be slowly losing market share. Patient administration is also growing relatively slowly. Both
markets are relatively concentrated and appear to offer lower margins than physician support. The physician support market is
large and less concentrated, and could potentially provide higher margins, but would require a larger investment. The hospital
market itself is becoming more concentrated and is pushing to consolidate vendors. The purchasing agent is often the same for the
three types of software.

Looking at our client's competitors, two, HTI and HCS Software Systems, appear to be particularly threatening. Each has a
dominant position in one segment and is branching out into other areas. They appear to be marketing their products and services
as a bundle and are using service as a key point of differentiation.

The client offers only one type of system and appears to have some weaknesses in its marketing organization, particularly in
marketing to the larger hospital networks, which offer the most promising market opportunities.

How would you recommend proceeding?

The first priority should be to fix the marketing organization, particularly for the large hospital networks. MedCount will have
trouble expanding into new markets if it can't defend its current position and shore up its existing customer relationships. There
should be a team dedicated to each of the major chains. The client should also look at improving customer tracking so that it is
clear when its customers are going to upgrade. There should also be clear contacts so that the customer can easily keep in touch with
MedCount.

Next, I would recommend that the client explore entering the other market segments by leveraging its dominant position in
administrative systems. At first glance, patient administration does not appear to be very attractive, with slow growth, low margins,
and large, dominant competitors. There appears to be some advantage, however, in having products across the product range. I
would recommend that we interview some of MedCount's existing customers to better understand their needs and future IT
requirements. If the customer base is interested in one software provider for both back-office administration and patient
administration functions, this segment looks promising.

If the client does decide to enter this market, it should look at the lowest-cost method of entry, either developing a product internally
or acquiring a competitor. The modular design of its existing administrative software suggests internal development of the patient
administration product may be the way to go, but we would need a more thorough comparison of the internal development and
acquisition options, including both cost and time to market. I think that physician support offers our client an exciting growth
opportunity, given its high margins, high growth, and fragmented competition. I would definitely think about an acquisition
strategy, since the client may lack the technical capabilities to enter this specialized market. I would recommend going for one of the
larger companies, as that would give the client a stronger position. Smaller companies would probably not offer an important
enough position in the market. More research would be needed, however, for us to better understand the intricacies of the market
and each potential acquisition.
INSEAD Consulting Book – 2006

- 88 -
This Practice Case was sponsored by:

Those are very interesting conclusions. Thank you.

INSEAD Consulting Book – 2006

- 89 -
9. Case examples
9.1 Discount retailer case (BCG)

Step 1: Actively listen to the case

Your client is the largest discount retailer in Canada, with 500 stores spread throughout the
country. Let's call it CanadaCo. For several years running, CanadaCo has surpassed the second-
largest Canadian retailer (300 stores) in both relative market share and profitability. However,
the largest discount retailer in the United States, USCo, has just bought out CanadaCo's
competition and is planning to convert all 300 stores to USCo stores. The CEO of CanadaCo
is quite perturbed by this turn of events, and asks you the following questions: Should I be
worried? How should I react? How would you advise the CEO?

Step 2: Establish understanding of the case

So, the client, CanadaCo, is facing competition in Canada from a U.S. competitor. Our task is to evaluate the
extent of the threat and advise the client on a strategy. Before I can advise the CEO I need some more
information about the situation. First of all, I'm not sure I understand what a discount retailer is!

A discount retailer sells a large variety of consumer goods at discounted prices, generally
carrying everything from housewares and appliances to clothing. Kmart, Woolworth, and Wal-
Mart are prime examples in the U.S.

Step 3: Set up the framework

Oh, I see. Then I think it makes sense to structure the problem this way: First, let's understand the competition
in the Canadian market and how CanadaCo has become the market leader. Then let's look at the U.S. to
understand how USCo has achieved its position. At the end, we can merge the two discussions to understand
whether USCo's strength in the U.S. is transferable to the Canadian market.

That sounds fine. Let's start, then, with the Canadian discount retail market. What would you
like to know?

Step 4: Evaluate the case using the framework

Are CanadaCo's 500 stores close to the competition's 300 stores, or do they serve different geographic areas?

The stores are located in similar geographic regions. In fact, you might even see a CanadaCo
store on one corner, and the competition on the very next corner.

Do CanadaCo and the competition sell a similar product mix?

Yes. CanadaCo's stores tend to have a wider variety of brand names, but by and large, the
product mix is similar.

36
Are CanadaCo's prices significantly lower than the competition's?

No. For certain items CanadaCo is less expensive, and for others the competition is less
expensive, but the average price level is similar.

Is CanadaCo more profitable just because it has more stores, or does it have higher profits per store?

It actually has higher profits than the competition on a per-store basis.

Well, higher profits could be the result of lower costs or higher revenues. Are the higher per-store profits due to
lower costs than the competition's or the result of higher per-store sales?

CanadaCo's cost structure isn't any lower than the competition's. Its higher per-store profits
are due to higher per-store sales.

Is that because it has bigger stores?

No. CanadaCo's average store size is approximately the same as that of the competition.

If they're selling similar products at similar prices in similarly-sized stores in similar locations, why are
CanadaCo's per-store sales higher than the competition's?

It's your job to figure that out!

Is CanadaCo better managed than the competition?

I don't know that CanadaCo as a company is necessarily better managed, but I can tell you that
its management model for individual stores is significantly different.

How so?

The competitor's stores are centrally owned by the company, while CanadaCo uses a franchise
model in which each individual store is owned and managed by a franchisee that has invested
in the store and retains part of the profit.

In that case, I would guess that the CanadaCo stores are probably better managed, since the individual
storeowners have a greater incentive to maximize profit.

You are exactly right. It turns out that CanadaCo's higher sales are due primarily to a
significantly higher level of customer service. The stores are cleaner, more attractive, better
stocked, and so on. The company discovered this through a series of customer surveys last
year. I think you've sufficiently covered the Canadian market-let's move now to a discussion of
the U.S. market.

How many stores does USCo own in the U.S., and how many does the second-largest discount retailer own?

USCo owns 4,000 stores and the second-largest competitor owns approximately 1,000 stores.

37
Are USCo stores bigger than those of the typical discount retailer in the U.S.?

Yes. USCo stores average 200,000 square feet, whereas the typical discount retail store is
approximately 100,000 square feet.

Those numbers suggest that USCo should be selling roughly eight times the volume of the nearest U.S.
competitor!

Close. USCo's sales are approximately $5 billion, whereas the nearest competitor sells about
$1 billion worth of merchandise.

I would think that sales of that size give USCo significant clout with suppliers. Does it have a lower cost of
goods than the competition?

In fact, its cost of goods is approximately 15 percent less than that of the competition.

So it probably has lower prices.

Right again. Its prices are on average about ten percent lower than those of the competition.

So it seems that USCo has been so successful primarily because it has lower prices than its competitors.

That's partly right. Its success probably also has something to do with a larger selection of
products, given the larger average store size.

How did USCo get so much bigger than the competition?

It started by building superstores in rural markets served mainly by mom-and-pop stores and
small discount retailers. USCo bet that people would be willing to buy from it, and it was right.
As it grew and developed more clout with suppliers, it began to buy out other discount
retailers and convert their stores to the USCo format.

So whenever USCo buys out a competing store, it also physically expands it?

Not necessarily. Sometimes it does, but when I said it converts it to the USCo format, I meant
that it carries the same brands at prices that are on average ten percent lower than the
competition's.

What criteria does USCo use in deciding whether it should physically expand a store it's just bought out?

It depends on a lot of factors, such as the size of the existing store, local market competition,
local real estate costs, and so on, but I don't think we need to go into that here.

Well, I thought it might be relevant in terms of predicting what it will do with the 300 stores that it bought in
Canada.

Let's just assume that it doesn't plan to expand the Canadian stores beyond their current size.

38
OK. I think I've learned enough about USCo. I'd like to ask a few questions about USCo's ability to succeed
in the Canadian market. Does USCo have a strong brand name in Canada?

No. Although members of the Canadian business community are certainly familiar with the
company because of its U.S. success, the Canadian consumer is basically unaware of USCo's
existence.

Does CanadaCo carry products similar to USCo's, or does the Canadian consumer expect different products
and brands than the U.S. discount retail consumer?

The two companies carry similar products, although the CanadaCo stores lean more heavily
toward Canadian suppliers.

How much volume does CanadaCo actually sell?

About $750 million worth of goods annually.

Is there any reason to think that the costs of doing business for USCo will be higher in the Canadian market?

Can you be more specific?

I mean, for example, are labor or leasing costs higher in Canada than in the U.S.?

Canada does have significantly higher labor costs, and I'm not sure about the costs of leasing
space. What are you driving at?

I was thinking that if there were a higher cost of doing business in Canada, perhaps USCo would have to
charge higher prices than it does in the U.S. to cover its costs.

That's probably true, but remember, CanadaCo must also cope with the same high labor costs.
Can you think of additional costs incurred by USCo's Canadian operations that would not be
incurred by CanadaCo?

USCo might incur higher distribution costs than CanadaCo because it will have to ship product from its U.S.
warehouses up to Canada.

You are partially right. CanadaCo has the advantage in distribution costs, since its network
spans less geographic area and it gets more products from Canadian suppliers. However, since
CanadaCo continues to get a good deal of products from the U.S., the actual advantage to
CanadaCo is not great-only about two percent of overall costs.

All this suggests that USCo will be able to retain a significant price advantage over CanadaCo's stores: if not
ten percent, then at least seven to eight percent.

I would agree with that conclusion.

39
Step 5: Summarize and make recommendations

I would tell the CEO the following: In the near term, you might be safe. Your stores have a much stronger
brand name in Canada than USCo's, and they seem to be well managed. However, as consumers get used to
seeing prices that are consistently seven to eight percent less at USCo, they will realize that shopping at USCo
means significant savings over the course of the year. Although some consumers will remain loyal out of habit or
because of your high level of service, it is reasonable to expect the discount shopper to shop where prices are
lowest. Moreover, over time your brand-name advantage will erode as USCo becomes more familiar to
Canadian consumers. You certainly have to worry about losing significant share to USCo stores in the long
term. You should probably do something about it now, before it's too late.

Can you suggest possible strategies for CanadaCo?

Maybe it can find ways to cut costs and make the organization more efficient, so it can keep prices low even if its
cost of goods is higher.

Anything else?

It might consider instituting something like a frequent shopper program, where consumers accumulate points that
entitle them to future discounts on merchandise.

What might be a potential problem with that?

Well, it might not be that cost-effective, since it would be rewarding a significant number of shoppers who would
have continued to shop there anyway.

Any other suggestions?

CanadaCo might want to prepare a marketing or advertising campaign that highlights its high level of service. It
might even institute a CanadaCo Service Guarantee that surpasses any guarantees offered by USCo.

Assuming the only way to keep customers is through competitive pricing, is there anything
CanadaCo can do to appear competitive to the consumer?

It might want to consider offering fewer product lines, so that it can consolidate its buying power and negotiate
prices with suppliers that are competitive with USCo's. It might lose some customers who want the variety of
products that USCo has, but it may be able to retain the customer who is buying a limited array of items and is
just looking for the best price.

All of your suggestions are interesting, and you would want to analyze the advantages and
disadvantages of each in more detail before making any recommendations to the CEO.

Additional Case Questions

By the time you've gone through the case interview example and the interactive case you
probably will have developed a good idea of what a case interview is all about. The best way to
prepare for a case interview is to practice a few. Ask a friend or career counselor to give you a
case using the sample business problems below.

40
1. A German luxury car manufacturer is interested in entering the sport-utility vehicle market
(for example, Jeep Cherokee) after noticing that the market has grown dramatically worldwide
in the past two years. How would you advise the manufacturer? What does it need to know
before making an entry decision? If it chooses to enter, what might a viable strategy be?

2. A North American manufacturer/retailer of high-end glassware experienced a dramatic


decline in same-store sales at its retail outlets last year. How would you begin to assess the
reasons for the decline? Using your analysis as a basis, what strategy would you recommend for
the manufacturer?

3. A large public utility formerly had a monopoly in the British electricity market. Now that the
market has been deregulated, small power-generation companies have already captured a five
percent share from the utility by offering to provide large businesses in the U.K. with their
own in-house power-generation capabilities. The CEO of the utility wants to understand
whether this trend will continue and how she can prevent further loss of share. How would
you answer her question?

4. A U.S.-based pharmaceutical company that focuses on discovering, developing, and selling


drugs for the treatment of cancer has been experiencing flat growth and is interested in
expanding into new businesses. In view of the growth and profitability of stand-alone cancer
treatment centers in the U.S., the company is considering establishing and operating similar
centers in China. This would be the company's first foray into the cancer treatment center
business. How would you evaluate the attractiveness of the opportunity?

5. The Swiss Ski Association has been petitioned by an international snowboarding club to
permit snowboarding on the ski slopes within its jurisdiction. (Assume that the association
currently forbids snowboarding on all Swiss ski slopes.) If the association is interested in
maximizing profits, how should it respond to the petition? What factors would the answer
depend upon?

9.2 Medical software industry case (BCG)

Step 1: Actively listen to the case

Your client is GenCo, a large, international, diversified company with a health care division
that produces a wide variety of medical instruments and related services. Five years ago, it
expanded into the health care software industry by purchasing MedCount, which markets
administrative systems to large U.S. hospitals. These systems are designed primarily for back-
office functions; they are not designed for managing patients or providing other physician and
technical support. Since it was purchased, the software division has failed to deliver the growth
needed to justify the multiple GenCo paid for it. GenCo feels it has already squeezed margins
as much as possible, and now is looking for new sales opportunities. MedCount turned to
BCG to help identify potential ways to increase revenues. How would you approach this
problem?

41
Step 2: Establish your understanding of the case

First, let me make sure I understand the problem. The parent company produces medical devices and services,
but before the acquisition was not involved in health care software. The company it purchased, MedCount, sells
only administrative systems software to large hospitals. It is now looking for opportunities to increase revenues.

That is correct.

Could I take a moment to jot down a few thoughts?

Sure, that would be fine.

Step 3: Set up the framework

I would suggest using the following framework: First, I'd want to understand the market size and growth rates
for MedCount's market and related software markets. Next, I would like to explore the competition and their
market shares. Third, I would like to examine customer requirements and then, given those external conditions,
look at the division's capabilities to understand how well prepared it is to meet the needs of the marketplace.

That sounds fine. So what do you want to know about the market?

Step 4: Evaluate the case using the framework

Well, the first hurdle would be to identify the markets the company would be interested in. Besides
administration systems, what other types of medical software systems do large hospitals purchase?

There are many software systems, but for the sake of time, the team focused on three primary
markets: administration systems, patient administration, and physician support systems.

What do those systems do?

Patient administration includes systems like admissions and tracking. Physician support
systems are more specialized, for individual physician procedures.

I would like to know how large each market is and how fast each is growing. I would use secondary sources such
as press releases, analyst reports, and published market studies, to obtain this information.

Great! That is what we did during the market study. Our information revealed the following
market sizes and growth rates.

Administration Patient administration Physician support


Market size ($M) 1,500 1,000 1,200
Growth rate 5% 5% 12%

42
From a size and growth perspective, physician support systems look like a very attractive market. I'd like to
know a little about the customers themselves. The client is currently targeting large hospitals. Approximately
what percentage of the market do they represent?

We were unable to get an exact breakdown, but we know that these hospitals make up the vast
majority of the total medical software market.

That would make sense, since the more sophisticated procedures at a hospital might necessitate more advanced
software solutions. I know that there have been a lot of changes in the industry as a result of managed care. I
don't know much about the industry, so I would want to look at market studies and press clippings to get a
better sense of the hospital market in general and any technology or software trends more specifically.

Okay. Let's say that you did that and were presented with this summary of market trends:
• Consolidation in the industry, with three to four large hospital networks dominating 45
percent of the market
• Cost controls instituted, particularly as these large hospital networks acquire smaller
hospitals (centralization of functions being a key cost issue)
• Many hospitals seeking to consolidate their vendor base
• With regard to technology, many hospitals upgrading their older systems

If hospitals are consolidating vendors, perhaps our client has an advantage in being part of a larger medical
company. Maybe the client could also gain some advantages by expanding into other software segments. Are the
people responsible for purchasing software at the hospital the same for all three segments?

Like all things, it differs by hospital, but the larger hospital networks have tried to consolidate
their purchasing not only within but also across hospitals.

Is the decision maker for medical software the same as for medical instrumentation and devices?

In some cases, the head of purchasing influences both decisions, but the person who makes
the final choice is different. Software decisions are usually made by the hospital IT function,
and those for instrumentation by the medical staff.

I think I have a pretty good understanding of the market for now. Let's look at competition next. We could
identify all the competitors and build up the market shares using a combination of public data and estimates.

Well, let's assume that you don't have an infinite amount of time to look at all the competitors.
You can only look at the top five competitors in each market. You are given the following
data:

Administration Systems Sales ($M) Growth (%)

MedCount 700 4%
HCS Software Systems 100 7%
Morningside Software 80 3%
Admin Systems Solutions 70 2%
HTI Software 50 15%

43
Patient Administration Sales ($M) Growth (%)

HTI 300 5%
Registration Software Solutions 240 4%
Signup Software 60 3%
HCS Software Systems 30 16%
Patient Software 20 -1%

Physician Support Sales ($M) Growth (%)

HCS Software Systems 150 16%


Physician Support Systems 100 11%
Medical Technology Inc 25 18%
HTI 20 32%
MedSys 5 15%

Very interesting. The first thing I would note from the data is that the market concentrations are very different.
In administrative systems, the top five competitors control 66 percent of the market and in patient
administration, they control 65 percent. But in the physician support market, they control only 25 percent.

I would want to know what gross margins look like in each of these markets as well. I might turn to analyst
reports and look at competitors' financial statements to deduce whether they are making money in each market.

Gross margins vary, of course, but the analyst reports have margins of 25 to 30 percent for
administrative systems and for patient administration. For physician support, the margins tend
to be higher, more like 45 to 50 percent.

I see that two competitors, HTI and HCS Software Systems, have very large revenue growth in all three sectors,
although they each dominate one. I would want to look at their financials, annual reports, and press releases to
find out a bit more about their strategy in each of these areas.

You'd find that they recently entered these non-core markets. Why might they have done that?

Perhaps, like our client, each had a strong position in its own segment, HTI in patient administration and
HCS Software Systems in physician support. Maybe they too decided to branch out into the other segments to
find additional growth.

That is a very good hypothesis. Let's say there is evidence in the sources you consult that
supports your assertion.

Well, if that were true, these two companies could be a threat not only in the other two segments, but also in our
client's segment, administrative systems. It looks as if the client is slowly losing market share in its segment,
since it is growing more slowly than its market.

44
Good observation.

The market and competitor trends could also suggest that the client may want to enter these other markets. In
particular, the physician support market looks attractive; given it has high growth and lack of a dominant
competitor. The higher gross margins may provide attractive returns on the necessary investment in software
development. However, the patient administration market may also be attractive. Although it is more
concentrated and offers lower margins than physician support, the client may be able to enter this segment with a
smaller up-front investment. Given the trend toward upgrading existing computer systems, it may be important
for MedCount to have a product offering in each of the three market segments. That should not be too difficult,
since the company is already in the software industry.

Perhaps, but you should think a little more closely about these types of software. Are all
software systems alike?

Well, let me think about that for a moment. I suspect patient administration would have relatively low entry
barriers. From your earlier description, these systems appear to be pretty basic, dealing primarily with
admissions and patient tracking. However, the entry barriers in physician support might be higher, since these
systems are more complex and there are probably multiple systems for the various physician procedures. I guess it
would be harder to get into those types of systems.

That would make sense.

Since the company might want to go into only some of the segments, I would want to know how important it is
to have products in all three segments. Do we know if the competitors are marketing their products as a bundle?

How might you find that out?

Since it would be difficult to talk to a competitor directly, I would probably target a competitor's customer,
particularly one that just converted from our client's software.

Let's say you get an interview with a customer that recently switched to HTI. You discover
that the competitor was offering it a better pricing deal and service for software products in all
three segments.

How were MedCount's software and service perceived in relation to those of competitors?

The customer thought that its administrative systems were adequate, "the old standby," but not
stellar.

Were there any other key reasons it switched from MedCount's system?

When it decided to upgrade its systems, it tried to contact MedCount, but could never get a
representative to describe its options.

Interesting. How did HTI perform?

The HTI representative had heard that the company was considering switching software
vendors and provided a sales representative to pitch HTI's administrative product the next day.

45
It definitely sounds as if there was a problem with the sales function and that customer relations need to be
improved, particularly for the larger hospital chains. There also seems to be an advantage from both a marketing
and sales perspective in having multiple software products. I would want to confirm those views by doing further
interviews.

Let's say further interviews support those assumptions.

Since we have already looked at the external conditions, I would like to move on to the client itself. I'd like to
know more about its marketing and selling organization as well as its software development skills.

So far, we know that our client offers administrative software and that there may be a problem with sales and
marketing. Could you tell me a little about the marketing department?

The marketing department is organized regionally. Teams are assigned to hospitals within each
state or geographic region, such as New England.

That could explain some of the problems with MedCount's marketing and sales. If hospital purchasing is
centralized, the marketing organization may be outdated. Does the company have any teams dedicated to the
four or five biggest hospital networks?

No, there are no dedicated teams. They talked about doing that for a while, but it conflicted
with the regional structure it had in place.

With regard to software, does the company feel it has any strengths or weaknesses?

It feels that their administrative product is very strong ("best of breed") and is the dominant
technology. Also, the product is modular in design, which allows for easier upgrades. Although
the company has never branched out into other market segments, the software developers
believe that certain modules could be used to build the foundation for other administrative
software programs. The company feels customer support is also an area in which it excels.

Step 5: Summarize and make recommendations

Let's start with our client's market. The client dominates the administrative software market, which is fairly
large but growing slowly, and the company appears to be slowly losing market share. Patient administration is
also growing relatively slowly. Both markets are relatively concentrated and appear to offer lower margins than
physician support. The physician support market is large and less concentrated, and could potentially provide
higher margins, but would require a larger investment. The hospital market itself is becoming more concentrated
and is pushing to consolidate vendors. The purchasing agent is often the same for the three types of software.

Looking at our client's competitors, two, HTI and HCS Software Systems, appear to be particularly
threatening. Each has a dominant position in one segment and is branching out into other areas. They appear to
be marketing their products and services as a bundle and are using service as a key point of differentiation.

The client offers only one type of system and appears to have some weaknesses in its marketing organization,
particularly in marketing to the larger hospital networks, which offer the most promising market opportunities.

How would you recommend proceeding?

46
The first priority should be to fix the marketing organization, particularly for the large hospital networks.
MedCount will have trouble expanding into new markets if it can't defend its current position and shore up its
existing customer relationships. There should be a team dedicated to each of the major chains. The client should
also look at improving customer tracking so that it is clear when its customers are going to upgrade. There
should also be clear contacts so that the customer can easily keep in touch with MedCount.

Next, I would recommend that the client explore entering the other market segments by leveraging its dominant
position in administrative systems. At first glance, patient administration does not appear to be very attractive,
with slow growth, low margins, and large, dominant competitors. There appears to be some advantage, however,
in having products across the product range. I would recommend that we interview some of MedCount's existing
customers to better understand their needs and future IT requirements. If the customer base is interested in one
software provider for both back-office administration and patient administration functions, this segment looks
promising.

If the client does decide to enter this market, it should look at the lowest-cost method of entry, either developing a
product internally or acquiring a competitor. The modular design of its existing administrative software suggests
internal development of the patient administration product may be the way to go, but we would need a more
thorough comparison of the internal development and acquisition options, including both cost and time to
market. I think that physician support offers our client an exciting growth opportunity, given its high margins,
high growth, and fragmented competition. I would definitely think about an acquisition strategy, since the client
may lack the technical capabilities to enter this specialized market. I would recommend going for one of the
larger companies, as that would give the client a stronger position. Smaller companies would probably not offer
an important enough position in the market. More research would be needed, however, for us to better
understand the intricacies of the market and each potential acquisition.

Those are very interesting conclusions. Thank you.

9.3 Jet fighter manufacturing case (BCG)

Step 1: Actively listen to the case

Your client is a U.S. defense contractor that manufactures the Mohawk Light Fighter Jet for
the British Royal Air Force. The company has produced the $20 million fighter jet for the past
12 years. The British government has decided to put the contract out to bid, however, and to
win the program, the client's purchasing agents have estimated, the company will need to cut
its costs by 5 percent. It has asked BCG to help it reduce costs.

Step 1: Establish understanding of the case

Let me first clarify the question. The client manufactures a $20 million jet and, because of competitive forces,
has to reduce its cost by 5 percent. Is BCG's role also to verify the purchasing department's estimate?

No, you can assume that the purchasing estimate is correct. BCG's role is to find the cost
savings to meet that estimate.

Could I take a few minutes to think about the case?

47
Sure, please do so.

Step 2: Set up the framework

First, I would like to understand the cost structure of the jet to see what we should look at first. Next, I would
like to look at major factors driving the costs we are targeting. Finally, I would like to explore potential ideas to
reduce cost.

That sounds like a very logical approach. Let's proceed.

Step 3: Evaluate the case using the framework

Because the time for the interview is limited, I think we should try to identify those areas most responsible for the
cost of the jet.

Time is limited on real projects as well, so I think that would be a good idea! You have the
following cost information for the jet. How would you interpret it?

The major cost driver for the jet appears to be purchased materials. Within manufacturing, direct labor is a
fairly large component of cost, as are program management and corporate overhead within overhead. I think we
would want to concentrate most on materials, however, since that's where most of the costs can be found.

That sounds like a good place to start. Where would you look within materials?

48
I see that materials are broken down into purchased subassemblies, components, and raw materials. I
understand what raw materials would be, but what would be the difference between components and
subassemblies?

A subassembly functions on its own. An example is the pilot night vision system. A
component is a smaller part, such as a part of the engine.

I know that governmental agencies often have very strict guidelines about purchasing that could affect the cost of
materials.

For the sake of this case, you can assume that the British Ministry of Defense, MOD, allows
"commercial off-the-shelf" purchases, which means that the client is free to purchase from
whomever it wants, as long as it can ensure that the parts meet MOD quality guidelines.

I see that purchased subassemblies comprise more than 70 percent of materials. How many suppliers are there
for these subassemblies?

There are seven suppliers of major subassemblies that go into the fighter jet.

That seems like a relatively small number. Are there more suppliers that are qualified to do this type of work?

The manufacture of these parts requires a substantial investment in R&D, engineering, and
infrastructure. It would be very costly for new suppliers to make the required investment,
particularly if the client is trying to reduce the price it pays to the subassembly manufacturers.

Since there are only a few subassembly suppliers, and the investment hurdle would preclude bringing in
competing manufacturers, it would be difficult to reduce the price paid. Perhaps we should look elsewhere for
savings.

But remember, if your client loses the contract, it will lose its customer unless it is teamed with
the competing bidder. Even then, if the competitor is underbidding your client, there will be
even less room for it to profit.

Perhaps it would have an incentive to reduce its costs in order to maintain the contract. Are the majority of its
costs in materials as well?

How could you find that out?

I would want to interview the purchasing and engineering personnel of the different subcontractors in order to
understand their cost structures. If we had a better understanding of their economics, our client might be able to
reduce cost across the board, allowing it to compete more effectively for the contract without killing everyone's
margins.

Let's say that purchased materials average approximately 70 percent of the price paid to most
of the manufacturers.

If the cost of subassemblies represents 40 percent of the jet cost and 70 percent of that is purchased materials,
total purchased materials would be approximately 28 percent of the cost for subassemblies. Purchases of raw

49
materials and components represent another 15 percent, for a total of around 43 percent of the cost of the jet. If
our client could reduce the cost of raw materials by 20 percent, it could reduce the cost of the jet by more than 8
percent, more than enough to offset the 5 percent reduction it would need to win the contract.

That sounds reasonable, but 20 percent is a very lofty goal. How would you go about doing
that?

First, I would look at the number of suppliers. Are there a large number of suppliers to the subassembly
manufacturers?

The client estimates that there are approximately 125 suppliers of raw materials and
components among the manufacturers of the subassemblies and itself.

Well, that sounds like a large number of suppliers. Of course, they could be providing very specialized materials
to the subassembly manufacturers. Are these suppliers providing customized or more commodity products?

About 80 percent of these products are commodities, such as sheet metal and wire harnesses.
Even some of the electronics, such as printed wire boards and circuitry, are fairly generic.

That sounds promising, but I would need to know whether these commodities are interchangeable, so that our
client could concentrate spending with fewer suppliers. Are there many commonalities among the parts used by
the different subassembly manufacturers? We could talk to their engineers and look at the designs and bills of
material to determine how much overlap there is.

Let's say that you did this and discovered that approximately 30 percent of the cost of raw
materials is from similar materials used across the subassembly manufacturers.

It seems safe to assume that the client would need more commonality to be successful in concentrating its
purchasing and reducing costs. Do the engineers believe that the percentage of overlap could be increased if the
designs were modified?

They believe they could increase that percentage substantially, particularly with basic materials
such as screws and sheet metal, but also in other more customized areas.

That's great news, but we would still need to know whether the subcontractors are using the same suppliers. We
could analyze the number of suppliers for each of the areas of overlap.
Good suggestion. Although there are some common suppliers, the analysis indicates that the
subassembly manufacturers tend to use different suppliers.

Step 5: Summarize and make recommendations

Our client needs to reduce costs by 5 percent. The largest area of opportunity appears to be in purchased
materials, the majority of which comprise subassemblies manufactured by seven subcontractors. By looking at its
purchases in total, the client can target approximately 40 percent of costs. To achieve the 5 percent cost
reduction, it would need to reduce costs by 15 to 20 percent. It could try to do that by increasing commonality in
the design of the subassemblies and components and by shifting volume to a smaller number of suppliers.

50
Considering that the majority of the raw materials and components are purchased
commodities, do you think the 15-20 percent cost reduction is achievable?

Well, I know that commodities typically have lower margins than more customized products. I suspect it may be
challenging to hit the client's savings target by focusing only on these purchases. But since raw materials and
components represent about 40 percent of costs and there is an opportunity to concentrate purchasing, I think we
should start here.

Where else could you look for savings?

If I look back at the cost data on the jet, direct labor is another large cost component. As a contingency, we
could look into that area as well. I've read that other companies use outsourcing to lower their manufacturing
costs-perhaps our client could do the same. For example, it might want to increase its use of purchased
subassemblies and reduce the amount of direct manufacturing it does. Of course this would work only if it could
drive direct labor costs below the offsetting cost of these subassemblies. The client will be working closely with the
subassembly suppliers to implement its purchasing initiative. This may give it an opportunity to explore the
suppliers' capabilities at the same time.

That's an interesting suggestion. How would you recommend the company pursue both of the
initiatives you have discussed?

I would look first to combine purchases across the subassembly suppliers with our client's purchases. I suspect
that the client and the subassembly suppliers will need to share a great deal of information, including engineering
drawings and specifications, with potential suppliers of the raw materials and components. The Internet could
prove to be a very effective medium for forming a single "virtual" purchasing department to consolidate both the
flow of information and purchase orders across the companies. Our client might also want to use a bidding
system for those materials that are true commodities.

Next, I would turn to the engineering departments and form cross-company teams to look for areas in which to
increase commonality of design. At the same time, those teams could explore opportunities to use more purchased
subassemblies and decrease the client's direct labor costs.

That sounds great, and is very similar to a project we did. I would caution you, however, to
examine the upfront costs involved in your recommendations, both for the redesign and for
the implementation of the purchasing system, before going ahead.

9.4 Gas retail case (BCG)

Client

Your client is the major operator (monopolist) in one of the largest European gas market. His
business includes two major activities:
• Gas sales to households and firms (gas bought from large producers in Russia,
Norway, Algeria…)

51
• Gas transportation from the national border, where it is delivered by the producer, to
the end consumers. This implies the existence of a large ensemble of infrastructures:
transportation network, distribution network, storage equipment, methane terminals…

Let's discuss the challenges on the natural gas market after market liberalization in Europe.

Situation

Concretely, the market's deregulation means


• The end of the monopoly for the gas sales; the arrival of new competitors
• The preservation of the monopoly on transportation, but under the surveillance of an
independent authority that guarantees equal access to all competitors

Your client is at the head of the purchases/sales department. He is in the following situation:
• Today, company market share is 100%
• At a certain point in the next years the market will at once be opened to competition
(which is a simplified way of putting it since in reality there will be stages)

Client's question

About the gas sale activity that will be opened to competition


Ö What will be the level of competitive intensity at opening?
Ö What actors are likely to become my competitors?

Evaluate the case

According to you, how many and what types of competitors are likely to enter the market?
(Structure)

I believe I would need to evaluate the market attractiveness (market growth, profitability/margin, risks) and the
entry barriers (gas availability, brand). I would need to ask the following questions:
• What are the rules of the game/key success factors (access to suppliers, customer intimacy, cost
advantages, branding …)?
• How are other players positioned to enter the market?
• What are their competitive advantages thanks to synergies with other activities (electricity, services …)?

Let us focus on the gas retail sale activity's attractiveness. There are three dimensions you
should consider: the natural gas market's growth potential, the profitability of this activity and
the risks associated with it.

Let us start with the market's growth potential. What are the market's growth levers?
(Structure)

I would differentiate between firms and households. The key levers by client type would be:
• Households: network penetration, share of gas vs. other energies; consumption of gas/household
(climate, isolation …)
• Firms: same as households, plus industry growth, productivity, competitiveness with other energy forms

52
Given the market's main growth levers for the firms' segment and for the households'
segment, do you think that the market will strongly grow, stagnate, or decrease? (Judgment)

For the households, I would forecast???? the rise of penetration (network extension) but, overall, I think the
consumption will decrease due to global warming and to better built houses. For the firms, I think it will
decrease, especially in industries that consume a lot of gas (general price and risk issues).

So what is your conclusion? (Synthesis, So what)

I think there will be weak or inexistent growth. A new entrant will have to take clients from the major player.

Can you imagine what a gas retailer's cost structure is? (turnover = 100)? (Synthesis, Structure)

I believe it would include the energy itself (cost of goods – gas), the infrastructure cost and sales and marketing
costs (commercial).

Here is a simplified cost structure: gas – 50%, infrastructures – 40%, commercial costs – 7%
and the margin is around 3%. What cost advantage can a new entrant expect to build for each
one of these costs? (Judgment)

Most probably, there is a small opportunity of differentiation through costs:


• Gas is sourced at comparable prices
• Infrastructure prices are identical for all competitors
• New entrants have to invest rather more in marketing
• New entrants are not expected to have a productivity lever and only have a small pricing lever.

I would have to check these assumptions.

Let us put ourselves in the shoes of a household client whose yearly gas invoice amounts to €
500. What is the price reduction potential for a new entrant? Can you give a rough estimate?
(Judgment, Rigor)

If I assume I can reduce commercial/marketing costs by 33% (500 x 7% x 33% = 11.55) and I allow a
50% lower margin (500 x 3% x 50% = 7.5), then a new competitor can reduce the gas price around € 15–
20/year (11.55+7.5=19). This might allow it to compete with the established client. Marketing costs can be
reduced if the new entrant is already established in other energy markets and benefits from scale and known
brand name.

What can we conclude on a new entrant's margin level? (Synthesis)

Margin will necessarily have to be weak or inexistent to attract clients and draw away from established player.

Let us now consider the risks borne by our retailer. In order to simplify, let us focus on what is
called the climatic risk. The sales volumes will vary a lot depending on the year, whether the
winter is cold or not. During a "warm" year, let's suppose that the heating volumes decrease by
10%, that the cost of supply/gas are totally variable, that the commercial costs are totally fixed,
that the infrastructure costs are partly flexible, at 70%. What will be our gas retailer's margin?
(Structure, Rigor)

53
I am basing my analysis on the sales and cost structure of a normal year (turnover = 100). Then I calculate the
value of each cost block for a warm year, also the margin and compare with the margin in a normal year.

Cold vs. warm


Sales: 100 vs. 90 (-10%)
_____________________
Gas: 50 vs. 45 (-10%)
Infrastructure: 40 vs. 38.8 (30% of 40 is variable, makes 12, 10% reduction makes 1.2)
Commercial: 7 stay 7

Total cost: 97 vs. 90.8

Margin: 3 vs. -0.8

In a warm year, it is more expensive to sell gas, so it is a high risk business.

What can we deduce from this risk calculation? (Judgment)

The climatic risk is too high to justify the small margin in a normal year.

Your first meeting with your client is tomorrow morning. What can you tell him/her to answer
his/her question based on the analyses that we have just done together? (Synthesis)

Well, the market is not that attractive and new entrants are a weak threat.

Finally, it looks like our major player does not have to worry; the gas retailer activity's
attractiveness is so weak that one would have to be stupid to venture in it at its opening! But
why would it be a big mistake to tell our client not to worry? (Creativity)

We are not working on the right strategic segment: the gas retail sale segment in not independent of the electricity
sale and services, as soon as the monopoly disappears. We have been influenced by the client's historical view.

In fact there is a bias in our reasoning from the start. What is it? (Creativity)

We have looked at the gas market on a stand-alone basis. But we need to take into account that the rules of the
game might change and that other energy providers might enter the market. Those providers might offer
additional products to the gas client: electricity, oil, services or other products.

Are there other levers that would enable a player to enter the gas market in a profitable way?
(Creativity)

By offering other energy products or services and products, there can be synergies with the gas supply:
• Channel diffusion/delivery costs
• Margins from other services can cover production risk

On the other hand, there could be cost synergies on the commercialization:


• Client back-offices could combine gas and electricity sales
• Brand and client acquisition

54
Who could the other new players in the gas market be? (Judgment)

Potential new players that bring additional value to the client could be major electricity firms, major oil
producers and/or major retailers. For the electricity firms, synergies would be mainly based on the
commercialization cost synergies, also for retailers. For the oil producers, there are synergies on the supply side.

What can we finally say to our client? (Judgment)

The threat is real; the firm's traditional strategic vision must be questioned due to the emergence of the new
market conditions and rules of the game. Examples of dangerous players are large power firms, oil producers if
they don't have more profitable investments to make and a partnership between a large European energy player
and a large retailer.

9.5 Consumer ADSL services cases (BCG)

The situation is as follows:

ADSL is a technology that enables the implementation of broadband Internet services via the
existing telephony infrastructure. This telephony infrastructure is owned by the incumbent
telecom operator in Norway. As in most European countries, the regulator has ruled that new
entrants may offer ADSL services, using the existing incumbent-owned telephony
infrastructure via MDF access. This means, the new entrant can hire the copper wire from the
incumbent operator but has to purchase its own ADSL equipment.

The technical set-up that a new entrant would need in order to establish an ADSL connection
basically consists of three elements: MDF access (copper wire), ADSL equipment, Internet
uplink capacity (fiber access connecting the ADSL equipment to the Internet).

The costs involved in establishing the technical set-up are given (amounts have been converted
to euros):
• MDF access tariff is EUR 12 per line per month, set by the regulator
• Required investments for the ADSL equipment is EUR 120,000 per location,
depreciation period of 5 years. For simplicity reasons we assume linear depreciation
with no interest costs. In total, there are 250 locations where ADSL equipment could
be installed, covering all households in Norway
• Internet uplink capacity is commercially available. Depending on the required end-user
speed, costs are on average EUR 2 per end-user per month
• For simplicity reasons, we assume that the consumer ADSL modem is client-owned.

The case interview starts with an open question to test the candidate’s ability to apply structure
to a problem while a lot of information is still unknown.

Interviewer: Suppose you wanted to assess if a new entrant can run a profitable business in
offering consumer ADSL services, how would you approach this?

55
Candidate: Let’s see, since we are considering a new entrant, who has initially no customers, the company will at
first generate loss, and gradually will become more profitable as the number of customers increases. I could make
some assumptions on the pace at which the customer base will grow, but rather than making this too complex at
the beginning, I would start by calculating some sort of break-even point. I mean, if we never reach break-even,
this idea will certainly not fly.

The structure is the basis for the entire case. A good structure should enable the candidate to
systematically solve the case, to set up some easy calculations to verify initial hypotheses, and
navigate back and forth through the case. Now, the interviewer asks the candidate to do some
easy calculations to see if he/she can apply the structure. This way the interviewer tests if the
candidate can combine the elements to a useful outcome (Synthesis), how comfortable the
candidate is in doing some basic calculations and if he/she applies sanity checks to the
outcome (Rigor).

I: Sounds good, how would you calculate the break-even point?

C: Hmmm, we know all the costs, but we don’t know the revenue, which would be price times number of
customers. So I could assume a price and then calculate the number of customers we need. Subsequently, I would
need to check if the required number of customers would be a reasonable number. Do we know anything about
the price?

I: We know that the incumbent charges on average EUR 20 per month.

C: OK, let’s assume we could also ask EUR 20 to start with. Later we may need to reconsider this and see if
we need to lower the price in order to become more competitive. So let’s see if we can calculate this… hmmm,
wait, I see we have a lot of information on technical costs, but I don’t see anything on personnel costs, is this
correct?

I: Well spotted, we also need an organization and a small marketing budget. The new entrant
has done some pre-work and has calculated that an annual budget of EUR 4.8 million would
be sufficient to cover the entire country.

C: OK, then I can calculate the break-even point. Per customer, the new entrant earns EUR 20, of which it
uses EUR 14 to cover the expenses for MDF access and the Internet uplink. That means he has EUR 6 per
customer per month to cover all other costs. We write off the ADSL equipment in 5 years, so that’s EUR
2,000 per month per location. For 250 locations that’s EUR 500,000 per month. In addition, we have to
cover the organizational costs, which is EUR 400,000 per month. So for break-even we need EUR
900,000/EUR 6=150,000 customers.

After the basic set-up of the case, the interviewer assesses the candidate's ability to make some
basic assumptions in order to get to the next level (Judgment) and play around a little bit with
that to see if the candidate can really see through the drivers of the case (Synthesis).

I: OK, so what do you think, is that a realistic number to achieve? Let me add that we have
roughly 2 million households in Norway.

56
C: Hmmm, let's see...at first sight, 150,000 out of 2 million does not seem impossible, but I would like to
know a bit more about the market. For instance: is there a lot of growth in the market? How many competitors
are there? Can you tell me a bit more about that?

I: Yes, at the time of this situation, the incumbent operator was still the only provider of
ADSL services in Norway, but others have considered entering the market, just as we are
doing now. The distribution was as follows: 20% of the households had ADSL services, 20%
used Broadband via cable, 50% still used dial-up and 10% had no Internet. We know that
these percentages were established in 5 years time. That is: 5 years before no broadband
Internet existed. We have no data from in-between years.

C: OK… so that’s a fast growing market. I would say we start focusing on acquiring new customers from the
group that is currently using dial-up, since this is the largest group and since it is probably easier to acquire new
customers than to persuade customers from our competitors to switch. Now, if we would take a situation for
instance 5 years later and if we assume that by then the market has grown again with roughly the same amount,
the ADSL market increases from 20% to 40%, which means 400,000 new households will subscribe to
ADSL. We would be competing for these clients, against the incumbent operator, and maybe some other new
entrants as you mentioned. So… let’s say the incumbent operator gets at least a 50% share of that, since it has
a strong presence in the market already. Let’s assume we would be competing against 1 or 2 other competitors,
that means we could get 17–25% share of the new customers, or ~70,000–100,000 in total. So it seems that
it won't be possible to reach break-even from new customers only. We could also have a look at the existing
customers, both ADSL and cable customers. But then I would need to know how likely it is that they would
switch?

I: OK, that is something you may want to investigate. But what would be your upfront
judgment on this?”

C: I can imagine that customers would want to switch if they are very unsatisfied with the current service, or if
the new entrant would offer a much lower price. Do we know anything about customer satisfaction?

I: Most customers are very satisfied, the service is very reliable.

C: OK, so let’s look at lowering the price. For instance a 10% discount…. Hmm… but that would mean we
also would need more customers to reach break-even. A 10% discount means we drop from EUR 20 to EUR
18 contributing EUR 4 per customer instead of EUR 6 to the fixed costs. So we would need 225,000
customers in this case. Even if we could get a greater share from new customers, I still think it would be
unrealistic to assume we could persuade a large number of customers to switch for a EUR 2 discount,
particularly if they are satisfied with the current service. In addition, there’s the risk that our competitors would
lower their prices too. Concluding, I would say this is a risky business to step into.

When the basics of the case have been cracked, the interviewer would finally test if the
candidate is able to assess the problem from different angles and if he/she can come up with
alternative solutions (Creativity). If the case runs smoothly and there’s time left, the candidate
can even quantify part of these alternative solutions.

I: I agree. The way we have been looking at it, the business case looks very challenging indeed.
But let’s spend a minute to see if we can come up with some creative ideas here. Could you
think of some ideas that would make this business case more attractive?

57
C: I was struck by the negative effect a slight price decrease has. So, I was thinking: could we somehow increase
the price? Of course, this can only be justified by offering better service. Would it be possible to increase the speed,
for instance?

I: Yes, technically this would be possible. You would need to make sure that you buy enough
capacity on the Internet uplink. Let’s assume the speed you can offer is directly proportional to
the capacity you buy and the costs associated with it.

C: OK, so doubling the speed would increase the cost per subscriber by EUR 2 per month, while we could
probably charge a much higher tariff, maybe EUR 30 or so. If we could diversify our offering, and if part of the
customers subscribe to the premium packages, this would certainly improve the case. Furthermore, I am
considering offering additional services, like telephony or even video services if that’s possible. Of course we would
need to make a separate business case for that, but since EUR 12 out of the 20 we charge is spent on MDF
access, any other service we can offer, which re-uses this asset, would greatly improve the case. I mean, we would
spread our largest cost component across multiple services.

I: OK, sounds like these ideas are worth investigating. Anything else you can think of?

C: What about starting in a limited number of regions, instead of covering the entire country from the start? Is
the number of customers per location equal across the country?

I: Good point! The area of each region is more or less the same, which means…

C: You have a huge variation in the number of households per location. If we pick the high density areas, for
instance the Oslo area, plus some other larger cities, we can perhaps reach 50% of the households through, I
don’t know, maybe only 20% of the locations and thus only 20% of the ADSL equipment cost. That would
make a difference.

I: You’re absolutely right, and you know, looking back this is exactly what new entrants have
done when entering the ADSL market: start in high density areas, offer higher speeds at
premium prices, in particular to small and medium-sized enterprises, and – at a later stage –
expand the service by offering additional services.

9.6 Call center case (BCG)

The case interview

There is a company, let's say a travel agency, that sets up a centralized call center where all
incoming phone calls from all branches are diverted to. Shortly after setting up the call center,
the travel agency is virtually unreachable because of the large amount of phone calls received.

Before answering any questions, the candidate should check if he/she completely understands
the problem stated. The candidate can ask additional questions if the problem is not
completely clear.

58
The first question tests the candidate's creativity and ability to come up with more than one
idea, at the same time it tests the use of a structure, which helps coming up with more ideas.
First, the candidate should think of main causes, and then specify these main causes in more
detailed possibilities.

Interviewer: What could cause the overload of the call center?

Candidate: I believe there could be different types of reasons:

Caused by supply
• Lack of call center agents (by sickness, by poor planning)
• Poor organization of the call center (unstructured call-diverting)

Caused by technical difficulties


• Calls are not transferred to the centralized call center

Caused by demand
• Peak in client demand (more calls than expected due to international affairs, high-season, bad
weather…)

I: Let us assume that the overload in the call center is not caused by technical problems or by
peaks in demand, but is solely caused by the fact that the amount of required agents does not
match the number of agents in the center. How would you calculate the required number of
call center agents needed to fix the problem?

For this question, it is important to set up a good structure to tackle it. Not a general structure,
but one tailored to this specific situation. This is one of the main things the interviewer tests
with this question: can the candidate set up a good structure, tailored to the problem.
Furthermore, the interviewer tests if the candidate understands the problem and has some
(business) judgment (e.g. in this case: a call center agent is not productive 8 hours per day).

A possible answer could be as shown in the next figure:

59
Number of diverted
customers
x
Call volume Number of calls per
customer
x
x Seasonal pattern (%)

Required number of
agents Call time (sec)
Average settlement time
+
Wrap-up time (sec)

: Working hours per agent


(sec)
x
Capacity per agent
Gross/net ratio (%)

x
Working Time Duration
(%)

This answer first divides the required number of agents in volume of calls, average time spent
on a call and capacity per agent. The next step would be to divide these three main parts into
smaller parts. For the volume of calls, the first important driver is the amount of diverted
customers. Furthermore, customers can call up more than once; this is called the average
number of calls per customer. The third driver of the volume of calls is the seasonal pattern.
The average time spent on a call is simply the duration of the call and the time needed by the
agent to wrap up the call (e.g. enter information in a computer).

The capacity per agent starts of course with the working hours per agent, but an agent is not
100% productive. There are several ways to define the productivity, in this example the
productivity is split up in two parts. The first is gross versus net working hours due to lunch
breaks etc. The second is "working time duration", this is the percentage of the time the agent
actually is busy with a phone call. An agent can be busier during prime time and less during
other hours of the day.

There is now a clear structure to show what drives the amount of agents needed, and therewith
a way to calculate the required amount of agents.

I: How many agents are required in week 20?

Most case interviews have some computation in them. To solve it, the candidate will have to
figure out the numbers for all parts of the "driver-tree" to compute the required number of
agents. There are several ways to figure out the numbers. First of all, an interviewer might give
the candidate some data to start with, and he/she will have to pick out the numbers to use in
the calculation. Secondly, the candidate can ask the interviewer for data, and thirdly the
candidate can make assumptions using common sense. The candidate should always clearly
state when he/she is making an assumption and how he/she comes up with it.

60
In this case interview, the following data is given by the interviewer:

Cumulative number of customers who are


diverted to the centralized call center
Number of 1.200.000
customers

1.000.000

800.000

600.000

400.000

200.000

0
0 10 20 30 40 50 60

Week

Note: At the moment of set-up of the call center, the graph starts with 0.

From this graph, the candidate can deduct the cumulative number of diverted customers in
week 20: approximately 140.000 customers. So in week 20, there are 140.000 clients "in the
database of the call center". To calculate the required number of agents, other data in the
driver-tree is needed, and the candidate will have to figure it out. He/she can start off with
asking the interviewer for data, but it's always best to make assumptions and test these with the
interviewer. For example: "I assume that on average, a client has contact with an agent of the
call center twice per year, so the average number of calls per customer is 2". And: "I wonder if
there are many more calls in week 20 versus the other weeks of the year, for now I assume the
seasonal pattern to be 0%". The candidate then calculates the calls' volume: 140.000 / 52 * 2 +
0% = approximately 5.400 calls in week 20.

To determine the average settlement time, the candidate can ask the interviewer for
information. The interviewer may either tell the candidate to make an assumption, or give the
numbers. In this case, these are given: "Monitoring talk- and wrap-up-time gave an average call
time of 240 seconds and an average wrap-up-time of 60 seconds per call". This information
gives the candidate the average settlement time: 240 + 60 seconds = 5 minutes = 1/12 hour.

The last driver is the capacity per agent. The candidate can assume that the number of hours
an agent works is 40 per week. To determine the gross/net ratio, the candidate could ask the
interviewer, though he/she could also easily make an assumption on own experience: "I
assume that of the 8 hours worked in a day, approximately 1.5 hour is lost to lunch and other
breaks. This makes the gross/net ratio ~80% ((8-1.5)/8 = 81%). The last number to figure out
is the working time duration, this is a hard number to guess without knowledge of call centers,
but the candidate could still give it a try. The candidate should then ask the interviewer if the

61
assumption is more or less right. From experience, the number is ~50%. This gives the
capacity of an agent per week: 40 * 80% * 50% = 16 hours per week.

Now, the candidate has all numbers to calculate the amount of agents needed in week 20. The
candidate should not forget to use the same units for all parts of the equation (all in seconds,
hours, or weeks) and not make any unnecessary mistakes. In this case, the number of agents
needed in week 20 is: 5.400 calls *1/12 hours per call / 16 hours per week = 28.1, so either 28
or 29 agents are needed.

I: There are no additional trained call center agents available. What other ways can you think of
to help the travel agency with its telephone problem?

It is stated that the amount of required agents does not match the number of agents in the
center. However, since there are no additional trained call center agents available, the candidate
has to find ways to decrease the amount of agents needed.

This question will test if the candidate really grasps the problem and if he/she uses his/her
structure in the right way. Looking again at the structure, the answer is clear: There are three
levers to decrease the amount of agents needed:

1. Reduce the call volume


2. Reduce the average settlement time
3. Increase the capacity per agent

To determine specific solutions, the candidate needs creativity and understanding of the
problem.

On the first lever:


• Stop diverting the calls from branches to call center (and divert back)
• Insert a computer-based menu which answers the most frequently asked questions

A possible solution to the second lever is:


• Reduce the wrap-up time by monitoring it, determine the most common wrap-up
actions and shorten these actions where possible. For example introduction of a
computer application that helps the agents with the standard wrap-up actions such as
"sending a brochure"

For the third lever, a possible solution would be:


• Align planning of agents with actual demand for agents by mobilizing more agents
during peaks in client demand and less during a low. This solution can be for example
achieved by stimulating the agents to work part-time.

62
9.7 Supermarket deli turnaround case (BCG)

Questions and Facts

1. Client’s deli financials

See Exhibit 1

2. Overall industry/ customers

Deli meat category has been flat to slightly declining recently. Prepared foods category has
been growing at roughly 10% per year as people have less time to cook at home.

3. Competitors

Increasing competition from deli departments of other supermarkets, discounters, etc. – e.g.,
expanding product lines, increasing advertising. Also competes with fast food restaurants in
prepared foods category.

4. Client’s product mix and recent events

Mix has remained constant, with the exception of two products introduced a couple of years
ago – BBQ chicken wings and “made to order” sandwiches. Both products have been a major
boost to prepared foods revenue.

5. Info on new products

BBQ wings are similar to the chicken wings the company already sells, although they take a
little longer to fry and are tossed in BBQ sauce after frying. “Made to order” sandwiches is
client’s response to Subway, etc. – for two hours during lunchtime and two hours during
dinnertime, one employee’s sole task is to make sandwiches to order for customers.

6. Financials of new products

Revenues for each product are $40M annually. Costs are not broken down at the product level.
See Exhibit 2.

63
SUPERMARKET DELI TURNAROUND
Exhibit 1

$M
2002 2003 2004
Deli meats Revenues 260 255 260

COGS 160 155 160

Prepared foods Revenues 360 400 440

COGS 190 230 270

Overall Revenues 620 655 700

COGS 350 385 430

Gross margin 270 270 270

SUPERMARKET DELI TURNAROUND


Exhibit 2

BBQ wings “Made-to-order” sandwiches


Price $5 for 20 pieces Price $4 per sandwich

Total material cost $0.10 per piece Avg. sales/store 20 sandwiches


per day
Prep time 15 minutes per
batch of 200 Total material cost $2 per sandwich

Employee cost $20 per hour Employee cost $20 per hour
(fully loaded) (fully loaded)

Total COGS $2.50 per 20 pieces Dedicated hours 4 hours per day

Margin $2.50 per 20 pieces Revenue $80 per store per day

Total COGS $120 per store per day

Margin $(40) per store per day

Note: Boxes indicate figures that should be calculated by the interviewee

64
SUPERMARKET DELI TURNAROUND
Sample Approach

Main question What should supermarket do to turn around deli?

Key areas to Revenue and profit breakdown within deli External factors influencing the overall deli
explore market

• Deli meat revenue and profits flat - consistent • People have less time to cook at home –
with overall category prepared foods category growing, deli meats
• Prepared foods showing revenue growth (10% category flat
consistent with category) but no profit growth • Increasing competition from other deli
Analysis – therefore declining margins – why? departments – starting to expand product lines,
- made-to-order (MTO) sandwiches losses increase advertising, etc.
offsetting profit growth from BBQ wings

• Eliminate made-to-order sandwiches (at least in low-traffic stores or during non-peak hours)
Recommend- • Raise or lower prices on MTO sandwiches (depending on demand elasticity)
ation • Boost demand for MTO sandwiches (e.g., advertising, promotions, merchandising)

• Eliminating MTO sandwiches or boosting demand can impact overall traffic in store and deli
Other factors

Framework and Analysis

There are three main questions asked to the candidate:


• Which part of the business is responsible for the lack of profit growth – deli meats,
prepared foods, or both?
• Is the lack of profit growth caused by flat revenues, increasing costs, or both?
• What is causing the flat revenues or increasing costs (and what should the client do)?

Based on Exhibit 1, the candidate will see that gross margins for both business lines are flat.
Furthermore, deli meat sales have been basically flat while prepared foods sales have been
growing at 10%.

The candidate should recognize that the client’s deli meat and prepared food sales have been
growing at about the category averages; therefore, revenues are not the main issue here. Deli
meat COGS have been more or less flat, mirroring sales. However, despite robust growth in
prepared food sales, prepared food profits have been flat, implying deteriorating margins.

At this point, the candidate is asked for some potential reasons for deteriorating margins (e.g.,
change in product/sales mix, rising material costs, rising labor costs).

If the candidate asks about changes in product mix, the interviewer informs him/her about the
BBQ chicken wings and the “made to order” sandwiches. The candidate should be suspicious
at this point and ask to learn more about these products.

65
By doing a back-of-the-envelope analysis of product profitability (based on data in Exhibit 2),
the candidate can find that BBQ wings have a 50% margin, indicating that they are not a
problem. On the other hand, he/she will find that the client is losing a lot of money on the
“made to order” sandwich concept.

The candidate is then asked for recommendations, which could include:

1) Eliminating the “made to order” sandwich


2) Restricting the “made to order” sandwich to busier stores or during busier times of the day
(e.g., lunch hours only)
3) Raising or lowering prices (to either increase profit per sale or units sold – will depend on
demand elasticity)
4) Boost demand (through increased advertising, promotions, better merchandising, etc.).

The candidate can also consider the second-order effects of eliminating the product or
boosting sales (the effect on traffic in the deli and the overall store).

9.8 China outsourcing case (BCG)

Problem set-up

The client is a national manufacturer of plastic consumer products that are sold in a variety of
retail formats, including supermarkets, discounters, club stores, and dollar stores. The company
has three main product lines: 1) freezer bags, 2) plastic plates and utensils, and 3) specialty
plates and utensils.

The CEO has been reading for some time about American companies outsourcing their
production overseas to low-cost countries such as China. She wonders whether this makes
sense for her company as well. It worries her that none of her main competitors have
established foreign production capabilities; on the other hand, this could be a tremendous
opportunity to gain a competitive advantage.

We have been asked to help the client understand the benefits and risks of moving its
production capabilities to China and to provide a recommendation.

Question and Facts

All three product lines have similar cost structures and savings (see Exhibit 1)

1. Freezer bags

Plastic bags are used mainly to store food items in freezers. According to customers, top
purchase criterion is quality, since low quality bags will result in food spoilage. Client is #3 in
category, with 200 million lbs. sold. The category leader has a strong brand and strong
innovation.

66
2. Plastic plates and utensils

Disposable plates and utensils; intended for single/limited use. According to customers, top
purchase criterion is price. Client is #2 in category, with 300 million lbs. sold. Client is at cost
parity with category leader but has a weaker brand.

3. Specialty plates and utensils

Plastic plates and utensils produced for specific retailers, customized to their design specs.
According to customers, top purchase criterion is style/design. Because many products are
new and untested, demand is highly variable. Client is #1 in category, with 100 million lbs.
sold. No strong competitors.

4. Current client production capabilities

All products are made in a single factory in Ohio. The factory is at capacity and the company is
considering building or acquiring a nearby facility.

5. Chinese production options

Client has no previous experience in building and managing a factory overseas. Client has met
with several Chinese manufacturing partners and has done initial product testing.

All three product lines have similar cost structures and savings (Exhibit 1).

Quality: lower quality on freezer bags, equal quality on plastic plates and utensils (both regular
and specialty).

Lead time: need 3-4 weeks of additional lead time for each product line for transportation
from China to U.S. distribution center.

6. Chinese market, current client presence

All three categories are relatively underdeveloped but growing, dominated by local
manufacturers. Client does not currently have any sales in China, although a few of their U.S.
customers (e.g., Wal-Mart) do have presence there.

67
CHINA OUTSOURCING OPPORTUNITY
Exhibit 1

Costs in Costs in China Costs in


Costs U.S. ($/lb.) relative to U.S. China ($/lb.)
Labor 0.30 8% of wage rate
80% of productivity
Material
• Plastic resin 0.30 80%
• Other material (incl. packaging) 0.20 75%

Variable overhead 0.05 140%

Fixed overhead 0.10 60%

Transportation
• China to U.S. distribution center N/A $6K to ship 40K lbs.
• U.S. distribution center to customer 0.05 Same

Total 1.00

CHINA OUTSOURCING OPPORTUNITY


Sample Approach

Main question Should plastics manufacturer move production to China?

Key areas to Cost savings Consumer behavior and Effect on current


explore purchase criterion production capabilities

• Would save $0.25/lb. (25% of • Quality is top purchase • Current plant is at capacity
current costs) criterion for freezer bags - outsourcing would
• At current production levels, - lower quality from China eliminate need to build
would save: • Price is top criterion for plastic additional capacity
Analysis - $50M in freezer bags plates and utensils • Plastic plates and utensils are
- $75M in plastic plates and • Style is top criterion for 50% of total production
utensils specialty plates and utensils - outsourcing may create too
- $25M in specialty plates and - highly variable demand much extra capacity
utensils requires short lead times

• Outsource plastic plates and utensils to China


Recommend- • Do not outsource specialty plates and utensils
ation • Do not outsource freezer bags (although further analysis may be warranted)

• To compensate for extra capacity that would be created in current plant, could produce new product
Other factors
line, rent out spare capacity, or move to smaller facility

68
Framework and Analysis

The candidate should start with a brief overview of the potential benefits and risks of
outsourcing to China. The main benefit is lower costs, mostly driven by inexpensive labor. A
secondary benefit is a possible springboard into the emerging Chinese (and other Asian)
market. Risks include lower labor productivity, possible quality issues, longer lead times,
additional transportation costs, and potential communication/coordination issues. The
candidate can be asked about the ramifications of longer lead times – they include greater
carrying costs, higher cycle and safety stock, greater forecast error, and less responsiveness to
demand.

There are three main questions asked to the candidate:


• How much cheaper is producing in China?
• What do consumers value and how would outsourcing affect those criteria?
• What are the client’s current production capabilities and how would outsourcing
part/all of their production affect the remainder?

First, the candidate should size the opportunity – is this a $5 million or a $500 million
opportunity? By solving for the last column in Exhibit 1, the candidate will find that the client
would save $0.25/lb. (25% of current costs) by outsourcing to China. Given current
production levels, the client would save $50 million by outsourcing freezer bags, $75 million by
outsourcing plastic plates and utensils, and $25 million by outsourcing specialty plates and
utensils. Two notes: 1) costs may increase if the Chinese Yuan rises versus the dollar and 2)
these estimates do not include a profit margin for the Chinese outsourcing partner.

The candidate should also recognize that cost savings alone are not sufficient to make a
decision. It is important to understand how an outsourced product will affect sales. The
candidate should suggest market research to understand consumer behavior.

Freezer bags: since customers’ top purchase criterion is quality and outsourcing would produce
lower quality bags, the candidate should raise a red flag here. A more sophisticated
recommendation would be to conduct market research to see the impact on sales of the lower
quality bag at lower prices – even though quality is more important than price, the magnitude
of a price change may override the drop in product quality.

Plastic plates and utensils: the top purchase criterion here is price, which makes this product
line an attractive outsourcing opportunity. The candidate can be asked what the client should
do with the cost savings – potential recommendations include dropping price to steal share,
investing to defend its position in case competitors begin outsourcing (e.g., brand, innovation,
customer service), and milking the product line as a cash cow.

Specialty plates and utensils: the highly variable and unpredictable demand for these products
means that shorter lead times are critical in order to adjust production quickly. Longer lead
times will result in greater forecast errors, higher safety/cycle stock, and more unsold
inventory and/or out-of-stocks. Therefore, specialty plates and utensils should not be
outsourced.

69
An analysis of customer purchase behavior indicates that plastic plates and utensils should be
outsourced, specialty plates and utensils should not be, and freezer bags probably should not
be. The final step is to understand the impact of outsourcing on the client’s current production
capabilities. For example, will it lead to plant closings (resulting in closing costs and possible
negative publicity)? Will it lead to underutilization of current facilities?

Since the current plant is already near capacity, moving plastic plates and utensils offshore
would actually save the client from investing in new facilities. However, since that product line
makes up 50% of total production (in terms of lbs.), removing it may create too much extra
capacity in the current plant for the two remaining lines. To compensate, the client could
produce a new product line, rent out the extra capacity, or move to a smaller plant.

9.9 Specialty paper sales case (BCG)

Problem set-up

Your client is a leading manufacturer of specialty papers sold to commercial printers. The
client produces self-adhesive sheeted papers that are ultimately used in a variety of labeling
applications – including the labeling of consumer goods and the printing of self-adhesive signs.

Your client’s operations are profitable, but the business has failed to grow over the past few
years. The client would like to invest in the business and you have been asked to identify
opportunities for growth.

GROWING SPECIALTY PAPER SALES


Questions and Facts (I)

Topic Information
Supply chain
Raw mats Layering Sheeting Packaging Distrib.
• Rolls of paper • Adhesive and • Rolls of layered
• Adhesive liner applied paper cut into
• Non-stick liner to rolls sheets

• The client is not capacity constrained in its manufacturing processes


• The client’s manufacturing and packaging operations are currently configured to package
specialty papers in boxes

Customers • There are approximately 24,000 commercial printers in the United States
• Printers are roughly categorized into three groups: small, medium, and large
• Differences among the groups are driven by the type of printing technology employed and the
size of print jobs that the printers are able to serve
• Printers prefer to receive product from the specialty paper manufacturers in different forms,
primarily driven by the type of printing technology employed
- Small printers prefer to receive their specialty paper in boxes
- Medium printers prefer cartons of specialty paper
- Large printers prefer to receive palletized shipments of specialty paper

Market share • The client has approximately 30% market share with small printers and only 10% share with
medium and large printers

Client financials • Margins are currently acceptable but management is against cutting price to gain market share,
knowing that competitors can match price cuts
• Price and cost to serve per equivalent box are different for each customer type

70
GROWING SPECIALTY PAPER SALES
Questions and Facts (II)

All figures are per equivalent box


Small (boxes) Medium (carton) Large (pallets)
Price to printer 20.00 18.00 15.00

Materials 5.50 5.50 5.50

Coating 1.00 1.00 1.00

Sheeting 0.50 0.50 0.50

Packaging (direct costs) 3.00 2.00 1.00

Gross profit/($ per equivalent box) 10.00 9.00 7.00

Number of printers 20,000 3,000 1,000

Annual usage (number of equivalent 100 500 3,000


boxes)

Total potential profit pool ($) 20,000,000 13,500,000 21,000,000

Framework and Analysis

The problem set-up indicates that the client wants to invest in this business. Investment can
take many forms including expansion of manufacturing operations and capacity, expansion of
customer-facing activities, and acquisition of competitors. It is interesting to note that this is
currently a profitable, no-growth business for the client. Investment decisions cannot be made
unless the management team (and the candidate) understands the market conditions as well as
the client’s internal capabilities

This case does not lend itself well to traditional “case solution structures”. A strong initial
response is to list a set of internal and external factors that must be understood and evaluated.
Ultimately, the candidate should decide whether investment is warranted, and if so, where.

Strong hypotheses might include:


• Assuming the client is not capacity constrained, there are likely groups of customers
that represent opportunities for profitable growth
• Depending upon the current go-to-market strategy, the client may need to re-evaluate
the way that it is configured to serve existing and potential customers

The client can expand its packaging operations to better serve medium or large customers, but
not both. Client economics and cost to serve each customer group are shown on Exhibit 1.
For simplicity, taxes and depreciation are ignored and SG&A is assumed to be fixed.

71
The candidate should recognize that a comprehensive solution evaluates the required
investment to serve a particular market segment (packaging line, manufacturing operations,
additional SG&A) against the expected return.

The candidate should evaluate the profit pool from serving medium and large customers. This
should be based upon an assumption about the size of the market that the client can capture.
Assuming the client can match its small printer market share, the client could capture an
additional 20% of the medium or the large printer customer segment

The following is given to the candidate:


• Investment and operation of carton packaging line would cost $675,000 per year
• Investment and operation of the palletizing line would cost $2,300,000 per year

A logical conclusion would be that an investment in a carton packaging line would be a


superior investment compared to the palletizing line

Potential conclusions:
• The carton packaging line is a less-risky investment (requires less up-front capital)
• The solution assumes a static environment. If large printers are growing in number and
or usage of specialty paper, this may change the answer
• The investment in a new carton packaging line would need to be evaluated against
other potential investments to understand if it is the optimal use of the client’s capital

9.10 Oil tanker case (Booz Allen Hamilton)

Step 1: Background and question

My grandfather has just died and left me an oil tanker. I need a valuation for tax purposes, and
I have hired you to tell me what it is worth. For your information, there are 3 types of tankers
in the world: small, medium, and large. Within these three classes, each tanker is identical to
every other. I have just inherited a medium tanker

Step 2: To be given as a response to student inquiries:

Supply-side information

Small Medium Large


Number 100 100 100
Capacity 1 unit 2 units 4 units
Number of trips per year 1 1 1
Operating cost $50,000/trip $75,000/trip $100,000/trip

Demand-side information

Scenario I: fixed demand for 500 units of capacity per year (transport costs are a negligible part
of total oil-cost structure, and demand is completely inelastic for purposes of this analysis).

72
Case 13: Street Sweepers
BACKGROUND
Firm: BCG
Round: 2006 FT, Final
Content: Mainly qualitative

CASE QUESTION

Our client is a large industrial conglomerate that has $60-70 million in annual revenues and is
extremely profitable. They have engaged BCG to examine one of their business units that is
underperforming. The unit manufactures and distributes street sweepers (zamboni/lawnmower-
like machines that a person would ride and which uses two large rotating brushes to sweep up the
street as it moves by) and has been doing so for 20 years. While never a standout division, until
recently it had always been profitable. 4-5 years ago the profit margins started to fall and it is
currently just above breakeven.

Management of the conglomerate had made a decision 2 years ago that they did not want to
invest in new features for their product lines and has the approach that they don’t want to invest
significant amounts of capital now (unless they can be convinced otherwise).

BCG’s task is to analyze the unit’s performance and recommend to the management of the
conglomerate what should be done. Tell me some of the areas you’d consider looking into here
knowing that we only have a few weeks to finish this engagement.

INTERVIEWER BRIEFING

The interviewer did not allow time for development of a framework – that isn’t the point of the
case. Instead, the interviewer simply starts asking questions and they should be answered as
thoughtfully as possible on short notice. Being thoughtful but remaining structured is vital here:
sticking to profits = revenue – costs and drilling down on costs and revenues. This is a great case
b/c if you understand basic drivers- it’s completely rational.

EXAMPLE DIALOGUE

Interviewee: That’s a tight timeline, so let’s focus on big areas. So profit is revenues minus
costs. Can you tell me about recent trends with regard to costs?

Interviewer: Sure. A quick look at the numbers showed costs have been fairly constant over the
past 20 years, only increasing with inflation.

Interviewee: So I’m thinking I’ll then move into revenue. How has that stood up?

Interviewer: Revenue has fallen steadily over the past 5 years. Why might this be happening?
Columbia Business School
Case Book 2005

44
Interviewee: It might be happening because of trends in the market, changes in customer
preferences…

Interviewer: Since you mention it, who are the customers?

Interviewee: I suppose they would be municipalities, governments, etc. I don’t see this as
being something for individual use.

Interviewer: Usually not. That’s right, it’s mostly state and local governments. So we looked
into the customers and they are keeping with past trends, replacing their street sweepers every 4-
7 years, and the number of municipalities purchasing street sweepers has remained constant.
What else might be driving revenues down?

Interviewee: Prices could be coming down…

Interviewer: Prices have actually remained constant.

Interviewee: So how about competition? Have new competitors entered the market or stolen
market share?

Interviewer: There have been no new competitors. These are what sales have looked like 20
years ago to now. What do you want to know after looking at this?

100%
90%
80%
70%
60% C
50% B
40% Client (A)
30%
20%
10%
0%
10 yrs ago 5 yrs ago Today
Interviewee: It looks like B has taken share from us. I’d like to know what B is doing
differently. And for that matter what C is doing differently so as to be unaffected.

Columbia Business School


Case Book 2005

45
Interviewer: About 5 years ago, B introduced a new technology that used air vacuums to clean
streets instead of the mechanical collecting methods that had always been used in the past.
These air machines are more effective at picking up small debris like sand and small litter, and
work more quickly and efficiently than the mechanical ones previously offered. Price points are
about the same. B still sells mechanical machines, but the drop in our market share was directly
related to the new air offering.

Interviewee: So how about C?

Interviewer: C makes machines that are far more heavy-duty, it’s really a different type of
offering.

Interviewee: So we don’t compete with C in reality?

Interviewer: No, but we could. Do you think we’d want to?

Interviewee: Not likely without knowing much about the market. They seem to have been
stable and have an expertise, so unless we can offer something new to their customers I’d guess
they’ll defend their position in a niche market at all costs.

Interviewer: Good, that’s the conclusion we quickly came to on C. Now, what other
information do you need to recommend something for our client?

Interviewee: I need to know if we can replicate the air technology.

Interviewer: We can, but it will take 2 years and cost $25 million.

Interviewee: Since you said management does not want to make a significant investment, that
seems unlikely. So other than that, what trends do we expect from this market going forward?

Interviewer: What would you think?

Interviewee: (Pause) I guess the encouraging news is that we’re still selling despite the new
technology and the similar price point. So do municipalities need both?

Interviewer: Good observation. In fact they do need both. So we broke the country into 7
regions, all of which needed the same amount of street sweepers overall. In the north, on
average there was a 22/8 ratio of mechanical to air sweepers needed. In the south, it was 22/18
(these numbers do not include C’s models). For the country, what % of street sweeper sales
were going to be that are mechanical?

Interviewee: Roughly 66% (It’s actually 64%).

Interviewer: Yes. So why would municipalities need mechanical sweepers if air ones are more
efficient, and what does this tell you?

Columbia Business School


Case Book 2005

46
Interviewee: I’d think they’d need mechanical ones to deal with larger, more solid things on
their streets. It looks like in the south they can use more air machines which would make sense
b/c they’d have sand and small debris. In the north you might get larger rocks, chunks of ice,
etc. This tells me the market isn’t going to zero, so maybe the company can simply protect the
market share that it has, scale back production to the point where the market will be in
equilibrium between air and mechanical sweepers, and keep decent margins going forward.

Interviewer: Right, so that’s exactly what we recommended. Once we recognized these trends,
we also looked for a strategic buyer, and in fact the conglomerate sold the unit to a foreign
company that already had air technology developed. That was outside the scope of this case,
though – just an interesting follow-up.

Columbia Business School


Case Book 2005

47
Case 14: Eye Surgery
OVERVIEW
Firm: BCG
Round: 2006 FT, First
Content: Market sizing, qualitative and quantitative

CASE QUESTION

Our client is a manufacturer of equipment for eye surgery. Specifically, the machines measure
deficiency in eyes, and the company also produces lasers for post-operational procedures and
adjustments. They don’t actually make the lasers or devices used for Lasik – rather, they are
complementary products for this procedure.

The global market for these devices is growing, but at a declining rate. As a result, the client
wants to get into a higher growth area, so they are looking at acquiring a company that makes
inter-ocular devices. These devices are used instead of Lasik but with similar effectiveness, and
they are used for two major categories of patients:

1 – patients with cataracts


2 – refractive surgery (to correct near or far sightedness)

How would you approach this opportunity? What would you look at?

Additional information provided during questioning

Interviewee asked about the specifics of what the machines were so as to consider synergies
between the two companies and product offerings. There would be significant synergies and that
is a component of answering the case.

INTERVIEWER BRIEFING
Recommended Approach
Given that we’re looking at a company with an existing product line that is exploring moving
into a related product line, we need to understand any links between the two. It is vital that the
interviewee demonstrates his acknowledgement of the risks of cannibalization and the benefits of
synergy between the old and new lines. Also key is to show an understanding of some of the
basics of M&A. High level, a framework looking into internal factors of both the target and
acquirer (such as culture, finances, and the synergies there might be between the two), external
factors such as market trends and competition, and customer factors (both doctors and patient
segments) is necessary. The interviewee should also remember the significance of the valuation
of the target- is it worth the asking price.

This case is fairly simple if you hit the numbers – take your time and get them right. The overall
framework was very helpful as interviewee was able to reference it multiple times during a fairly
Columbia Business School
Case Book 2005

48
focused case discussion. The key is identifying that there will be different types of customers for
each offering, so suggesting IDing customer segments up front seemed to be a major plus.

Key Facts
• US population is roughly 300M
o 75% of the US population over 65 has cataracts
o US population is evenly distributed over 80 years, the same number of people are
each age
o When someone turns 65, they have a 75% chance of getting cataracts, and if they
don’t get it immediately they will never get it
o 1/3 of the population is near-sighted and ¼ of the population is far-sighted->
175M people need vision correction of some kind
• There are government caps on pricing for cataracts surgery and that there is substantial
competition from major national players.
• The refractive market is still very fragmented and growing rapidly – 1.5M surgeries/year
will grow to 3-4M as procedures become safer. Also, the patient pays 10x as much for
refractive surgery as a cataracts patient would pay.

EXAMPLE DIALOGUE

Interviewee: I would look into internal factors of both the target and acquirer (such as culture,
finances, and the synergies there might be between the two), external factors such as market
trends and competition, and customer factors (both doctors and patient segments). Related to all
of these would be the valuation placed on the company. If we could, I’d like to start with drilling
down on the customers.

Interviewer: OK, I like that. So let’s talk about the cataracts patients. If I were to tell you that
75% of the US population over 65 has cataracts, how many potential patients are we talking
about?

Interviewee: Well I know that 12% of the population is 65+, so let’s call that 10% for
simplicity. 10% of 300 million is 30 million. 75% of that is 22.5 million. But some of those
people might already have had surgery.

Interviewer: Good point. And it gets a little dicey because the segment would be skewed
towards 65. So here is a simplifying assumption – assume the US population is evenly
distributed over 80 years, the same number of people are each age. When someone turns 65,
they have a 75% chance of getting cataracts, and if they don’t get it immediately they will never
get it. What’s the market size thinking this way?

Interviewee: OK, so we have 300 million people over 80 years. That’s 3.75 million people in
each year age bucket. So it would be 3.75 million people turning 65 every year. If 75% of them
get cataracts, that’s… roughly 2.9 people a year. Plus some percentage of the population already
over 65, I’m thinking right around 3 million people a year.

Columbia Business School


Case Book 2005

49
Interviewer: Does that make sense?

Interviewee: I don’t know a lot about cataracts, but it seems to. I’m not sure all of those people
currently get laser eye surgery currently, though.

Interviewer: Right. OK, now let’s turn our attention to the refractive surgery market. So your
research tells you that 1/3 of the population is near-sighted and ¼ of the population is far-
sighted. Assume that those numbers already include those who’ve had their vision corrected.
How many people are we talking about for the potential market size?

Interviewer: (Works out 4/12 + 3/12 = 7/12; 7/12 * 300 million people total = 175 [shortcut:
1/4 of 100 = 25, * 7 = 175]) 175 million people.

Interviewer: Right. And it turns out that it translates to 1.5 million people a year actually
getting refractive surgery. So if we acquire this company and can position it as a cataracts
provider or a refractive surgery provider, which should we position it as? (note: the machinery
would be slightly different, enough so that it would be beneficial to go after one market or the
other).

Interviewee: OK, so I know that the cataracts market is around 3 million a year and the
refractive market is 1.5 million a year. But I don’t know anything about profitability so I can’t
really say. Can you tell me a bit about the markets?

Interviewer: What do you want to know?

(Key information: There are government caps on pricing for cataracts surgery and that there is
substantial competition from major national players. On the other hand, the refractive market is
still very fragmented and growing rapidly – 1.5 will grow to 3-4 as procedures become safer.
Also, the patient pays 10x as much for refractive surgery as a cataracts patient would pay).

Interviewee: So based on what we just discussed I’d like to target the refractive market.

Interviewer: Is there anything else you would want to know before making a decision to buy the
company?

Interviewee: I’d need to know more about the financials to give a clear answer. I’d also need to
better understand the synergies and how they’d be perceived in the market. However, it looks
promising given our examination of the market segments.

Interviewer: Excellent.

Columbia Business School


Case Book 2005

50
Case 17: CPG Company
OVERVIEW
Firm: BCG
Round: 2006 FT, Final
Content: Qualitative and quantitative

CASE QUESTION

A large food and beverage consumer packaged goods company (CPG) is the client. They have a
40% market share in traditional CPG channels, like large format grocery. They have a 17%
market share in food service. They hired us to help them grow that food service business. How
would you structure that?

INTERVIEWER BRIEFING
Recommended Approach
This is really a 3 Cs case. If you use the 3 C’s, develop questions around them that are relevant
to the specific question, and probe the interviewer for new, relevant information, then you can
crack it. This is not a numbers case. Some knowledge of marketing and consumer packaged
goods would be helpful. Later in the case, using the 4 Ps to help think through the questions
about how specifically to grow market share would be helpful in generating ideas.

Key Facts
There’s a lot of additional information in this case. The interviewee should probe for this
information because it’s essential for solving the case.

- The client has one major competitor in beverage, but no major competitor in their foods
portfolio. The beverage portfolio consists of sodas, waters, juices, teas, health drinks,
and new specialty bottled drinks—no alcoholic beverages. The food portfolio consists
primarily of snack foods—for throughout the day (ie, breakfast, lunch, and later). The
client’s main competitor is a major beverage company—but they don’t have any sort of
food portfolio.

- The client’s share is 17% across the food service market. But within this market, the
market share for the client varies in the different segments and subsegments. There are
three primary segments in the food service market: 1) cafeteria/ workplace food service,
2) restaurants (which is comprised of fast food, slow fast food/ alternative fast food, and
other), 3) entertainment/ theme park venues, and 4) misc. Amongst these segments,
restaurants is definitely the largest—70% of the market.

- Within restaurants, there are a few subsegments: 1) fast food (Wendy’s, BK,
McDonald’s), 2) slow fast food/ alternative fast food (Chipotle, Au Bon Pain, Cosi,
California Pizza Kitchen Express, Wolfgang Puck’s), and 3) Misc. (corner deli, mom and
Columbia Business School
Case Book 2005

59
pop, etc.). The fast food segment is dominated by our major competitor, who has a 70%
market share in the segment. Slow fast food/ alternative fast food is a smaller segment,
but it is growing at 30% a year, as a part of a trend of eating healthier.

EXAMPLE DIALOGUE

Interviewee: I’d like to make sure that I understand the case. So, our client is a traditional
CPG, with a beverage and foods portfolio. Can you tell me a bit more about that portfolio? I’m
assuming that they have sodas and juices? Do they have other types of drinks?

Interviewer: Sure. The beverage portfolio consists of sodas, waters, juices, teas, health drinks,
and new specialty bottled drinks—no alcoholic beverages.

Interviewee: What about their food portfolio? What is that comprised of?

Interviewer: The food portfolio consists primarily of snack foods—for throughout the day (ie,
breakfast, lunch, and later). That’s one thing that makes us distinctive from our main
competitor—we have both a food and beverage portfolio. Our main competitor only has
beverages.

Interviewee: Interesting. So, they have a pretty comprehensive portfolio. Can I have a minute
to structure my thoughts?

Interviewer: Sure. Take your time.

(Now the interviewee should take a few minutes to structure the case. This will require more
than just laying out a framework but also identifying key questions that will help them drive to an
answer.)

Interviewee: I would like to know about three main things: 1) customers, 2) our company and
capabilities, and 3) the competition and how we can differentiate between them. In those three
main categories of information, there are a few key questions that I’d like to answer. This is how
I might structure it:

Columbia Business School


Case Book 2005

60
•Who are the potential customers in the food service
market?
¾Cafeteria/ Workplace food service
Customers
¾Restaurants—fast food, slow fast food, other?
¾Entertainment venues—movie theaters, theme parks
¾Other?

•What’s in our portfolio?


¾Food portfolio
Company ¾Beverage portfolio
•Do we have any capabilities that are specific to us?

•Who are our competitors?


•What do we have that differentiates us from our
Competitors competition?
•Where are our competitors strong? Where are we
stronger than them? (Mkt. shares?)

Interviewer: That’s good. What would you like to start with first?

Interviewee: Well, we already talked about our company and portfolio a bit; is there anything
else that I should know about our company. Like, do we have any capabilities that our
competition doesn’t have?

Interviewer: Not really.

Interviewee: Well, I’d like to know a little more about customers in the market. I don’t know
much about the food service market, but I imagine that there’s a lot that falls into the group of
customers.

Interviewer: You’re right. You’ve listed a few. Why don’t we discuss those.

Interviewee: Sure. I imagine that food service includes things like cafeterias, both at schools
and universities. And, I know that the company that I worked at before school had a cafeteria—
maybe that as well?

Interviewer: Yep. What else do you have?

Interviewee: Well, then, there’s movie theaters and theme parks—maybe places like bowling
alleys?

Interviewer: Sure. What else?

Columbia Business School


Case Book 2005

61
Interviewee: Yes, there’s also restaurants. And then, there might be customers that I’m
forgetting. Is there anything else?

Interviewer: Sure, there’s probably some sort of miscellaneous group—but you’ve covered the
main ones. So, of those main three, what do you think the sizes of the segments are?

(The Interviewer is testing the interviewee’s business judgment/ common sense in asking for the
interviewee’s sense of the sizes of the segments.)

Interviewee: I think that miscellaneous is pretty small. And then, the entertainment one is also
pretty small and stable. But the cafeteria/ workplace one, I imagine that that’s a little bigger—
and maybe it’s growing. And then, the restaurant one, I bet that’s the largest. Though, I’m not
sure how fast it’s growing.

Interviewer: You’re basically right. The restaurant segment is definitely the largest—it’s about
70% of the food service market.

Interviewee: If I think back to the original question of helping them grow their share in the food
service segment, I think that I would focus on the restaurant segment. Can you tell me a bit more
about that segment of the food service market?

Interviewer: Well, what do you think about that segment?

(Part of what the Interviewer is testing here is the interviewee’s ability to use common sense to
lead the case and see what they can deduce on their own.)

Interviewee: Well, I imagine that restaurants includes a few subsegments. For a CPG, high-end
restaurants are out. But in the low-end, there’s probably fast food, delis and mom and pop shops,
and miscellaneous.

Interviewer: That’s right. There’s one big, new one that you’re missing. It’s referred to as
“slow fast food.” It’s new chains with more made-to-order, healthier food. It’s restaurants like
Chipotle, Boston Market, Cosi, etc. It’s part of people’s need to eat healthier.

Interviewee: Interesting. I imagine that this new segment is growing a lot faster than the other
segments. But it’s small. If I think about the original question of the case, if I was going to try to
grow market share, I might try to grow it throughout the restaurant segments. However, before I
make a recommendation, I’d like to know a bit more about the competition—we haven’t talked
much about that.

Interviewer: You’re right. Well, the competition is pretty fierce in the fast food subsegment. E
have one main competitor. They only make beverages, and they have a 70% market share in fast
food.

Columbia Business School


Case Book 2005

62
Interviewee: Wow. Well, given that, I would focus on the other two segments: this new slow,
fast food segment and the mom and pop segment.

Interviewer: Great. What would you do to try to focus on them and grow your market share
with them?

(The interviewee might want to take a few seconds to think about the answer to this question.
It’s kind of the crux of the answer to the case. Since it’s a marketing question—how to grow
market share—the 4 Ps come in as a handy framework to use to think through the case.)

•Are there customers in the restaurant segment that we’re


not taking care of right now? New sub-segments?
Placement

•Can we offer specialty products/ tailor-made products to


restaurants?
Product •Can we offer new product packaging that is specific to our
restaurant customers?

•Should we compete on price? Do we want to compete


on price? (Price competition could grow our unit share,
Price but hurt of $ volume market share—and our profits
overall)

•Can we cross-promote with our restaurant customers?


Promotion •Will cross-promotion help grow restaurant customers’
loyalty to us?

Interviewee: Well, there are a few things that we could to grow our share with these customers.
In terms of price, well, we could change our price, but I don’t think that we want to compete on
price. That could help us grow our unit volume share, but it might hurt our dollar volume share
and definitely would hurt our profits. In terms of placement/ distribution, is there a way that we
can grow our share by distributing to new customers/ locations?

Interviewer: Maybe. But we already distribute to a majority of the market. What else could you
do to grow share.

Interviewee: In terms of promotion, maybe they can create partnerships with certain customers
or customer chains and run cross promotion campaigns—like advertising campaigns, or coupons.
And in terms of product, maybe they can create customized products for their restaurant
customers? Like new packaging? Specialty sizes, etc?

Columbia Business School


Case Book 2005

63
Interviewer: Yep. That’s great. That’s a good way to grow share. But can you just quickly give
me some of the pros and cons of specialty or custom products?

(Here, the Interviewer is testing the interviewee’s ability to identify some business risks with new
ideas—testing business judgment and risk identification.)

Interviewee: Well, on the pros side, it would build a special relationship with the customer; it
might differentiate us from our competition. On the cons side, our profit margins would
definitely decrease on custom products. So, we might grow our share, but not our overall bottom
line.

Interviewer: That’s right. So, is there anything else—other than custom products or any of the
ideas that you mentioned—that can help us grow our share in the restaurant segment?

(Here, the Interviewer is looking for a specific answer. If the interviewee doesn’t get it, don’t
worry about it. You can still get the job without getting the answer to this one.)

Interviewee: Well, when I think about the information that I got early in the case, the only thing
that I have that really differentiates us from our competition is the fact that we have a food and
beverage portfolio. Maybe we can leverage the fact that we can distribute food products and
beverage products to restaurant customers to grow our market share?

Interviewer: Yes, that’s right. So, how would you summarize your recommendation?

Interviewee: Sure. The client wants to grow their market share in the food service segment.
The best way to do that would be to focus on the restaurant segment of the food service market,
because that’s the largest part of the market—nearly 70%. Within that, the client should focus
on the non-fast food segments, as those segments aren’t dominated by our major competitor and
are still up for grabs. And they’re growing faster than the traditional fast food segment. The best
ways to grow market share within those sub-segments—without hurting the bottom line—would
be to leverage the breadth of our portfolio, having both food and beverage products to offer
which our competition does not, and perhaps by making custom products, where it doesn’t hurt
our profit margins.

Columbia Business School


Case Book 2005

64
Case 18: Institutional Asset Manager Fees
OVERVIEW
Firm: BCG
Round: 2006 FT, Final
Content: Qualitative and quantitative

CASE QUESTION
BCG has been hired by an institutional asset manager client. The client is very profitable and
doing well. However, they know that they are leaving money on the table: they know that they
have a lower average percent fee of assets under management than their competition—lower
than the industry average. They’ve hired BCG to tell them why that is, but in the end we told
them more than that.

INTERVIEWER BRIEFING
Recommended Approach
This case dives right into data and question and answer. There’s a question that the client asked,
but the interviewer hinted up front in the case question that he/she wants the interviewee to do
more than just answer the client’s question. This is a good case for practicing looking at data
and taking the case interview past answering the question and more towards “fixing” the
business problem.

The interviewer should expect the interviewee to drive the case, but should provide the data
(exhibits 1 &2) up-front pretty quickly and make sure that the interviewee understands the
industry and how the fee structure works very early on.

Key Facts
- Institutional asset managers’ revenues are based on fees from clients. Clients are charged
a fee, which is structured by some percentage of assets under management. You can
assume 5% if it makes the discussion easier—though, the percent fee never comes up.

- Institutional asset management companies are companies like Fidelity. They can serve
any sort of large, investing clients—pension funds, CalPers, TIA-CREF, hospitals,
foundations, universities, etc. They do have retail division that serve individuals with
their 401Ks and mutual funds, b ut this case focuses on the institutional investor side of
the business.

EXAMPLE DIALOGUE
Interviewer: So, first do you have any questions?

Interviewee: Sure, I want to make sure that I understand how the client’s business works. I
don’t know too much about institutional investors or asset managers; so, I’d like to just make
sure that I understand the business model before we dive into the problem.
Columbia Business School
Case Book 2005

65
Interviewer: Sure, that makes sense. What can I tell you?

Interviewee: I imagine that an institutional asset manager is a company like a Fidelity, or


Capital Group Companies.

Interviewer: That’s right.

Interviewee: And I bet that their clients can range from anyone—you and me investing our
savings in a mutual fund to large investors like pension funds.

Interviewer: Yep, that’s right. Pension funds like TIA-CREF and CalPers, or foundations and
universities are some of the bigger clients. We’re going to focus on that part of the business—
the institutional investor part of the business, not on you and me and our individual 401-Ks.

Interviewee: Great. And you said that they generate revenues from fees which are based on a
percent of assets under management?

Interviewer: Right. What’s your sense of how that percent might change?

Interviewee: My guess is that larger clients—clients with larger amounts of assets under
management—might have a lower percent fee charged to tem, because they still generate more
revenues with a smaller percent fee.

Interviewer: That’s right. In fact, I have an exhibit that shows you what that looks like.
Exhibit 1
Revenue Structure for Institutional Asset Management

Fee on
Assets
Under
Management
(%)

$ Under
Management

It’s a stepped function. What would you image the cost structure of different size clients is like?

Columbia Business School


Case Book 2005

66
Interviewee: Huh. Well, I don’t know the business to well, but I imagine that it costs about the
same to service a large client as it does a small one.

Interviewer: Yes, that’s correct. So, what does that tell you?

Interviewee: That tells me that large clients—in terms of assets under management—are highly
profitable because they generate more in revenues—even with smaller percentage fees—at
basically the same cost as a small client. Thinking back to the original question of the case: why
does the client have a lower average fee than the rest of the industry, I now want to know more
about the fees that they’re charging different customers.

(If the interviewee did not get these points, it’s not a big deal. But it is important to show them
how the fees are structured and explain that larger clients generate more revenues and cost the
same as any other client—therefore, they’re highly profitable clients.)

Interviewer: That’s right on track. Let me show you some data that we collected on the percent
fees—and discounts—that they’re giving to their customers.

Exhibit 2
Discounts by Deal

100

90

80

70
%
Discount 60

50

40

30

20

10

10 100 1,000 10,000

$ Under Management
(in MN)

What does this data tell you?

Interviewee: Well, it’s not what I would expect. It looks like they’re giving some of the highest
discounts on their fees to their mid-sized clients, not to their largest ones. This may be keeping
the profitability high on their large clients, but it’s probably destroying margins on their mid-

Columbia Business School


Case Book 2005

67
sized clients, and bringing down their overall fee average. I think this shows the answer to the
client’s question.

Interviewer: Sure. Now here’s another question: if you were the client, what would you want to
see here?

(Here, the interviewee should generate a few ideas. A bunch are listed—if they get one or two,
that’s great, and you can move on. Also, it might help the interviewee to give them the exhibit so
that they can draw on it—if that makes them more comfortable or helps them at all.)

Interviewee: Well, I would want to see a few things. First, I would want to be giving the
biggest discounts, or the most discounts, to my larger clients. Here, it looks like they’re giving
the biggest discounts to the mid-sized clients. Second, there doesn’t seem to be any sort of cap
on the amount of discount that can be given. They’re discounting up to 90% in some cases—I
would think that that would destroy all profit margins. I would expect for there to be a cap
around 30% or something. Last, it all seems pretty random where the discounts are given. I
wonder whether the clusters of discounts represent a certain salesman/ account executive.

Interviewer: That’s right. So, what are the key steps that you need to take to fix the problem?

Interviewee: Well, first, you have to diagnose why the discounts are the way they are right now.
So, look-up who discounted what—see whether it’s saleperson-driven, or if it’s driven by the
type of client. Then, you would need to monitor the discounting and keep track of it. Maybe
you can incent the salespeople not to give discounts?

Interviewer: Yep. That’s right: monitoring and tracking, and then put in whatever systems are
necessary to get the company’s average fee up to at least the industry average. Great. That’s it.

Columbia Business School


Case Book 2005

68
CASE 2:
DRYWALL COMPETITIVE THREAT
Firm Style Interview Round
BCG / Bain 1/2

Case Question:
Our client is a dry wall manufacturer and is concerned about a new player coming into the
market. The new player is perceived to be a low price player and our client is considering
lowering its price by 10%. The client seeks our advice regarding its options.

Clarifying Questions & Answers


Provide the following answers only if the interviewee asks the corresponding questions.
Question Answer

How many products does the client produce? One

Is the new competitor’s product similar to ours? For practical purposes we can assume that our
product is similar to the new competitor’s
product.
How are “dry-wall” products differentiated? For our analysis assume all the “dry-wall”
products in the market are similar.
DRYWALL COMPETITIVE THREAT

Framework / Structure
This is a strategy/competitive type case. A comprehensive plan should
include the following areas:
• Client’s impact of reducing price by 10%
• Market Structure (# of competitors and their share)
• Customer:
• Who Are They?
• Customer Preferences
• Price Sensitivity
• New Competitor:
• Size
• Strengths
• Weaknesses
• Our Client’s Position:
• Market Position
• Duration in the Market
• Brand Strength
DRYWALL COMPETITIVE THREAT

Question 1: After the candidate lays out his or her plan, ask the
candidate to analyze the impact of 10% price reduction on our client’s
bottom line. Give the following information when asked:
a. Number of units sold = 100,000
b. Selling Price Per Unit = $10
c. Variable Cost Per Unit = $3
d. Allocated Fixed Cost Per Unit = $2

Key Assumption:
If the candidate asks regarding the volume change with the price cut,
state that the volume will remain same with the price cut. If the
candidate does not ask about the volume change it is expected that
the candidate explicitly state the assumption before proceeding to the
calculation.

Expected Calculation:
• Total Fixed Cost = $100,000 * $2 = $200,000
• This is a key insight: the allocated fixed cost needs to be converted
into total fixed cost.
DRYWALL COMPETITIVE THREAT

Original Case:
• Revenue = $10 * 100,000 = $1,000,000
• Variable Cost = $3* 100,000 = $300,000
• Contribution Margin = $1,000,000 - $300,000 = $700,000
• Profit = $700,000 - $200,000 = $500,000

Price Cut Scenario:


• New Price = $9
• Revenue = $9 * 100,000 = $900,000
• Variable Cost = $3 * 100,000 = $300,000
• Contribution Margin = $900,000 - $300,000 = $600,000
• New Profit = $600,000 - $200,000 = $400,000

Impact On Bottom Line:


[(New Profit – Old Profit)/(Old Profit)]
Impact on bottom line = ($400,000 - $500,000)/$500,000 = -$100,000/$500,000 = -
20%

The candidate is expected to state that the impact on the bottom line is 20%
and thus is magnified compared to the 10% price cut.
DRYWALL COMPETITIVE THREAT

Question 2: Now ask the candidate if our client maintains the price how
much volume it potentially needs to lose to make it equivalent to
cutting the price by 10%.

If the candidate takes the initiative and explores this question then it needs
to be noted and encouraged (this differentiates a very strong candidate
from others)

Expected Analysis: Let the new volume of units the client sells be “x”, and if
the client sells “x” units @$10:
• Revenue = 10*x Variable Cost = 3*x Fixed Cost = $200,000
• Profit = 10x -3x - $200,000
A classic mistake is to take the contribution per unit as $5 (effectively
treating the allocated fixed cost as a variable cost too) and using $5*x as
the profit.
DRYWALL COMPETITIVE THREAT

So for this option to be equivalent to cutting the price by 10% the


equation would be
• 10x – 3x – 200,000 = 400,000
• 7x = 600,000
• x = 600,000/7 = 85,700 (approx)
Loss in market share = (85,700-100,000)/100,000 = 14.3%

It is expected that the candidate explains what this 14.3% number


represents; expected answer
would be that the 14.3% is the maximum market share our client can
afford to lose before the option of cutting price by 10% appears to be
a better choice.

A candidate can differentiate himself or herself by stating that the key


is to estimate how much the client can potentially lose if it does not
reduce price by 10%.
DRYWALL COMPETITIVE THREAT

Question 3: Now ask the candidate that if our client needs to make a
choice between these two options how you would help our client make
the choice.

Now the candidate is expected to make use of his or her plan to help make a
decision. If candidate asks the following questions provide the following
information:
• What is the market structure? (In terms of number of competitors and their share)
• Show Exhibit 1, with the shares
• What is size of the competitor?
• They are a small to mid size player
• Who are our customers?
• They are primarily contractors/small business owners. They buy directly from us.
• What are our customer’s preferences or how price sensitive they are?
• Customers usually buy based on relationships and they build preference on a
particular brand. We do not have any information regarding price sensitivity.
DRYWALL COMPETITIVE THREAT

• How do we sell to our customers?


• We sell based on their orders. There are no long term contracts.
• What are the strengths of the new competitor?
• Not much information, they are a new player in the market. They have
presence in other market our client does not compete in.
• What are the weaknesses of the new competitor?
• Not much information, they are a new player in the market. They have
presence in other market our client does not compete in.
• What is our brand strength?
• As a number 3 player, we have strong brand presence and we have
been in the market for a significant period of time.
• How long we have been in the market?
• We have been in the market for a significant period of time.
• Is there much differentiation in the product?
• Not much, for practical purposes consider all products to be similar.
DRYWALL COMPETITIVE THREAT

Key Expected Insights


Based on the information provided the candidate is expected to
come up with at least the following insights:
• Dry wall market is a very fragmented, and we are a dominant
player

• Considering the size of the competitor, they would be one of the


small 30+1 players

• That would mean on an average the new competitors’ market


share would be around:
• 70/31 = 2.3% approx
• New Competitor assuming fair share draw would take 2.3% of our
volume, although one can argue that they may not take any
volume and compete with the other small/mid size 30 players
DRYWALL COMPETITIVE THREAT

Recommendation
The recommendation should include the following:
• The answer: Client should not reduce price, because this is not a serious
threat and cutting price in this market would lead to a price war where
everyone would lose.
• The number(s): Reducing the price by 10% would impact profit by 20%.
20% impact on profit is equivalent to losing 14.3% market share at the
current price levels and at most the client can lose 2.5% volume.
• Risks or considerations:
• We might underestimate the size or strength of the new competitor
(new competitor might be bought by a bigger player)
• One of the major competitors (A or B) may have a knee jerk reaction
and reduce price
• Next steps:
• To mitigate some of the risks our client should aggressively strengthen
its client base by providing additional services
• To create a barrier for the new competitor to poach our client’s
customers
• Client can enter into long term contracts with its customers
• Even if A or B reduces its price, our client should maintain price and
aggressively provide value added services to the customers.
DRYWALL COMPETITIVE THREAT

Exhibit 1
CASE 6:
HOSPITAL PROFITABILITY
Firm Style Interview Round
BCG 1

Case Question:
A hospital group has been experiencing growing pains and has asked our firm for help.

Clarifying Questions & Answers


Provide the following answers only if the interviewee asks the corresponding questions.
Question Answer

What do you mean by growing pains? The hospital group originally consisted of one
hospital and then it acquired 3 other hospitals,
and its profits are going down.
Any other change in organization? Yes the CEO of the main hospital changed; it
was originally an MD doctor now it is a business
experience MBA.
Any other changes or events impacting the None.
hospital group?
Any external market condition changing? No.
HOSPITAL PROFITABILITY

Framework / Structure
This is a profitability case with lots of twists and turns. The analysis may include,
but is not limited to, the following areas:
Phase 1
• Identify the root cause of profitability decline.
• If the candidate makes an assumption that the acquisition led to
profitability decline, steer candidate away by stating that the
acquired hospital did not experience any decline in profits.
• Key expected response/insight: So the main hospital is
experiencing profit decline
• Identify the root cause of the profitability decline in the main hospital
• Costs – No change
• Revenue – Decline
• Key expected response/insight: So the primary reason for profits to
decline is that the revenue for the main hospital is going down.
HOSPITAL PROFITABILITY
Explore the revenue decline
• If the interviewee asks the revenue stream for the hospital, this should be the
response:
• The hospital treats patients and mainly collects payments from the
insurance companies based on the service provided.
• Key facts to be provided to the interviewee if asked:
• Volume of patient change: No
• Any change in mix of services performed by the hospital (lower of number
of surgeries etc.): No
• Key Expected Insight: In some way the revenue per patient on average is
decreasing, which implies that the patient mix is changing in terms of
revenue. Which implies that the patient mix in terms of insurance they cover is
changing
• If patient asks to explain the insurance coverage, provide the following facts:
Primarily the patients can be categorized into the following categories:
• Private Insurance Covered Patients
• Public Insurance Covered Patients
• No Insurance Covered Patients (Usually homeless people)
• Payments from private insurance is greater than public insurance.
HOSPITAL PROFITABILITY

• So expect some insight from candidate:


• Key Insight Expected: Looks like then the patient mix for the hospital is
shifted, with less patients with private insurance coming to the hospital.
• On providing the insight, ask the candidate what does the candidate wants to
do with this information.
• Key Question Expected: What are the key drivers to getting people with
private insurance to the hospital.( Note to the interviewer: Guide the person
to explore the driver)
• For the key drivers that drive the patients with private insurance to the hospital is
referral by private physicians.

Phase II
• Interviewer should prompt the candidate to make some hypothesis with regard
to lower number of patients with private insurance coming to the hospital.
• The key expected hypothesis: Because of change of management the
perception of physicians may have changed –resulting in lower number of
referrals of patients with private insurance. Interviewer should guide the
candidate to make the hypothesis.
• Interviewer should then ask the candidate how he/she can test the hypothesis.
• Key expected answer: Survey of physicians who normally refer patients.
HOSPITAL PROFITABILITY

• Direct the candidate to the exhibits 1 and 2 for survey results


• Key Insight expected: Majority of doctors are indifferent about the hospital
which may not be good news when it comes to referrals. One of the major
issues plaguing the doctors is complex billing.
• Interviewer should then prompt the candidate as to what should be the next
course of action given some of the insights.
• Key Insight Expected: Candidate should try to highlight the need to strengthen
relationships with physicians and “complex billing” may be an opportunity.
• Interviewer should then highlight that the hospital has a outsourcing billing service
which it does not aggressively market to the physicians. Pose the question how will
judge whether this is a viable service for the physicians.
• Expected question from candidate: What is the cost of the service to the
physicians and the economics of the physicians office.
• Interviewer to provide the following data:
• Price of the service: $50,000
• Physician Annual Profit: $175,000
• Physician time: 75% on patient care, 15% on billing, 10% on administrative
overhead
Ask the candidate to provide a plan for the approach before calculation
HOSPITAL PROFITABILITY

• Key Insight expected:


• If physician outsources the billing to the hospital, then what benefit
does it derive.
• To derive the benefit, I assume that for 15% of the time the physician is
not involved in billing he/she can see patients.
• So the candidate should ask for the revenue number:
• ONLY IF THE CANDIDATE ASKS FOR REVENUE, PROVIDE IT, IF NOT TRY TO
GUIDE THE CANDIDATE TO THE REVENUE NUMBER.
• Revenue of the doctor’s office : $425,000
• The candidate should either clearly ask whether the physician incurs
additional cost when he or she adds 15% additional physician time
• Interviewer state that no additional cost is incurred.
• Now the interviewer should guide the candidate to perform the cost
benefit analysis: Refer to the calculations section
• Ask the candidate to calculate ROI, New Profit, Increase In profit %

See the next slide for calculations.


HOSPITAL PROFITABILITY

Calculations
$425,000
Revenue
$175,000
Profit
$250,000
Cost
Patient Care 75%
Billing 15%
Administrative 10%
Revenue earned if billing outsourced $85,000 (425000*15%/75%)

$50,000
Cost of Outsourcing
Return on Investment 70% (85000-50000)/50000

((425000 +85000)(Revenue) –
$210,000 (250000 + 50000)(Cost))
New Profit
(210000-175000)/(175000)
Increase In profit 20%
HOSPITAL PROFITABILITY

Case Performance Assessment


Strong Plan
• Candidate clarifies key terminology – “growing pain”
• Candidate quickly focuses on the profitability of the main hospital
• Candidate keeps exploring reasons for revenue decline for main hospital and takes
hints from the interviewer
• Candidate keeps asking questions or states assumptions aloud
• Candidate clearly establishes hypothesis linking revenue decline and change of
management
• Candidate clearly articulates the plan, asks for the revenue to calculate the cost
benefit analysis and gives some insight after calculations

Weak Plan
• Candidate keeps getting distracted and ask questions about acquisition
• Candidate starts focusing on cost instead of revenue decline
• Candidate fails to identify change in patient mix with regard to insurance
coverage
• Candidate fails to clearly state the hypothesis of lower referrals with physician
dissatisfaction with management change
• Candidate does not ask about the revenue when calculating cost benefit analysis
HOSPITAL PROFITABILITY

Recommendation
• The recommendation should include the following:
• The hospital is facing revenue decline because of lower number of
patients with private insurance which may be due to a change in
perception of private physicians after management change.
• To reverse the revenue decline the main hospital needs to
establish strong relationship with the physicians by aggressively
marketing the billing outsourcing service as it provides 70% return
on investment within one year and increases profits of physicians
by 20%.
HOSPITAL PROFITABILITY

Exhibits

Exhibit 1
HOSPITAL PROFITABILITY

Exhibit 2
CASE 8:
E-COMMERCE IN THE AIRLINE INDUSTRY
Firm Style Interview Round
BCG, Bain, A.T. Kearney 2
Case Question: The year is 1999, in the middle of the .Com-boom. An airline was
approached by a group of entrepreneurs. The entrepreneurs offered the airline to
invest $100MM in their software company for 20% equity, and also be the first customer
for the software they develop. The software is an e-market place where airliners and
spare parts suppliers could meet to improve the purchasing capabilities of the airlines
and to reduce the purchasing costs. Aircraft parts are obviously a hugely important
part of the airline business, as airlines struggle to keep planes in the air as much as
possible, minimize downtime and control costs. What should the airline do?

Clarifying Questions & Answers


Provide the following answers only if the interviewee asks the corresponding questions.
Question Answer

How does the airline currently purchase spare The airline currently has a legacy system. The
parts? system is not that suitable to what’s is actually
happening and therefore there is a lot of extra
phone calls and process work happening.
Cont’d on next slide
E-COMMERCE IN AIRLINE INDUSTRY

Clarifying Questions & Answers


Provide the following answers only if the interviewee asks the corresponding questions.
Question Answer

How much does a purchasing transaction cost to an $50 per transaction


airline? (supply this number rather easily, even if you are
not asked directly)
How many transactions does each airline perform per 500,000 for each of the airlines in the
year? states
How many airlines are there in the states? 20

How much will a transaction cost us in $6 payment for the software company
the new system? (try to supply only one of the numbers $9 internal cost (labor, systems, etc.)
and see if the interviewee asks for the other)
Any other cost a customer needs to pay to join? $20MM licensing fee

What is the cost structure for the software company? No variable costs. $10MM a year to
pay for corporate expenses.
Cont’d on next slide
E-COMMERCE IN AIRLINE INDUSTRY

Clarifying Questions & Answers


Provide the following answers only if the interviewee asks the corresponding questions.

Question Answer

Will the E-Market allow us an increased bargaining The aircraft industry in general, and the
power? spare parts industry specifically, are
highly regulated. To become an
approved FAA supplier costs money
and therefore the barrier for entry are
significant. The result is that each part
has one or two suppliers at the most
and hence the bargaining
power will not improve with the E-
Market.
E-COMMERCE IN AIRLINE INDUSTRY

Framework / Structure
This is a investment assessment case. The analysis may include, but is not
limited to, the following areas:
• Investment framework. Market and competitors.
• Company – availability of capital but also fit.
• Current situation – how is the airline currently manages the spare parts
sourcing.

Strong Plan
The candidate has de-coupled effectively both issues requiring decoupling
– One analysis is to be done from the perspective of an investor (who is the
customer here) and from the perspective of a customer. This is actually THE
most important issue here – decoupling the problems effectively. The
candidate also asked questions about the current situation and what
advantages will the system provide, and also talked about the market and
competitors.
Weak Plan
The candidate is thinking about the problem as one problem. It seems that
he will perform cash flow analysis for customer and investment together
and doesn’t seem to realize the “first customer” is not the same as
“customer”. In addition, candidate didn’t talk about the current situation
or the market at all.
E-COMMERCE IN AIRLINE INDUSTRY

Exhibit
Exhibit 1
Interviewee: Could you tell me more about the E-Market?
Interviewer: (draw the following picture on a piece of paper and talk a little bit
about it)

Expected Insight:
• For the customer to get value out of the system, many suppliers need to be
connected to the system.
• Do the suppliers need to pay anything to use the system?
Tell the candidate for this analysis assume suppliers pay nothing.
E-COMMERCE IN AIRLINE INDUSTRY

Calculations
The numbers are provided in the answers to questions. There are two
calculations here:
1. Calculating the cash flow for investment
• Best in class answer will utilize the cash flow framework (drawing years
from 0 to 10 and writing beneath the cash flow every year).
• –100MM for year 0 (provided in the initial question)
• Calculating the resulting cash flow
• Revenues every year - $6 (payment per transaction) * 500,000
(transactions a year) * 20 (airlines) = $60MM a year.
• – 10MM a year (fixed costs for software company – case fact)
• = $50MM.
• We only own 20% of that number = $10MM, the resulting cash flow.
E-COMMERCE IN AIRLINE INDUSTRY

• The interviewee should now say “so what” and talk about one or
both of:
• There will be $10MM annual return for $100MM investment. Without
discounting, we know it is a 10 years till breakeven. In the software
industry that’s unacceptable
• $10MM till perpetuity. Discount rate for the software industry is high.
Let’s assume 20%. $10MM/20% = $50MM.
• So what? Meaning that the moment you investment the $100MM it
is worth $50MM. this is a bad investment.
• A interviewee can be confused and try and use the airline
discount rate (which is extremely low). This is obviously a mistake as
discount rate is directly correlated to risk.
E-COMMERCE IN AIRLINE INDUSTRY

2. Calculating the cash flow for customer


• $-20MM investment (licensing fee).
• Revenue stream (cost savings)
• $50 transactions will only cost $15 (=$9+$6).
• Savings of $35 per transaction
• 500,000 transactions.
• $35*500,000 = $17.5MM savings a year.
• So what?
• The investment in the software for the customer could be
extremely good investment as it is returned in less than a year and
a half.
• However, first customer can be something else all together as
there are risks than can decrease the returns:
• Not all suppliers will join right away or at all.
• The software will have bugs in the first few months /years.
• The software company can go bankrupt, taking our licensing
fee away and never give any returns.
E-COMMERCE IN AIRLINE INDUSTRY

Recommendation
• The recommendation should include the following:
• The answer – Candidate suggests that investment in the company
is not a good option as it takes 10 years to recoup investment
(without discount rate). Being a customer looks good, though not
necessarily first customer.
• The number(s) – NA
• Risks or considerations –
• Joining as a first customer will be considered if the client will be
compensated for the risks.
• After few airlines will join the system, and the software proven
itself, the client should definitely join as it will save costs.
• Next steps – NA
E-COMMERCE IN AIRLINE INDUSTRY

Questions to Further Challenge the Interviewee


The following question tests creativity further:
• Let’s assume there is another feature in the software that allows airlines to view
other airlines‟ inventory by location and part. So, for example, if I’m southwest and I
have a plane down in NYC. I contact the supplier and get an answer that I’ll have
the part in three days. I can look into American Airlines inventory and see if they
have the part in NYC. If they do, I can buy it from them at cost and ask the supplier
to send the part their way. Is it a good feature?
• A full answer (push interviewee till he reaches all conclusions or gives up):
• The feature is great for the industry, as the entire industry will be able to reduce
downtime and share the value in its companies‟ bottom line and with the
customers.
• However, the risk is that the feature actually encourages the airlines to carry no
inventory (why should I carry inventory when everyone shares the benefits?)
• A solution to mitigate the risks through the system is to create a system of
checks and balances and create a part exchange account for every airline.
So every time an airline takes a part from another airline then the airline's part
exchange account is reduced by the part cost. Every time an airline provides a
part to another, its parts exchange account is increased by the part cost. If we
cap the part exchange account to say minimum of -$5MM, then we solved the
problem.
• This will actually encourage better inventory management throughout the
industry as airlines will not want to stay away of this program as it provide huge
cost benefits.
CASE 10:
NEW RUBBER PLANT INVESTMENT
Firm Style Interview Round
BCG, Bain, A.T. Kearney, Deloitte 1

Case Question:
The federal government of a country in certain part of the world is investigating whether to
restart a rubber factory in the western part of the country. The factory was operational in
past but has not been used for 7 years. The plant was closed due to terrorism in the area
which has now come down significantly though there are still issues and skirmishes reported
in the area. If rejuvenated the plant may become a target for the rebels. All the equipment
is considered useable but the government still estimates to spend $12M to rejuvenate the
plant which would enable the plant to produce up to 10M lbs of rubber per month. The
demand of rubber worldwide is strong but rubber must be transported to an export port via
trains; up to 2 trains per day can used for this.
Clarifying Questions & Answers
Provide the following answers only if the interviewee asks the corresponding questions.
Question Answer

What are the raw materials? Need gum resin. 3lb of resin after processing
results in 1lb of rubber
Where are the resins coming from? They need to be transported from the capital.
Up to 4 trains can be used for the same (This is
a key question, a candidate not asking the
question misses out a key element in the case)
NEW RUBBER PLANT INVESTMENT

Recommendation
The recommendation should include the following:
• The answer – Go ahead with investment in the plan as it seems
highly profitable
• The number(s) – With production at 5M lbs of rubber we make a
profit of $12M a year. Production is limited by transportation, an
area that can be looked at and addressed. This should further
increase our profits in future.
• Risks or considerations – We have highlighted a lot of the risks, key
is that the government takes steps to mitigate the risks , the
government can take some steps based on our analysis of the
mitigation. Some of the steps could mean a long term
investment.
• Next steps – Assess how plant can be staffed, whether
transportation bottleneck can be alleviated, the level of terrorism
threat and steps to mitigate.
CASE 13:
CAPITAL INVESTMENT FOR UTILITY
Firm Style Interview Round
Booz, BCG 1

Case Question:
A major East Coast, vertically integrated and regulated electric utility has
received a permit to build its first nuclear power plant. It wants to know if this
will be a good investment and possible risks associated with this venture.

Clarifying Questions & Answers


Provide the following answers only if the interviewee asks the corresponding questions.
Question Answer

Can you describe the structure of the A vertically integrated utility owns the entire
company? What does “vertically integrated” value chain – generation, transmission and
and “regulated” mean? distribution. It is somewhat of a monopoly and
is overseen by state regulators.
What is the competition like? There is virtually no competition because it is a
regulated industry.
Cont’d on next slide
CAPITAL INVESTMENT FOR UTILITY

Clarifying Questions & Answers


Provide the following answers only if the interviewee asks the corresponding questions.
Question Answer

What is the size of the plant? The new plant is expected to supply about
8,760 GWh (Gigawatt hours) per year and it is expected
to be utilized for 30 years.
What is the current demand? The current annual demand in the utility’s service area is
17,000 GWh. For this case assume this demand will
remain constant in the near future.
What is the expected revenue on a $80/MWh (1000 MWh = 1 GWh)
variable basis?
What is the expected variable cost? $10/MWh

Is there any public opposition? No

Why is the utility building this new Great question. Currently the company has about 9,000
plant? GWh of supply that comes from old, dirty and inefficient
coal fired power plants. It plans to retire them in the
near future.
Cont’d on next slide
CAPITAL INVESTMENT FOR UTILITY

Clarifying Questions & Answers


Provide the following answers only if the interviewee asks the corresponding questions.
Question Answer

Who will build the plant? The utility plans to outsource the building of the nuclear
plant and the price quote is about $7 Billion
What is the market cap of our client? Top question. The market cap of this company is $6
Billion. The candidate should note that the fixed cost of
the plant is more than the entire company is worth.
CAPITAL INVESTMENT FOR UTILITY

Framework / Structure
This is a cost-benefit analysis case along with elements of assessing risk
associated with developing a new technology.
• Understanding the company structure and industry:
• This is a regulated industry and hence no competition exists.
• The main barriers to any project are the cost, subsequent benefits and
most importantly the ability to assess the need for this project.
• Internal barriers can be significant especially with regard to operating
this new plant.
• Market information
• The candidate should quickly be able to calculate the contribution
margin generated from this venture.
• Calculating the estimated value of the fixed cost will give the
candidate enough information to figure out the breakeven period.
• Risks
• There are significant risks associated with this project including
financial and organizational. Utilization of the new plant is not an issue
because it will be used to capacity.
• There is a construction cost overrun risk as well.
• There is also the risk of possible regulatory changes.
CAPITAL INVESTMENT FOR UTILITY

Phase 1
Understand the cost of this new venture. When asked by the candidate, provide the
following information:
• The cost of the plant is estimated to be about $7 Billion. However the company’s
management believes that there are cost overrun risks involved. They are as
follows:
Cost of
Construction Type Quoted cost Risk of cost overrun overrun
Technology and $4 Billion 50% 800 M
engineering
Structure $2 Billion 50% 600 M
Other overhead $1 Billion 20% 250 M
Total $7 Billion

What is the expected value of the cost of the plant?

Expected value of the fixed cost:


$7 Billion + (50%*800M)+(50%*600M)+(20%*250M) = $7B +$400M+$300M+$50M
= $7.75 Billion
CAPITAL INVESTMENT FOR UTILITY

Phase 2
Evaluate the revenue streams and contribution margin. The capacity of the plant will be
8,760 GWh per year.

The price for each MWh (not GWh) is fixed at $80/MWh. The variable cost is $10/MWh.
Contribution = $80-$10 = $70/MWh which translates to $70,000/GWh
Annual contribution = 8,760 GWh *$70,000/GWh = $613.2 M
The breakeven point of the investment is $7.75B/$0.6132B = 12.6 years ~ 13 years

Phase 3
Qualitatively evaluate the financial and organizational risks.

There are some critical financial risks associated with this investment. The market cap
of the firm is $6Billion and this investment alone is estimated to be $7.75Billion. The
breakeven period for the plant is almost 13 years which is much longer than the
industry average breakeven period of 6-7 years. If the firm goes ahead with this
investment it will need to rely heavily on debt financing or more expensive equity. This
will likely prevent other capital investments in the near future.
Nuclear technology is complicated and one major risk is the shortage of personnel
with the necessary skills to run the plant. The company will have to spend a
considerable amount of time and money on training and hiring new personnel. Safety
and reliability are issues to consider as well.

The candidate should address most of these issues in evaluating the qualitative risks. If
they don’t, keep giving them clues and push them to consider the above risks.
CAPITAL INVESTMENT FOR UTILITY

Recommendation
The recommendation should include the following:
• The answer – At this point it does not make sense for the firm to go
ahead with this project.
• The number(s) – The expected cost of building the plant is $7.75B and
the breakeven period is 13 years.
• Risks or considerations – The firm should look closely at the financial
and organizational risks involved with this project.
• Next steps – The client should look carefully at its options and see if
they can go with a less expensive technology to replace their old
plants
CAPITAL INVESTMENT FOR UTILITY

Strong Recommendation
The client should not go ahead with this project. The expected value of the
project is $7.75B and the breakeven period is approximately 13 years, which
means that a significant amount of capital will be tied to this investment for
a very long time. With a market cap of $6B which is less than the cost of this
plant, the financial risk is quite high especially with the large amount of
capital needed to be raised. Finally, the organization will need to adjust to
the high demand on human resources with regard to running such a
complex technology as well as safety and reliability concerns. The client
should evaluate other generation technologies which are less expensive
with a shorter pay-off period.

Weak Recommendation
The company should not go ahead with this investment because it will be
very expensive and will take a long time to recuperate the costs. Also the
organization will have a tough time running this new plant because of the
complex technology.
CASE 15:
MINER’S DILEMMA
Firm Style Interview Round
BCG, Bain 1
Case Question:
Your client is in the mining industry. They have just purchased a mountain that has high
concentrations of a valuable metal ore. Unfortunately, the mountain is made up of an
unusually hard type of rock and none of the commercially available drills are able to
penetrate the mountain’s surface. Your client’s engineers have built a prototype of a
drill that could be used to extract the metal from the mountain. The client is not sure
about whether to manufacture the drill and has doubts regarding profitably mining at
the mountain. Help the client think through the issue.
Clarifying Questions & Answers
Provide the following answers only if the interviewee asks the corresponding questions.
Question Answer

Is there any way to mine without having to manufacture No


the drill?
Is it possible to manufacture the drill that the client has Yes, the client can manufacturer the
prototyped? drill in-house.
Is it possible to outsource the production of the drill to Yes, there is a supplier who is capable
one of our suppliers or other manufactures? and willing to manufacture the drill.
MINER’S DILEMMA

Framework / Structure
This is a cost-analysis and profitability case.
Phase I – Understand more about the client’s plan for the mine
• Can you tell me more about the useful life of the mine and the extraction rates the
client is hoping to pursue?
• The client plans to extract ore from the mine for 20 years from the date drilling
commences. After that the mine will be retired with no salvage value.
• Additionally the client is concerned about flooding the market and will cap
annual extraction as per the following schedule:
• Years 1 – 5: < 10% of total US production
• Years 6 – 10: < 15% of total US production
• Years 10 – 15: < 20% of total US production
• Years 15 – 20: < 25% of total US production
• For logistical reasons (transportation, etc.) they must also cap annual extraction
to 12,000 tons.
• What is the annual US production?
• The annual US production is 64,000 tons.
MINER’S DILEMMA

Phase II - Determine which of two drill options is more cost effective.


Manufacture Drills In-House
• What are the costs to the client if they decide to manufacture their own drills?
• There will be a $30MM fixed setup cost and a variable cost of $100,000 per drill.
• How many drills will the client need if they produce their own drills?
• Using their own design, each drill will be able to mine 500 tons of ore per year. b.
Each drill that the client produces will last 4 years at the given rate of mining.

Outsource Drill Production


• What are the costs to the client if they outsource production of drills?
• Each drill will cost the client $250,000 but there will be no setup cost.
• How many drills will the client need if they buy from a supplier?
• The manufacturer estimates that each drill of their design will be able to mine
400 tons of ore per year.
• The manufacturer estimates a 2.5 year lifespan for their drills at the given rate of
mining.

Phase III - Determine which option is more profitable.


• Can you tell me more about the ore? How much is it worth?
• For every ton of ore extracted the client will realize $325 in revenue.
• How much does the mine cost to operate?
• $1MM per year.
MINER’S DILEMMA

Strong Plan
The candidate has successfully identified both the issues in the case – drill cost
effectiveness and profitability of mining at the mountain. The candidate also asked
questions about the client’s mining plan and associated constraints.

Weak Plan
The candidate makes basic assumptions about the size of the mine and number of
drills typically needed. He starts working to a solution without considering the
outsourcing option.

Calculations
Total amount of ore to be mined over the life of the mountain:
• Years 1 – 5: 10% of 64,000 tons = 6,400 tons per year
• Years 6 – 10: 15% of 64,000 tons = 9,600 tons per year
• Years 11 – 15: 20% of 64,000 tons = 12,800 tons per year, but client is
constrained to a maximum of 12,000 tons per year
• Years 16 – 20: 25% of 64,000 tons = 16,000 tons per year, but client is
constrained to a maximum of 12,000 tons per year
• Total to be mined over 20 years: (6,400 x 5) + (9,600 x 5) + (12,000 x 10) =
200,000 tons
MINER’S DILEMMA
Number of drills needed if manufactured by client:
(200,000 tons) / (4 years per drill x 500 tons per drill per year) = 100 drills

Number of drills needed if outsourced:


(200,000 tons) / (2.5 years per drill x 400 tons per drill per year) = 200 drills

Total cost if client manufactures drills:


$30MM fixed cost + $100,000 variable cost per drill x 100 drills = $40MM

Total cost if client outsources drills:


$0 fixed cost + $250,000 variable cost per drill x 200 drills = $50MM

Total revenue from mining activities:


200,000 tons x $325 revenue per ton = $65MM

Profitability if client manufactures drills:


$65MM in revenue - $1MM operating cost per year x 20 years - $40M drill cost = $5MM

Profitability if client outsources drills:


$65MM in revenue - $1MM operating cost per year x 20 years - $50M drill cost = $-5MM
MINER’S DILEMMA

Recommendation
The recommendation should include the following:
• The client should manufacture their own drills to make the operation profitable.
Over the life of the project, outsourced drills cost $10MM more than drills
manufactured in-house.
• Risks or considerations
• There is a risk of cost overrun in setting up in-house manufacturing facility. It will
be important to secure the $30MM construction cost with the contractors.
• Additionally, our client may want to pursue the sale of drills to other mining
companies that own property with the same geologic makeup.

Questions to Further Challenge to Interviewee


• Do you think it may strain relations with the client’s drill supplier if they begin to
manufacture their own drills? What can the client do to mitigate this possibility?
• Do you think there might be a third solution? What about working with the supplier
to have them manufacture the drills that your client’s engineers have designed?
How could that be good for your client?
• Based on what you’ve calculated, what could change that would make
outsourcing more optimal?
CASE 20:
GAS LIQUEFACTION
Firm Style Interview Round
BCG 2

Case Question:
Our client is a U.S. regional power company. They currently use natural gas fired power plants to
produce electricity which is then transmitted via the existing power grid. Our client currently gets
their gas directly from a field located in the Gulf of Mexico, but they are concerned about the
capacity of this supplier going forward. They are considering investing in capabilities to allow them
to utilize liquefied natural gas.
Natural gas is a cleaner burning and relatively low cost fuel, however in gas form cannot be
transported over long distances. The gas can be liquefied essentially through a combination of
dropping the temperature and pressurizing the gas. The compression factor for LNG vs. normal
gas is 1/600th of the gas volume.
How would you go about analyzing this decision and what would you recommend.

Clarifying Questions & Answers


Provide the following answers only if the interviewee asks the corresponding questions.
Question • Answer

How do you • Extraction


utilize LNG? • Liquefaction
• Transport
• Gasification
GAS LIQUEFACTION

Additional Information to Give when Asked


• Our client currently just pays for the extraction costs. Cost of gas from their current
supplier is shown in the graph on the following page. Interviewer should be
directed to use a cost of $5 per mcf.
• Unit of measure is 1000 cubic feet: mcf.
• We are looking to utilize gas from a field in Nigeria.
• Extraction cost is $1/mcf
• Liquefaction cost is $1/mcf
• Transport- Interviewer should push back…ask ‘How would you go about estimating
transport costs?’
• Transport breakdown:
• Charter tanker cost: $60,000/day
• Speed: 10 mph
• Miles: 6000 miles
• (So total tanker cost per trip comes to $1.5 M)
• Tanker volume: 300,000 meters cubed.
• So per mcf of GAS you would have to run the following conversion:
• 300,000 meters cubed liquid X ((3 ft ^3)/(meter ^3)) X (1/1000) X (600 gas/1 liquid)
= 4,860,000.00
• Which is the total mcf of Gas per tanker trip.
• Transport cost per mcf = $3,000,000/4,860,000 = Approximately 0.31.
• Gasification is $1/mcf
GAS LIQUEFACTION

Recommendation
• Because of the compression factor, the transport costs for LNG
are relatively low and as such there are significant savings to
be had by switching to LNG. We would need to look at our
current volumes to see if this would justify investments in
gasification facilities. Because LNG is an already traded
product, we shouldn’t necessarily invest in our own
liquefaction capabilities, but treat it as a variable cost
included in what we pay LNG suppliers.
• Conclusion: This was a second round BCG case, given by a
principal. The main thing she seemed to be trying to test was
the ability to keep up with all of the conversions. I had done
two relatively low-quant cases earlier in the day and this was a
number crunching exercise.
0
1
2
4
5
6
7
8

3
Exhibit
1995
1996
1997
1998
1999
2000
2001
2002
2003
2004
Gas Prices

2005
2006
2007
2008
GAS LIQUEFACTION

Gas Prices
Case 12:
12: Mining in Brazil

BACKGROUND

Firm: BCG
Round: 2007 Summer, Second
Content: Qualitative and quantitative

CASE QUESTION

Our client is a US industrial conglomerate, with major investments in South America,


India and China. One of these investments is a mining operation in Brazil. At this mining
operation, our client produces only one metal, which is considered to be an international
commodity product. This metal has hundreds of applications. In Brazil there are only two
other producers.

The CEO has hired BCG to help identify new opportunities for this business as well as
understand the market dynamics. He wants to know whether he should divest the mining
business or invest in an additional facility. This afternoon, the BCG team is going to meet
with the CEO to discuss our initial hypotheses. We have been provided the following
information to assist us:

Local Players Plant capacity (tons) Cost/ton Sales


Local Export
Client (A) 600,000 $450 600,000 -
Competitor B 4,500,000 $420 2,800,000 1,200,000
Competitor C 3,200,000 $420 2,000,000 700,000

The world-wide production (supply) of this product is estimated to be 100,000,000 tons.


Furthermore, it is known that demand for this product is greater than current supply. As a
commodity, the international price (export price) is $450/ton and the local price is
$600/ton.

INTERVIEWER BRIEFING

Recommended approach: This is a typical BCG case. BCG often (i) uses real life cases,
(ii) presents available data upfront in a table of during the case question, and (iii) drives
the interview to be a conversation between the interviewer and the interviewee.
Therefore, be sure to engage the interviewer as you approach the case.

Management Consulting Association Case Book 2007


44
The interviewer was expecting the interviewee to first provide a structure/framework,
which would look at the big picture and then start hypothesizing. A strong candidate
would:
- Discuss market dynamics (local and international and supply/demand).
- Discuss the expected competitive response to any action, e.g., a price war.
- Summarize all findings in a presentation format.

Key facts: All given beforehand, but the following would help in the discussion:
o An efficient plant should have a 1,000,000 ton capacity (but, not all plants are
operating efficiently), however, from this information the interviewee should be
able to assume that therefore competitors are operating more than one plant each.
o Market grows with GDP.
o There is a strong demand for this product internationally.
o The competitors are probably located away from the coast, adding transportation
costs.

EXAMPLE DIALOGUE

Interviewee: [Summarize the case and work on a framework]. In this case it is important
to look at the competition (specifically, understand the different cost structure of the 3
producers), estimate the market demand and discuss the international trade environment.
We should also discuss the specifics of a metal commercialization.

Interviewer: OK. Where should we start?

Interviewee: Clearly, our client is running on full capacity, but its competitors appear to
have some idle capacity. Furthermore, I note from the data provided that the competitors’
cost structures allow them to sell in the international market while our client does not
currently export any of the products. Our client would experience zero margin if it was
to export since the international price is given at $450 which is equal to our client’s cost
of production. Nevertheless, the local price is considerably higher than the international
price, so the producers would rather sell as much as possible in the local market. Given
this information, I would hypothesize that there is not enough demand in the local
market.
o Competitors would prefer to sell in the local market ($600/ton) instead of export
($450/ton)
o Competitors are running with idle capacity, but we know that demand exceeds
current supply

Interviewer: That is correct. But what about the international market?

Management Consulting Association Case Book 2007


45
Interviewee: This is a good point. You did mention that there is a strong demand
worldwide. Now we have to find out why the competitors are not selling their full
capacity. We can think of many possible reasons. Geographical distance, transaction
costs, transportation costs, export taxes, etc.

Interviewer: They all make sense, but as we see competitors are able to export some
metal, right?

Interviewee: Competitors might have operations abroad so it makes easier to export to


their international facilities. Or they might produce part of their capacity close to harbors.
Considering that the international price is much lower than the local one, I would expect
some barriers for international trade.

Interviewer: That is correct, Brazil does have some taxes for foreign products and
producers struggle with transaction costs. Let’s look at the cost structure. Why do you
think there is a difference in costs?

Interviewee: I could consider geographic location, technology, economies of scale,


supply chain synergies, etc.

Interviewer: You mentioned some important elements. Our client has only one location
and the competitors have multiple plants, each in the countryside. Our client was the first
player in this market, having built its facilities 10 years earlier than its competitors. The
industry has realized that a plant must have a minimum of 1,000,000 capacity in order to
be efficient.

Interviewee: But our client is still making a lot of money, right?

Interviewer: Yes, enough to convince the CEO to invest in a new production facility.
This would be a $400M investment in year 0 for a capacity of 1,000,000 tons with a cost
of $420/ton. How would you evaluate this investment if this new production would be
traded in the international market? Would you recommend this investment?

Interviewee: Basically the margin will be $30/ton ($450-$420) * 1,000,000 equal to


$30M per year. By using a 10% discount rate this investment will generate $300M in
total. The $400M investment would not be worth it.

Interviewer: That is true. So, are you going to advise him not to make the investment?

Interviewee: I would advise him against it unless he is willing to engage in a price war.

Interviewer: That would be the minimum price he could go to turn this investment
profitable?

Management Consulting Association Case Book 2007


46
Interviewee: We should be cautious because a lower price would impact the current
profitability. The current margin is ($600 – $450) * 600,000 = $90M. For instance, if we
drop the price to $513/ton we would be losing $52.5M [($600-$513) * 600,000] from the
current 600,000 ton production but generating $93M [($513 - $420) * 1,000,000] from
the new production, a net gain of $40.5M. With the 10% discount the minimum profit
required from the new plant would be $40M ($400M*10%). Therefore, the minimum
price would be $513.

Interviewer: But if you drop the price to $513/ton, what would the competitors’ reaction
be? Would they also increase production? Remember, they have idle capacity and you
don’t.

Interviewee: That is correct, but the competitor has a lot more to lose with a price
reduction. In our client’s case we found out that it would lose money as the margin of the
current production drops. However, the client only sells 600,000 tons right now while the
competitors sell 4,800,000 tons combined. They would probably reduce their production
to avoid a higher price reduction.

Interviewer: Really? So you are recommending our client to invest $800M in a


2,000,000 tons capacity plant?

Interviewee: I haven’t done the math but I guess this would be too risky. I would
recommend our client to invest the $400M and see how the market reacts.

Interviewer: That is a fair recommendation; after all they will be playing a game with no
real expected result.

The interviewee should now expect to summarize the findings from this discussion for the
client, highlighting the approach and key recommendations.

THIS IS THE LAST PAGE


OF THE CASE BOOK

Management Consulting Association Case Book 2007


47
Case 25: Stew’s Connections
Market Entry BCG Round 2
Our client is a start-up with the ability to deliver broadband internet to commercial airlines. How
Prompt would you help them think about their offering?

About the case: This is a market entry case where candidate are required to evaluate the feasibility
of a new product in conjunction with the airline industry. The candidate should use a comprehensive
framework, walk the interviewer through it and be prepared for analytical detours throughout the flow
of the case.

The calculations represented here are only one approach and interviewees may take other
approaches, depending on the assumptions made. The interviewer should be mindful of this and
allow for flexibility.

Industry & Market Size Discussion: The interviewee should have mentioned this as a major bucket
in his/her framework.

Use the information below to provide guidance as necessary.


Guidance
Broadband for the airlines
There is general interest in broadband internet from the airline industry. The start up would have to
invest relatively little up front and would keep most of the revenues. They would charge the
customers on a per flight pricing model.

Size of the Market


Ask the candidate to estimate the market size and hand over Exhibit 1. Inform the candidate that
there are 3,000 planes. Full answer in chart below

Pricing
In order to finish the market size, the candidate should ask for the price per
flight. Hand out Exhibit 2 and ask candidate to set the price.
114
Case 25: Stew’s Connections
Market Entry BCG Round 2
Class First Coach
Seats/Plane 20 180
Load Factor .75 .75
Full seats/plane 15 135
Analysis Biz Travelers 100% 30%
Laptop users/plane 15 40.5
Total laptop users/plane 55.5
3000 planes x 2000 legs/plane x ~50 laptop users/plane = 300,000,000 approximate annual
potential user-legs.

115
Case 25: Stew’s Connections
Market Entry BCG Round 2
Assume 100 passengers (for ease) at the various price/penetration combinations.

30 users at $5=$150/flight

25 users at $10=$250/flight

Analysis 20 users at $15=$300/flight

10 users at $20=$200/flight

5 users at $25=$125/flight

Set price at $15.


Breakeven Analysis
Given the information already revealed in the case and the information below the candidate should
calculate a break-even point.
Guidance
Information to be given if asked:
The company has discovered that if they can generate $250,000 per plane in annual
revenue, they will be profitable installing the technology on that plane.
250,000/2,000 legs/plane = $125/leg

$125/$15 about 8 users/leg


Analysis
50 laptop users/leg, and at $15, there‘s a penetration rate of 20%, so we estimate 10 users/leg.

Response: Yes, they should break even.


116
Case 25: Stew’s Connections
Market Entry BCG Round 2
Other factors
Guidance Probe the candidate for breadth and understanding of new market entry. Ask him/her about the
following aspects of this project.

Competition
The interviewer should probe deeper into the competition especially with regard to Intellectual
Property. For this case, the company has the patent on the high speed connection.

Analysis What about low-speed internet connections?

Risks
Ask the candidate which risks are associated with the business model. Use your
judgment when considering their answers.
Candidate should provide a crisp recommendation – A crisp recommendation should be roughly 30 -
45 seconds long and should include clear bullet points that support an overall recommendation. For
example, ―Enter the market for reasons 1, 2 and 3‖

Expected: Accurate arithmetic with solid profitability and breakeven calculations.


Performance
Evaluation Good: Candidate provides a framework that includes exploration of all possible options and offers a
clear recommendation that summarizes key findings in under 30 seconds.

Excellent: Candidate considered sufficient number of other factors while maintaining poise through
―what else‖ line of questioning at the end of the case.

117
Case 25: Stew’s Connections
Market Entry BCG Round 2

Exhibit 1

118
Case 25: Stew’s Connections
Market Entry BCG Round 2

Exhibit 2

119
Case 15: Convenience Store
Profitability BCG Round 1
Our client is a major convenience store chain with 5,000 stores in the US and $25B in annual
revenues. About one year ago their CEO hired our firm to help increase profits. Since then, we
have worked with the client to aggressively control costs through negotiating larger discounts from
Prompt suppliers, restructuring the client‘s labor force, and cutting other operating expenses by 10%. As a
result of our work, our partners believe there is little room for cost reductions going forward. How
else can we increase profits for our client?

1) Background data (TO BE GIVEN ONLY IF ASKED):


• There are 4 major companies (including the client) in the industry that control 95% of the market.
Each has a roughly similar share of the market.
• Client is not interested in expanding/changing product line.
• Client does not want to expand internationally due to logistical concerns.
• We studied possible M&A activities, but there are no desirable targets.
• Client charges same price per item in each of its stores. (i.e. no difference between prices of one
item in Manhattan versus Durham)
• Consumers consider Motrin and Tylenol to be very different brands, with each having very loyal
customers. It would take a substantial change in price of one brand to convince customers to
switch to the other. Buyers of both brands would consider the store brand product , however
Guidance they feel that it is a slightly inferior product and would need to be incentivized to do so.
• Advil and Motrin suppliers charge the same price to every convenience store chain
• All other costs (overhead, etc.) should be considered the same across the 4 major chains
2) Case delivery: Initially try to get candidate to brainstorm about how they might be able to
increase profits. Present candidate with Exhibit 1 if they inquire about pricing, products sold, or
sales volumes. Depending on whether candidate wants to focus on Food&Bev or Pain Relievers
first, present them with Exhibits 2 or 3 respectively.

73
Case 15: Convenience Store
Profitability BCG Round 1

Exhibit 1: Candidate should immediately focus on Pain Relievers and Food &Beverage segments.
Pain Relievers have lowest margin and are most inelastic, which suggests opportunity to increase
prices. Food&Bev is our best seller in terms of Sales and Scan Margin, so look for ways to sell
more or gain share. Cleaning Product sales are $4B. Price elasticities are meant to be illustrative
and are not needed for any calculations. (Note: Scan Margin is essentially the same thing as profit
margin, however it also includes funding from producers. It is a term frequently used by retailers).

Exhibit 2: Candidate should recognize that client has highest unit price and lowest volume/share
relative to competitors in Food&Bev segment. This would imply that there is little room to increase
prices in this category. He/she should recognize that this is not the solution and should go explore
Pain Relievers .

Analysis Exhibit 3: Candidate is presented with Total Dollar Sales, Price and Cost data for each of the three
brands of pain relievers. He or she can then calculate profit per unit, total unit sales, and total
$profit per brand

Exhibit 4: The key insight is that our client charges a lot less for Advil that the competition. We
charge the same for Motrin and slightly more for Store Brand. A good candidate will inquire about
market share data at this point, and should be presented with Exhibit 5

Exhibit 5: Candidate should determine that we have a very strong share in Advil (which accounts
for over 50% of industry-wide pain reliever sales). This combined, with our significantly lower price
and profitability in Advil, as well as slim likelihood that customers would switch brands or to our
competitors, means that we should raise prices.

74
Case 15: Convenience Store
Profitability BCG Round 1
Expected:
•Candidate recognizes that we need to focus on revenues going forward
•Identify that revenues are a function of price, quantity, and mix and ask for more data
on each component
•Recognize that priority should be on investigating Food&Bev and Pain Relievers
when viewing Exhibit 1 and ask for more information on each
•Determine that raising price of Advil is the solution to the case

Good:
• Quickly determine that Food&Bev and Pain Relievers drive profitability
• Candidate performs calculations quickly and accurately
• Recognize that Food&Bev is not the solution after viewing Exhibit 2
Performance • Candidate will ask about competitor prices and market share data when viewing
Evaluation Exhibit 3

Excellent:
• Quickly recognize that client has highest unit price and lowest volume/share relative
to competitors in Food&Bev segment. No need to perform market share calculations
• Inquire about shifting share to Store Brand to generate more profits (since it has
highest profit per unit)

Deeper Insights (Second Years)


• Attempt to determine profit impact of increasing price of Advil
• Have insight that charging higher prices for same item in certain markets would be
an opportunity to generate additional profits 75
Case 15 : Convenience Store
Exhibit 1 - Products Sold by Client

Profitability BCG Round 2

Total Client Sales Total Scan Margin Price Elasticity

Pain Relievers $10 B 0.3

Cleaning Products $0.5 B 1.1

Food & Beverage $9 B $3 B 1.4

Other $2 B $0.4 B 1.8

Total $25 B $4.9 B

76
Case 15: Convenience Store
Exhibit 2- Convenience Store Industry Food &
Beverage Sales
Profitability BCG Round 2
Average
Total Unit Volume Total $ Sales Share
Price/Unit

Client $1.50 $9 B

Competitor 1 10 B $14 B

Competitor 2 $1.35 12 B

Competitor 3
$1.45 8B

Total 36 B $50.8 B

77
Case 15 : Convenience Store
Exhibit 2 ANSWER KEY – DO NOT SHOW
CANDIDATE
Profitability BCG Round 2

Average Total $Sales Share of Total


Total Unit Volume
Price/Unit Volume Market

~18%
Client $1.50 6B $9 B

$1.40 ~28%
Competitor 1 10 B $14 B

$16.2 B ~32%
Competitor 2 $1.35 12 B

Competitor 3 $11.6 B ~22%


$1.45 8B

Total 36 B $50.8 B

78
Case 15: Convenience Store
Exhibit 3 - Client Pain Reliever Sales

Profitability BCG Round 2

Total $
Price/Unit Cost/Unit
Sales
Advil $8 B $3.99 $3.79

Motrin $1 B $4.99 $4.49

Store $1 B $1.99 $1.29


Brand

79
Case 15: Convenience Store
Exhibit 3 ANSWER KEY – DO NOT SHOW
CANDIDATE

Profitability BCG Round 2

Total $ Total Total


Brand Price/Unit Cost/Unit Profit/Unit
Sales Units Profit

Advil $8 B $3.99 $3.79 $0.20 ~2 B $0.4 B

Motrin $1 B $4.99 $4.49 $0.50 ~200 M $0.1 B

Store
$1 B $1.99 $1.29 $0.70 ~500 M $0.35 B
Brand

80
Case 15: Convenience Store
Exhibit 4- Convenience Store Pain Reliever Prices

Profitability BCG Round 2

Brand Client Price Average Competitor Price

Advil $3.99 $4.69

Motrin $4.99 $4.99

Store Brand $1.99 $1.89

81
Case 15: Convenience Store
Exhibit 5 - Pain Reliever Market Share $ by Brand
Profitability BCG Round 2

18 10 7

10%
14%

Client 44%

90%
86%

3 Competitors 56%

Advil Motrin Store Brand


82
Case 28: Stew’s Connections
Market Entry BCG Round 2
Our client is a start-up with the ability to deliver broadband internet to commercial airlines. How
Prompt would you help them think about their offering?

About the case: This is a market entry case where candidate are required to evaluate the feasibility
of a new product in conjunction with the airline industry. The candidate should use a comprehensive
framework, walk the interviewer through it and be prepared for analytical detours throughout the flow
of the case.

The calculations represented here are only one approach and interviewees may take other
approaches, depending on the assumptions made. The interviewer should be mindful of this and
allow for flexibility.

Industry & Market Size Discussion: The interviewee should have mentioned this as a major bucket
in his/her framework.

Use the information below to provide guidance as necessary.


Guidance
Broadband for the airlines
There is general interest in broadband internet from the airline industry. The start up would have to
invest relatively little up front and would keep most of the revenues. They would charge the
customers on a per flight pricing model.

Size of the Market


Ask the candidate to estimate the market size and hand over Exhibit 1. Inform the candidate that
there are 3,000 planes. Full answer in chart below

Pricing
In order to finish the market size, the candidate should ask for the price per
flight. Hand out Exhibit 2 and ask candidate to set the price.
131
Case 28: Stew’s Connections
Market Entry BCG Round 2
Class First Coach
Seats/Plane 20 180
Load Factor .75 .75
Full seats/plane 15 135
Analysis Biz Travelers 100% 30%
Laptop users/plane 15 40.5
Total laptop users/plane 55.5
3000 planes x 2000 legs/plane x ~50 laptop users/plane = 300,000,000 approximate annual
potential user-legs.

132
Case 28: Stew’s Connections
Market Entry BCG Round 2
Assume 100 passengers (for ease) at the various price/penetration combinations.

30 users at $5=$150/flight

25 users at $10=$250/flight

Analysis 20 users at $15=$300/flight

10 users at $20=$200/flight

5 users at $25=$125/flight

Set price at $15.


Breakeven Analysis
Given the information already revealed in the case and the information below the candidate should
calculate a break-even point.
Guidance
Information to be given if asked:
The company has discovered that if they can generate $250,000 per plane in annual
revenue, they will be profitable installing the technology on that plane.
250,000/2,000 legs/plane = $125/leg

$125/$15 about 8 users/leg


Analysis
50 laptop users/leg, and at $15, there‘s a penetration rate of 20%, so we estimate 10 users/leg.

Response: Yes, they should break even.


133
Case 28: Stew’s Connections
Market Entry BCG Round 2
Other factors
Guidance Probe the candidate for breadth and understanding of new market entry. Ask him/her about the
following aspects of this project.

Competition
The interviewer should probe deeper into the competition especially with regard to Intellectual
Property. For this case, the company has the patent on the high speed connection.

Analysis What about low-speed internet connections?

Risks
Ask the candidate which risks are associated with the business model. Use your
judgment when considering their answers.
Candidate should provide a crisp recommendation – A crisp recommendation should be roughly 30 -
45 seconds long and should include clear bullet points that support an overall recommendation. For
example, ―Enter the market for reasons 1, 2 and 3‖

Expected: Accurate arithmetic with solid profitability and breakeven calculations.


Performance
Evaluation Good: Candidate provides a framework that includes exploration of all possible options and offers a
clear recommendation that summarizes key findings in under 30 seconds.

Excellent: Candidate considered sufficient number of other factors while maintaining poise through
―what else‖ line of questioning at the end of the case.

134
Case 28: Stew’s Connections
Market Entry BCG Round 2

Exhibit 1

135
Case 28: Stew’s Connections
Market Entry BCG Round 2

Exhibit 2

136
BIG RED CASE BOOK JOHNSON SCHOOL CONSULTING CLUB

CONSECO

Type of case: Strategy


Company: BCG

Description: All of the data in this case is public domain. Conseco is a company at the financial
services industry and more specifically at the business of life and health insurance. During the
years 83-98 Conseco was a great performer and lead the S&P 500. Conseco’s main growth
engine was its successful acquisitions. On average, the company acquired a target every 6
months. During 98, Conseco acquired Green Tree Financials. Surprisingly, the day after the deal
was announced Conseco share price dropped 20% and a year after the share was down 50%
from its price the day before the announcement. You were hired by the CEO to explain this drop in
the share price and to suggest a course of action.

Additional data:
 Green Tree Financial is a provider of loans for homebuyers.
 Green Tree Financial is charging higher interest rates than Conseco.
 Green Tree deal was much larger than Conseco’s previous deals.
 Conseco share price before the acquisition was $57.7.
 Green Tree Financial share price before the deal was $29.
 The deal was a fixed equity exchange deal where 0.9165 shares of Conseco were
awarded for every share of Green Tree Financial.
 Conseco’s market cap before the deal was $7B.
 Green Tree owned approximately 50% of the company created by the M&A transaction.
 A year after Green Tree needed an additional investment of $1B.

Solution Structure:
 Identify the players attributes.
 Identify the exact deal structure.
 Identify misalignments in the deal that might cause the share price drop.
 Try to predict what will happen next and suggest course of action accordingly.

Solution Analysis:
 Problems with the deal structure:
o Misalignment in the companies business.
o The almost 1:1 stock exchange didn’t reflect the different market values of the two
companies.
o Conseco’s expertise was in smaller and more rapid acquisitions and this
acquisition wasn’t something they could handle.
 Problems with the acquisition target:
o From the last bullet in the additional data section it is obvious that Green Tree was
at a difficult situation before the acquisition and wasn’t a good target for
acquisition.

The market adjusted Conseco’s share price to reflect these misalignments.


 What to do now (after a year)?
o Investigate the financial state of Green Tree after a year (it is evident it wasn’t
good).
o If Green Tree continues to be in distress suggest dumping it.
 Conclusion
o Green Tree continued to suffer big loses and dragged Consico with it

© 2003 The Johnson School Consulting Club Page 74


BIG RED CASE BOOK JOHNSON SCHOOL CONSULTING CLUB

o After several years Conseco was unlisted from the S&P.


 Additional questions
o What was Conseco’s management thinking?
o Where was Conseco’s board of directors?

© 2003 The Johnson School Consulting Club Page 75


BIG RED CASE BOOK JOHNSON SCHOOL CONSULTING CLUB

INSURE ME!

Type of case: Strategy


Company: BCG

Description: Insure me is a Global Financial Services company at the insurance business.


Recently, the CEO of the company was fired and took with him all of the 10 employees of the
company’s private funding division, which was his pet project. No one that is left in the company
knows what is going on in that division, and there is no reporting system to rely on (the CEO took
all of the data with him). How would you go about managing this division?
Additional data:
 The company is operating in the US and Europe.
 The company provides car, life and other type of insurance.
 The company is one of the 4 leading players at its market with over $1B of annual
revenues.
 The private funding division is type of a VC.
 We have a data sheet (see appendix) which list 4 of the division’s current investment.
 These 4 investments are only around 20% of the number of investments but form 80% of
their value.
Solution Structure:
 Identify the company’s business and core competency.
 Identify the assets under the division management.
 Identify any financial and strategic synergies between the division’s assets and the
company.
 Analyze ways to leverage the division and its assets moving forward.
Solution Analysis:
 As mentioned the company’s core competency is in the insurance field.
 As could be observed from the appendix two assets are not complimentary to the
company’s business.
 From the remaining ones one is forecasted to lose money next year.
 As such there is one company it make sense to keep and the other are not a real asset to
the company.
Recommendations:
 Keep the company with the strategic fit that makes money and try to sell the others (for a
good deal).
 For the one that makes sense try to increase the company’s holding in it.
 The company with the fit will serve both to hedge the bets and in order to keep the finger
on the pulse of the new market needs.
 As for the division, try to find what would be needed (funds, time, efforts, HR etc.) in order
to bring it to an operational mode.
 Find what are the estimated operation costs.
 If it makes sense from the financial aspect you might want to keep this division as it hedge
your bets.

© 2003 The Johnson School Consulting Club Page 76


BIG RED CASE BOOK JOHNSON SCHOOL CONSULTING CLUB

Name of A B C D
company
Field High Value Stadiums Golf clubs Executive
commodities renovation design insurance
insurance
This year’s $150M $300M $100M $70M
revenue
This year’s $100M $280M $150M $50M
expenses
Next year’s 300% 200% 100% 300%
revenue growth
(additional on top
of the current)
Next year’s 500% 200% 150% 400%
expenses growth
(additional on top
of the current)

© 2003 The Johnson School Consulting Club Page 77


Too Many Books: Storage and distribution dilemma

Context

Books Galore is a company in the publishing industry with stable sales, in regards to both
volume and value.

Its distribution warehouse, which is used for storage and order preparation, is reaching
maximum capacity, resulting in a rapid deterioration in the quality of service.

The Head of Books Galore’s distribution subsidiary wishes to build an extension to the
distribution warehouse, at a £10m cost, though the project would bring no additional
revenues.

The CEO has asked you to determine whether the investment is necessary or whether it can
be better used elsewhere

Question 1

Why is the warehouse reaching maximum capacity?


The logical approach would be to determine the drivers of warehouse capacity. It should be
assumed that each book or product has a unique allocated place.

That is correct. Each title has a unique ‘SKU’ – Stock Keeping Unit, which necessarily
corresponds to a specific storage location in the warehouse. Storage areas will have their
particular SKUs in varying number.

The capacity of the warehouse can therefore be expressed as:

(# of SKUs) X (Volume per SKU)


Capacity of the warehouse

There are three possible explanations for increasing capacity:


• Increasing number of SKUs (more titles)
• Increasing number of volumes per title (assuming that books are not changing size)
• Decrease in overall capacity (perhaps warehouse is being used for other
purposes?)
54
These three possible explanations indicate causes for the increasing space usage and
possible levers to address them:

Number of Number of volumes / Overall warehouse


SKU capacity
SKUs / Titles

ƒ Overestimate of
ƒ Many new titles potential sales
Potential Causes for ƒ New legislation
increased space ƒ Old titles not ƒ Longer
usage ƒ Safety issues
destroyed replenishment
cycle

ƒ Eliminate part of
overstock

ƒ Reduce initial
stock ƒ Extend the
ƒ Eliminate number warehouse
Potential levers to of titles stored
ƒ Reprint on request (client’s
increase capacity
proposed
ƒ Shorten solution)
replenishment
times

The first potential lever – eliminating the number of titles stored - seems like a promising way
to start. Has there been a recent increase in the number of new titles stored?

No, but the company acts as service provider of storage and order preparation services for
third parties (other publishers). This business represents 50% of the warehouse flows. The
warehouse charges these third parties for the delivered services. It might be conceivable to
terminate all or part of the third-party business

With this in mind, in makes sense to stop serving some of the third parties to decrease the
number of SKUs being stored and increasing the space for the company’s own usage.

Question 2

Which criteria would you use to determine which third parties the
company should no longer serve?
The objective should be to maximise the positive impact for the warehouse, which depends
on the impacts brought about by a publisher’s exit:
• Released space
• Suppressed costs
• Lost revenues

We can therefore prioritise the third-party publishers through two simple criteria:
55
• Space released
• Economic impact

Question 3

How could you assess the impact on the warehouse P&L statement of a
particular publisher’s exit?
The impact on profit would be the changes in revenue minus the changes in costs.

Let’s first proceed with costs – how would you analyse the impact on costs?

Total costs are composed of:


• Fixed costs (independent of the number of publishers)
• Variable costs (decreasing when a publisher leaves)

In this example, fixed costs would be the building itself, sets of shelves and other
infrastructure, as well as some of the administrative and IT staff that coordinate operations in
the warehouse.

Variable costs would include handling staff and packaging.

I would like to determine how much each of these costs contributes to the overall expense of
running the factory.

After investigation, you establish the following cost structure:

10
10

15

25 100

40

Handling Admin + Premises Packagin Fork-lift Total


staff trucks
(deprec.)
Variable costs
Fixed costs

56
Question 4

What are the drivers of our variable costs?


Variable costs are encompassed by handling staff and packaging. In terms of time utilisation,
that requires filling title / SKU spots, fetching one or several volumes from these places, and
then packing these books for distribution.

Cost drivers for these actions would be:

• Number of movements to be performed = number of times operators have to


fetch one or several books of a same type, which means that personnel
expenses will vary depending on the number of order "lines"
• Packing - Number of boxes / packages, which corresponds to the number of
books (assuming that the size and type of book does not change)

This gives us a good idea of potential areas for cost savings. But what about the impact of
decreased number of publishers on revenue?

Question 5

If revenues are based on the price of the book (fixed at 5%), which
publishers should be terminated?
Our goal should be to maximise profits through a publisher’s exit from the warehouse. Since
revenues are a function of the price, two possibilities would be:
• Eliminating publishers with low average price and rotation
• Eliminating publishers which have many references selling in small quantities

We have identified an external publisher that would be a good candidate to reconsider.

Publisher's characteristics:
• 10% of volumes
• 20% of order lines
• Average selling price = that of the warehouse

Current warehouse margin: £5m, which is 25% of sales

Question 6

What would be the impact on the margin of this publisher’s exit?


Change in margin will be the change in revenue with the publisher’s exit subtract the change
in costs

Revenue will decrease by 10%


• Total sales of £20m (£5m / 25%)
• £20m revenues x 10% = £2m lost

57
Expected revenue would be £18m

Total costs currently £15 (= sales of £20m – margin of £5m)


Current cost breakdown:
• Fixed (50%) = £7.5m
• Personnel (40%) = £6.0m
• Packaging (10%) = £1.5m

Projected cost without publisher:


• Fixed = £7.5m (no change because fixed cost)
• Personnel = £6.0m – 20% = £4.8m
• Packaging = £1.5m – 10% = £1.35m

Total costs without publisher expected to be £13.65m

New margin = expected revenue – expected cost


= £18m - £13.65m
= £4.35m

Margin would thus decrease by £0.65m by eliminating services for that publisher

Question 7

If eliminating that publisher would reduce margins by £0.65m per year,


what is our best option - terminating a publisher’s contract or building
an extension?
To assess which is best, we could compare the potential annual loss of eliminating this
publisher with the cost of the investment proposed initially
• Loss: = £0.65m / year
• Investment: £10.0m
• £10m / (£0.65m / year) = 15 year
The proposed extension is the same cost of 15 years of eliminating the publisher in question,
assuming everything else remains constant. Thus, it would be better NOT to build the
extension.

Yet, giving up a publisher generates sales and margin losses. Are there other conceivable
solutions?

Question 8

Are there any opportunities to manage the capacity issue?


Looking back to our initial framework and potential levers to increase capacity, there are
several options:
• Ensure old stocks of unused titles / SKUs are destroyed
• Eliminate over-stocks
• Reduce size of initial stocks
• Re-print on request
• Shorten replenishment times

58
Whiskey Brand Turnaround
Context

The client has been in business close to 90 years. The original founder started in the whisky
business, although, over time, successive leaders expanded the product line so that today,
the company is a multi-billion dollar diversified liquor manufacturer, with less than 5% of its
sales in .

This expansion strategy made particular sense over the last 20 years, as total U.S. sales
have been declining by ~2% per year, and the client’s brand has been declining at the same
rate.

While the company has enormous passion for the brand (as the founding basis of the
company), in recent years they have been paying limited attention to the brand. Last year,
however, events caused our client to suddenly pay attention to their brand. While the
market declined by 2% again, our client’s brand declined by 15%, despite selling 10m bottles.

The client comes to you because they A) want to find a way to grow the brand again; but B)
want to do this without lowering the total profits they were taking out from the brand.

Question 1

What is going wrong that has made sales of the client’s brand suddenly
decline?
To frame the problem, it would be helpful to establish internal vs. external causes for the
decline of the brand.

Some of the possible internal reasons for the brands decline include:
ƒ Reduction of marketing spend
ƒ Decline in quality of marketing
ƒ Decline in product quality
ƒ Product availability (production / supply chain issues)

External reasons are also very likely. Some external factors which could hasten the decline
of the brand are:
ƒ Negative PR
ƒ New competitor product launch
ƒ Increase of competitor marketing
ƒ Pricing effects

65
To establish which of these factors is decisive in the decline of the brand, further examination
will no doubt be required.

Some members of the board have voiced their concern about our marketing spend vs.
competitors, but company marketing spend hasn’t changed in the past several years.
Perhaps competitor marketing has changed significantly.

Question 2

How could we determine if there has been change in relative marketing


spend against major competitors?
First we’ll need to gather information on our own marketing spend, so that we have a basis of
comparison.

Secondly, we need to determine our competitors’ marketing spend, which we could do


through several sources:
ƒ Internal interviews
ƒ External benchmarking through competitor company reports
ƒ Count of number of ad pages in magazines
ƒ Interviews of experts within the advertising and marketing industry

Interviews with advertising experts and an examination of competitors’ ad pages have shown
that while marketing spend hasn’t changed significantly, there has been significant change in
prices within the industry. The following profitability data was gained from the above
investigations:

Place of Cost to
Brand 2005 price 2006 price 2007 price
sale produce

Grocery,
Client Speciality £8.00 £14.75 £14.75 £15.00
stores

Competitor A – Speciality
£8.25 £15.00 £17.00 £19.00
Premium stores

Competitor A – Grocery
£6.50 Not launched Not launched £ 9.00
Own brand stores

Competitor B – Specialty
£8.25 £16.00 £17.75 £19.75
Premium stores

Competitor B – Grocery
£6.50 Not launched Not launched £ 9.75
Own brand Stores

66
Question 3

What conclusions can we draw from this client and competitor


profitability data?
It appears as though competitors A and B have been slowly increasing their prices on
Premium brands, aiming at a segment of consumers who are willing to pay more for their at
specialty stores. It is possible that they have succeeded in capturing a more lucrative share
of the market

At the same time, they have introduced ‘Own Brand’ labels for selling in grocery stores that
are significantly cheaper than the client brand.

Thus, it seems that competitors have been capturing the lucrative top end while also
launching new brands that have captured the low price-end, effectively squeezing the client
brand out of the market.

Considering our production costs and the price of Competitor A and Competitor B ‘Own
Brands’, we cannot compete on price in supermarkets. With our strong brand, we must aim
to recapture the high end of the market, hence increasing prices.

Question 4

What price should we set and how will it impact profits?


In order to understand the impact of an increase in price on profits, I would need information
on the number of bottles sold and some info on the cost structure.

We already know that 10M bottles of our were sold at an average price of £15 per bottle.

Of total costs, 75% was variable costs and 25% fixed costs

Total profit last year was total revenue subtract total costs. Total revenue was 10M X £15 =
£150m. Total costs, as seen already was £8 per bottle or £80M in total. Total profit at the
current price was therefore £70M in 2007.

First, we need to establish a new price for our re-branded ‘premium’ . With competitors
selling at £19.00 and £19.75 for their premium brands, I’d guess a similarly high amount
would be appropriate, say £20 per bottle.

In order to determine how this will affect profits, I’d need more information on price elasticity,
to see how a higher price will affect sales.

At an increased price of £20 per bottle, sales would decline from 10M units sold to 8M units
sold

At £20 per bottle, 8M units would yield revenue of £160M.

Fixed costs would stay the same as before, which were 25% of £80M, or £20M.

Variable costs would be 8M X 75% of £8, or 8M X £6m = £48M.

67
Total costs would therefore be £68M, yielding a total profit of £92M. A higher price would
therefore position our brand in the higher-end market, decreasing the amount produced but
increasing profits by £22M with a higher price.

Question 5

What complications might arise in our attempt to capture this higher-


end market?
In the short-term, we need to be aware of how competitors may react to the move, by
adjusting their pricing, branding or advertising. Our move could provoke a change in prices
amongst competitors that will leave us positioned as before. We should also be aware of
possible difficulties in marketing our brand in the premium category as the third mover.

Further complications could arise given that our brand exists across a wide portfolio of
products. The price increase of our could negatively impact the customer base for other
products, if the customers who stop purchasing our at a higher price develop negative
feelings regarding the brand.

In the longer-term, we should examine whether it makes sense to re-invest earnings into the
business since it is a declining market. Investments might be most profitably put to use by
growing other parts of our business.

68
50 London Business School Case Book

BCG
Iceberg PROFIT

Europe

Case at a glance (for the interviewer only)

Part A Part B Part C


Structure & hypothesis Interpretation & numeracy Recommendations & summary

Opening statement: Present the candidate with the slide Ask the candidate:
“Our client is Iceberg, a major titled: “UK ice cream tub prices” and “What strategies could Iceberg use
global branded ice cream producer. tell them: to address the performance issue in
Iceberg develops, manufactures and “The Associate on this case prepared Europe and how would you prioritise
markets ice cream products and this slide. What is causing the them?”
sells to retailers who, in turn, sell to performance issue in Europe?”
the end consumer. Ice cream is one (If the candidate is struggling, ask:
of the most profitable products that “How should Iceberg segment the Tests strategic thinking, creativity
Iceberg makes. The business has market and what is happening in each and ability to prioritise and provide
grown at 5% led by North America segment?”) reasons
and developing markets. However,
Iceberg has recently seen poor
growth and competition intensifying Tests business intuition and “What are your recommendations for
in the European ice cream market, in the ability to interpret data, Iceberg’s management?”
particular from supermarkets’ own- draw conclusions and identify
brand ice cream. Iceberg management implications
are sure they have great products: Tests ability to synthesize and
they continue to win in consumer taste structure recommendations,
tests, there is a strong pipeline of “How does the market opportunity business intuition and empathy
planned product launches, and they compare to Iceberg’s business
have strong brands in many markets. today?”
Management believes this allows them (If the candidate is struggling,
to sell their products at a higher price ask: “Which segment would you
than the competition. What could recommend Iceberg focus on and
be causing the performance issue in what is the margin and volume
Europe?” potential in that segment?”)

Tests ability to structure a Tests numeracy, ability to make


problem and state and explain a reasonable assumptions, degree
clear hypothesis of confidence/insecurity and
personality in the face of challenge
to their work (ask “Are you sure
you’re right?”)
London Business School Case Book 51

BCG

Iceberg

Key case insights an excellent candidate might uncover (for the interviewer only; do not tell the candidate)
• There are 3 market segments: • Iceberg is winning market share in • In any given segment
economy, mass market and a shrinking mass market recommended: Iceberg’s volume,
premium-priced products margin or profit potential; its
• Premium segment is likely growing, competitiveness to customers
• Iceberg competes primarily in the as brands distinguish themselves and consumers (realising they are
mass market segment (defined as from the mass market to retain and different); and its ability to win
price points €2.00 – 3.99), with a grow margins against branded and private label
~38% market share by value (€46m products
out of €122m), ~34% by volume • To compete, Iceberg should:
(15m L out of 44m L) —— Drive volume to improve plant
utilisation (~35% in Western This case is long and candidates
Europe, vs. ~60% in North would not necessarily be expected
If the candidate delineates 3 America) and reduce unit to finish it
segments slightly differently, their costs, so that it is better able to
market size and share numbers compete on price in the mass
would differ accordingly market
—— Increase presence in premium
(relying on taste performance
• Mass-market consumers are and strength of brand)
becoming more price conscious —— Optimise drivers of consumer
(sales of €2.00-2.59 are strongest purchasing behaviour besides
in the mass market category) price (e.g. packaging /
advertising / shelf placement)
• In the mass market and economy —— Potentially expand in the upper
segments, Tesco is undercutting end of the economy market,
Iceberg and other competitors although its retailer purchase
on price, growing the economy price may be less competitive
segment and pushing down
Iceberg’s revenue in the mass
market
52 London Business School Case Book

BCG

Iceberg

Exhibit: UK ice cream tub prices (show to candidate)

Market price architecture Example products

7.00-7.9 9

6.00-6.99 Haagen Dazs


Ben & Jerrys
5.00-5.99 Tesco Finest
4.50-4.99
Retail price segment €/L

4.00-4.49

3.50-3.99

3.00-3.49

2.50-2.99 Iceberg 1L Vanilla €2.82

2.00-2.49 Tesco 1L Vanilla €2.22

1.50-1.99

1.00-1.49

0.00-0.99 Tesco Val. 2L Vanilla €0.78

0 5 10 15 20 25 30 35 40
Market value of price segment at retail price 2012 (€m)

Iceberg ice cream tubs Other company ice cream tubs

Source: Nielsen: Store research; BCG analysis

Exhibit: Iceberg cost structure of 2L vanilla ice cream tub in UK


(show to candidate if this data is requested)

1.0

0.18 0.89

0.8
0.11

0.10
Product cost €/L

0.6
0.08 0.51

0.19
0.4

0.11
0.2
0.13

0.0
Ingredients Packaging Production Distribution Product Advertising Overhead Profit Retailer
cost & promos purchase
price
Source: Client data; BCG analysis
London Business School Case Book 53

BCG

Iceberg

Exhibit: Iceberg global ice cream production plant utilisation (show to candidate if this data is requested)

Europe N America
100% 100%
80 80
Max Max
60 70% 60 70%
40 40
20 20

0 0
A B C D E F G H IJ K L A B C D E F G H I

Western Europe CEE

Production plant size Production plant size

Productive capacity Unused capacity (based on 8760 hrs per year)

Example of a possible case structure (for review after the case interview)

What can Iceberg do to improve their competitiveness in Europe?

Product Pricing
Cost structure Competition
(compare with) (compare with)

Other branded ice creams Other branded ice creams Fixed costs Distribution channels

Own label ice creams Own label ice creams Variable costs Shelf positioning

Substitutes Substitutes
Promotions
(other desserts) (other desserts)
54 London Business School Case Book

BCG

Iceberg

Differentiation between poor, average and superior performance (for review after the case interview)

Poor Performance Average Performance Superior Performance


Framing problem / Suggests what Sets out a structure for Sets out a clear, logical
prioritising issues supermarkets are doing analysis; identifies 3 price structure for analysis;
without clear rationale segments, and possibly recognises that market
or structure; does not that supermarkets have has three segments,
consider differences power because Iceberg is with Iceberg strongest in
across the range of reliant on them to sell its the mid-price segment;
supermarket products products identifies need to
understand Iceberg's
ability to compete

Identifying relevant Starts asking for a variety Asks a series of specific Defines information
information of information – no clear questions related to a needed, including
logic single logical line; identifies rationale; identifies key
some key points from points and explains their
the graphs; can process implications from the
answers and move on graphs presented

Running calculations / Calculates incorrectly that Correctly calculates Realises lowering price
drawing conclusions from Iceberg cannot compete at Iceberg can compete at may dilute margins and
facts supermarket price points lower price points except suggests ways to avoid;
Tesco Value and quantifies identifies production
margin utilisation issue and
proposes solution;
Calculates volume /
revenue / profit potential

Identifying key implications Limited or Needs to be asked Identifies the key case
and next steps; illogical additional for ideas on potential insights; drives to
demonstrates creativity recommendations solutions; has a few ideas solutions on their own;
on where to improve; for how to improve prioritises a list of alternate
formulaic approach opportunities; goes beyond
(e.g. spend more on the obvious throughout the
marketing) case process
London Business School Case Book 55

The Interview Process


BCG

Market
Cupid’s Arrow Entry

North America v United Kingdom

Case at a glance (for the interviewer only)

Part A Part B Part C


Structure & numeracy Analysis & business judgement Recommendations & summary

Do not share any exhibits until Candidate is expected to continue Ask the candidate:
Part B with their case analysis. Share facts of “So, what recommendations would
the case or exhibits (see the following you make to Cupid’s Arrow’s
1) Structuring the case pages for details) when these are management?”
specifically asked for by the candidate.
“Our client is Cupid’s Arrow, a
successful subscription-based online When sharing an exhibit, ask the Tests ability to synthesize and
dating agency. They currently operate candidate: structure their recommendations,
exclusively in the US market, where “What does this exhibit tell us? How business intuition and empathy
they are the market leader. Cupid’s might this affect Cupid’s Arrow’s entry
Arrow are considering entering the into the UK market?”
UK online dating market. What are
the main factors that they should
consider?” Tests business intuition and
the ability to interpret data,
draw conclusions and identify
Tests ability to structure, implications
hypothesise and think creatively
around a problem

2) Market size estimation

“How would you estimate the size of


the UK online dating market?” (if the
candidate is struggling, clarify this as
being “revenue per year”)

Tests structure, numeracy and


ability to make reasonable
assumptions

“What does this tell us so far about


the attractiveness of the market for
Cupid’s Arrow? What else do we need
to think about?”
56 London Business School Case Book

BCG

Cupid’s Arrow

Key case insights an excellent candidate might uncover (for the interviewer only; do not tell the candidate)
• The UK market will nearly double —— The UK soul mates segment • Entry into the UK market could be
in size over the next 2 years and is may already be quite via organic growth or syndicated
quite fragmented with at least a few competitive: HappyHearts (33% from the existing US Cupid’s Arrow
new entrants share and 20% p.a. growth) and site, but would be fastest via
Lovebirds (23% share) together acquisition and rebranding of a
• Profit margin is healthy at 75% per have ~75% share and the soul smaller site, for instant network
customer (£180 p.a. per customer) mates segment is only 25% of effects between subscribers. Given
the UK market the anticipated pace of growth in
• Cupid’s Arrow may struggle in —— UK may increasingly shift the UK market and the likely lock-in
entering the UK market (candidate towards soul mates, like the effect in this market based on the
may take a slightly different view US as online dating loses its size of a subscriber base,
of the future direction of the UK stigma, but it is not there yet acquisition and rebranding of a
market and optimal strategy, but is smaller site would be advisable
expected to support their position • Overall, the UK market is attractive,
with similar insights): but may require Cupid’s Arrow to
—— There is greater stigma around adapt its image / focus in the UK This case is long and candidates
online dating in the UK (65%) more towards the interests of UK would not necessarily be expected
than in the US (35%), although customers (socialising / casual to finish it
this is declining over time dating) and to form a clear strategy
—— Cupid’s Arrow’s core strength to compete against the aggressive
in the soul mates segment in growth of HappyHearts and the
the US (60% of the US market), threat of new entrants
is less applicable in the UK
where this segment comprises • Along with a clear strategy,
only 25% of the market aggressive marketing campaigns
(socialising and casual dating and friend referral benefit schemes,
segments comprise 75% of etc. are key to establishing a
the market) presence in the UK market

Facts to share with the candidate if asked for specifically (for the interviewer only)
• Cupid’s Arrow currently has US • Set-up costs for Cupid’s Arrow • “Exhibit: Running costs for a
revenues of USD$30m per year in the UK for organic growth typical UK online dating agency”
are minimal (e.g. IT equipment, – share if asked about costs /
• Cupid’s Arrow currently focuses on customer survey) profitability
finding “soul mates” / life partners
for its subscribers in the US • HappyHeart’s growth is due to • “Exhibit: US vs. UK perceptions
aggressive marketing campaigns of online dating” – share if asked
• Expected revenue for Cupid’s and friend referral benefit schemes about market segmentation /
Arrow in the UK is £20 per month demographic differences / types of
per customer • “Exhibit: Historic and projected online dating sites in the US versus
growth of the UK online dating the UK
market “ – share only in Part B of
the case (after the market sizing)
– if asked about market growth or
competition
London Business School Case Book 57

BCG

Cupid’s Arrow

Exhibit: Historic and projected growth of the UK online dating market


(show to candidate if this data is requested)

300 CAGR
’12-’14 278
Other (~ 20 players <£2m)
+36%
250 Table4Two
TakeAChance
CAGR 204
200 TakeMeOut
UK revenue (£m)

’09 -’12
+19% MatchMeUp

150 150 Lovebirds


133
30 HappyHearts
30 5
100 99 10
89 10 10
30 10 23
30 22
50 20 23
20 22
20
20
30 40 50
0 20
2009 2010 2011 2012 2013F 2014F

Exhibit: Running costs for a typical UK online dating agency (Show to candidate if this data is requested)

60
5

50
15
£ per customer/year

40

30 60

20 40

10

0
Marketing IT support Admin support Total costs per customer
58 London Business School Case Book

BCG

Cupid’s Arrow

Exhibit: US vs. UK perceptions of online dating (show to candidate if this data is requested)
Responses to questions from a survey

Question 1: Do you believe there is a stigma Question 2: What are you looking for from an
around online dating? online dating agency?

50 50 60 60

40 50
40
% respondents

% respondents
40 40
30 35
25 25 30
20 20 25 25
15 20
15
10 10
10
5
0 0
US UK US UK

Yes Meet new people for socialising


Yes, but less than it used to be Find someone for casual dating
No, not anymore Meet my soulmate/life partner
No, there never was

Source: Survey of a random sample 20-45 year olds from the US and UK (n=100 in each country)

Example of a possible case structure (for review after the case interview)

Main factors affecting the attractiveness of the UK online dating market for Cupid’s Arrow

Overall attractiveness of Market attractiveness Competition in relevant


Market entry method
the UK market to Cupid’s Arrow segment

Value proposition in the Syndicated from US site


Size Current competition
US vs UK into UK

Growth rates New entrants Organic growth

Profitability Acquisition
London Business School Case Book 59

BCG

Cupid’s Arrow

Example calculation for the size of the UK market (For review after the case interview)

Drivers Assumptions

UK population 60m

% in target age range for


50%
dating websites (20-60 yrs)

X
Number of UK
% of target range that
customers 33%
are single
0.5m
X
% of single potentials
Subscription revenue: that are interested in 25%
Total size of the UK internet dating
X
market (£/year) X
£120m % willing to pay for a
20%
subscription service

Revenue per customer Subscription fee


£20/month
per month
£240/year

A superior candidate may also identify other revenue


streams (e.g. advertising and events)
60 London Business School Case Book

BCG

Cupid’s Arrow

Examples of creative ideas to maximise success in the UK (for review after the case interview)

Candidate may take different views of optimal strategy – not all of these will apply

Potential views of challenges Potential creative solutions

The UK has a stigma around online dating, compared Adapt marketing to integrate with the UK market
to the US market • Be less overt about finding “The One”
• Emphasise socialising and meeting new people
• Supplement UK sites with in-person social events

HappyHearts is expanding aggressively through Analyse the target segments of HappyHearts


marketing campaigns • Survey the target customers to understand their needs
and identify those met by HappyHearts
• If this segment is attractive to Cupid’s Arrow in the
context of its new brand, offer initial sign-up deals (e.g.
first 2 months free) and some free events

UK customers are looking for a different type of online Rebrand in the UK towards a more social focus
service (socialising / casual dating), less geared • Appropriate branding to attract a wider pool of singles
towards finding a life partner • Modify the website to emphasise meeting friends/
casual dates as well as partners
• Offer regular managed events to get single people
together in a fun setting (e.g. ice skating, bowling)

Although 75% UK market is today focussed on Expand existing US site directly into the UK with
socialising / casual dating, with rapid UK market strong branding to reduce online dating stigma
growth, online dating is expected to rapidly lose • Maintain focus on finding life partners
its stigma and customers will increasingly seek life • Aggressive, wide marketing base showing real
partners online, as has been the case in the US members and matches to emphasise that “everybody’s
using it”
• Expect potentially slow growth until stigma reduces
London Business School Case Book 61

BCG

Cupid’s Arrow

Differentiation between poor, average and superior performance (for review after the case interview)

Poor Performance Average Performance Superior Performance


Structuring the analysis Only identifies one or Sets out a good structure Sets out a clear, logical
two factors that affect for analysis- identifies at structure for analysis;
the attractiveness of minimum three factors. touches on wider issues
the market (e.g. market Is able to provide a few such as the attractiveness
size, growth) and needs explanatory points about of the UK in the wider
significant prompting to each factor context of the client's
think of other factors. business (e.g. compared to
May focus exclusively on other potential markets)
revenues/costs

Making a market size Struggles to identify the Makes a clear structure Makes a clear structure for
estimate main drivers of the market. for estimation, makes estimation and completes
Does not have a rough no / very few errors with analysis with confidence
idea of UK population. numerical steps and enthusiasm.
Struggles to provide Makes insightful
rationale for estimates. commentary around
Makes basic numerical estimate assumptions.
errors Acknowledges potential
other revenue sources.

Interpretation of graphical Needs significant Correctly interprets main Identifies all main trends
figures; identifying key info prompting to understand competitor trends from plus more subtle features
output. Draws only basic graph, is able to calculate of graphical outputs,
conclusions from the data; profit margin, understands asks probing questions
little insight some of the survey findings (e.g. Do we know what is
with little prompting driving the doubling of the
market size?) and suggests
hypotheses; synthesizes
clearly between the market
and survey exhibits

Synthesizing key Poor recollection of main Can correctly draw Summary is a well
findings and making findings; laundry list together key findings with synthesized and structured
recommendations; recall with little synthesis reasonable synthesis of view that incorporates
demonstrating creativity / insight. Unable to ideas; needs prompting all the main findings.
provide creative ideas for to come up with creative Drives independently to
success in the market (e.g. ideas for Cupid’s Arrow to the need for a change
suggests just offering a be successful in strategy for entry into
low subscription price) the UK, gives a clear
strategy recommendation
and rationale and makes
creative suggestions
Table of Contents

Practice Case 9: Champions league


(BCG - Round 1)
I. Case scope provided by interviewer:

Our client is a German television broadcasting company. It has rights


for broadcasting Champions League Soccer Tournament in Germany.
We have been asked to decide whether to broadcast it on pay-TV
channel owned by the client or on the free channel also owned by the
client.

II. Additional information provided after relevant questions:

Interviewee has to come up by himself with notion of incremental revenues


and cost of serving and acquiring customers.

• The revenue sources for the two options are: Ads for free TV and
additional subscribers for pay TV.
• In first round, there are 4 groups with 4 teams each. Each team plays
with each other team within there group in the first round. There are 3
German teams. They are all in different groups. We are going to
broadcast only games with their participation. Second round is
quarterfinals with home and guest games. Then semifinals come. And
then final – one game. 30 seconds of ads are $50K at any stage. A
game has 25 min of ads.
• Can assume that the German teams advance through all rounds
• If we decide not to broadcast the game on free TV, we will still get the
same minutes of advertising at $20k/minute
• If we decide to broadcast on pay-TV, we will get 1M more subscribers.
• Pay TV: Subscription fee is $10. Price per month $5.
• No customers will leave if you decide not to broadcast on pay-TV.
• Project only for 1 year.

III. Sample Solution:

1 round = 3 × 6 = 18 games broadcasted


2 round – 8 games
3 round – 4 games
Final – 1 game
Total: 31 games.

Free TV

Ad revenue per game = 25 × 2 × 50K = 2.5M

We have also the opportunity cost. With no game broadcasting we can get
$20K per minute.

21
Table of Contents

Incremental revenue per game = 25 × (100 – 20) = 2M


Total incremental revenue = 31 × 2 = 62M
No additional cost.

Pay TV

Incremental revenue will be 1M × (10 + 5 × 12) = 70M.


But the acquisition cost is $5 per new customer and it costs $6 per year to
serve one customer.

Incremental profit will be 70M – 1M × (5 + 6) = 59M.

Given this information, the incremental profit for pay TV is $59M and for
free-TV is $62M. Our client should broadcast the games on free-TV.

Push the interviewee to come up with exact numbers.

22
Table of Contents

Practice Case 10: Orthopedics company


(BCG - Round 1)
I. Case scope provided by interviewer

Your client is the CEO of a US Orthopedics company (that


manufactures and markets joints/shoulder/hip replacement parts).
The company is a global company with revenues of $2 billion. They
are merging with a global European Orthopedic manufacturer with
Revenues of $ 1.5 billion. You have been hired to assist with Post-
Merger Integration issues. What opportunities do you see and what
advice can you give to the client?

II. Additional information provided after relevant questions:

• Value chain: R&D-Sourcing- Manufacturing-Distribution-Sales


(+HR/Finance & IT)
• Plant A manufactures products selling for $0.5b at $0.3b cost. By
transferring production to Plant B they can manufacture the same
products for $0.2 b. However they would have to incur a shutdown
cost of $50Million and Plant B expansion cost of $ 200 Million.

III. Sample solution

As soon as I knew this was a merger case I followed the standard structure
of evaluating synergies in value chains and highlighting risks and risk
mitigation strategies. I examined three things:

• Value chains of both companies


• Synergies in value chain that would provide Cost Savings and
Incremental Revenues
• Cultural Integration issues.

Opportunities for cost savings

R&D
Patents
Combining Research Staff and cutting costs on Labs.
Knowledge transfer on existing projects.

Sourcing
Supplier consolidation : Higher buying power.
Access to new supply sources.
Knowledge of supplier previous contracts (transparency) with the other
entity.

Manufacturing:

23
Table of Contents

Economies of scale
More optimum capacity utilization ( if current plants were under-utilized)
Knowledge exchange on production best practices

Distribution:
Access to new distribution channels and new markets
Cross-Selling opportunities
Higher supplier power

Consolidating plants:
We would increase our profitability by $100 M per year at an additional
investment of $250M ($200M+$50M).

Other issues we have to consider if we shutdown Plant A.


Labor union issues.
Any Government Regulations/Labor Laws
Cultural integration.

Other issues: There are also significant pricing advantages that can be
gained through this merger. One of the firms charges more and has very
strong brand equity among the doctors. While no numbers were given,
discovering revenue synergies wins bonus points.

24
Table of Contents

Practice Case 29: Hedge Fund


(BCG - Round 1)

I. Case scope provided by interviewer:

Your client is $2B hedge fund. Previously they selected investments


based on a complex mathematical model. However, about 2 years
ago they started seeing lower returns. To address this problem they
added a research team to help evaluate investments.

If the model recommends an investment money is invested. If the


model and the analysts agree on an investment a larger percentage
is invested than if only the model recommends the investment.

The research team consists of 4 Senior Analysts and 3 Junior


Analysts. The CEO is extremely pleased with the new research team
and the uplift in returns they have seen as a result of the new
investment methodology. However, he does not feel that the team is
properly structured and has asked you to advise him on structuring
the team. What issues would you consider and how would you
structure the team.

II. Additional information provided after relevant questions:

• The overall goal of the hedge fund is to maximize returns and they
tolerate a high amount of risk
• Individual investors invest between $20M-$100M
• All seven researchers currently report to the head of trading
• When the trading department has a question on an investment the
head of trading will contact one of the Senior Analysts and ask him to
evaluate an investment. The Senior Analyst will then prioritize the
request and divide it between him/herself and the Junior Analysts
• Investment research (even an individual request) can usually be
broken into several discrete pieces
• The CEO does not want to fire anyone

After a brief discussion of compensation/rewards and reporting lines, the


interviewer directed me to the issue of leverage. Upon asking if the work
could be broken up between Junior and Senior analysts he provided the
following data:

• Both Junior and Senior analysts feel that they are doing work that is
beneath them
• If investment research work was pushed down to the lowest
competent level the work could be divided as follows

65
Table of Contents

Sr. Analysts Jr. Analysts Research Assistants


Data Gathering 25% 25% 50%
Analysis 20% 80% 0%
Synthesis 80% 20% 0%

If there is 100 hours of research work to do it would be broken up as follows


Data Gathering 20%
Analysis 60%
Synthesis 20%

Hot tip:
Considerations
How to compensate
Reporting lines
Appropriate leverage

III. Sample Solutions: Please synthesize how you cracked the case.

Using data provided on work allocation I calculated how many of each level
of worker would provide appropriate leverage to the senior analysts.

If a research project takes 100 hours it would be broken down between (in
hrs):
total time Sr. Analyst Jr. Analyst Assistant
Data Gathering 20 5 5 10
Analysis 60 12 48 0
Synthesis 20 16 4 0
TOTAL HOURS 100 33 57 10

Therefore, one senior analyst should have roughly two junior analysts and
one assistant can be shared between 3 Senior analysts.

Thus, based on the current four Senior Analysts our client currently has they
should create the following organizational structure

Sr. Analyst Sr. Analyst Sr. Analyst Sr. Analyst

Jr. Jr. Jr. Jr. Jr. Jr. Jr. Jr.


Analyst Analyst Analyst Analyst Analyst Analyst Analyst Analyst

Asst. Asst. Shared resources

66
Table of Contents

The following additional implications/points provide additional insight:


There will be additional capacity for the assistants; however, because they
are the cheapest resources you do not want them to be the bottleneck
There should be little resistance by any involved parties. While the Jr.
Analysts will no longer report directly to the Trading Head, they will no longer
be doing work that is “beneath them” and will have a manager trained in
their field.

67
Table of Contents

Practice Case 30: Paper manufacturer


(BCG - Round 2)
I. Case scope provided by interviewer:

Our client is a paper manufacturing company. Their product is


grease-proof paper which they sell in rolls to “bag converters.” Bag
converters are companies which combine 2-ply paper, popcorn, oil,
and a “receptor” to make microwaveable popcorn. (When
microwaved, the receptor gets hot and pops the corn.) Converters
supply retailers.

Our company uses wood pulp and chemicals to make paper. A recent
innovation at the R&D department has increased the grease
resistance of our paper by 10 times. The client needs help in
deciding on whether to switch its production facilities to manufacture
this new type of paper. (This is an either/or decision).

II. Additional information provided after relevant questions:


• The company is operating at capacity
• The market is pretty small—4 primary competitors dominate it
• Customers don’t care much about how grease-proof the paper is
• The costs to a converter are (currently):
• 30 cents-2-ply paper bag
• 10 cents-receptor
• 5 cents-popcorn and oil
• 5 cents—assembly
• Our current cost of producing the paper are (per 2-ply paper bag) 25
cents
• This cost is 50% fixed and 50% variable

III. Sample Solution:


My approach was to evaluate the additional value to customer and compare
it with the cost of switching to the new process.

First, I wanted to understand the benefit of this innovation. As end


consumers don’t care about the grease, we have to look to converters. I
suggested that they might be able to switch to 1-ply bags now, an important
change because bags represent 60% of their costs.

The interviewer confirmed this.

Proceeding from this assumption, I stated that as long as we could charge


less than the current 30 cents per bag, the converters would be willing to
purchase our product (there is no additional cost to them to use 1-ply instead
of 2-ply).

68
Table of Contents

The interviewer told me to ignore our switching costs and to look at


maintaining profitability.

I then evaluated change in our costs:


I assumed that fixed cost per bag will remain the same (50%*25 cents=12.5
cents)
I assumed that variable cost per bag would be cut in half (50%*50%*25
cents=6.25 cents)

The interviewer confirmed these assumptions.

The total cost per bag to us becomes 18.75 cents.


In order to maintain the current profit margin (30 cents-25 cents=5 cents)
we need to charge 18.75+5=23.75 cents
This is a good deal to the converter, who currently pays 30 cents

(You can look on this on per-sheet basis, instead of per-bag basis. In that
case, you need to make twice as much profit per sheet because you are
selling only half the volume. You should then charge 18.75+2*5=28.75.
This is still better than 30 cents.)

Strategic Implications:

This is a profitable proposal for both us and our customers

As we are currently at capacity, this innovation would in effect double our


capacity

We could capture some new business and increase our market share by
selling this cheaper, better paper, and now we have the capacity to meet this
increased demand

We should implement as soon as possible and lock in converters in long-term


contracts before our competitors can replicate this innovation

There will be switching costs, such as downtime, training, etc.

69
Table of Contents

Practice Case 31: Energy producer


(BCG - Round 1)
I. Case scope provided by interviewer:

Our client is in the energy business and makes electricity. The


CEO is deciding on making a capital investment to expand
capacity. How should we advise him?

II. Additional information provided after relevant questions:

• $100M investment to produce 2,000 units/week (units are


Megawatts of electricity).
• Client uses natural gas fired generators.
• Variable costs are gas and labor, gas costs $40/unit and labor is
$10/unit.
• Market price is $100/unit.

III. Sample Solution

What are the fixed costs involved in the investment?

$100M investment to produce 2,000 units/week (units are Megawatts of


electricity).

Before assessing the fixed costs, I want to know what are the major
inputs (i.e. raw materials) used in generating electricity.

Our client uses natural gas fired generators.

Ok, so, I think that the major variable costs involved would be the
following:
Natural Gas
Labor
Distribution – infrastructure to connect utility grid
What are the actual costs for each of these buckets of costs?

Let’s say that the variable costs are gas and labor, gas costs $40/unit and
labor is $10/unit.

Ok, so now that we have looked at both the fixed and variable costs
involved, I would like to assess the potential revenue that may be
generated from this investment.
Given that electricity is a commodity product, I would like to know the
current market price of electricity in our client’s market.

70
Table of Contents

$100/unit. Given the data you have now, will this investment be
profitable in the long-run and how long would it take to pay-back?

Calculate contribution margin and assuming they are able to sell the full
capacity of 2,000 units/week.

Revenue = 2,000 units/week * $100/unit = $200,000/week


Variable Cost = $50/unit * 2000 units/week = $100,000/week
So, they would make a profit of $100,000/week or $5,200,000/yr

So, the payback period for this investment is $100M/$5.2M ~ 20 yrs

How would you evaluate whether this was an appropriate investment?

Based on the payback period of 20 years, I would compare the payback


period to the useful life of the new plant.

The useful life is approximately 30 years. What else could you do?
Look at the payback period of similar investments our company has made
to determine if this was in-line with this payback period of those
investments. I could also use industry benchmarks.

Good. The payback period for similar investments is between 15-20


years. What else should you consider in assessing this investment.

Since this analysis assumed 100% capacity utilization, I would want to do


further sensitivity analysis to test that assumption. For example, what
would be the impact on our model if revenue if there was not enough
demand to justify the new capacity?

That’s a good point, how would you go about estimating new demand for
electricity.

So, the things that drive new electricity demand would be new home
starts, business growth – i.e. industrial complexes, growth in consumer
devices that use electricity, such as white goods.

What other trends or areas would you look at?

I would look at consumption trends such as the move towards low energy
appliances and how that might impact the projections.
I would also look at the moves that competitors are making, i.e. the
impact that new capacity from competitors might have on my ability to
set prices.

71
Table of Contents

Synthesis and Recommendation: Given that the payback period is within


same range as similar investments, I would recommend that the CEO go
ahead with the investment after considering some of the risks mentioned
(the sensitivity of demand and potential competitor moves).

72
Table of Contents

Practice Case 32: Consumer package goods


(BCG - Round 1)

I. Case scope provided by interviewer:

Our client is one of the world’s leading consumer packaged goods


producers. They are going to launch a new product and need to
decide which of two alternative methods to use to go to market. One
is the “direct store” system, the other is “warehouse delivery”.
Which one should they chose?

Interviewer Hot Tip: Profitability is the main driver. Therefore you need to
compare the cost structures of both thorough the value chain and potential
revenues. But never forget to consider other issues.

II. Additional information provided after relevant questions:

Currently both systems are used by 50%.


Per product costs through the value chain for both options:

Direct Store Delivery:


plant To distribution In dist. To In SG&A TOTAL
center Center store store
$1.75 $0.40 $0.35 $1 $1 $0.50 $5

Warehouse Delivery:
plant Retailer dist. Retailer Store clerk TOTAL
center ships merchandiser
$1.75 $0.30 $0.70 $0.25 $3

III. Sample Solution:

My recommendation to our client will be based on the most profitable


delivery system. Therefore I will first compare both systems in terms of costs
and revenue potential.
Good.

(Then I got the above data). So direct store delivery seems more costly.
What are the prices we charge for both?
Price per product charged in direct store delivery is $7 and $6 for warehouse
delivery.

So contribution margin for direct store delivery is $2 and it is $3 for


warehouse delivery.
So will you recommend warehouse delivery?

73
Table of Contents

Not so quick. I presume that direct store delivery might have some
advantages over warehouse delivery that affects sales positively. So at the
end, direct store delivery may be more profitable.
Like what?

We probably attach more importance to keep our products in healthy


conditions in our own distribution centers. We can take additional precautions
to prevent waste. However the retailers might not be that sensitive because
most of the cases they return the waste products to the manufacturers.
Second, we can also ship with less damage, because our current trucks that
we use in direct delivery are designed for our own products. However the
retailer is carrying lots of stuff.
In direct delivery, our own employees will put our products to the shelves.
They take care of shelf design more than the retailer.
All those factors might contribute to the sales increase.
Good. Thank you.

The important point in this case was not to limit the analysis with the
profitability per product. There are also qualitative issues (which can be
quantified if the interviewer has data) to consider in order to come up with a
recommendation.

74
Table of Contents

Practice Case 33: Hybrid corn producer


(BCG - Round 2)
I. Case scope provided by interviewer:

Corn farmers usually save a portion of the harvest to use as seed for
planting it next year. However, in countries with high GDP, farmers
typically purchase and plant hybrid corn, which is bred for specific
characteristics, such as insect resistance. Another important
characteristic is yield—how many bushels per acre can be grown
from the seed.

Our client has developed a new hybrid with higher yield and would
like to know how to price it.

II. Additional information provided after relevant questions:


• This hybrid increases the yield from 80 bushels per acre (industry
average) to 90 bushels per acre.
• Farmers sell corn at $10 per bushel.
• 2 bags of seed corn are needed per acre, regardless of whether regular
or hybrid seed corn is used.
• Cost to farmer per bag is $80.
• No additional costs (fertilizer, equipment, time, etc.) are incurred by
farmers when switching from regular seed corn to hybrid corn.
• There is no cost to client to switch to producing this new hybrid. The
development costs have been taken care of in the R&D budget. (You
can make up these numbers when giving the case to make it more
interesting).

Complication:

The interviewer complicated matters by telling me that a weather event


occurred 30 times in the last 150 years, which caused the crop to be
completely wiped out. Determine how this would impact pricing

III. Sample Solution:


I suggested determining the pricing strategy by looking at value to customer
and comparing that to our cost. I asked questions related to this approach
(understanding the customer’s business model)

Value to customer is:

Extra 10 bushels/acre*$10/bushel=$100/acre
• This is pure profit, because the customer still only has to buy two bags
of seed corn. The additional value to customer per bag is therefore
$50 (because 2 bags are needed per acre).

75
Table of Contents

• Based on these data and the current cost per bag, we can charge up to
$80+$50=$130/bag.

When I asked whether the value of this hybrid is apparent to the customer, I
was told that research has been published in a magazine that farmers trust,
so this shouldn’t be hard to sell.

The cost to the client is the same, so we can charge up to $120 per bag (we
would want to leave some money on the table to induce purchase and gain
additional market share).

We should launch this as soon as possible to lock in contracts with farmers


before our competitors get to market with their version of this product.

Complication:

All of the above seemed pretty obvious. The interviewer complicated matters
by telling me that a weather event occurred 30 times in the last 150 years,
which caused the crop to be completely wiped out. I was asked whether and
how this would impact pricing.

As a result, the value to the farmer is reduced (the payoff is uncertain:


30/150=20%). The value becomes 80%*$50=$40, so we can charge up to
$120 per bag.

76
Table of Contents

Practice case 34: Bank merger


(BCG - Round 1)

I. Case scope provided by interviewer:

Bank A and Bank B are negotiating to merge their check processing


facilities. Check processing consists of all the steps required to
process a check received in any bank branch, from collecting all the
checks in all the bank branches and sorting them through making the
necessary balance adjustments to the accounts involved and filing
them away.
You were hired to analyze this merger.

How would you do it?

First tell me the major data / information you would need to make a
thorough analysis and then whether or not you would recommend it.

II. Additional information provided after relevant questions:

• Investment to make the merger = $200 M (shared in half by both


banks)
• Bank A current check processing unit cost = $ 0.4
• Numbers of check processed by bank A in a year = 200 M
• Bank B current check processing unit cost = $ 0.5
• Numbers of check processed by bank B in a year = 200 M
• Cost of capital = 10%
• Bank A and B estimated the check processing cost per unit if the
merger occurs = $ 0.36

III. Cracking the case:

First, I mentioned all relevant points necessary to analyze the case (financial-
>investment needed, cost of capital, expected benefits due to merger;
operational-> how the checks are collected from the branches, how they are
processed, synergies in terms of process and best practices, etc, so we could
see whether there is any room for improvement. Then I said that we should
do an NPV analysis to verify the viability or not of the merger.

If you work the numbers you will get the NPV for bank A -20M and for B
+80M. So the merger would be good for bank B but not for bank A. However,
since the overall combined unit cost is reduced by the merger, it creates
value and should be made if the NPV for A were also positive. To do this we
should reduce the investment for bank A and increase the investment for

77
Table of Contents

bank B. The best case scenario would be splitting the added value by
merging the banks. That is, making both banks realize the same NPV
through the merger. For this to happen, bank A should invest 50M and bank
B 150M. This way both banks would end up having NPV of 30M and the
merger will likely go through.

78
Table of Contents

Practice Case 40: Telecom provider


(BCG - Round 1)

I. Case scope provided by interviewer:

Our client is large telecom service provider. In the past the telecom
industry has enjoyed a monopoly as various regional telecom
companies established monopolies under government protection.
Recently the US government has passed a new law that allows
competitors to enter other regional areas where there is already an
incumbent present. The competitor (new entrant) can lease
incumbents infrastructure and target incumbent’s customers. The
incumbent will HAVE to charge the price set by the govt. The client
has seen some cannibalization in their region so they set up a new
division to penetrate other regions. However, for the last 2 years that
venture/division has been unprofitable. The CEO wants your
judgment before he makes a decision to shut the division down.

II. Additional information provided after relevant questions:

• Number of current customers signed up by new group: 1MM


• The avg. price per customer per month of $38.60. It is 10% less than
the competition.

Cost structure
• SGA is $5.10 – half of which is FC and other half is VC.
• Client believes that they can reduce SGA by 10% at most without
affecting quality of business.
• Government has set a price of $36.90 that client has to pay to the
competitor per customer per month.

III. Sample Solution:

I would like to understand the revenue structure of the client. Can you tell
me how many customers does the client have?

1 MM

I think that client has done really well by gaining 1MM customers in just 2
years in a new territory. Seems like the client has good service offerings.

The client offers a very good quality service.

What about prices? Is there also a standard price for each customer or is
there some price discrimination?

91
Table of Contents

You are right there is some price discrimination but I have the avg price per
customer per month of $38.60. This price is 10% less than the incumbent
player.

Why is client charging 10% less than the incumbent?

Interviewer: What do you think?

Me: I think this is an aggressive pricing policy to penetrate a new region

Interviewer: Sounds good

Let me go to the cost drivers now. Could you tell me the FC and VC drivers?
Obviously the govt set price will be a big factor.

SGA is $5.10 – half of which is FC and other half is VC. Also, govt has set a
price of $36.90 that client has to pay to the competitor per customer per
month.

Ok, looks like the client is not even making money at an operating level. I.e.
ignoring FC client is still losing money per customer. Obviously, the $36.90
that client pays to incumbent cannot be changed because it is set by the
government policy. I’d like to look at SGA and understand whether client is
operating at industry average or not.

Good point. Client believes that they can reduce SGA by 10% at most
without affecting quality of business.

OK, reduction of 10% on $5.10 of SG&A does not even let the client break
even. You mentioned that there is some price discrimination and the price
you gave me was an avg price. Could you tell me about customer segments?

Yes, there are non-business consumers and business consumers.

Probably the price to business customers is higher as compared with non-


business. Because, of peak load pricing and inelastic nature of business
consumers.

You are right.

Could you tell me what percentage of client customers are business vs. non
business?

90% non business and 10% business.

The client is heavily relying on a very price sensitive segment of the market
and thus pressured to reduce margins. I recommend that the client should
not close the non-business division but try to switch from targeting non-

92
Table of Contents

business to targeting business consumers and create an opportunity to


charge higher prices.

This is what BCG recommended the clients. Thank you.

93
Practice Cases – Case 5: GenCo BCG
(Source: Sample case from BCG website)

Context
Your client is GenCo, a large, international, diversified company with a health care
division that produces a wide variety of medical instruments and related services.
Five years ago, it expanded into the health care software industry by purchasing
MedCount, which markets administrative systems to large U.S. hospitals. These
systems are designed primarily for back-office functions; they are not designed for
managing patients or providing other physician and technical support. Since it was
purchased, the software division has failed to deliver the growth needed to justify
the multiple GenCo paid for it. GenCo feels it has already squeezed margins as much
as possible, and now is looking for new sales opportunities. MedCount turned to BCG
to help identify potential ways to increase revenues. How would you approach this
problem?

Establish understanding of the case

Interviewee: First, let me make sure I understand the problem. The parent company produces
medical devices and services, but before the acquisition was not involved in health care
software. The company it purchased, MedCount, sells only administrative systems software to
large hospitals. It is now looking for opportunities to increase revenues.

Interviewer: That is correct.

Interviewee: Could I take a moment to jot down a few thoughts?

Interviewer: Sure, that would be fine.

Set up the framework

Interviewee: I would suggest using the following structure to approach the problem:

• First, I'd want to understand the market size and growth rates for MedCount's market
and related software markets.
• Next, I would like to explore the competition and their market shares.
• Third, I would like to examine customer requirements and then, given those external
conditions, look at the division's capabilities to understand how well prepared it is to
meet the needs of the marketplace.

Interviewer: That sounds fine. So what do you want to know about the market?

Evaluate the case using the framework

Interviewee: Well, the first hurdle would be to identify the markets the company would be
interested in. Besides administration systems, what other types of medical software systems
do large hospitals purchase?

Interviewer: There are many software systems, but for the sake of time, the team
focused on three primary markets: administration systems, patient administration,
and physician support systems.

Interviewee: What do those systems do?

42
Practice Cases – Case 5: GenCo BCG
Interviewer: Patient administration includes systems such as admissions and
tracking. Physician support systems are more specialized, for individual physician
procedures.

Interviewee: I would like to know how large each market is and how fast each is growing. I
would use secondary sources such as press releases, analyst reports, and published market
studies, to obtain this information.

Interviewer: Great! That is what we did during the market study. Our information
revealed the following market sizes and growth rates.

Patient Physician
Administration
administration support

Market size
1,500 1,000 1,200
($M);

Growth rate 5% 5% 12%

Interviewee: From a size and growth perspective, physician support systems looks like a
very attractive market. I'd like to know a little about the customers themselves. The client is
currently targeting large hospitals. Approximately what percentage of the market do they
represent?

Interviewer: We were unable to get an exact breakdown, but we know that these
hospitals make up the vast majority of the total medical software market.

Interviewee: That would make sense, since the more sophisticated procedures at a hospital
might necessitate more advanced software solutions. I know that there have been a lot of
changes in the industry as a result of managed care. I don't know much about the industry, so
I would want to look at market studies and press clippings to get a better sense of the hospital
market in general and any technology or software trends more specifically.

Interviewer: Okay. Let's say that you did that and were presented with this
summary of market trends:

• Consolidation in the industry, with three to four large hospital networks


dominating 45 percent of the market
• Cost controls instituted, particularly as these large hospital networks acquire
smaller hospitals (centralization of functions being a key cost issue)
• Many hospitals seeking to consolidate their vendor base
• With regard to technology, many hospitals upgrading their older systems

Interviewee: If hospitals are consolidating vendors, perhaps our client has an advantage in
being part of a larger medical company. Maybe the client could also gain some advantages by
expanding into other software segments. Are the people responsible for purchasing software
at the hospital the same for all three segments?

Interviewer: Like all things, it differs by hospital, but the larger hospital networks,
have tried to consolidate their purchasing not only within but also across hospitals.

Interviewee: Is the decision maker for medical software the same as for medical
instrumentation and devices?

43
Practice Cases – Case 5: GenCo BCG
Interviewer: In some cases, the head of purchasing influences both decisions, but
the person who makes the final choice is different. Software decisions are usually
made by the hospital IT function, and those for instrumentation by the medical staff.

Interviewee: I think I have a pretty good understanding of the market for now. Let's look at
competition next. We could identify all the competitors and build up the market shares using a
combination of public data and estimates.

Interviewer: Well, let's assume that you don't have an infinite amount of time to
look at all the competitors. You can only look at the top five competitors in each
market. You are given the following data:

Administration Systems Sales ($M) Growth (%)

MedCount 700 4%
HCS Software Systems 100 7%
Morningside Software 80 3%
Admin Systems Solutions 70 2%
HTI Software 50 15%

Patient Administration Sales ($M) Growth (%)

HTI 300 5%
Registration Software Solutions 240 4%
Signup Software 60 3%
HCS Software Systems 30 16%
Patient Software 20 -1%

Physician Support Sales ($M) Growth (%)

HCS Software Systems 150 16%


Physician Support Systems 100 11%
Medical Technology Inc 25 18%
HTI 20 32%
MedSys 5 15%

Interviewee: Very interesting. The first thing I would note from the data is that the market
concentrations are very different. In administrative systems, the top five competitors control
66 percent of the market and in patient administration, they control 65 percent. But in the
physician support market, they control only 25 percent.

I would want to know what gross margins look like in each of these markets as well. I might
turn to analyst reports and look at competitors' financial statements to deduce whether they
are making money in each market.

Interviewer: Gross margins vary, of course, but the analyst reports have margins of
25 to 30 percent for administrative systems and for patient administration. For
physician support, the margins tend to be higher, more like 45 to 50 percent.

Interviewee: I see that two competitors, HTI and HCS Software Systems, have very large
revenue growth in all three sectors, although they each dominate one. I would want to look at
their financials, annual reports, and press releases to find out a bit more about their strategy
in each of these areas.

44
Practice Cases – Case 5: GenCo BCG
Interviewer: You'd find that they recently entered these noncore markets. Why
might they have done that?

Interviewee: Perhaps, like our client, each had a strong position in its own segment, HTI in
patient administration and HCS Software Systems in physician support. Maybe they too
decided to branch out into the other segments to find additional growth.

Interviewer: That is a very good hypothesis. Let's say there is evidence in the
sources you consult that supports your assertion.

Interviewee: Well, if that were true, these two companies could be a threat not only in the
other two segments, but also in our client's segment, administrative systems. It looks as if the
client is slowly losing market share in its segment, since it is growing more slowly than its
market.

Interviewer: Good observation.

Interviewee: The market and competitor trends could also suggest that the client may want
to enter these other markets. In particular, the physician support market looks attractive,
given it has high growth and lack of a dominant competitor. The higher gross margins may
provide attractive returns on the necessary investment in software development.

However, the patient administration market may also be attractive. Although it is more
concentrated and offers lower margins than physician support, the client may be able to enter
this segment with a smaller up-front investment. Given the trend toward upgrading existing
computer systems, it may be important for MedCount to have a product offering in each of the
three market segments. That should not be too difficult, since the company is already in the
software industry.

Interviewer: Perhaps, but you should think a little more closely about these types of
software. Are all software systems alike?

Interviewee: Well, let me think about that for a moment. I suspect patient administration
would have relatively low entry barriers. From your earlier description, these systems appear
to be pretty basic, dealing primarily with admissions and patient tracking. However, the entry
barriers in physician support might be higher, since these systems are more complex and
there are probably multiple systems for the various physician procedures. I guess it would be
harder to get into those types of systems.

Interviewer: That would make sense.

Interviewee: Since the company might want to go into only some of the segments, I would
want to know how important it is to have products in all three segments. Do we know if the
competitors are marketing their products as a bundle?

Interviewer: How might you find that out?

Interviewee: Since it would be difficult to talk to a competitor directly, I would probably


target a competitor's customer, particularly one that just converted from our client's software.

Interviewer: Let's say you get an interview with a customer that recently switched
to HTI. You discover that the competitor was offering it a better pricing deal and
service for software products in all three segments.

45
Practice Cases – Case 5: GenCo BCG
Interviewee: How were MedCount's software and service perceived in relation to those of
competitors?

Interviewer: The customer thought that its administrative systems were adequate,
"the old standby," but not stellar.

Interviewee: Were there any other key reasons it switched from MedCount's system?

Interviewer: When it decided to upgrade its systems, it tried to contact MedCount,


but could never get a representative to describe its options.

Interviewee: Interesting. How did HTI perform?

Interviewer: The HTI representative had heard that the company was considering
switching software vendors and provided a sales representative to pitch HTI's
administrative product the next day.

Interviewee: It definitely sounds as if there was a problem with the sales function and that
customer relations need to be improved, particularly for the larger hospital chains. There also
seems to be an advantage from both a marketing and sales perspective in having multiple
software products. I would want to confirm those views by doing further interviews.

Interviewer: Let's say further interviews support those assumptions.

Interviewee: Since we have already looked at the external conditions, I would like to move
on to the client itself. I'd like to know more about its marketing and selling organization as
well as its software development skills.

So far, we know that our client offers administrative software and that there may be a
problem with sales and marketing. Could you tell me a little about the marketing department?

Interviewer: The marketing department is organized regionally. Teams are assigned


to hospitals within each state or geographic region, such as New England.

Interviewee: That could explain some of the problems with MedCount's marketing and sales.
If hospital purchasing is centralized, the marketing organization may be outdated. Does the
company have any teams dedicated to the four or five biggest hospital networks?

Interviewer: No, there are no dedicated teams. They talked about doing that for a
while, but it conflicted with the regional structure it had in place.

Interviewee: With regard to software, does the company feel it has any strengths or
weaknesses?

Interviewer: It feels that their administrative product is very strong ("best of breed")
and is the dominant technology. Also, the product is modular in design, which allows
for easier upgrades. Although the company has never branched out into other
market segments, the software developers believe that certain modules could be
used to build the foundation for other administrative software programs. The
company feels customer support is also an area in which it excels.

46
Practice Cases – Case 5: GenCo BCG
Summarize and make recommendations

Interviewee: Let's start with our client's market. The client dominates the administrative
software market, which is fairly large but growing slowly, and the company appears to be
slowly losing market share. Patient administration is also growing relatively slowly.

Interviewer: Both markets are relatively concentrated and appear to offer lower
margins than physician support. The physician support market is large and less
concentrated, and could potentially provide higher margins, but would require a
larger investment. The hospital market itself is becoming more concentrated and is
pushing to consolidate vendors. The purchasing agent is often the same for the three
types of software.

Interviewee: Looking at our client's competitors, two, HTI and HCS Software Systems,
appear to be particularly threatening. Each has a dominant position in one segment and is
branching out into other areas. They appear to be marketing their products and services as a
bundle and are using service as a key point of differentiation.

The client offers only one type of system and appears to have some weaknesses in its
marketing organization, particularly in marketing to the larger hospital networks, which offer
the most promising market opportunities.

Interviewer: How would you recommend proceeding?

Interviewee: The first priority should be to fix the marketing organization, particularly for
the large hospital networks. MedCount will have trouble expanding into new markets if it can't
defend its current position and shore up its existing customer relationships. There should be a
team dedicated to each of the major chains. The client should also look at improving customer
tracking so that it is clear when its customers are going to upgrade. There should also be clear
contacts so that the customer can easily keep in touch with MedCount.

Next, I would recommend that the client explore entering the other market segments by
leveraging its dominant position in administrative systems. At first glance, patient
administration does not appear to be very attractive, with slow growth, low margins, and large,
dominant competitors. There appears to be some advantage, however, in having products
across the product range. I would recommend that we interview some of MedCount's existing
customers to better understand their needs and future IT requirements. If the customer base
is interested in one software provider for both back-office administration and patient
administration functions, this segment looks promising.

If the client does decide to enter this market, it should look at the lowest-cost method of entry,
either developing a product internally or acquiring a competitor. The modular design of its
existing administrative software suggests internal development of the patient administration
product may be the way to go, but we would need a more thorough comparison of the internal
development and acquisition options, including both cost and time to market. I think that
physician support offers our client an exciting growth opportunity, given its high margins, high
growth, and fragmented competition. I would definitely think about an acquisition strategy,
since the client may lack the technical capabilities to enter this specialized market. I would
recommend going for one of the larger companies, as that would give the client a stronger
position. Smaller companies would probably not offer an important enough position in the
market. More research would be needed, however, for us to better understand the intricacies
of the market and each potential acquisition.

Interviewer: Those are very interesting conclusions. Thank you.

47
Practice Cases – Case 6: Sugar Cereal BCG
(Source: Sample case from BCG website)

Context
Your client is the sugar cereal division of Foods Inc., a U.S.-based distributor and
manufacturer of packaged foods. According to the division president, Foods Inc.'s
traditional strength has been with grocery stores, which still account for the
majority of its $1.1 billion in sugar cereal sales. But Big M Mart, a discount chain, has
been growing at a healthy rate of almost 15 percent per year and has now become
Food Inc.'s largest customer. Your client is not sure how to react, and has asked BCG
for assistance with its distribution strategy

Establish understanding of the case

Interviewee: First, let me make sure I understand the problem. Our client specializes in
sugar cereals traditionally distributed through grocery stores. Sales to Big M Mart, a discount
chain, have been growing at 15 percent per year, and the chain has recently become the
largest distributor of the client's product nationwide. We are here to help evaluate the
distribution strategy in light of Big M Mart's growth.

Interviewer: That is correct.

Interviewee: Could you explain to me how grocery stores differ from discount stores?

Interviewer: Sure. Grocery stores generally specialize in food, as well as selling


some household goods and over-the-counter pharmaceuticals. Discount stores, on
the other hand, offer food alongside a wide variety of merchandise, including
clothing, home electronics, and housewares.

Interviewee: Does Big M Mart market its food products differently than do grocery stores?

Interviewer: Discount stores advertise lower prices for a wide variety of foods,
particularly staple, nonperishable foods.

Interviewee: Could I take a moment to write a few notes to myself?

Interviewer: Please feel free.

Set up the framework

Interviewee: Before making recommendations, I think we would need to evaluate


whether sales growth at Big M Mart is good or bad for Foods, Inc. To do that, I would
first look at how its sugar cereal performance at Big M Mart compares with that in
other distribution channels. Second, I would look at its performance at Big M Mart in
relation to competitors' performance. Next, I would determine what drives customer
purchases. Finally, I would want to understand the supply chain.

Interviewer: That certainly sounds like a reasonable approach. Let's proceed.

Evaluate the case using the framework

Interviewee: First, I would like to get a better sense of where Big M Mart stands in relation
to our client's other distribution channels by examining the client's sales data and margins, by
distributor.

48
Practice Cases – Case 6: Sugar Cereal BCG
Interviewer: The marketing department does not have margins by channel, but
tracks sales and volume for its top five distributors.

Sales ($M) 1997 1999 2001 5-Yr CAGR


Big M Mart 142 162 246 14.7%
R.J.'s 157 185 200 6.2%
Bozo Mart 143 175 189 7.3%
Ace Grocery 101 109 153 11.0%
Shoppers Mart 57 62 67 4.0%
Total Top 5 600 693 856 9.3%
Total All Distributors 1,000 1,079 1,150 3.6%

Volume (M boxes) 1997 1999 2001 5-Yr CAGR


Big M Mart 65 74 113 14.7%
R.J.'s 72 81 85 4.2%
Bozo Mart 65 77 80 5.2%
Ace Grocery 46 47 64 8.8%
Shoppers Mart 26 27 28 2.0%
Total Top 5 274 307 370 7.8%
Total All Distributors 450 468 487 2.0%

What does this imply about Big M Mart as a distribution outlet?

Interviewee: It looks as if the top distributors have been growing more important, but
particularly Big M Mart, which is growing faster than all the others. This is particularly true
when we look at volume, where Big M Mart's growth is much higher than that of the other four
channels.

Interviewer: And how could you interpret what these data says about margins?

Interviewee: While the client's sales through other distribution channels are growing faster
than volume, Big M Mart volume and sales growth are the same, so the average price paid by
Big M Mart has remained constant. That implies that sales growth at Big M Mart could have
negative implications for our client's margins.

Next, I would like to look at how our client is doing in relation to the competition within Big M
Mart. Have they been gaining or losing market share?

Interviewer: How might you find that out?

Interviewee: I would try to interview Big M Mart's purchasing personnel, since they would
probably track those data for their own purposes.

Interviewer: Why would they want to talk to you? How might you approach such an
interview?

49
Practice Cases – Case 6: Sugar Cereal BCG
Interviewee: I would approach the purchasing personnel and suggest that our client and Big
M Mart work together to identify best practices to reduce costs and increase sales of sugar
cereals at Big M Mart.

Interviewer: Let's say in a perfect world you could get a breakdown of Big M Mart
sales for the four largest competitors (see market shares below).

What can we infer about our client's competitors within this channel? Who should
they be worried about?

Interviewee: It looks like our client is losing market share, as is Tasty Breakfast, while
Cereal Co. and Private Label are gaining share. Private Label, however, looks to be growing
from a very small base.

I would like to explore why our client is losing market share to Cereal Co. at Big M Marts. Are
their prices better than those of our client?

Interviewer: After a period of price wars six to seven years ago that lowered
industry margins, the cereal companies have refrained from price competition within
the same channel.

Interviewee: If prices are not driving the difference, I would look at other factors such as
brand selection, percentage of shelf space, product placement, and in-store promotions.

Interviewer: Visits to Big M Marts indicate that each name-brand company holds 30
percent of the shelf space, while private label has 10 percent. Cereal Co. brands,
however, tend to be placed lower on the shelf than your client's products.

Interviewee: Well, I suspect that children are a large target market for the sugar cereal
manufacturers. The lower shelf placement could be especially important to children who are
looking at the different types of cereals. Are there any other promotions?

Interviewer: Some Cereal Co. brands have sales promotion tags, and the team notes
that store flyers advertise specials on Cereal Co. brands for Big M Mart customer
cardholders.

Interviewee: So, even if all the companies are maintaining product prices, maybe Cereal Co.
is strategically discounting prices to gain market share.

It seems as if there is evidence of cooperation between Cereal Co. and Big M Mart. Do we
know anything about their relationship?

50
Practice Cases – Case 6: Sugar Cereal BCG
Interviewer: During earlier discussions with Big M Mart, you discovered that your
client's competitors have 50 sales representatives dedicated to the Big M Mart
account. Your client has seven.

Interviewee: Cereal Co. appears to be dedicating more resources to its relationship with Big
M Mart than our client is. This may explain its better product placement and promotion
programs.

I think I have a good sense of distribution and competition. I would now like to look at the
customers and understand why they select the products they do. One hypothesis I have is
that shifting brand loyalties are hurting our client's market share at Big M Mart.

Interviewer: That's interesting. What do you think might motivate purchases of


sugar cereals?

Interviewee: There are lots of factors, such as the games in the boxes, the price of the
cereal itself, how it tastes. To better understand consumer behavior, we might conduct market
research, possibly through focus groups, customer observation, and price sensitivity studies.

Interviewer: BCG teams often do such research. Let's assume your team conducts
some analysis. Your research concludes that most buyers tend to fall into two
categories. Approximately 60 percent of buyers go straight to one cereal and grab it.
We can call this group the "brand-loyal" shoppers. Another 40 percent of shoppers
look at all the cereals and then select one that interests them. Let's call this group
the "impulse" buyers.

Interviewee: For the brand-loyal shopper, the priority would be product availability, while
product placement would be important for consumers who like to shop around.

Within these groups, are consumers price sensitive such that one brand can lure shoppers
loyal to another brand?

Interviewer: In general, your research indicates that consumers are not price
sensitive and are extremely loyal to their preferred brand. But when the preferred
cereal is unavailable, the brand-loyal customers will purchase discounted cereals
approximately 35 percent of the time.

Interviewee: Well, from that information, it appears that price is not a major driver of
purchases unless the preferred cereal is out of stock. In these stock-out situations, you said,
brand-loyal customers will purchase discounted cereals 35 percent of the time. What happens
when the customer does not purchase a discounted cereal?

Interviewer: In approximately 25 percent of cases, the customer walks away


without purchasing any cereal at all. In the remaining 40 percent of cases, the
brand-loyal customer will act like an impulse shopper and select another brand.

Interviewee: Interesting. It seems as if product availability could be a major driver of total


cereal volume for Big M Mart. Of course, we would need to know how often stock-outs occur
that cause consumers to walk away without purchasing cereal occur.

Since I have a pretty good understanding of customer motivation, I'd now like to ask a few
questions about the client's supply chain. I would want to talk to our client's distribution
personnel to understand the distribution process and to determine how often stock-outs occur.
Can you describe how our client's cereal is distributed at Big M Mart?

51
Practice Cases – Case 6: Sugar Cereal BCG
Interviewer: Cereals are distributed from the factory to the distributor's warehouse
twice monthly. The retailer then stocks the shelves itself.

Interviewee: Do we have any knowledge about when the individual stores are out of stock?

Interviewer: No, we do not, since our client only delivers to the warehouses and has
no direct access to in-store inventory information.

Interviewee: Since we identified product availability as a key success factor earlier on, I
would want to make sure that the stores were stocking the product correctly.

Interviewer: Let's say that in your earlier in-store investigations, you found out that Big M
Mart stores averaged 15 percent of sugar cereal brands out-of-stock, across all brands.

Interviewee: Stock-outs would be a major problem for our client, since 60 percent of
customers look for a specific brand of cereal and 35 percent of them would buy a discounted
brand in a stock-out situation. Big M Mart would also have an incentive to reduce out-of-stock
incidents, since 25 percent of the time, a brand-loyal customer will walk away without buying
anything.

Summarize and make recommendations

Interviewee: Big M Mart is our client's leading customer, accounting for more than 20
percent of our client's sugar cereal revenue. Although sales to Big M Mart are increasing on an
absolute basis, our client's margins there are lower than in its other channels and its
competitive position is eroding in that channel.

At Big M Mart, our client faces competition from both private label and Cereal Co., although
the latter appears to be the greater threat. There appears to be a relationship between Big M
Mart and Cereal Co. as evidenced by their joint promotions, the superior placement of the
Cereal Co. product, and the substantial resources that Cereal Co. has dedicated to the Big M
Mart account.

We learned that 60 percent of customers are brand-loyal, implying product availability is most
important. However, 40 percent like to try different kinds of cereal, indicating product
placement is also important. Purchasers do not appear to be price conscious, unless the type
of cereal they are looking for is out of stock, in which case there is a stronger tendency to
base purchases on price promotions.

In terms of distribution, our client is making deliveries twice a month to Big M Mart's
warehouses. Big M Mart, in turn, is responsible for stocking the shelves. We currently have no
direct knowledge of when our client's items are out of stock at the individual stores, but there
is evidence that stock-outs do occur with some frequency.

Interviewer: Well, it sounds as if you understand the situation. What would you
recommend the client do?

Interviewee: The sales through Big M Mart appear to have a negative impact on the bottom
line, as they have lower margins than sales through grocery stores. The client could work with
grocery stores to ensure that they are able to compete effectively with Big M Mart in the sugar
cereal market. This strategy could be risky, however, since Big M Mart is a large and important
customer. Therefore, I would recommend that our client work more collaboratively with Big M
Mart.

To defend its current position at Big M Mart stores, the client should move toward a
partnership with Big M Mart and dedicate more resources to the relationship. The customer

52
Practice Cases – Case 6: Sugar Cereal BCG
and competitor data indicate that our client's first priority should be to improve distribution to
ensure better product availability. In addition, it should push for product placement equal to, if
not better than, that of its competitors.

Interviewer: Why would Big M Mart be willing to enter into a partnership with Foods
Inc?

Interviewee: Foods Inc could offer to share its information about customer behavior to help
increase revenues for both itself and Big M Mart. Stock-outs hurt Big M Mart in two ways. First,
some brand-loyal customers simply walk away without purchasing cereal whenever their
preferred brand is unavailable. Second, we know that other brand-loyal customers purchase
lower-priced cereal whenever they encounter a stock-out of their preferred brand. Both of
these instances lower Big M Mart's revenue.

By eliminating stock-outs, Big M Mart could increase its sales by simply ensuring that
customers don't walk away without making a purchase. Converting these purchase occasions
to sales would increase Big M Mart's sales of sugar cereals by more than 2 percent(1).

Better availability also helps Big M Mart and our client increase their revenue by deterring the
brand-loyal shoppers from trading down to lower-priced cereals. Recall that 35 percent of the
brand-loyal shoppers purchase a discounted cereal if their preferred brand is not available. If
improved distribution now makes the preferred brands more consistently available, the
customers will pay a higher price for these products.

Finally, we could use the information about consumer purchase behavior to help persuade Big
M Mart to share information about product availability in its individual stores. We could work
with our client and Big M Mart to improve the current distribution system to allow for more
economical deliveries, while at the same time ensuring that our client's product is consistently
available in the store.

Interviewer: Thank you. Those sound like solid recommendations, but I would
suggest that you fully understand the root cause of the stock-out situations and the
cost to eliminate them before moving ahead.

(1)
15 percent out of stock x 60 percent brand-loyal customers x 25 percent willing to forgo
purchase = 2.25 percent

53
Practice Cases – Case 7: Fighter Jet BCG
(Source: Sample case from BCG website)

Context
Your client is a U.S. defense contractor that manufactures the Mohawk Light Fighter
Jet for the British Royal Air Force. The company has produced the $20 million fighter
jet for the past 12 years. The British government has decided to put the contract out
to bid, however, and to win the program, the client's purchasing agents have
estimated, the company will need to cut its costs by 5 percent. It has asked BCG to
help it reduce costs.

Establish understanding of the case

Interviewee: Let me first clarify the question. The client manufactures a $20 million jet and,
because of competitive forces, has to reduce its cost by 5 percent. Is BCG's role also to verify
the purchasing department's estimate?

Interviewer: No, you can assume that the purchasing estimate is correct. BCG's role
is to find the cost savings to meet that estimate.

Interviewee: Could I take a few minutes to think about the case?

Interviewer: Sure, please do so.

Set up the framework

Interviewee: First, I would like to understand the cost structure of the jet to see what we
should look at first. Next, I would like to look at major factors driving the costs we are
targeting. Finally, I would like to explore potential ideas to reduce cost.

Interviewer: That sounds like a very logical approach. Let's proceed.

Evaluate the case using the framework

Interviewee: Because the time for the interview is limited, I think we should try to identify
those areas most responsible for the cost of the jet.

Interviewer: Time is limited on real projects as well, so I think that would be a good
idea! You have the following cost information for the jet. How would you interpret it?

54
Practice Cases – Case 7: Fighter Jet BCG

Interviewee: The major cost driver for the jet appears to be purchased materials. Within
manufacturing, direct labor is a fairly large component of cost, as are program management
and corporate overhead within overhead. I think we would want to concentrate most on
materials, however, since that's where most of the costs can be found.

Interviewer: That sounds like a good place to start. Where would you look within
materials?

Interviewee: I see that materials are broken down into purchased subassemblies,
components, and raw materials. I understand what raw materials would be, but what would be
the difference between components and subassemblies?

Interviewer: A subassembly functions on its own. An example is the pilot night


vision system. A component is a smaller part, such as a part of the engine.

Interviewee: I know that governmental agencies often have very strict guidelines about
purchasing that could affect the cost of materials.

Interviewer: For the sake of this case, you can assume that the British Ministry of
Defense, MOD, allows "commercial off-the-shelf" purchases, which means that the
client is free to purchase from whomever it wants, as long as it can ensure that the
parts meet MOD quality guidelines.

Interviewee: I see that purchased subassemblies comprise more than 70 percent of


materials. How many suppliers are there for these subassemblies?

Interviewer: There are seven suppliers of major subassemblies that go into the
fighter jet.

55
Practice Cases – Case 7: Fighter Jet BCG
Interviewee: That seems like a relatively small number. Are there more suppliers that are
qualified to do this type of work?

Interviewer: The manufacture of these parts requires a substantial investment in


R&D, engineering, and infrastructure. It would be very costly for new suppliers to
make the required investment, particularly if the client is trying to reduce the price it
pays to the subassembly manufacturers.

Interviewee: Since there are only a few subassembly suppliers, and the investment hurdle
would preclude bringing in competing manufacturers, it would be difficult to reduce the price
paid. Perhaps we should look elsewhere for savings.

Interviewer: But remember, if your client loses the contract, it will lose its customer
unless it is teamed with the competing bidder. Even then, if the competitor is
underbidding your client, there will be even less room for it to profit.

Interviewee: Perhaps it would have an incentive to reduce its costs in order to maintain the
contract. Are the majority of its costs in materials as well?

Interviewer: How could you find that out?

Interviewee: I would want to interview the purchasing and engineering personnel of the
different subcontractors in order to understand their cost structures. If we had a better
understanding of their economics, our client might be able to reduce cost across the board,
allowing it to compete more effectively for the contract without killing everyone's margins.

Interviewer: Let's say that purchased materials average approximately 70 percent


of the price paid to most of the manufacturers.

Interviewee: If the cost of subassemblies represents 40 percent of the jet cost and 70
percent of that is purchased materials, total purchased materials would be approximately 28
percent of the jet cost subassemblies. Purchases of raw materials and components represent
another 15 percent, for a total of around 43 percent of the cost of the jet. If our client could
reduce the cost of raw materials by 20 percent, it could reduce the cost of the jet by more
than 8 percent, more than enough to offset the 5 percent reduction it would need to win the
contract.

Interviewer: That sounds reasonable, but 20 percent is a very lofty goal. How would
you go about doing that?

Interviewee: First, I would look at the number of suppliers. Are there a large number of
suppliers to the subassembly manufacturers?

Interviewer: The client estimates that there are approximately 125 suppliers of raw
materials and components among the manufacturers of the subassemblies and itself.

Interviewee: Well, that sounds like a large number of suppliers. Of course, they could be
providing very specialized materials to the subassembly manufacturers. Are these suppliers
providing customized or more commodity products?

Interviewer: About 80 percent of these products are commodities, such as sheet


metal and wire harnesses. Even some of the electronics, such as printed wire boards
and circuitry, are fairly generic.

56
Practice Cases – Case 7: Fighter Jet BCG
Interviewee: That sounds promising, but I would need to know whether these commodities
are interchangeable, so that our client could concentrate spending with fewer suppliers. Are
there many commonalities among the parts used by the different subassembly manufacturers?
We could talk to their engineers and look at the designs and bills of material to determine how
much overlap there is.

Interviewer: Let's say that you did this and discovered that approximately 30
percent of the cost of raw materials is from similar materials used across the
subassembly manufacturers.

Interviewee: It seems safe to assume that the client would need more commonality to be
successful in concentrating its purchasing and reducing costs. Do the engineers believe that
the percentage of overlap could be increased if the designs were modified?

Interviewer: They believe they could increase that percentage substantially,


particularly with basic materials such as screws and sheet metal, but also in other
more customized areas.

Interviewee: That's great news, but we would still need to know whether the subcontractors
are using the same suppliers. We could analyze the number of suppliers for each of the areas
of overlap.

Interviewer: Good suggestion. Although there are some common suppliers, the
analysis indicates that the subassembly manufacturers tend to use different
suppliers.

Summarize and make recommendations

Interviewee: Our client needs to reduce costs by 5 percent. The largest area of opportunity
appears to be in purchased materials, the majority of which comprise subassemblies
manufactured by seven subcontractors. By looking at its purchases in total, the client can
target approximately 40 percent of costs. To achieve the 5 percent cost reduction, it would
need to reduce costs by 15 to 20 percent. It could try to do that by increasing commonality in
the design of the subassemblies and components and by shifting volume to a smaller number
of suppliers.

Interviewer: Considering that the majority of the raw materials and components are
purchased commodities, do you think the 15-20 percent cost reduction is achievable?

Interviewee: Well, I know that raw materials and components typically have lower margins
than more customized products. I suspect it may be challenging to hit the client's savings
target by focusing only on these purchases. But since raw materials and components
represent about 40 percent of costs and there is an opportunity to concentrate purchasing, I
think we should start here.

Interviewer: Where else could you look for savings?

Interviewee: If I look back at the cost data on the jet, direct labor is another large cost
component. As a contingency, we could look into that area as well. I've read that other
companies use outsourcing to lower their manufacturing costs—perhaps our client could do the
same.

For example, it might want to increase its use of purchased subassemblies and reduce the
amount of direct manufacturing it does. Of course this would work only if it could drive direct
labor costs below the offsetting cost of these subassemblies. The client will be working closely

57
Practice Cases – Case 7: Fighter Jet BCG
with the subassembly suppliers to implement its purchasing initiative. This may give it an
opportunity to explore the suppliers' capabilities at the same time.

Interviewer: That's an interesting suggestion. How would you recommend the


company pursue both of the initiatives you have discussed?

Interviewee: I would look first to combine purchases across the subassembly suppliers with
our client's purchases. I suspect that the client and the subassembly suppliers will need to
share a great deal of information, including engineering drawings and specifications, with
potential suppliers of the raw materials and components.

The Internet could prove to be a very effective medium for forming a single "virtual"
purchasing department to consolidate both the flow of information and purchase orders across
the companies. Our client might also want to use a bidding system for those materials that are
true commodities.

Next, I would turn to the engineering departments and form cross-company teams to look for
areas to increase commonality of design. At the same time, those teams could explore
opportunities to use more purchased subassemblies and decrease the client's direct labor
costs.

Interviewer: That sounds great, and is very similar to a project we did. I would
caution you, however, to examine the upfront costs involved in your
recommendations, both for the redesign and for the implementation of the
purchasing system, before going ahead.

58
Practice Cases – Case 8: CanadaCo BCG
(Source: Sample case from BCG website)

Context
Your client is the largest discount retailer in Canada, with 500 stores spread
throughout the country. Let's call it CanadaCo. For several years running, CanadaCo
has surpassed the second-largest Canadian retailer (300 stores) in both relative
market share and profitability. However, the largest discount retailer in the United
States, USCo, has just bought out CanadaCo's competition and is planning to convert
all 300 stores to USCo stores. The CEO of CanadaCo is quite perturbed by this turn of
events, and asks you the following questions: Should I be worried? How should I
react? How would you advise the CEO?

Establish understanding of the case

Interviewee: So, the client, CanadaCo, is facing competition in Canada from a United States
competitor. Our task is to evaluate the extent of the threat and advise the client on a strategy.
Before I can advise the CEO I need some more information about the situation. First of all, I'm
not sure I understand what a discount retailer is!

Interviewer: A discount retailer sells a large variety of consumer goods at


discounted prices, generally carrying everything from housewares and appliances to
clothing. Kmart, Woolworth, and Wal-Mart are prime examples in the United States.

Set up the framework

Interviewee: Oh, I see. Then I think it makes sense to structure the problem this way: First,
let's understand the competition in the Canadian market and how CanadaCo has become the
market leader. Then let's look at the United States to understand how USCo has achieved its
position. At the end, we can merge the two discussions to understand whether USCo's
strength in the United States is transferable to the Canadian market.

Interviewer: That sounds fine. Let's start, then, with the Canadian discount retail
market. What would you like to know?

Evaluate the case using the framework

Interviewee: Are CanadaCo's 500 stores close to the competition's 300 stores, or do they
serve different geographic areas?

Interviewer: The stores are located in similar geographic regions. In fact, you might
even see a CanadaCo store on one corner, and the competition on the very next
corner.

Interviewee: Do CanadaCo and the competition sell a similar product mix?

Interviewer: Yes. CanadaCo's stores tend to have a wider variety of brand names,
but by and large, the product mix is similar.

Interviewee: Are CanadaCo's prices significantly lower than the competition's?

Interviewer: No. For certain items CanadaCo is less expensive, and for others the
competition is less expensive, but the average price level is similar.

Interviewee: Is CanadaCo more profitable just because it has more stores, or does it have
higher profits per store?

59
Practice Cases – Case 8: CanadaCo BCG
Interviewer: It actually has higher profits than the competition on a per-store basis.

Interviewee: Well, higher profits could be the result of lower costs or higher revenues. Are
the higher per-store profits due to lower costs than the competition's or the result of higher
per-store sales?

Interviewer: CanadaCo's cost structure isn't any lower than the competition's. Its
higher per-store profits are due to higher per-store sales.

Interviewee: Is that because it has bigger stores?

Interviewer: No. CanadaCo's average store size is approximately the same as that of
the competition.

Interviewee: If they're selling similar products at similar prices in similarly-sized stores in


similar locations, why are CanadaCo's per-store sales higher than the competition's?

Interviewer: It's your job to figure that out!

Interviewee: Is CanadaCo better managed than the competition?

Interviewer: I don't know that CanadaCo as a company is necessarily better


managed, but I can tell you that its management model for individual stores is
significantly different.

Interviewee: How so?

Interviewer: The competitor's stores are centrally owned by the company, while
CanadaCo uses a franchise model in which each individual store is owned and
managed by a franchisee who has invested in the store and retains part of the profit.

Interviewee: In that case, I would guess that the CanadaCo stores are probably better
managed, since the individual storeowners have a greater incentive to maximize profit.

Interviewer: You are exactly right. It turns out that CanadaCo's higher sales are due
primarily to a significantly higher level of customer service. The stores are cleaner,
more attractive, better stocked, and so on. The company discovered this through a
series of customer surveys last year. I think you've sufficiently covered the Canadian
market—let's move now to a discussion of the United States market.

Interviewee: How many stores does USCo own in the United States, and how many does
the second-largest discount retailer own?

Interviewer: USCo owns 4,000 stores and the second-largest competitor owns
approximately 1,000 stores.

Interviewee: Are USCo stores bigger than those of the typical discount retailer in the United
States?

Interviewer: Yes. USCo stores average 200,000 square feet, whereas the typical
discount retail store is approximately 100,000 square feet.

Interviewee: Those numbers suggest that USCo should be selling roughly eight times the
volume of the nearest United States competitor!

60
Practice Cases – Case 8: CanadaCo BCG
Interviewer: Close. USCo's sales are approximately $5 billion, whereas the nearest
competitor sells about $1 billion worth of merchandise.

Interviewee: I would think that sales of that size give USCo significant clout with suppliers.
Does it have a lower cost of goods than the competition?

Interviewer: In fact, its cost of goods is approximately 15 percent less than that of
the competition.

Interviewee: So it probably has lower prices.

Interviewer: Right again. Its prices are on average about ten percent lower than
those of the competition.

Interviewee: So it seems that USCo has been so successful primarily because it has lower
prices than its competitors.

Interviewer: That's partly right. Its success probably also has something to do with
a larger selection of products, given the larger average store size.

Interviewee: How did USCo get so much bigger than the competition?

Interviewer: It started by building superstores in rural markets served mainly by


mom-and-pop stores and small discount retailers. USCo bet that people would be
willing to buy from it, and it was right. As it grew and developed more clout with
suppliers, it began to buy out other discount retailers and convert their stores to the
USCo format.

Interviewee: So whenever USCo buys out a competing store, it also physically expands it?

Interviewer: Not necessarily. Sometimes it does, but when I said it converts it to


the USCo format, I meant that it carries the same brands at prices that are on
average ten percent lower than the competition's.

Interviewee: What criteria does USCo use in deciding whether it should physically expand a
store it's just bought out?

Interviewer: It depends on a lot of factors, such as the size of the existing store,
local market competition, local real estate costs, and so on, but I don't think we
need to go into that here.

Interviewee: Well, I thought it might be relevant in terms of predicting what it will do with
the 300 stores that it bought in Canada.

Interviewer: Let's just assume that it doesn't plan to expand the Canadian stores
beyond their current size.

Interviewee: OK. I think I've learned enough about USCo. I'd like to ask a few questions
about USCo's ability to succeed in the Canadian market. Does USCo have a strong brand name
in Canada?

61
Practice Cases – Case 8: CanadaCo BCG
Interviewer: No. Although members of the Canadian business community are
certainly familiar with the company because of its United States success, the
Canadian consumer is basically unaware of USCo's existence.

Interviewee: Does CanadaCo carry products similar to USCo's, or does the Canadian
consumer expect different products and brands than the United States discount retail
consumer?

Interviewer: The two companies carry similar products, although the CanadaCo
stores lean more heavily toward Canadian suppliers.

Interviewee: How much volume does CanadaCo actually sell?

Interviewer: About $750 million worth of goods annually.

Interviewee: Is there any reason to think that the costs of doing business for USCo will be
higher in the Canadian market?

Interviewer: Can you be more specific?

Interviewee: I mean, for example, are labor or leasing costs higher in Canada than in the
United States?

Interviewer: Canada does have significantly higher labor costs, and I'm not sure
about the costs of leasing space. What are you driving at?

Interviewee: I was thinking that if there were a higher cost of doing business in Canada,
perhaps USCo would have to charge higher prices than it does in the United States to cover its
costs.

Interviewer: That's probably true, but remember, CanadaCo must also cope with the
same high labor costs. Can you think of additional costs incurred by USCo's Canadian
operations that would not be incurred by CanadaCo?

Interviewee: USCo might incur higher distribution costs than CanadaCo because it will have
to ship product from its United States warehouses up to Canada.

Interviewer: You are partially right. CanadaCo has the advantage in distribution
costs, since its network spans less geographic area and it gets more products from
Canadian suppliers. However, since CanadaCo continues to get a good deal of
product from the United States, the actual advantage to CanadaCo is not great—only
about two percent of overall costs.

Interviewee: All this suggests that USCo will be able to retain a significant price advantage
over CanadaCo's stores: if not ten percent, then at least seven to eight percent.

Interviewer: I would agree with that conclusion.

Summarize and make recommendations

Interviewee: I would tell the CEO the following: In the near term, you might be safe. Your
stores have a much stronger brand name in Canada than USCo's, and they seem to be well
managed. However, as consumers get used to seeing prices that are consistently seven to
eight percent less at USCo, they will realize that shopping at USCo means significant savings
over the course of the year.

62
Practice Cases – Case 8: CanadaCo BCG
Although some consumers will remain loyal out of habit or because of your high level of
service, it is reasonable to expect the discount shopper to shop where prices are lowest.
Moreover, over time your brand-name advantage will erode as USCo becomes more familiar to
Canadian consumers. You certainly have to worry about losing significant share to USCo stores
in the long term. You should probably do something about it now, before it's too late.

Interviewer: Can you suggest possible strategies for CanadaCo?

Interviewee: Maybe it can find ways to cut costs and make the organization more efficient,
so it can keep prices low even if its cost of goods is higher.

Interviewer: Anything else?

Interviewee: It might consider instituting something like a frequent shopper program, where
consumers accumulate points that entitle them to future discounts on merchandise.

Interviewer: What might be a potential problem with that?

Interviewee: Well, it might not be that cost-effective, since it would be rewarding a


significant number of shoppers who would have continued to shop there anyway.

Interviewer: Any other suggestions?

Interviewee: CanadaCo might want to prepare a marketing or advertising campaign that


highlights its high level of service. It might even institute a CanadaCo Service Guarantee that
surpasses any guarantees offered by USCo.

Interviewer: Assuming the only way to keep customers is through competitive


pricing, is there anything CanadaCo can do to appear competitive to the consumer?

Interviewee: It might want to consider offering fewer product lines, so that it can consolidate
its buying power and negotiate prices with suppliers that are competitive with USCo's. It might
lose some customers who want the variety of products that USCo has, but it may be able to
retain the customer who is buying a limited array of items and is just looking for the best price.

Interviewer: All of your suggestions are interesting, and you would want to analyze
the advantages and disadvantages of each in more detail before making any
recommendations to the CEO.

63
Practice Cases – Case 9: HVAC Service Provider BCG
(Source: Interview case from BCG Round I, RSB 2005 Casebook Case#3)

Context
Your client is an energy firm that has a lot of extra cash and wants to know if they
should purchase all the HVAC (heating, ventilation and cooling) service firms in the
Atlanta area and consolidate them. The client would like to know if this is a viable
investment they should consider.

It’s important that interviewee asks the following clarifying questions:

- What is the rationale behind this investment?


- How many HVAC service firms in the Atlanta area?
- What criteria does the client use to evaluate investments?

Information given upon request:

- The client wants to diversify into other industries.


- There are 500 HVAC service firms in the Atlanta area.
- Client’s finance department requires a 3-yr break-even on any investment.

Interviewee will frame the issue. The following is a sample framework.

To evaluate if the investment will break even in 3 years, we need to find out what projected
annual profit is from the consolidated entity, the acquisition price and cost of capital. If the
investment will break even in 3 years, I’d like to further look into some non-financial risks involved
in the investment. If the investment won’t break even in 3 years, I’d like to discuss our alternatives.

- Projected profit from the consolidation


- Projected revenue
o Current revenue
o Projected revenue growth
o Revenue synergies from consolidation (if any)

- Projected costs
o Current cost structure (ask interviewee to identify the cost structure)
o Cost synergies from consolidation
- Acquisition price

- Risks involved in the acquisition such as post integration issues (cultural difference,
management issues, etc.), and regulation issues.

- Alternatives

Information given upon request

- Average annual revenue of each is $10M and average revenue growth is 3%


- No revenue synergies will occur because they serve the same market
- Current cost structure (% of revenue)
o Labor: 50% (Technicians are 100% utilized)
o Equipment: 25%
o Administrative: 20%
- Cost synergies (let interviewee brainstorm cost saving areas)
o 5% decrease in labor due to labor dispatching efficiency
o 5% decrease in equipment due to strengthened bargaining power against
suppliers
o 1% decrease in admin due to consolidated common functions such as IT
and accounting

64
Practice Cases – Case 9: HVAC Service Provider BCG
- Cost of capital: 13%
- Acquisition price of each firm: $5M
- Assume all cost savings occur in year one

Interviewee should calculate the cost savings

Interviewee should conduct the break-even analysis

A great answer will discuss opportunities to improve the payback and brainstorm alternatives
The investment won’t break even in 3 years. However,
- we could negotiate with the firms to reduce the acquisition price. It will depend on
our bargaining power versus theirs and if there are other bidders who offer
competitive price.
- We could also further look into the cost savings areas to see if more cost synergies
can be achieved.
- We could also look into opportunities to improve revenue through advertising
efficiency, optimizing brand portfolio, setting up better incentive systems, etc.

If after trying all these efforts, we still can’t break even on this deal. We can look into other
regions to identify better HVAC investment opportunities, or we can look into other industries
that offer more attractive opportunities.

65
Practice Cases – Case 10: Termite Control BCG
(Source: Interview case from BCG Round I, RSB 2005 Casebook Case#18)

Context
Your client is a termite control company that provides solutions to eliminate termites
from homes. Their current solution is to set up a baiting system (a similar concept to
mouse traps) that attracts and kills termites.

A competitor has come up with spray system that uses liquid spray to kill termites.
They have just started selling this system. How should the client respond?

Interviewee should first assess how credible the threat is and if it’s credible then how the
client should respond.

A sample framework:

To assess the importance of the threat, we can conduct a market research to find out how
likely customers will convert and how much market share we will lose to the new product. To
determine the conversion rate, I’d like to do a comparison of our product and the new solution
on price and other elements that are important to customers.

If the threat is significant, we can respond by:


- Improve our product to make it more competitive.
- Producing the new solution. It depends on our capability, the profitability of the
new solution, and financial/operational impact on our current product.
- Purchasing the competitor.
- Producing a new product that’s better than the new solution.

I’d like to discuss the feasibility of each of these options.

When discussing the market research, interviewee should be able to identify some possible
customer preferences such as price, ease of use, and effectiveness.

Information given upon request

- Client has 20% share with 100K new installations/yr


- The new solution is cheaper than our product
Baiting system Spray system
Initial installation $1100 $750
Annual renewal & prevention $250 $100

- Both products are equally easy to use and equally effective


- Some customers think that the new solution has health risk
- Market research found out that existing customers will not switch, but a high number
of new customers will use the new system because of the lower price (% unknown)
- Historically the client’s customer retention rate is shown below(% of customers in year
0 that renew their subscription in subsequent years)

Year 1 Year 2 Year 3 Year 4 Year 5 Year 6 Year 7 Year 8 Year 9 Year 10
90% 80% 70% 60% 50% 40% 30% 20% 10% 0%

Conclusion: The threat is significant because we will keep losing market share because we will
lose some new customers and keep losing existing customers each year as well.

Now interviewee can proceed to discuss how our client should respond

Information given upon request

- It’s unlikely to improve our current product to make it more competitive.

66
Practice Cases – Case 10: Termite Control BCG
- It’s also unlikely to produce a new product that’s better than the competitor’s in
short term.
- We have the capability to produce the new system quickly (no patent on formula)
- The competitor is a small company and therefore acquisition is possible. However,
direct interviewee to discuss the “producing the new system” option

A good discussion on producing the new system could include the following:

To produce the new system, we need to assess the profitability of the new system, impact on
our current product (for example, cannibalization), capacity, production process, and
sales/marketing. We also need to assess competitor reaction.

Information given upon request

- Profit margin on both products is:

Baiting Spray
Initial installation $400 $250
Annual renewal & prevention $50 $100

- Assume the new system has the same customer retention rate.

Expected profit per customer over 10 years, taken into account the retention rate (ignore
discount rate in the calculation):

Conclusion from the calculation: The accumulated profit per customer is higher for the spray
system only after Year 5. It will make the client depend more on customer retention rate.
Customer retention rate could decrease if other products come out in the future or customer
preference changes over the years. Considering the time value of money and the possible
change in retention rate, our client should minimize the cannibalization of the baiting system.

Sample recommendation:

Given that a high number of new users will use the new system, we will lose market share
each year. Therefore, the best response would be to produce this new system since we have
the capability and are able to do so quickly. However, because having less drawn-out per
customer profit makes the baiting system an more attractive product to us, we should direct
more sales/marketing effort to promote it to minimize the cannibalization.

In the next a few days, we will further analyze the financial and operational impact of
producing the new system and possible competitive reaction to our new strategy.

67
Case 5: PlastiCo -medium- The Boston Consulting Group, Round 2

(Source: Case Interview from The Boston Consulting Group, Round 2)

Context:
PlastiCo currently sells thin lamination film used to make advertisements and signs (think ads on the
side of the city busses, buildings or billboards) – sold in all manners of colors and finishes (matte,
gloss, shiny, etc)
PlastiCo currently sells its material in the USA at 10cents / sq. ft.
A European competitor has just finished building a lamination factory in Los Angeles and is
approaching our customers and offering to sell them product at 9cents /sq. ft.

PlastiCo hired BCG to help them figure out if they should match the competitor’s price.

Establishing the case and asking for more information


So our client is facing competition in the US from a European manufacturer. Our task is to evaluate this
threat and advice our customer on a strategy to match or not match the competitor’s price, Before that I
need some more information on what are the products our customer sells and who are the customers of
our client.

Interviewer:
This plastic film is manufactured in giant rolls and cannot be economically transported by air and
cannot survive the harsh conditions of shipment by sea.
Currently the majority of sales are to independent regional suppliers. The reminder of sales is shipped
directly to major accounts (major Ad Companies, Home Dept., etc).

A good structure should include the following:


There are two options for our client: matching the competitor’s price or not.
For assessing the feasibility of each of the options, I want to investigate the following areas:
- the profits in both cases and the long term implications of either approach
o In order to do that, we need to look at the revenues and costs. We need to analyze the
type of products our customer sells and at what price. Then we need to assess the
variable costs along with the fixed costs
- the market:
o market size;
o the market share of our customer and how much of it would be threatened by the
European competitor

Interviewer: (information given only when requested)

Market size = 1 billion sq. ft.


Current market share = 100% - Interviewee should immediately realize that it is a Monopoly

This lamination film is essentially a commodity (the candidate should immediately know that it means
that the customers are sensitive to price and would switch to a cheaper version)

The new competing plant has only capacity to produce 250 million sq. ft. per year (25% of the market)

21
Case 5:PlastiCo -medium- The Boston Consulting Group, Round 2

75% of PlastiCo’s products can be manufactured by the competitor with no discernable difference. The
candidate should realize that 25% of the products are differentiated products b/c it uses high tech
coating process that prevents colors from fading.

Additional candidate insights:

Because it is a Monopoly, our client should attempt to price as high as possible to consume all
customer’s willingness to pay.

Our client should concentrate on the 25% of the product that cannot be copied:
• Increase the price of these specialty items
• Without substitutes, higher margins here can offset the lost revenue on the other 75% of the
products

Do we have any information about the profit margins, so we can estimate the profits?

Interviewer should show the following slide:

Profit Margins

22
Case 5:PlastiCo -medium- The Boston Consulting Group, Round 2

Sample solution calculation:

Do not match price:


Market share lost: 25%
Profit loss= $0.035/sq. ft. * 250M sq. ft. = 8.75M
We made the assumption of 100% conversion to the competition

Long term implications: This situation leaves PlastiCo open to further mkt. erosion if competitor
increases capacity. Margins can also be eroded due to loss of scale (not a good long term solution)

Match Price:
Profit loss: 1c/sq. ft. on every sale
= 1,000M sq. ft * $0.01 = $10M
We made the assumption that no market share will be lost (which is not necessarily true)

Long term implications: it is more expensive in the short run but defends the market position

Bonus questions: how much market share would need to be lost before it would cost more (in the
short term) to match the 9c/sq ft. price?
Solution:
1,000M * $0.035 * (x%) = $10M
X = 28.57%

Recommendation:

PlastiCo must match prices to stave off possible market share decline of 25% in the first year and
potentially more if competition expands its existing plant. The remaining 25% of products should retain
their high prices and perhaps even raise them to maximize producer surplus. There are some risks
though as even with the price match, they can lose some market share.

23
ChairCo – BCG Round 1
Guidance for interviewer and
Problem statement narrative
information provided upon request
Our Client, ChairCo manufactures metal parts* that  If the candidate asks, tell them that there are no
are used to manufacture chairs. ChairCo primarily specific financial targets.
sells these parts to US based chair manufacturers.  Give the exhibits in the subsequent slides only
They are facing declining revenues and the CEO has when the candidate asks for the relevant data.
asked us to evaluate the problem and suggest
corrective measures.

*Metal bases that are used in the revolving office


chairs.

Difficulty: Easy Industry: Manufacturing Type: Profitability, Operations, Chart Based

- 37 -
ChairCo – BCG Round 1

Additional Questions to Steer Discussion

Questions for the candidate

After seeing Exhibit 1, the candidate should make an observation that prices and volume are
decreasing and both these issues need to be addressed.

 Why did ChairCo have to decrease prices? Because competition has decreased prices.

 Why did competition decrease prices? Because metal parts are a commodity and they might
have a lower cost structure than us.

 Why do you think our competitor has a lower cost structure? Material and labor could be the two
major reasons.

 Why is our client loosing unit sales despite decreasing price? Because their customers are
moving to low cost countries.

 Can the client reduce costs? Client is already very efficient and cannot decrease their costs
without shifting operations to China, Indonesia etc.

- 38 -
ChairCo – BCG Round 1

Suggested Solution and Structure

Solution Guide

 Exhibit 1 – Volumes have decreased and so have prices ($10 to $9.5). Ask candidate why he/she
thinks the price must have gone down. The most logical answer should be that since this is a
close to commodity product, prices for the entire industry have fallen down and ChairCo had to
respond. Competitors might have become more cost competitive because their operations are
located outside US.

 Exhibit 2 - Competition has significant cost savings in material and labor. The most logical
reasons are that they are based in low wage counties such as China, Indonesia and that they are
using an inferior/cheaper metal.

 Exhibit 3 – ChairCo customers are moving geographically away which explains the drop in
volume despite the drop in price.

- 39 -
ChairCo – BCG Round 1

Conclusion

Recommendation Next Steps

To become cost competitive and gain proximity to  Analyze which country has low cost
customers (chair manufacturers), ChairCo has base, high proximity to customers and
to shift manufacturing to Asia. low barriers (regulations, etc.) to set up
manufacturing.
Risk – downsizing in US will lead to a PR
backlash.

- 40 -
ChairCo – BCG Round 1

Exhibit 1 – ChairCo Sales

$500 M
50M Units
$380 M
40M Units

2010 2011

- 41 -
ChairCo – BCG Round 1

Exhibit 2 – ChairCo Vs. Competitor cost

ChairCo Costs
Cost Structure
SGA COGS
ChairCo Competitor

Materials 4.9 2.5

Labor 2 1.5

Transportation 0.5 1.5


$7.5 $7.4

Tax 0 1

IT 0.5 0.6 $1.5 $1.6


Overhead 1.1 1 2010 2011

- 42 -
ChairCo – BCG Round 1

Exhibit 3 - Manufacturers of Finished Chairs selling in US

Europe Canada US Asia

10.0%
30.0%
50.0%

80.0%
60.0%
40.0%

5.0% 5.0% 5.0%


5.0% 5.0% 5.0%
2009 2010 2011
- 43 -
Midwest Hospital – BCG Round 2

Guidance for interviewer and


Problem statement narrative
information provided upon request
Midwest Hospital is a research-based hospital and If the candidate asks tell them that there are no
takes pride in its joint replacement surgery financial targets.
department. Recently they did a P&L analysis for all Give the exhibits in the subsequent slides only when
departments and found that the joint replacement the candidate asks for the relevant data.
surgery department is making losses. The CEO has
asked for our help. Candidate should figure out during the course of the
case that there are several levers that can increase
profitability:
1. Increase price
2. Change patient mix
3. Increase total number of surgeries
4. Decrease costs
5. Provide post surgery services such as
physiotherapy (vertical integration)

Difficulty: Hard Industry: Healthcare Type: Profitability

- 70 -
Midwest Hospital – BCG Round 2

Additional Questions to Steer Discussion

Questions for the candidate

At some point near the start of the case, interviewer should take the lead and ask these questions
after exhibits has been given

1. Exhibit 1: Would it be advisable to not cater to Medicare patients (assume no backlash)?


2. Exhibit 2: What is the number of surgeries that Midwest needs to conduct in a year to
breakeven?
3. Exhibit 3: Why is Company D able to stay profitable despite having fewer patients and
unfavorable patient mix?

- 71 -
Midwest Hospital – BCG Round 2

Suggested Solution and Structure

Solution Guide

1. On fully cost allocated basis Medicare patients are unprofitable but they are still paying $1K
above the variable cost (marginal cost). This helps cover the fixed costs of the department.
So, it is not recommended to stop conducting surgeries for Medicare patients.
2. Average revenue per patient is 19K. Average variable cost is 14K. Gross margin per patient is
5K. Fixed costs are 7M, so 1400 surgeries are required for breakeven. Assuming same
proportion as in Exhibit 1 the hospital requires 140 commercial, 420 insurance, and 840
Medicare patients.
3. Comp D might have a lower cost structure or are able to negotiate better pricing from payers.

- 72 -
Midwest Hospital – BCG Round 2

Conclusion

Recommendation Next Steps

 Increase total number of patients.  Analyze scope for cost reduction, starting with
 Change mix of patients to have a higher competitive benchmarking.
proportion of commercial and insurance  Analyze scope for increase in price, starting
customers. with competitive benchmarking.
 Analyze profitability of post care services
provider.

- 73 -
Midwest Hospital – BCG Round 2

Exhibit 1: Patient Mix

Payer Type # Surgeries List Price Invoiced price

Commercial (Enterprises) 100 $40,000.0 $40,000.0

Insurance 300 $40,000.0 $20,000.0

Medicare (Government ) 600 $40,000.0 $15,000.0

- 74 -
Exhibit 2: Joint replacement department P&L

Revenue 19M

VC Physician 5M
Materials 5M
Others 4M

FC Facilities 3.5M
Others 3.5M

Costs 21M

Profit (2M)

- 75 -
Midwest Hospital – BCG Round 2

Exhibit 3: Competitive Benchmarking

Surgeries Commercial HMO Medicare Profitable

Midwest Hospital 1000 10% 30% 60% No

Comp A 1200 20.0% 20.0% 40.0% Yes

Comp B 800 30.0% 20.0% 50.0% Yes

Comp C 900 10.0% 20.0% 70.0% Yes

Comp D 1000 5.0% 25.0% 75.0% Yes

- 76 -
University of Michigan Business School
Case Book 1 - 2001-2002 Edition
Michigan

Practice Business Case 1


Company: BCG

1. Our client, AEM Inc, is an agricultural equipment manufacturer, with 4


product lines, facing declining sales. We have been brought in to examine the cause
and recommend initiatives to improve their profitability.

Interview Hot Tips: This is a profitability case

There are two broad drivers contributing to declining sales for AEM – decreasing unit
sales, decreasing unit price, or a combination of both of these drivers. To proceed, we
would examine the macro and micro perspectives to understand the reasons for declining
sales. For the micro perspective, we would need to understand details of AEM’s business.
For the macro perspective, we would want to study the changes in the industry over the
last few years due to new competitors, emerging technology, changes in consumer
demands, and other factors.

Firstly, we study AEM and its sales trends across the 4 product lines.

⇒ The sales trends for AEM across its product lines are detailed in this exhibit.
Sales Sales % Gross Margin
Product Lines 1995 1996 1997 1997 1997
Direct Combines $ 289 m $ 267 m $ 235 m 67% 43%
Tractortronics $ 73 m $ 71 m $ 65 m 19% 21%
Crossmaries $ 25 m $ 28 m $ 34 m 10% 17%
Hewittrangers $ 15 m $ 15 m $ 16 m 5% 19%
Total $ 402 m $ 381 m $ 350 m

⇒ Also, AEM has been facing declining market share in a mature, slow growth
industry.

Sales
Market Players 1995 1996 1997
AEM 49% 47% 45%
John Drill 36% 39% 42%
Cagen Thomas 3% 3% 4%
Others 12% 11% 9%

The exhibits on sales trends and market shares over the last 3 years indicates that we have
been losing sales steadily for the Direct Combines and Tractortronics product lines which
are significant contributors to AEM’s top-line and bottom-line. The question that leads
from here is – what is causing sales of the Direct Combines product line to decline – new
technology, new competitors, change in consumer demands, change in distribution
channels or any other cause.

5
University of Michigan Business School
Case Book 1 - 2001-2002 Edition
Michigan

The industry has seen an emergence of a new technology in Direct Combines with
John Drill leading the initiative. This has been the primary driver for change in
consumer demands. We should not worry about Tractortronics since is a declining
marketplace and its products are being replaced by Direct Combines.

The last statement above is a strong hint from the interviewer to ignore Tractortronics in
our further analysis, and focus the discussion on the Direct Combines product line. This
emergence of the new technology explains John Drill’s increased market presence – we
can now see that this new technology in Direct Combines may be a primary driver for the
market share shift. The next step is to study the impact of this technology on the industry
and AEM – is the technology here to stay, how mature is this technology, how
sustainable is the advantage due to this technology and what are its switching costs for
AEM. This will help us establish if the technology is worth adapting for AEM. If it is
worth adapting, we need to address how would we implement this technology change for
AEM’s products – in terms of switching costs, new product development, marketing and
promotion initiatives, and other issues. If the technology is not worth adapting or cannot
be adopted (due to patent reasons, for example), then we can examine discontinuing the
Direct Combines product line over time.

This technology helps significantly reduce maintenance costs and maintenance


related downtime of agricultural equipment by enhancements to the engines and drive-
train. Given our interviews with AEM’s engineers and marketing managers, as well as
industry analysts, we understand that this technology is becoming a market standard and
will be here to stay.

We recommend that AEM introduce the new technology in its Direct Combines range of
products. Given that this product line is an important source of revenue and we see
existing sales from this product line, we should introduce products with this new
technology and plan on a new product introduction strategy. We would need to examine
how AEM would go about implementing this recommendation.

How would you go about implementing this recommendation?

This would lead the discussion towards product development, manufacturing switching
costs, product launch and marketing related issues. A number of issues would need to be
covered. Firstly, should this technology be included in existing product lines or should
new products be introduced with this technology. This decision would be driven by
demand for existing product lines and costs associated with either decision. Secondly, in
terms of obtaining this technology, we can examine either developing it in-house or
acquiring a smaller company that has this technology. Finally, if we have adequate data,
we can do some quick “back-of-the-envelope” calculations to work out market-share
trends and break-even analysis.

6
University of Michigan Business School
Case Book 1 - 2001-2002 Edition
Michigan

Practice Business Case 9


Company: BCG

9. A regional banking client wants to go into the online brokerage business. What
would you advise them to do?

Interview Hot Tips:


o This is an open ended, market-entry case. One option is to use the 4C
framework (customers, competitors, cost, capabilities)
o Stay focused on the question. Interviewer gave a lot of extraneous data on
several sheets.
i. First and foremost: Should the client enter the industry?
ii. Second: How should the client enter?
o In terms of prioritization: Understand the market first!
i. How does an online brokerage make money?
ii. What are the relevant segments?
iii. Are there any unmet needs?
o Then look at the competitors.
i. Do they cater to certain customer segments?
ii. What are cost structures?
iii. What competitive advantages do they possess?
o Wrote the 4Cs on a sheet of paper but only to keep track of own thoughts.

Customers

Interviewee started with the customers (from the 4 Cs) and wanted to look at their size
and segmentation. The interviewer provided data on people with brokerage accounts. He
then asked the following question:

What are the revenue streams?

The three main revenue streams would be - commission, spread, and online fees. I’d like
to see if I could segment the customers. What are the different customer types and how
do they contribute?

There are three types of customers: active day traders, retired, and in between.
Day traders are the most profitable. (The interviewee then gives a quick summary of all
the information given: online customers and different revenue customers.)

Competitors

What are competitors doing? Is there any differentiation?

What do you think?

21
University of Michigan Business School
Case Book 1 - 2001-2002 Edition
Michigan

Well, since we are providing trading services, I don’t think there would be too much
differentiation. Perhaps, some competitors might provide more research capacity.

Any other ways

Advertising for the most part. We can observe from the info that there has been a huge
increase in advertising. Moreover there are no real differences in product features. How
about differences in cost structures?

What does it mean to you that there is only limited differentiation?

Well, this would imply that demand in this market could be very sensitive to price, which
could lead to a low-margin, high-volume kind of environment. In that scenario, the cost
structure would be very important.

Interviewer provided all this cost data

Well, we don’t have too much information on that. I don’t think we would have a cost
advantage though. The biggest portion of total operating costs tends to be advertising
advertising.

Capabilities

How do customers perceive the client?

How do you find that out?

Surveys.

Well, the customer is quite happy with the bank and view it as reliable. They view
it as a convenient, middle of the road regional back. Local bank on the corner.
Geography is key driver.

What are the Internal capabilities of the bank to provide these services?

Like any other bank, they have IT, but no web based system.

Hence, if we decide to do this we must partner up for IT. We would have to commit to a
new investment in IT infrastructure.

Would there be any synergies that we could realize?

There would be some synergies because the bank already has an existing customer base,
and it should be easier to expand the service offering to a given customer base than it
would be to go out and seek customers from scratch. I also think that the bank already
has a brand because of the advertising from the other divisions, and this would allow us

22
University of Michigan Business School
Case Book 1 - 2001-2002 Edition
Michigan

to expand the brand at a relatively lower cost. However, both the brand and customer
base are local, and this would not be of tremendous benefit in an Internet based business
where the goal is to draw customers from across the country.

Well, we have to wrap up now. What do you think overall?

Summary of Case

Based on the customer segmentation, there is somewhat of a misfit between the


bank’s clientele and the profitable online segments.
In addition the bank has a mostly local presence, so it cannot benefit from
synergies on a national scale.
Moreover, the bank does not have the right capabilities and would need to invest
in additional infrastructure

I would therefore recommend the client not to do it. We could explore some kind of a
link up where we partnered with an online brokerage and generated some kind of referral
fee based on sending customers their way.

Interview Hot Tips: Always try to provide your key points or recommendations
in a structured, bullet like fashion.

23
University of Michigan Business School
Case Book 1 - 2001-2002 Edition
Michigan

Practice Business Case 10


Company: BCG

10. Client is a mail order catalog company that has been experiencing declining
profit/catalog. Give me five different possible causes.

Interview Hot Tips:


o Try to be focused - I started to give 5 answers. I probably should have
been a bit more organized with my answers because I sounded like I just
mentioned anything that came to mind. Based on the interviewers
background (PhD Economics), I expected an economics related case. As it
turned out, the case went into economics later on. I used the MR = MC
equation to structure my case at the later stage.
o Prioritize the issues and filter out unneeded information – I started with
MC first, since that was easier to estimate. I then moved on towards
figuring out the MR equation. I needed information on the demand and the
revenue by customer groups.
o Additional information sought – I asked for MC information for
catalogues and MR information for customers.
o I use a graph to illustrate the MR-MC lines (they love Price Demand
graphs).

Interviewer drew a chart showing declining profitability/catalogue over the past 4 years.

Can you provide with 5 possible causes for this decline?

I used a clipboard and drew up five bulleted numbers (1-5) and provided the following
five possible causes:
Starting to target unprofitable customers
Items in catalog were lower margin
People are not ordering from catalogs
Shipping Costs could be higher
Products are not popular products

It turns out that they are shipping to customers who order less. How should the
company determine who to send to and whom not?

In a competitive industry such as mail order catalogues, MR = MC (Know the


background of the interviewer, I thought this would be a good place to start). Let’s start
with the marginal costs. Do you have any information on this?

MC = $2

I will need some information on MR. I would like to rank customers in deciles based on
the revenue each provides.

24
University of Michigan Business School
Case Book 1 - 2001-2002 Edition
Michigan

The interviewer had actually had done a regression and gave the equation of the
curve - MR = 50 – 2 * “customer percentile.” (Customer percentile: Profit by customers
ranked. This was based on the customer database of the client).

Solving for MC = MR. Gives the percentile. – Answer !

25
University of Michigan Business School
Case Book 1 - 2001-2002 Edition
Michigan

Practice Business Case 11


Company: BCG

11. Client is a yearbook publishing company that is experiencing declining


profitability. What should they do?

Interview Hot Tips:


o This is a standard “Profit = Revenues – Costs” case
o Prioritize issues and filter out unneeded information - Started to query the
revenue side first. Nothing there. Unit prices constant, revenue flat. Next,
explored costs.
o Framework used – “Profit = Revenue – Cost” framework
o Visual Representation - Drew out the manufacturing process and went
through each stage to explore the costs.
o Solution hints provided by interviewer
• Cause: Sales force incentives led to a change in order mix.
• Should change manufacturing process to do small batches.

I would like to explore to the revenue and costs.


Revenue - Could you tell me what happened to the prices?

Prices have stayed constant in real terms.

How about quantity sold?

Pretty much flat.

Then there must be something going on with the cost structure. This is publishing so
there are high FC. (Be careful with assumptions in cases) Did we make any investments
or do anything else that could have increased the fixed costs?

No. The client has old machines that are pretty much depreciated.

Because they are old machines could the maintenance costs have increased or is there
more downtime?

No

I will then look at the VC: Did the unit labor or raw material costs change significantly?

No

TIP - (It wopuld help if you had sketched out what a cost structure might look like
for this type of business to see if there were any key areas you would like to investigate
further)

26
University of Michigan Business School
Case Book 1 - 2001-2002 Edition
Michigan

At this point I felt I was at a dead-end. I needed some more information about the
client to figure out the problem. Hmmm. Could you explain how the production
process works so I can understand the cost structure better?

Set-up costs are very high. It is optimal to do longer runs so to lower costs.

Is it therefore some kind of mix issue. Could you tell something about the changes in
demand?

Yes. We have small schools and large schools that use us. The smaller schools are
ordering fewer yearbooks while larger schools are ordering larger quantities. We
therefore started to do more uneconomic batches.

Why are small schools ordering less?

Well it is certainly not the case that the # of students has changed dramatically at
smaller institutions. Can you think of some solutions to increase the order quantity from
smaller schools?

What drives sales?

One key driver: having your picture in the yearbook.

Do the schools order all of their yearbooks at one time?

Good questions. Actually they tend to order them at one, but often call us up after
and ask if they can order just a few more. We have always provided them as a customer
satisfaction compromise.

Have we considered charging a premium for these ‘extra’ orders.

Well, we haven’t in the past, but we could certainly consider doing so

Any other ideas

I guess that we should help schools with advertising to increase yearbook participation/
awareness.

Anything else

Well, this may be a radical proposal, but we could consider trying to shift our customer
base more towards large schools. We should look at the trade-offs involved in the new
customer acquisition costs versus the improved operating margins.

Good. Time is up!

27
Eye Glasses (1 of 4)
BCG, Round 1

Guidance for interviewer and


Problem statement narrative
information provided upon request(1)

The client is an eyeglass (lens) manufacturer from Japan. -They currently have operations and sales in Asia,
They are a well known brand in Japan and are Europe and Canada.
contemplating entering the U.S. market.
- They make the thinnest lens that is also anti-reflective.
This is patented.
They’ve hired BCG to help them decide if entering the
U.S. market is a good idea. - Their criteria for entering markets is if they can capture
2% of the market within the first five years and be
profitable.
Eye Glasses (2 of 4)
BCG, Round 1

Eye Glasses Market Distribution Channels

Somewhere in the student’s framework, they should ask The student should brainstorm channels and you should
about what the U.S. market is like or try to size the U.S. guide them to the following data:
market. You can provide the following information to
help them do this: - Eye care/doctor distribution = 50% of market
- Mass merchandisers (like Wal-Mart) = 20% of the market
-The U.S. market has one player owning 40% of the - Retail (like lens crafters) = 20% of the market
market, two others that each own 20%, and the rest is - Other = 10% of the market
fragmented amongst 10 players
Ask the student what they think of this and if they have
Ask the student what next? any thoughts on which distribution channels the client
They should recognize that 20% of the market is very could focus on?
fragmented and that this = opportunity. They should
focus on penetrating this 20% and should see that Good answers will recognize that the client has a special
logically getting 2% in 5 years is feasible. lens and you’d want to go to the channel in which people
are buying that lens and that mass merchandisers may
A good student will try and inquire about how people have long standing relationships or contracts that make
buy glasses in the U.S. and you will share with them that penetration for a new company difficult.
that (unlike Japan) they are not loyal to a brand, but
rather the place where they go to buy their contacts or In general, guide the student to focus on Eye care
glasses. because it’s here that people buy special lenses most
often and the client would also like to focus where most
A good student will then ask about distribution channels. of the volume is. Dismiss other saying there isn’t enough
info available on this channel.
Eye Glasses (3 of 4)
BCG, Round 1

Volume Profitability

Once the student focuses on the two channels, they Information on costs and revenues:
should ask what the volume going through Eye care is.
-Tell them its 80MM pairs (this means 160MM lenses). - They sell single vision for $25/pair
- They sell progressive for $100/pair
They should next consider how much of this can the
client capture. They should ask about the different Ask student to brainstorm costs. The cost information
products being sold in this channel: you have is:

- 50% of total volume is single vision lenses -Variable:


- 25% of total volume is a “buy and try” lenses -single vision: $12.50/pair
- 25% of total volume is progressive lenses -progressive vision $50/pair

The client has products in single vision and progressive -Fixed: can be allocated to single vision at 25% of
lenses. They don’t make “buy and try” lens types (so variable and to progressive as 30% of variable
student should see that the total market size is now 75%
of 80MM pairs or 75% of 160MM lens). -Ignore all other fixed costs right now

The student should now check for profitability0


Eye Glasses (4 of 4)
BCG, Round 1
Now ask HOW they would go about penetrating
the U.S. market broadly? What would they look Summary
at to actually execute upon this?
Student should see that the company goals on Student recommendation should include:
profitability and market share can be met.
1) Answer: Yes, you can enter U.S. market and reach
A good answer will look @ the following dimensions: your goals0

Focus on Distribution: - We looked at market, and 20% is fragmented, you


- focus on eye care and retail for reasons discussed can get 2% of that by specifically focusing on eye
(mass merchandisers may be locked into contracts and care (which is majority of market)
in eye care/retail you will get more specialty consumers - Looked at costs/revenues and profitability is there
who will be interested in the client’s special product)
- pursue benefits they could provide to the channel, 2) Next Steps: Build Distribution, Hire Sales People
maybe they can deliver with better speed or provide
better terms (something that their competitors in the 3) Risks: Brand name doesn’t go a long way and this one
market aren’t doing) of the assets you have. Basis of competition in the
U.S. is different than in Japan and what the client is
Marketing: used to.
- focus on channel, not on end consumer. End consumer
doesn’t care about brand (unlike client’s home country
and other Asian markets)

Price:
- ensure price is competitive
New Airline Routes (1 of 4)
BCG, Round 2

Guidance for interviewer and


Problem statement narrative
information provided upon request
This first question requires the candidate to assess all
In the 1960s, PanAm had a monopoly on routes within the potential reasons for wanting to stop in Kansas City.
the United States from the West Coast to the East Coast. A sample of responses could be:
These routes originated from three points on the West
Coast – Seattle, San Francisco, and Los Angeles, with • Using different planes (shorter range) – the same plane
terminating points in NYC, Washington, and Atlanta. is used on all routes.
(See Exhibit 1) • Picking up additional passengers – no additional
passengers are picked up from KC
The airline was presented with the option to introduce a
• Picking up additional cargo – no additional cargo
stopover point in Kansas City (see Exhibit 2).
• Attempting to open up a new hub – No need; KC is a
small market
1. Why would the airline want to do this? • Load factors are low; attempting to consolidate new
routes – Yes, load factors are low. Press the candidate
on why load factor is important (used as a utilization
factor for scheduling and purchasing aircraft, also
increases revenue per aircraft)

All of the above answers are acceptable, but load factor


is the critical area. The candidate should specifically ask
for load factor amount (currently at 50 percent).
New Airline Routes (2 of 4)
BCG, Round 2

Additional questions for candidate Solution guide

Provide candidate with Exhibit 2 and have him/her The candidate should realize that, at 50 percent load
explain the new route network he/she would utilize. factors, planes can be consolidated.

• If the load factor is 50 percent on all of PanAm’s A shortcut the candidate should use: examine all
routes from west to east, what will be their new load three routes from ONE city and extrapolate.
factor from KC to the East Coast if they now stop in
Kansas City? • Assuming 100 seats per plane (but can be used for
any number of seats), and equal distribution, the new
• What if the load factor was 80 percent? load factor will be 75%.

• At which point does the load factor become • Applying the same approach, the candidate should
irrelevant (i.e., consolidation of a/c will not be discover that the load factor is still 80 percent.
economically beneficial)?
• Once the loads of aircraft exceed the capacity of two
• Based on the above calculations, what does this planes (or 66.7%), then three planes will be needed
suggest about PanAm’s decision? per KC-east coast route.

• What recommendations would you make to Above a certain load factor, the benefit is moot.
management? Several factors should be considered:
Will load factors increase on west coast-east coast
routes beyond 66.7%?
Are additional opportunities for KC expansion
available?
New Airline Routes (3 of 4, Exhibit 1)
BCG, Round 2

CURRENT ROUTES

SEATTLE

SFO NYC
WASHINGTON, DC

LAX

ATLANTA
New Airline Routes (4 of 4, Exhibit 2)
BCG, Round 2

PROPOSED ROUTES

SEATTLE

SFO NYC
KANSAS CITY WASHINGTON, DC

LAX

ATLANTA
Optical Fibers (1 of 5)
BCG, Round 2

Guidance for interviewer and


Problem statement narrative
information provided upon request
The client is a fiber optical manufacturer. (Fiber optics
are wires that transmit data)
The CEO of this fiber optical firm is new and is trying to - Data types transmitted over these wires are internet, TV,
assess the firm’s business situation. At a surface levels, phone, etc. .
she knows the firm is experiencing a 50% decline in
revenues, but has hired BCG to dig further. - The client specifically sells these wires to big firms like
AT&T and Sprint and they put them underground and
use them to transmit data over long distances.
She has 3 specific questions:
-Why is the firm experiencing this revenue decline?
-When will the firm rebound?
-How does the firm relate to its competition?
Optical Fibers (2 of 5)
BCG, Round 2
Student should tackle each one of the three questions in this order:
The answer to this question is qualitative in nature. The industry as a whole is suffering because 4
1. Why is the firm years ago more optical fibers were put in the ground then their was demand for. Each fiber optical
experiencing this cable can support about 1 tera bit of information, and the demand per second (unit of time we’ll
revenue decline? use) is less than 1 tera bit per hour.

The student should be looking for data that will help them see when the market or demand for
these cables will come back.

When prompted, the interviewer will reveal:


2. When will the firm
-Candidate should use the market of fibers between D.C. and NY as representative of the world for
rebound?
the purposes of this case
-There are 1000 fibers between these two cities and the peak of demand is 100 terabits/sec.
-Every year the peak demand doubles. Use time zero of demand as 100 terabits, year 1 as 200 and
so on.

Demand will exceed capacity in year 4 so somewhere between year 3 and year 4, there fore more
Candidate should
cables will have to be installed and the company can rebound. This is sufficient for this part of the
determine that:
case (don’t need to get into how much of that market they can capture, etc.)
Optical Fibers (3 of 5)
BCG, Round 2
3. How does the firm Ask the candidate what they would look at to compare this firm to the competition. Don’t let them
relate to its go until they’ve exhausted most of the things on the list below. First have them list all things, then
competition? give them information on the firm as it relates to each category on their list.

Category Information on Firm


Price Commodity good - same price as everyone else
Product Quality Not relevant to customers, commodity market
Manufacturing Capacity Largest capacity, most idle capacity
Cost Structure Lowest cost structure/best profits
Share of Market #1 in market, have scale benefits

Customer Base Sell to the major companies in market like AT&T and Sprint
Geographic Footprint/Distribution Same as competitors
Cash/Assets (Balance Sheet) Lot of cash on hand, no debt

Some of these are things you would always look at, but some of them are important to this case.
Candidate should For example, capacity matters because the firm with idle capacity will be able to grab the new
identify that: business when the market picks up again. Also the firm with cash on hand and no debt will be able
to get through tough times when the market is slow.
Optical Fibers (4 of 5)
BCG, Round 2

Tell the candidate that he/she’s done a good job getting through the 3 questions put forth, but you
After answering the 3
still have one more. Someone on the CEO’s team wants to get a quick sense of the firm’s variable
initial questions:
and fixed costs. This is an ad hoc question they want a quick answer on.

Two months of data:

Provide this data: Month 1 Month 2


Produce 3MM km of fiber Produce 2MM km of fiber
Costs are $50MM Costs are $40MM

1MM km fiber costs $10MM (difference between Month 1 & 2 costs)


Candidate should
determine that: Thus fixed costs are $20MM per month
Optical Fibers (5 of 5)
BCG, Round 2

Tell the candidate that the CEO has a couple minutes between meetings and is looking for a quick
Lastly: summary and recommendation. They should use this opportunity to update on questions 1, 2 and
3 (not the variable/fixed question).

1) Your revenues have declined because the industry as a whole has declined. The current fiber
optic supply exceeds demand.
2) The market demand will bounce back in 3 years
Candidate should 3) You’re in a great position to capture the new demand (excess capacity, largest player, stellar
respond with: customer base) and weather the market slow down (have cash on hand, no debt.)

Student can also offer next steps or risks, but this is not critical.
Pay Phone Company Acquisition (1 of 2)
BCG, Round 2
Guidance for interviewer and
Problem statement narrative
information provided upon request
A private equity firm has approached • Pay phones are located throughout the US in all metropolitan areas
BCG about their views of an • There are 1 million installed pay phones
acquisition of a pay phone business. • The average pay phone produces $75 in revenue. There is great variation in the
How would you advise the PE firm? revenue by phone.
• The use of payphones has been declining 3-5% per year over the last several
years. It is 2008 (i.e. cell phone penetration in the US is nearly 100%).
• Gross margin is about 1/3 of revenue
• Costs are 60% fixed, 40% variable
• There are no potential synergies between this potential acquisition and other
holdings of the PE firm

Suggested Solution and Structure

Private equity firms make their money by improving the performance of the business (i.e. profitability/cash flow) and then
making an exit. Therefore, the way to think about this question is whether there is an opportunity to increase profitability
and eventually exit. A structure need not be more than cost and revenue. A strong candidate can also recognize the
possibility of synergies with other businesses and that reversing the decline in usage is very unlikely.
Revenue: How could you increase revenue?
1) Increase usage;
2) Raise rates;
3) Utilize the space- advertising is 80% of the pay phone revenue in Manhattan
(1)Costs: Major
If detailed exhibitscosts arewillrepairs,
exist, they maintenance,
be referenced in this box, collection of money, line costs, etc. There is not too much opportunity for
and included in fullhere.
improvement on the following slide(s)
Pay Phone Company Acquisition (2 of 2)
BCG, Round 2
Analytics & Summary Solution guide

1. Based on your suggestion, we decide to 1. Close the lines that are losing money and don’t
shut down half of the existing lines. How seem to offer enhanced revenue potential (such
as advertising) that would make them profitable.
should we determine what lines to close?
2. We will no longer be profitable and will lose
2. Let’s assume that we close 50% of the $2.5 mn per month.
lines, choosing at random (i.e. not
selecting those that are losing money Today After Cuts
first). What is the effect on our profitability Lines (mn) 1 given 0.5
per month? Revenue ($75 per line) 75 given 37.5
Gross Margin 0.67 given
3. How would advise the CEO? Total Cost 50 2/3 of Rev
Fixed Cost (.6 * 50) 30.0 .6 * Cost 30.0 stays constant
Variable Cost 20.0 .4 * Cost 10 reduced by half
b/c of line
reduction
Profit (mn) 25 (2.5)

3. While the Pay Phone business offers significant


opportunity to increase profitability by closing
lackluster lines, I would be cautious about
investing. Pay phones are a shrinking and
increasingly obsolete business and therefore
increasing or finding an equivalent exit multiple
will likely be difficult.
Telecom (1 of 3)
BCG, Round 2

Guidance for interviewer and


Problem statement narrative
information provided upon request(1)
The client is a telecom manufacturer. Specifically they -There are 3-4 major players in PBX, the client is #1.
make the phones and supporting wiring/systems that
support offices telephone systems. -Key is more commodity driven. There are 14-15 players,
the client is somewhere in the top.
The client’s net margin is at 5-6% while that of -PBX more profitable ~15% margin
competition is 12-13%, the CEO has hired BCG to see if
this can be fixed.
Review the students framework and guide them to focus
on costs. This is a margin problem and the client would
They segment their customers into two groups: like to look internally at its cost structure. They should
also focus on Keys segment since we know PBX is at
PBX which is customers who have 30+ employees, and 15% margin.
KEY which is customers who have 3-30 employees.
If they breakdown into variable and fixed costs, ask them
to brainstorm around different types of costs.

Within fixed costs ask them what they could do to trim


back?
Within variable costs, guide them to focus on the
variable sales costs (costs required to sell units).

Guide them to look at sales channels and then give them


Exhibit A.
Telecom (2 of 3)
BCG, Round 2

Once student looks @ slide they should see that the client should exit direct sales.

Ask the student how the client should exit direct sales? How would they think about this?
The student should ask:
-Is it realistic to migrate all sales/revenue from direct sales to the indirect sales channel?
Further Questions -Are there any benefits to doing direct sales?
- Indirect sales is heavily saturated by competition. Are there any other channels
that could be low cost the client could pursue? The answer is internet.
- The client has feels that doing direct sales with Keys customers is good for
relationship building and leads to higher retention. When the Keys customers
convert/grow into PBX customers this is good, since PBX is more profitable

Answer:
Think problem can be fixed if we trim back on direct sales to key customers. We should
segment key customers into high and low potential and only do direct sales for high
potential ones. For low potential we should move them to internet sales.

Summary Next Steps:


Develop segmenting process, explore and develop internet sales channel.
Telecom (3 of 3)
BCG, Round 2

Exhibit A

Channel % of Costs % of Revenue Comments


Direct sales 65% Sell directly to customer, client installs systems etc
Sell to a company like Office Max who then sells to end
Indirect sales 80% consumer
Gas station (1 of 7)
BCG, Round 2

Guidance for interviewer and


Problem statement narrative
information provided upon request(

Your client is a gas company that operates in a town with a 1. What are the proposed groceries the gas station would now
population of 1,000. There is only one other gas company in this sell? (turn question around and ask the candidate).
town, and it is 1 mile away. The other nearest gas stations are Likely answers include cigarettes, milk, snack foods;
outside town, and they are 20 miles away (see picture on next probably not fresh produce or healthy foods.
page).
2. Currently, the gas station is barely scraping by. Profit is
essentially 0.
Recently, our client was approached by a supermarket with the 3. If asked anything about the other gas station, the answer is “we
idea of selling groceries in the gas station. Our client is a simple don’t know, but assume they are identical”
businessman and has hired us to evaluate this proposal. What
should our client consider? Candidate should recognize that each gas station serves
500 people.

- 15 -
Gas station (2 of 7)
BCG, Round 2

Map of area (not to scale)

- 16 -
Gas station (3 of 7)
BCG, Round 2

Framework

Additional questions for candidate Solution guide

What information would you want to consider when • Revenue: Increased revenue from selling groceries in store;
deciding whether or not to sell groceries at this gas station? more people coming to buy gas from this station instead of
(You as interviewer are trying to get candidate to provide station B.
this framework)

• Costs: Up front investment costs such as a freezer, shelves,


etc.
• Recurring costs such as labor will be minimal since
same staff can handle gas and groceries.
• Will there be any profit-sharing with the grocery
store?

• Competition: What is keeping the grocery store from going


to gas station B as well?

- 17 -
Gas station (4 of 7)
BCG, Round 2

Costs

Additional questions for candidate Solution guide

What kind of costs could there be? • Investment in freezers, shelves, utilities.

• No increase in labor expected.

• Total increase in costs: one-time cost of $1.25MM.

- 18 -
Gas station (5 of 7)
BCG, Round 2

Revenue stream

Additional questions for candidate Solution guide

• Groceries: • People who shop here are probably not health conscious,
• What kind of people will shop for groceries here? since gas station groceries are not healthy. They are
probably also in a rush, and will make small purchases
• How much do they spend per year? (milk, cigarettes, jerky, etc.)
• Out of 1000 people, assume 500 will buy groceries
• What else can be a source of revenue? here at $200 per year per person
• Candidate should recognize that more people will • -> $100,000 per year
buy gas here now. • Increase in gas sales: very important distinction: people
won’t buy more gas, but more people will buy gas here
(stealing customers from station B)
• What is the total increase in sales?
• We can steal 50% of station B’s customers (250
people), who purchase $100 in gas per year ->
• The gas station owner is a simple man and wants to look at $25,000 per year
this like a perpetuity. What does this work out to? • Total increase in sales per year: $125,000.
• (discount rate) r = 10% • As a perpetuity: $1.25MM.
• (growth in cash flows) g = 0%

- 19 -
Gas station (6 of 7)
BCG, Round 2

Competition and Should they do it?

Additional questions for candidate Solution guide

What would you advise the gas station owner to do? • This is a prisoner’s dilemma.

• If station A does it, it is value neutral, BUT, we can take


$250,000 away from station B (good). But what is keeping
station B from retaliating and doing the same thing?
Remember, station A is barely scraping by. If station B
responds, we could go out of business since revenue would
no longer cover the one-time cost (bad). If station B does
not respond, then we can steal all customers from station B
and be profitable (good).

• If A does not do it, station B might do it. Then station A


would lose $250,000 and it could go out of business (bad).

• No matter which scenarios the candidate chooses, push


hard for the other option and play devil’s advocate.

- 20 -
Gas station (7 of 7)
BCG, Round 2

Final Recommendation

Recommendation Risks Next steps


(based on what candidate decides) (based on what candidate decides) Try to form an exclusive contract with the
grocery store.

Do it, and try to push station B out of Station B might respond and bring both
business. stations down.

Don’t do it, and hope station B doesn’t Or Station A might miss out on the
do it either. opportunity and lose first mover
advantage.

- 21 -
Bidding on the Olympics (1 of 4)
BCG, Round 1

Guidance for interviewer and


Problem statement narrative
information provided upon request(1)

Our client, a major TV Network, wants to know how much to bid • This only includes the rights to U.S. programming, overseas
on the TV rights for the 2016 Olympic Games. Bid will need to be programming is out of scope
paid in 2010. • Assume they will only show the Olympics on their one flagship
channel
The amount of programming is as follows: • Olympic programming will replace regularly scheduled
programming

16 days total • Prime Time is considered anytime after 7pm on a weekday,


and all day during the weekends.
Opening Ceremonies on a Friday: 8pm – 11pm
• Interviewee should point out that the network will earn ad
14 days of programming for 10 hours a day, revenue while the coverage is on, and hopefully will consider
9am – 12 pm, 2pm – 5pm, and 7pm -11 pm M-F added viewership to the network as a whole due to Olympic
coverage.
11am – 9pm on the Weekends
• Interviewee should consider the costs to put on the event,
Closing Ceremony on the following Saturday from 8pm – 11pm and opportunity cost of ad revenue from other program.

- 22 -
Bidding on the Olympics (2 of 4)
BCG, Round 1

Breakdown of Ad Revenue during Olympics

Prime Time Non Prime Time


Revenues $400k / 30 $200k / 30
seconds of ads seconds of ads

Amount of Ad time 10 min / hour 10 min / hour

Breakdown of Costs

Prime Time
Cost associated with $428 MM
coverage*

Opportunity Cost of $1M / hour


ad revenue from other
programming

* This includes all fixed and variables costs for travel, equipment, salaries, etc. Don’t let the candidate get
caught up in these costs.

- 23 -
Bidding on the Olympics (3 of 4)
BCG, Round 1

Candidate should Revenues Programming hrs Minutes o f Ads Ad Revenue


calculate the total Prime Time 86 860 860*400K *2=$688M
revenue to be received Non Prime Time 60 600 600*200K*2=$240M Total Revenue
=$928 Million
in 2016.. The *2 is because you
make$400K per 30 seconds

Event Costs Opportunity Cost


$1 M per Total Cost Total Profit in 2016
$428M
… and the total costs programming =$574 Million =$928M -$574M
hour ($146M) = $354 Million

Now, the candidate should think about Time Value of Money

• Profit of $354 Million (Plus any additional bump for future viewership or required profit margin candidate may include) will be received in
2016, but bid must be paid in 2010.
• If asked, give them a Cost of Capital of 12%

• A great candidate will recognize rule of 72, which states that you divide 72 by the interest rate to determine how long (in years) it will
take an investment to double. 72 / 12 = 6, which is how long in the future we will receive our profits. Therefore, $354M in 2016 is worth
$177M in 2010.

- 24 -
Bidding on the Olympics (4 of 4)
BCG, Round 1

Answer Guidelines

• The key takeaways from this case are to:


• Recognize time value money
• Understand and identify Opportunity Costs
• Make judgment on unknown information (how much value having the Olympics will bring to other
programming on the network)
• Great candidates will:
• Understand there are 10 weekday days and 4 weekend days (plus the opening and closing) and not list
out every day to calc hours for programming
• Know and use the Rule of 72

• Exact answers are irrelevant, though should be around $177 million, with well thought out adjustments to
increase or decrease the number.

- 25 -
Household Cleaners Growth (1 of 6)
BCG Round 1

Guidance for interviewer and


Problem statement narrative
information provided upon request

Your client is a global consumer packaged goods company —Grime This case is about growth both through internal actions and
Co. through acquisition. Initially, the candidate should brainstorm an
array of possible growth strategies. Eventually, he or she will have
Grime Co. makes paper products (like paper towels), home to drill down on new products and acquisition, in addition to
cleaning products, and laundry care products. The company's considering market growth. Then, she or he will have to evaluate
Board of Directors has set an aggressive net sales target of $2 two targets, demonstrating an understanding of positive and
billion by 2015 (four years). Currently, net sales are at $1 billion. negative synergies. Without considering market growth, organic
The CEO has come to you to ask for help. growth, and inorganic growth —and without exploring synergies in
acquisition — the candidate will not be able to solve the case.
Specifically, our client would like you evaluate the company's
position and to help develop a strategy to deliver top-line results of Additional information:
$2 billion by 2015.
• The company has a strong stance on sustainability
Net sales: Retail sales minus trade spend. Trade spend is what • Sales are divided evenly between the three categories — 33%
manufactures pay distributors or retailers to incentivize them to • The company has low profit margins and does not want to
sell their products to end consumers. take on additional debt, so cash available for investment is
about $300 million. (Be sure not to say “for acquisition”.)

If the candidate asks which growth strategies Grime Co. has


considered, the interviewer should prompt her or him to
brainstorm various options — see next slide.

- 55 -
Household Cleaners Growth (2 of 6)
BCG Round 1

The candidate must touch on market growth, new products, and acquisitions to solve the case — in any order. The
following structure is how the candidate may organize information. Profit or cost should not be part of the structure.

Area the candidate should explore: Information provided upon request


Market growth: Growth through maintaining When the candidate asks, reveal that market growth alone will bring sales to $1.5
market share in a growing market billion by 2015. Specifically, the company is growing overall at 10% and expects to
1 maintain a constant market share. (10% compounded over four years is roughly
$500 million incremental.) In the interest of keeping this case shorter, the
candidate does not have to calculate this. If the candidate asks about categories,
tell him or her that growth is about the same in all three.

Organic growth: Actions taken within the The candidate must cite new products: it is the only organic growth strategy that
organization to drive revenue. Examples: is viable for our client in this case. The interviewer should provide logical reasons
to why the other options are not available at this time.
• Price adjustments to drive volume
• Increased advertizing Our client has a new toilet cleaning product in development that analysts believe
• Expansion into new geographies will do well. The following details should be provided by request:
2 • Vertical integration
• Promotions and deals • Product is near launch — hits shelves in a months
• Negotiation for better placement • Price will be $5 a unit, but requires 20% trade spend per product to reach
• New products volume target
• Expected to sell 40 million units on average per year
Interviewer should steer candidate to explore
new products No other investment is required — sunk cost. (See slide 3 for calculations.)

Inorganic growth: Growth through acquisition The candidate must identify growth through acquisition: Our client’s Corporate
3 or joint venture Development department has identified two high-priority acquisition targets —
Organoclean and Home Defense Inc. (See slide 4 for detailed information.)
- 56 -
Household Cleaners Growth (3 of 6)
BCG Round 1

The candidate must determine how much top line growth can be achieved
2 New Product Calculations
through the launch of the new product.

Current Net Sales $1 billion Price2 $5 per unit

Net Sales in 20151 $1.5 billion


Trade spend - 20% = $4 per unit
Deficit $500 million

Avg. units per year x 40 million

New product $160 million Net sales per year = $160 million

New deficit $340 million

1. $500 million incremental achieved through market growth, as cited on the previous slide 2. Data presented to candidate from previous slide; these are just calculations
- 57 -
Household Cleaners Growth (4 of 6)
BCG Round 1

Acquisition Calculations: Sales The candidate must determine which property our client should purchase.
3
Forecast Candidate should request each data set: sales, products and growth

Target Organoclean (Private) Home Defense Inc. (Public)

Products Organic household cleaners Household cleaners, bug control

Sales $150 million $200 million

Growth rate 10% 20%

2011 sales $165 million $240 million


$181.5 million $288 million
Instruct candidate to round
$199.65 million $345.6 million
to the nearest $10 million
$219.615 million $414.72 million

Rounded 2015 sales $220 million $410 million

With a deficit to the 2015 sales target of $340 million, the candidate might be tempted to
However, the candidate must
choose Home Defense Inc. as the better acquisition target. (Remember that our client only
also consider positive and
has $300 million available for purchases, so a quick 1.5x sales multiple as a potential
negative synergies before
acquisition price suggests our client can only buy one of the two.) If asked, confirm that our
choosing a target…
client can only buy one.

- 58 -
Household Cleaners Growth (5 of 6)
BCG Round 1

Acquisition Calculations: Synergy The candidate must determine which property to purchase.
3
Considerations Candidate should brainstorm synergies and calculate financial impact.

A good interview will cite several of these Organoclean


as potential synergies. Push the Our client believes it can leverage its Europe distribution network to
candidate along until he or she lands on generate additional sales:
both distribution and values: $500 million deficit
- $160 million new product
P • Distribution synergies
• $40 million year one 1 - $220 million Organoclean 2015
O
• Procurement synergies • $40 million year one
S • Should triple in 4 years - $120 million Europe sales
I • Manufacturing synergies • Should triple in 4 years
T
• Back-office synergies = 0 deficit to 2015 target
I
V • Co-branding new products
E • Scale synergies Home Defense Inc.
N
Our client will not sell harmful chemicals, and all of Home Defense’s bug
E
G
• Corporate culture mismatch killers fall into this category. They cannot be reformulated or sold. Our
A • Anti-trust issues client would have to discontinue these products
T
I
• Mission or values clash
V • Brand dilution $500 million deficit
E
• 25% of sales are - $160 million new product
bug killers - $410 million Home Defense 2015
Synergies impacting Organoclean and Home Defense
• 25% of sales are bug
killers + $102.5 million lost sales

1. Candidate can consider year 1 today = $32.5 million deficit to 2015 target
- 59 -
Household Cleaners Growth (6 of 6)
BCG Round 1

At this point, the candidate should realize that Organoclean is the best option between the two, and that together with the
launch of the new product and market growth, Grime Co. will hit its 2015 net sales target of $2 billion

Although market growth and the launch of a new toilet product should get Grime Co. to $1.66 billion in net
sales by 2015, the $2 billion net sales target will not be met. Therefore, Grime Co. will have to pursue growth
Recommendation through acquisition. Of the two targets preferred by the client — and since Grime Co. can only buy one — we
recommend purchasing Organoclean. The growth of the company coupled with positive distribution synergies
will allow Grime Co. to reach its 2015 target.

• Growth trajectory of target could change • Verify growth estimates


• Price of target could be too high to afford • Use DCF valuation to determine best price
Risks and Mitigation • Target could be unprofitable — risk of sales focus • Gain access to data room and review financials
• Using all available cash limits other investments • Compare purchase against NPV of other projects
• Doubling in size itself could be a risk — too fast • Make sure to update systems to match growth

• Verify assumptions and assign roles


• Draft pre-diligence plan
Next Steps • Establish contact with target
• Conduct detailed valuation and determine BATNA
• Roll out new product

- 60 -
Household Cleaning Services (1 of 8)
BCG, Mock interview

Guidance for interviewer and


Problem statement narrative
information provided upon request(1)

Your client, Restoration Co., is a fire and water remediation 1. The client is only looking at the US right now (all of it)
company, that specializes in extensive cleanup in the aftermath of 2. US population is 300MM
fires and floods. They are currently hired by insurance companies
on behalf of those affected by these disasters. While this existing 3. There are 100MM households in the US
business is quite profitable, they are looking to expand into the 4. US residential cleaning market is growing steadily with
residential cleaning market (typical household cleaning, such as inflation.
vacuuming, dusting, etc.). They have come to BCG with two
questions: 5. Their main goal is to enter the market profitably.

1. What is the size of the residential cleaning market?


2. Should the company enter this market?

- 61 -
Household Cleaning Services (2 of 8)
BCG, Mock interview

Market sizing question

Additional questions for candidate Solution guide

• If candidate comes to 100MM households, Candidate should realize that the higher income people will
• Question for candidate: How would you segment this have a higher WTP. Make the candidate guess first, then
market? give the answer:

• Answers may include: types of homes (response: • Above $75K: 40% willing to pay for service
good idea, but assume all home types are • Below $75K: 10% willing to pay for service
equivalent); geography (good idea, but assume all
geographic locations are same)
• Answer we are looking for: socioeconomic status (i.e.
household income): Assuming households can be
split in half according to whether they earn more or
less than $75,000, what would you estimate the %
willingness to pay (WTP) for each segment?

- 62 -
Household Cleaning Services (3 of 8)
BCG, Mock interview

Market sizing question (cont.)

Additional questions for candidate Solution guide

Given this information, what is the size of the market? • 50MM * 40% + 50MM * 10% = 25MM purchases per year.
• However 25MM is not the market size!
• Candidate should ask what typical cleaning prices are:
$2,000 per year on average.

• Market size: $50B.

• Interviewer should say, “That’s a pretty big number” simply


to test whether candidate is sure of his/her math. Look for
floundering.

- 63 -
Household Cleaning Services (4 of 8)
BCG, Mock interview

Should they enter the market?

Additional questions for candidate Solution guide

How would you go about figuring out whether to enter the Candidate should ask about competition:
market? • National players (10% of market)
• Regional players (20% of market)
• Individual players (70% of market)

Candidate should also ask about customer preference:


• Ranges on a scale from “quality” to “price”
• National players compete on quality
• Individual players compete on price
• Regional players compete on both

- 64 -
Household Cleaning Services (5 of 8)
BCG, Mock interview

Should they enter the market? (cont.)

Additional questions for candidate Solution guide

Where should Restoration Co. position itself on this spectrum? Restoration Co. should position itself based on quality:
• It currently does extensive cleanup after disasters;
surely it can handle regular cleaning.
• It does not want to dilute its brand name.
• It does not want to enter into a pricing war with a
highly segmented market (70% of market competes
on price, and these are individuals).

• Bonus: candidate should recognize that market size goes


from $50B to $5B (10%)

- 65 -
Household Cleaning Services (6 of 8)
BCG, Mock interview

Should they enter the market? (cont.)

Additional questions for candidate Solution guide

How would you figure out the profitability of national players?


• Revenue: $75
• Revenue: Average price for a cleaning service is $75. • Cost: $55 ($10 * 5 + 5)
• Cost: $10/hour on labor; $5/job on cleaning supplies (these • Profit: $20
are all-inclusive costs) • Ask candidate what the profit margin is: 27%
• When asked: average cleaning lasts 5 hours.

- 66 -
Household Cleaning Services (7 of 8)
BCG, Mock interview

Should they enter the market? (cont.)

Additional questions for candidate Solution guide

Anything else to consider before making your final • Candidate should come up with ways to separate
recommendation, such as how to separate ourselves from Restoration Co. from its competition:
the competition? • Using premium cleaning supplies (such as
sustainable chemicals, scented shampoo, etc.)
• Performing a survey of the house to assess for
potential fire or flood hazards.
• Offer a free cleaning first.
• Bad idea: enter into a pricing war in a segment that
competes on quality.

- 67 -
Household Cleaning Services (8 of 8)
BCG, Mock interview

Final Recommendation

Recommendation Risks Next steps


Enter the market: Market size is $50B 27% margin needs to be put in context of Review financing and up-front costs
(even better: $5B), and margins are a current business
healthy 27%, since we will compete on
quality with the national players. How can we use our current client book
Possible brand dilution to jump start business?
- But let’s offer only quality products
How will our sales model change
Need to separate our business (currently relies upon insurance
companies, will change to door to door
- But go with ideas mentioned on the marketing)?
slide before

- 68 -
Bio-Product Growth (1 of 7)
BCG, Round 1

Guidance for interviewer and


Problem statement narrative
information provided upon request(

Your client is a chemical company, ChemCo. Due to stagnant 1. Last year bio-products segment accounted for $300 million in
growth in the chemicals segment, they decided to create a high revenue
growth bio-products segment. This segment sells these products 2. Bio-products industry has been growing at 10%, hence the
to universities and labs primarily for use in drug development. CEO’s target
Revenue for the new segment has been growing at 3-5% per
annum for the last few years. The CEO is unhappy with this and
wants to achieve 10% growth in revenue in the next year. You have
been hired to help bridge this gap and achieve this target.

A good structure will include:


Industry (growth and trends)
Client (growth and trends, limitations on growth, product mix)
Growth Strategies (geographies, channels, acquisitions, product
mix, R&D)

- 83 -
Bio-Product Growth (2 of 7)
BCG, Round 1

Framework

Additional questions for candidate Solution guide

After the structure, when the candidate asks for • Some implications of Exhibit 1 include:
Industry/competitor information provide Exhibit 1. • ChemCo is “over-diversified”
• Could have achieved diseconomies of scope
Allow the candidate to make some insights, push them if • Scale operations could be more efficient
necessary. Eventually get them to consider the use of R&D. • Inefficient use of resources:
What are the implications of R&D from that slide? • Sales and Marketing
• Research and Development
Give them Exhibit 2. • Some implications of Exhibit 2 include:
• ChemCo is below market trend
• C7, C5 & C3 could be studied to understand efficient
R&D
• Inefficient use of R&D funds
• Could eliminate inefficient R&D or spend more due
to high correlation between R&D and revenue

- 84 -
Bio-Product Growth (3 of 7)
BCG, Round 1

Additional questions for candidate Solution guide

Lets look further into R&D Candidate SHOULD use Exhibit 2 to project revenues based on
new R&D/category. If they don’t, ask them for their assumptions.
We have: You can then choose to push them to use Exhibit 2 or continue
• 5 Strategic Products with their methodology.
• Revenue = $200M
• R&D = $10M Slide 2 indicates that if we spend $6 million in R&D per category
• 15 Non-strategic Products ($30 million for 5 strategic categories), then projected revenue
• Revenue = $100M per category is $70 million.
• R&D = $20M
See slide 6 of 7 for projected revenues using this forecast
So what should we do? methodology.

Answer: Reinvest R&D into strategic products

Then:
1. Revenue for non-strategic products will decrease to zero
over 5 years
2. Revenue for strategic products will ramp-up over 3 years

- 85 -
Bio-Product Growth (4 of 7)
BCG, Round 1

Exhibit 1: Competitor Revenues and Product Mix

400 380

350 316
300
300
250 240
250

200 185

150 125

100 67
50 36

0
C1 C2 ChemCo C3 C4 C5 C6 C7 C8

C1 C2 ChemCo C3 C4 C5 C6 C7 C8
Revenue ($ millions) 380 316 300 250 240 185 125 67 36
# of Product Categories 6 6 20 4 7 4 3 2 1

- 86 -
Bio-Product Growth (5 of 7)
BCG, Round 1

Exhibit 2: Industry Revenue against R&D Spend

70
C3 C1
Revenue per Category ($millions)

60

50 C5
C2

40 C4 C6
C7
C8
30

20 ChemCo

10

0
0 1 2 3 4 5 6

R&D per Category ($millions)

- 87 -
Bio-Product Growth (6 of 7)
BCG, Round 1

Since we are projecting $70M in revenue per strategic category we will ramp up from $200M to $350M over 3
years and then stay constant.
For the non-strategic products the revenue will decrease from $100M to $0 evenly over 5 years.

REVENUE PROJECTIONS
Year 0 Year 1 Year 2 Year 3 Year 4 Year 5

Strategic Categories $200 $250 $300 $350 $350 $350


Non-Strategic Categories $100 $80 $60 $40 $20 $0
TOTAL $300 $330 $360 $390 $370 $350

Note:
This is the best method for projecting revenue, however the candidate may choose another method. There
should at least be a logical reason for their methodology. Additional methods could include a linear increase -
$10M in R&D generated $200M in Revenue, so $30M in R&D will generate $600M in Revenue.

- 88 -
Bio-Product Growth (7 of 7)
BCG, Round 1

• Our client should reorganize their R&D efforts to focus on the 5 strategic products. This will achieve the 10%
Sample growth in year 1 from $300M to $330M. In the long run we should consider focusing our future R&D as well.
Recommendation

• We have only reviewed by strategic vs. non-strategic. We may be missing good products by not looking at the
A solid interview will individual product categories.
address other • Assumption that R&D is the only factor that influences revenue. If we don’t get $70M per category, the growth
potential risks… target will not be achieved.

• Look into revenue and R&D by product to get granular details.


And suggest next • Consider sixth product for future growth possibilities, but beware of over-diversification again.
• Consider competitive analysis to understand what 4-7 product categories everyone else focuses on.
steps…

- 89 -
Li-Ion Battery Separators (1 of 7)
BCG, Round 1

Guidance for interviewer and


Problem statement narrative
information provided upon request(1)

Your client is a U.S. Textile Manufacturer. They have recently • Your client has no prior experience or knowledge of the battery
developed a new technology for making Lithium Ion battery separator market
separators. Is this an attractive industry? And should your client • Battery separators are an integral part of the lithium ion
enter the market? batteries. They need to be thin and provide a medium for
charged particles to pass between the cathode and anode
A good structure will include: (positive and negative terminals).
• Industry analysis (five forces, profitability, competition, • We do not know if our technology is better than existing
customers) technology.
• Client (patents, experience, etc) • Your client had $250 million in sales last year.
• How to Enter (JV, Greenfield, Acquisition, Licensing) • No current patent, but we can get a patent on our technology.
• Exit strategies • Safety is a big issue in this industry and so there is a very
expensive 1-year certification process

- 90 -
Li-Ion Battery Separators (2 of 7)
BCG, Round 1

Additional questions for candidate Solution guide

After the structure, allow the candidate to ask for data. Feel A good analysis will consider the following:
free to push them on the definition of an attractive Industry – • Barriers to Entry (HIGH)
sustained profitability. • High switching costs between suppliers
• Expensive certification process
• Existing customer relationships
• Provide the exhibits when the candidate asks the • Supplier Power (HIGH)
appropriate questions. • Can charge premium price for certified safe product
• Existing relationships
• Rivalry (LOW)
• Limited competition due to existing relationships
• Pose the problem again after they have the exhibits - Is this • Profitable
an attractive industry? • Price drop in separators less than price drop for rest of battery
components
• Because component costs have decreased at the same rate this
indicates higher margins for separators

- 91 -
Li-Ion Battery Separators (3 of 7)
BCG, Round 1

Li-Ion Battery Separator Manufacturers

100% 1%
4%
90% 7%

9%
80%

70%
25% C7
60% C6
C5
50%
C4
40% 26% C3
C2
30%
C1
20%
28%
10%

0%
Market Share

- 92 -
Li-Ion Battery Separators (4 of 7)
BCG, Round 1
Customer Supply Base

100%

90%

80%

70% C7

60% C6
C5
50%
C4
40%
C3
30% C2

20% C1

10%

0%
Sony LCG Sanyo Other

•Customers generally have a primary supplier and a secondary supplier


•Industry characterized by strong supplier relationships given safety concerns

- 93 -
Li-Ion Battery Separators (5 of 7)
BCG, Round 1

Battery Component Prices

2000 2010
Cathode $8 $4
Anode $2 $1
Separator $3 $2
Other $7 $3
TOTAL $20 $10

Note: Costs have decreased at the same rate for all battery components.

- 94 -
Li-Ion Battery Separators (6 of 7)
BCG, Round 1

Additional questions for candidate Solution guide

A good analysis will consider the following:


So, should we enter the Li-Ion Battery Separator Market? (The
• Green Field:
candidate must realize that this question must also cover
• Pros – Management Control
“how should we enter”. If they do not go down this route,
• Cons – Expensive and timely
direct them with the additional questions.)
• Joint Venture:
• Pros – Easier and quick
• Cons – Limited control, finding partner
• What are the possible ways to enter?
• Acquisition:
• What are the pros and cons of each?
• Pros – Quicker, Management Control
• Cons – Culture clashes, buyer’s curse (overpaying)
• License/Sell
• Pros – Easier, less commitment to new market (get your milk
without the cow)
• Cons – finding buyers

- 95 -
Li-Ion Battery Separators (7 of 7)
BCG, Round 1

• Our client should License the technology due to a lack of knowledge and experience in the Li-Ion Battery
Sample Separator market, existing buyer-supplier relationships, and the expensive certification process inherent in the
industry.
Recommendation

• No industry experience
A solid interview will • Existing manufacturers will not want to change technology
address other • No discussion on benefits of new technology
potential risks…

• Understand technology and value proposition to manufacturers


And suggest next • Patent technology and determine licensing fee
• Identify interested manufacturers and develop interest from customers (Sony, LCG, Sanyo, etc.)
steps…

- 96 -
Case: Greeting Card Manufacturer
BCG, Mock Interview
64

Problem statement narrative


A greeting card manufacturer has experienced decreased profit. The CEO has asked you to figure out why.

Overview for interviewer Information to be provided upon request

This case involves a discussion of both the revenue and cost The greeting cards are stocked at grocery stores and
drivers of profit. Greeting card companies operate with a pharmacies like CVS
unique revenue system, and this will also affect the cost side
of the company. Greeting cards are good for one season only – if a card
does not sell by the end of the season, it will be shipped
back to the manufacturer at their expense and discarded

Case Type: Profitability The greeting card industry has experienced moderate
growth over the years

Competitors have experienced steady or slightly increased


profit
Case: Greeting Card Manufacturer
Potential Categories of Candidate’s Framework
65

Notable comments / potential discussion points


Industry Analysis
• Greeting cards market: growing, shrinking, stable?
• Competitors: market share, growth rate
• Consumers: needs, brand perception, differentiation?

Profit drivers
• Revenue
• Factors that affect sales volume
• Card selection
• Card supply
•Is it better to overstock or stock just the right amount?
• Overstock: customers like choices; it looks bad when there is only one card on the shelf –
creates more goodwill when there are additional cards even if they will not all sell
• Just the right amount: do not incur additional shipping costs to send unsold cards back to the
manufacturer; do not incur variable costs for unsold cards; why make more than you can sell?
• How does the greeting card manufacturer earn revenue? What are the revenue streams?
• Costs
• What are the fixed costs?
• What are the variable costs?
• Is there anything special about greeting card costs?

The candidate must figure out how the greeting card manufacturer earns revenue as well as how costs are incurred.
Case: Retirement Apartment Complexes
BCG, Round 1
87

Problem statement narrative

The client owns and operates 25 retirement apartment complexes for the 55-75 year old demographic in the south-east and
south-west states of FL, CA, NM and AZ. By and large the apartment complexes have a similar design and amenities. Should the
client company’s CEO consider expanding to other Northern cities or continue to focus on the South?

Overview for interviewer Information to be provided upon request

Even though this case starts out with a market entry problem candidate should probe deeper to understand the market in
question, the candidate should eventually realize (with or the North and why the client wants to consider expanding.
without guidance) that the case is about profitability,
specifically costs as there is clearly an unmet market in the • Any specific financial goals? – maximizing profitability
North and hence revenues wouldn’t be an issue. through new apartment existing properties.

Ultimately, the candidate should use some math to determine • What makes the client think that there is demand in the
that the financial justification for expanding in the North vs. North? – Survey conducted 2-3 years ago in states
South is roughly equivalent, so strategic benefits and risks such as NY, NJ, MA and IL revealed that the pre-
should guide the client’s focus. retirees /retirees want similar facilities in those
underserved (hence no competition exits) markets so
that they can stay closer to their families and friends.
Case Type: Profitability/ Market Entry
• A pilot apartment complex was erected in downtown
Chicago 2 years ago with more amenities than its sister
properties in the south.
Case: Retirement Apartment Complexes
Potential Categories of Candidate’s Framework
88

Notable comments / potential discussion points


Most candidates will tend to examine revenues and costs once they realize this case is about maximizing profitability. While this is
the eventual course of action it is important for the candidate to understand the current markets that the client operates in and glean
key learnings. Remember – if the candidate had asked the financial goal question, the client wants to maximize profitability through
new or EXISTING properties.

Potential points include, but are not limited to:

Industry analysis
• firm position / market share
• trends
• competition

Profit drivers
• revenue
• costs
Case: Retirement Apartment Complexes
Potential Issue Tree & Approach to Solving the Case
89

Key elements of analysis to solve the case

Market Size Revenue Cost


When dealing with apartment Identify sources of revenue from Identify cost buckets and distinguish fixed
complexes, examine the following: apartment complexes: from variable components

• Number of properties • Rent • Maintenance


• Number of units per property • Utilities, if managed by building co. • Amenities
• Occupancy Rates • Premium for amenities, if any • Leasing/Marketing
• Retail revenue, if any • Utilities
• Maintenance or other fees • Property Taxes

Possible follow-up and guidance Possible follow-up and guidance Possible follow-up and guidance to
to interviewer to interviewer interviewer
If the candidate has not yet asked for Recall that the Chicago property pilot
the current apartment complexes in the provides residents with additional
Southern states, give her/him details: amenities compared to properties in the
• Competition is strong in the south; south. The candidate should examine this
easy to build new apartments. aspect to accurately compare costs.
• Occupancy rates typically start out at
90%+ but eventually settle at 80% as
new complexes are built.
• 3-5% growth every year fed by new
people who migrate to the south
Case: Retirement Apartment Complexes
Question 1 – Math
90

Math Question: Which is more profitable – north or south?


1. First compute occupancy levels and revenue. For this discussion all apartments in a given region can be considered equivalent.

South: 800 units/bldg * 0.8 occupancy rate * $500 rent/mo = $320,000 per mo (let candidate know that rent includes
maintenance, amenities, utilities; each property generates identical revenue so leave calculations on an individual property basis)

North: 400 units * 0.9 occupancy rate * $1,000 rent/mo = $360,000 per mo
Rent includes amenities charges for a 24 hr. on-site medical facility that residents benefit from in a huge way

2. Next compare costs (per month)

Cost Item North South


Maintenance 400 units * $200 = $80,000 800 units * $125 = $100,000
Amenities $54,000 $48,000
Utilities $45,000 $38,000
SG&A $60,000 $80,000
Medical Facility $60,000 $0

Total Cost per mo $299,000 $266,000


Total Revenue per mo $360,000 $320,000
Profit per mo per bldg $61,000 $54,000
Case: Retirement Apartment Complexes
Question 1 – Math, cont’d
91

Math Question cont’d…


3. Next calculate profitability for both locations by using the formula
Profit per mo per building
÷
Total Revenue per mo
4. Turns out that the south and north are equally profitable at 16.8% approx.

Overall approach, good shortcuts & solution Information to provide up front


The key to this question is not to stop at calculating profit and N/A
jump to a conclusion then. A good candidate will take the next
step to calculate profitability and realize that the South and
the North are equally profitable relative to revenue
generated.

Now the candidate should consider the profitability relative to


the cost to implement various initiatives. Is it more expensive to Provide information if asked
open in the North or South? What is the incremental cost of
At this point the candidate may ask about the market
constructing medical facilities in the south (relative to increased
size in the North or the cost to open new facilities in the
earnings)? What is the cost of enhancing some amenities in the
North (or the cost to renovate / alter facilities in the
South to generate higher rent? Higher NPV is the goal.
South). There is no additional data available, but for the
purposes of this discussion we can assume that the return
We will not explore the capital cost in this case, but a return on
relative to cost is also comparable in North vs. South.
cost analysis should be part of next steps.
Case: Retirement Apartment Complexes
Sample Recommendation
92

To improve shareholder return and maximize profitability, the CEO of the Retirement Homes should
consider expanding in the north to capture unmet demand and be the first mover in this area. To increase
profitability in the South the following could be considered:

1. Increase revenues by opening medical centers in all apartment complexes and increase the rent. We
Recommendation have seen that people will pay for the convenience of such an amenity.
2. Decrease costs using best practices with the Chicago site, re-negotiate contracts, etc.

The key here is that the candidate should make a strategic argument for the recommendations he makes.
Given that the financial measures are roughly equivalent here (profit margin per region, profit relative to
capital investment (NPV) per region, strategic intuition will drive the client’s decision.

candidate should mention risks such as cost of expansion in the North, the commercial real estate issues
facing the industry however, pulling this business insight from the information provided during the case
Risks interview will distinguish a good response from an insightful one - 90% occupancy is not sustainable in the
long run given what happens in the South.

Evaluate cost to open new buildings in the North and cost to renovate in the South.
Size demand in major Northern cities and, cost of expansion notwithstanding, consider expanding rapidly
Next Steps to secure demand.
Examine how best practices can be applied to existing properties North to South and vice versa.
Establishing the case
49

Case2 : China Outsourcing Opportunity


BCG: Round 1, practice mock case

Guidance for interviewer and information to be


Problem statement narrative
provided on request
The client is a national plastic manufacturer in the US. 3 Product lines – freezer bags, plastic plates and
Their customers are supermarkets and discount retailers. utensils, and specialty plates and utensils
They are looking to outsource manufacturing to China
but the CEO is concerned because no one else in the
industry has done this yet. Should we recommend to our
client to outsource to China?
Sample solution element – issue tree & qualitative analysis
50

Candidate may propose analysis / action in:

Cost Savings Consumer Preferences PR / Brand Risks


• Fixed and variable costs of • Preferences regarding quality • Layoffs in the US
producing the US vs. China issues with products • Labor standards in China
• Suppliers? • Seasonality of products
• Transportation Costs
• Tariffs

Possible follow-up and Possible follow-up and Possible follow-up and


guidance to interviewer guidance to interviewer guidance to interviewer
• See handout to calculate •Quality :Freezer Bags – lower quality in • PR – not big issue, no real
differences in costs China information
•Plastic Plates – equal quality
•Specialty Plates – equal quality
• More important to show awareness
•Consumer Prefs: Freezer Bags – commodity, of these risks
but quality very important
•Plastic Plates – commodity, quality not issue
•Specialty Plates – seasonal business, trend
is important
China Outsourcing Opportunity – Handout
51

Calculate the Costs of Outsourcing in China

Math Question

Costs in Costs in China relative Costs in


Costs U.S. ($/lb) to U.S. China ($/lb)

Labor 0.30 8% of wage rate


80% of productivity
Material
• Plastic resin 0.30 80%
• Other material (incl. packaging) 0.20 75%

Variable overhead 0.05 140%

Fixed overhead 0.10 60%

Transportation
• China to U.S. distribution center N/A $6K to ship 40K lbs.
• U.S. distribution center to customer 0.05 Same

Total 1.00
China Outsourcing Opportunity – Solution
52

Calculate the Costs of Outsourcing in China

Math Question Looks like they will save 25% of costs by going to China

Costs in Costs in China relative Costs in


Costs U.S. ($/lb) to U.S. China ($/lb)

Labor 0.30 8% of wage rate =.3*(10/8)*(8/100) = .03


80% of productivity
Material
• Plastic resin 0.30 80% .24
• Other material (incl. packaging) 0.20 75% .15

Variable overhead 0.05 140% .07

Fixed overhead 0.10 60% .06

Transportation
• China to U.S. distribution center N/A $6K to ship 40K lbs. =6/40 = .15
• U.S. distribution center to customer 0.05 Same .05

Total 1.00 .75


Solution element – recommendation et al.
53

Recommend outsourcing the paper and plastic plates to China. Keeping the other lines in the US due to
quality and trend issues.
Recommendation

Change in fixed overhead costs currently if move production to China. Additional mfg capacity – other
Risks product lines, rent out to another company, close certain lines; PR / Brand image risks

Research risks mentioned above to determine whether beneficial to implement


Next Steps

BONUS
What might be some other benefits?

- Potentially easy access into the growing Asian


economies in the plastic market.
Establishing the case
70

Case7 : Jamaican Land Investment


BCG: Round 1

Problem statement narrative Information to be given upon request (cont‟d)

Our client is thinking about buying a piece of land in •Fixed Cost: $500 initial set up (first year only), $350 per year
Jamaica for $3000 and has asked us to determine for salaried labor
whether or not this is a good idea. •Market Demand per year
• 5000 Trees, 1000 shrubs, 1000 fruit, 2500 exotic
flowers
Information to be given only upon request •Penetration rate: competitors cannot meet current demand.
•The price of the land is $3000 • Competitors have 60% tree share, 20% shrub
•Total acreage: 10 acres share, 85% fruit share, 90% exotic flower
•Financial Target: $4,500 profit within first two years, excluding share…the remaining shares can be captured by
$3000 purchase price our client.
• When prompted about use of land, ask candidate to brainstorm •How many plants can fit on an acre? When asked about how
possibilities before giving him/her the answer: real estate much of each plant can fit onto an acre, throw the question back
development, farming, hold and sell it once it appreciates, etc.) and ask: “which plants do you think would have less of per acre?”
•Land will be used for agriculture (trees and shrubs b/c they take up a lot of room)
• Trees, Shrubs, Fruit, Exotic Flowers • 10 Trees/acre
•Cannot mix products (trees and shrubs) on same acre. Only one • 25 Shrubs/acre
type of plant allowed per acre. • 75 Fruit/acre
•Price per plant • 50 Exotic Flowers/acre
• Tree $50, Shrub $35, Fruit $15, Exotic $25
•Variable Cost per plant
• Tree $30, Shrub $25, Fruit $11, Exotic $17
Case Sequence
71

Candidate may propose analysis / action in:

Market Size/Est. Demand Margins Per Unit Profitability Per Acre


Demand P-C=Profit per unit # of units per acre x profit per unit
Market Size x Client Penetration Rate $50-$30= $20 per Tree 10 trees per acre x $20= $200/acre
•5000 trees x 40%= 2000 units $35-$25= $10 per Shrub 25 shrubs per acre x $10= $250/acre
•1000 shrubs x 80%= 800 units $15-$11= $4 per Fruit 75 fruit per acre x $4= $300/acre
•1000 fruit x 15%= 150 units $25-$17= $8 per Exotic 50 exotic flowers per acre x $8=
•2500 exotic x 10%= 250 units $400/acre

Supply
*Client‟s capability to meet estimated
demand is dependent on the number of
acres it has (10 acres).

Possible follow-up and Possible follow-up and Possible follow-up and


guidance to interviewer guidance to interviewer guidance to interviewer
Go From Most Profitable To Least 5 acres of Exotic Flowers x $400/acre= To make the case more challenging, ask:
•Exotic Flowers: 250 units demanded/50 $2000 -How would you price each unit? (cost-
units per acre= 5 acres used fpr Exotic 2 acres of fruit x $300= $600 based pricing, look at competitors prices,
Flowers 3 acres of shrubs x $250= $750 price by segment (premium supermarkets vs.
•Fruit: 150 units demanded/75 units per fruit stands, etc.)
acre= 2 acres used for Fruit Y1: $3,350-500-350= $2,500 (do not
•Shrubs: 800 units demanded/ 25 units per include $3000 investment) -If demand was there, would you use all 10
acre= all remaining acres (3 acres used for Y2: $3, 350-$350= 3000 acres for exotic flowers? Why or why not?
Shrubs) (important to diversify products!)
Total # of acres should add to 10! Total Profit For First Two Years: $5,500
Sample solution element – recommendation et al.
72

Recommended Approach
Clarify the Problem: Interviewee should always spend time clarifying the objective(s)
upfront. Find out what the land will be used for, how many acres, financial target of client,
etc. in order to come up with a more precise structure and to avoid stumbling later on in the
case.

Determine Demand & Assign Plants To # of Acres Based on Profitability: Interviewee


should determine total demand of each product and the client‟s penetration rate (market
share that client can grab). After finding estimated demand, interviewee should look to
assign each type of plant to a specific number of acres. This involves figuring out the margins
per unit, the profitability per acre of each plant (most profitable plants get first priority in
acre assignments), and how many plants can fit onto an acre.

Determine If Investment Would Meet Client‟s Financial Target: Once interviewee assigns
plants to a specific number of acres, he/she should calculate the total profitability for the first
two years. Interviewee should take into account the fixed costs for each year. Running the
numbers, he/she will find that the investment will exceed the client‟s financial target of
$4,500.
Sample solution element – recommendation et al.
73

“My recommendation to our client is to invest in the Jamaican property. My calculations show that we
Recommendation would achieve $5,500 in profit within the first two years, exceeding our client‟s financial target by
(Answer First) $1,000.”

“Before making a final decision, however, I would look into the growth rate and expected demand for these agricultural
Risks products, and any risks from natural disasters such as hurricanes, drought, plant diseases, etc.”

“At this point in time though, given my calculations, investing in this land looks like a great idea.”
Next Steps

Notes
•It is important to let the candidate drive the case!

•The objective of the case is vague and ambiguous. Candidates


must spend time clarifying what exactly a “good idea” is in the
eyes of the client.

•Before calculating the figures, it‟s always a good idea to tell the
interviewer what you‟re going to do next.

•Remember to label your units and keep your math neat. Use as
much paper as necessary.

•Once you arrive at a figure, step back and explain what the
number means for the benefit of both you and the interviewer.
Establishing the case

Case: Cash-Rich Energy Company


BCG: mock interview

Guidance for interviewer and information to be


Problem statement narrative
provided on request
Your client is an Atlanta-based energy company that sits The interviewee should focus on the (financial) soundness
on piles of cash. They are looking for a good investment of the proposed investment. There are several
and are currently considering an acquisition and investment criteria that could be applied: ROI (return on
consolidation of small service companies that service investment), breakeven point/payback time, opportunity
heating and cooling systems of other companies and costs (consider and compare with other investments) and
households. NPV (net present value or calculating discounted cash
flows).
The client would like you to help them understand
whether this investment financially makes sense. If the interviewee jumps into one investment criterion
(normally, that would be a simple one-year profitability
calculation), the interviewer should push back on the
investment criteria and the interviewee should briefly
explain how she would proceed under each criterion
(i.e. what data would she need, the process of
calculating and advantages/disadvantages).

After that, the interviewee should be asked to use


revenues and costs after the consolidation.
Sample solution element – issue tree & qualitative analysis

Candidate may propose analysis / action in:

Revenue Costs Industry


Target companies: Explore expected Target companies: Explore the costs of Customers: Understand the target
revenues of the target companies. the target companies; fixed costs companies‟ customers by exploring their
Consider how the consolidation would (SG&A) and variable costs needs, demographics, loyalty, satisfactio
affect the revenues (e.g. potential (labor, material), and the potential n with companies‟ services and any
decrease due to cannibalization or affect of the consolidation on these recent changes in their behavior that
additional streams due to increased costs. could affect their needs and/or use of
market power). Acquiring company: Explore the the services (e.g. temperature changes
Acquiring company: Consider the acquisition costs (deal price) and any in the environment) and how these could
effect of the acquisition on the acquiring other affects of the acquisition on the affect future revenue streams.
company and its subsidiaries, if any. costs (e.g. increased SG&A costs etc.). Competitors: Understand the
competitive landscape of the
Possible follow-up and Possible follow-up and industry, major market players, their
guidance to interviewer guidance to interviewer market shares, products, pricing
The interviewee should be specific in The interviewee should demonstrate stretegies and geographic focuses, as
asking for revenues streams, e.g. either understanding of cost structure for the well as expectations of potential new
per company/year (multiplied with the target companies and the acquisition market entrants.
no of companies) or per customer/year company, as well as how a Regulation: Understand anti-trust
(multiplied wtih the no of consolidation can decrease overall costs regulation and whether that could be a
customers/company and no of (cost synergies). deal-breaker.
companies) or ask for price and volume
(no of companies and no of
customers/company), and demostrate
understanding on how a consolidation
can boost revenues (revenue-synergies).
Sample solution element – Math

If the interviewer does not attempt to calculate costs and revenues (profits) after the consolidation, the
interviewer should ask her to calculate profits for the target companies. For simplicity reasons we
Math Question assume that there are no changes in the acquiring company‟s revenues and costs, or any other
acquisition costs apart from the deal price.

Overall approach, good shortcuts & solution Provide information if asked

Ultimately, the interviewee should calculate profits/cash flows. Given the Cost reduction (for all target
information it is best to start with a per-company calculation: companies): 5% decrease in labor
Cost savings: costs, 5% decrease in equipment costs
5% of $5M = $0.25M (labor) and 1% decrease in SG&A costs.
5% of $2.5M = $0.125M (shortcut: half of the previous number) (equipment) Curent cost structure/company: $5M
6% of $2M = $0.12M (SG&A) for labor, $2.5M for equipment and
Total cost savings/company: $0.25M+ $0.1.25M+ $0.12M=$0.495M (shortcut: $2M for SG&A or alternatively 50% of
round up to $0.5M and recognize this represents 5% of revenue) revenue for labor, 25% of revenue for
Revenue/company: $10M equipment and 20% of revenue for
Profit/company (shortcut): recognize that profits before consolidation were SG&A.
5%, cost savings increase the profits for additional 5%, therefore 10% of revenue or Number of target companies: 500
10% of $10M = $1M Revenue/company : $10M/year
Total profit: 500x$1M = $500M (per year) Profit growth (g) : 3%/year
NPV (assuming perpetuity): CF/(r-g) = $500M/(0.13-0.03) = $5B (shortcut: when Discount rate (r) : 13%
dividing by 0.1, just add another zero)

Alternative Note that there are several ways of calculating the NPV in this case, however, most of them are more
Calculation time consuming. Since none of the information is given upfront, note that the interviewee may make some
Approaches assumptions (e.g. discount rate), which is all right as long as they are reasonable.
Sample solution element – recommendation et al.
122

The decision whether the client should make the proposed investment should be based primarily on the
deal price. Given that we have just calculated the NPV of the investment to be around $5B, the client
should not pay more than that. The more they can negotiate the price down from $5B, the better the
Recommendation return on the investment will be. However, before they close the deal, they should first gather more data
and perform a more careful analysis on revenue, costs, and profit growth especially for the longer-term
projections. And second, they should consider other investments, and compare the NPVs and ROIs.

First, for the lack of data, we have assumed that there are were no anti-trust issues with the
consolidation, but that would have to be verified with an attorney before proceeding with the
negotiations. Second, we have not had a chance to analyze the competitive landscape and the customer
Risks
base which could both importantly impact the revenue projections. Finally, we have not probed and
verified any other assumptions on which the costs and revenue projections were made, which could also
significantly impact the NPV calculations.
As suggested above, verify the assumptions on which the revenue/cost projections were made, obtain an
Next Steps opinion from an anti-trust attorney and consider other investment opportunities.

BONUS This is not an easy


case, especially for people
The more investment criteria the interviewee considers at the beginning and the more who are not comfortable
shortcuts used while calculating the NPV, the better.
with finance. The
One other twist is to also evaluate pay-back. Say, price is $2B and client typically expects interviewer should be well
pay-back in 3 years for their projects. Answer will be do not purchase according to the prepared and provide
calculation, but see if interviewee tries to persuade you as to why the client should be using guidance where and when
DCF/NPV instead as the risk of this acquisition (reflected in discount rate) makes the return appropriate.
worthwhile and using same payback requirement across all project types is likely incorrect.
CASE 1: OPTICAL FIBRES (BCG – Round 1)
Our client makes optical fibre (has volume advantages to copper wire) which is mainly used by
the telecom, cable and mobile phone industries. It’s made in glass strands and rolled onto 25 km
spools. Their customers (who are generally major Telecoms networks) would buy a huge
quantity, bundle it up, dig a trench and put the bundle of fibres in the ground. They’ve seen a
50% decline in revenues, and have recently brought in a new CEO. The new CEO would like to
know:
a) Why did revenues drop 50% in one year?
b) Can he expect an improvement, and if so in what timeframe?
c) How does our company compare to our competitors?

Additional Information Provided After Relevant Questions:

• Optical fibre is similar to a commodity, so consumers do not purchase by brand and pricing is in
line with competition.
• There has been no significant change in supply from the company’s perspective.
• There are no direct substitutes and no regulation changes during this period.
• The “usage” of the optical fibres, for voice, data and internet, has actually doubled each year.
(meaning you would think you’d actually need twice as much each year)
• Customers are asking for less.
• No major changes in customer’s industry. Even spread across industries for product sales.
• Current usage is 5 T/second, but capacity is 100 T/second.

Interviewee’s Solution:

Why did revenues drop 50% in one year?


I determined the problem was that because it is such an effort and expense to dig the trenches to put
the fibre bundles in, the customer does it as infrequently as possible which leads to overcapacity.
Can he expect an improvement? If so when?
I found out that they buy a year in advance of needing it. Since usage doubled yearly, I worked out
that customers would reach capacity in 3.5 years, and therefore we would be impacted for 2.5 years.
The CEO wants a benchmark slide with how we compare to our competitors: what would be the
main items you’d want to see on that slide?
I asked for Market Share (leader), Cost (lowest cost), People (no advantage), Customer Base (no
advantage), Product (no real advantage), and Balance Sheet (highest)
He asked if we could weather the storm or if we were in trouble.
I said if we had a pretty healthy financial situation, we could weather the storm for 2.5 years but that it
would be worthwhile looking for other ways to diversify since this would be a cyclic problem.

3
Commentary/Recommendations:

The key to answering this question is understanding what is actually going on in the industry.
To get to the heart of this issue you could ask:

Clearly our client has lost a significant portion of their revenue. To understand why there are three key
things I would want to examine.
• First, what has changed about our Client – we haven’t heard of any major reason why things
would be going wrong internally but some of the things that I would want to look at are: Have
they lost key sales personnel with good customer relationships? Have they lost their good
reputation because of some incident or accident etc? Have there been perceived quality
problems? Etc
• Second, I would want to look at our key Competitors – Are they offering a better product?
(probably not if the industry is commoditized) Are they selling at a lower price? Are they
offering better service/delivery times etc? Are there now more competitors? Are our
competitors better placed geographically to meet the needs of our customers? Are their some
competitors that are doing better than us? If so how do they differ from us? Are they bigger
and leveraging their scale, etc?
• Third, I would want to look at the needs of our Customers and whether these have changed. –
If there is nothing wrong with our product (Client) and our Competitors haven’t suddenly got
better then the downturn in sales must have something to do with the needs of our Customers.
What is their current usage? What is their current capacity? What is their anticipated usage?
What are their purchasing patterns relative to capacity? etc

This structure should drive you to identify the key issue. The way the facts are presented it looks like
most of the questions on Client and Competitors will yield little return and the richest part of the
discussion will focus around Customer needs. This should provide the opportunity to discover all of
the facts about the growth in demand and the capacity constraint currently in place. It could also lead
to understanding the “lumpy” nature of these capital purchases.

This discussion should also draw out enough information to answer the follow up question on how our
client compares to its competitors.

4
CASE 6: PARTY GOODS (BCG – Round 1)
Our client is a manufacturer of party goods. They have two product portfolios: everyday and
seasonal. Everyday products pertain to occasions that happen throughout the year such as
birthdays and anniversaries. Seasonal goods are for occasions such as the 4th of July or
Thanksgiving. They have a design facility that comes out with different party themes and takes
care of licensing for goods using characters like Mickey Mouse or Shrek. They also have a
manufacturing and distribution facility and a sales force that provides support to retailers.
One of their large and new customers came to management complaining that their prices are
too high. They claimed that this is due to high distribution costs. After some analysis, the client
found this to be true. We have been asked to figure out how we can decrease distribution costs.

Additional Information Provided (spreadsheet):


Freight Calculation

Everyday Months Seasonal and Everyday Months


UPS Zone Number of Price per Box Boxes Shipped Total Cost Per Boxes Shipped Total Cost Per
Stores Per Month Month Per Month Month
1 1 3 200 600 800 2400
2 1 4 200 800 800 3200
3 1 5 200 1000 800 4000
4 1 6 200 1200 800 4800
5 1 7 200 1400 800 5600

Monthly Total 5000 20000


No. of Months 8 4
Annual Total 40000 80000
Supplier Alternatives:

Truckload - fill empty truck with goods up to 1000 boxes per truck, costs $1500 per truck, makes only one stop
Less than Truckload - fill part of truck with goods up to 500 boxes per truck, costs $1000 per truck, makes only one stop

Sample Computation:

For Everyday Months:


Use UPS for Zones 1 and 2 where Cost per month less than $1000 1400
Use less than truckload for Zones 3, 4, 5 3000
Total Monthly Cost 4400
Cost Savings per Month 600

For Everyday and Seasonal Months:


Use Truckload for all Zones since truckload cost of $1500 is less than UPS cost per month 7500
Cost Savings per Month 12500

Total Annual Cost Savings 54800

Interviewee’s Solution:
Interviewee: I first need to understand the distribution process and cost breakdown. Then, I want to
benchmark against competitors and find out what they are doing in terms of distribution. Then,
based on the info, I want to figure out what the best alternatives are for our client.
Interviewer: That’s a good framework. Let’s start with costs. What do you think are the costs related
to distribution?
Interviewee: Labor, Trucks, Sorting machines, OH, freight etc. I went through the entire value chain
and identified costs.
13
Interviewee’s Solution (cont.):

Interviewer: Here’s a breakdown of freight costs. They use UPS to distribute goods. (Explains table
attached) What do you think?

Interviewee: I’d like to get more information about other suppliers, the geographic concentration of
our customers and seasonal volume.

Interviewer: Other competitors use a mix of UPS and trucks to distribute goods. (See spreadsheet
for more info) How do you think we can save during our everyday months? How about during the
seasonal/everyday months?

Interviewee: (computes cost difference and savings)

Interviewer: You’re meeting with the CEO tomorrow. How will you summarize your findings?

Interviewer: There’s great opportunity to save using a combination of UPS and truckload services.
From our initial computations we can save as much as $54,800 annually. (see computation)

Commentary/Recommendations:

This case is relatively straightforward and, if you can do the math, then the case should practically
solve itself.

The key to being successful is to ensure that you ask the right questions upfront to obtain all of the
necessary information to calculate the cost savings and that you then logically and thoroughly drive
through those calculations.

The candidate has done a pretty good job of that here, first asking about the distribution process and
about the alternatives used by competitors to elicit information before starting the calculations.

If you are good with numbers then these cases can be a little boring as you are never called on to do
anything really creative and you will probably never feel out of your depth. That is not necessarily a
bad thing however as there is probably very little risk of you going too far wrong.

If you are not good at math, then this will be a good practice question. It is essential that you get to the
point where you are proficient and comfortable in analyzing data and running a series of computations
to calculate cost savings or profit improvements. These questions are not uncommon.

14
CASE 7: CPG – RAZORS & BLADES (BCG – Round 1) MARKET SEGMENTATION

Our client is a global CPG company, with multiple lines of business (toothpaste, batteries, hair
care, etc). They are the global market leader in every market they play in, and have come to us
specifically asking how they can grow their share in the razors and blades market. What do you
tell them?

Additional Information Provided After Relevant Questions:

• Market is divided by wet-shaving and dry-hair removal (e.g. waxing, electric, etc).
• Wet shaving is divided into cartridge-systems and disposables in shaving. Client has 70% of the
men’s shaving market; 35% of the women’s cartridge-system market.

Interviewee’s Solution:
Thru some basic math on sheets she gave me but then kept; I determined that Client has 50 of every
100 women who wet-shave; but only 35/100 of all women who remove hair. Certain countries had
different attitudes; people along coasts were more likely to remove hair than in middle of countries. 4
reasons that women weren’t wet-shaving:
• Fear of being cut
• Wet shaving considered a chore
• Masculine – shaving not seen as feminine
• Shaving considered “lower class”
BCG took these reasons & implemented various marketing programs to change consumer perception
& improve % of total market that client had.

Commentary/Recommendations:
We are not given a lot of information from the actual interviewee so I would recommend using the
following data to flesh out the detail of the case as needed:

Percentage of men who use hair removal products regularly: 95%


Percentage of men who shave as their primary form of hair removal: 90%
Client’s share of men’s shaving market: 70% (70% of disposables and 70% of cartridges)
Percentage of women who use hair removal products regularly: 90%
Percentage of women who shave as their primary form of hair removal: 70%
Client’s share of women’s shaving market: 50% (35% of cartridges and 65% of disposables)
Disposable systems cost on average half the amount per shave as cartridge systems.
Cartridge shaving costs on average half the amount per hair removal as dry-hair removal.
Case recipients should then have to ask relevant questions to extract this data. An analysis could then
determine that there is little room for growth in the men’s market and the client would need to focus
on growing its share in the women’s market.

Potential strategies to explore would include:


• Grow the total women’s market – encourage more women to shave
• Grow market share (especially cartridge share) – encourage existing customers to migrate from
cheaper disposables to more expensive cartridge systems
• Increase price – introduce marketing aimed at shifting the image of shaving up-market to
enable the client to sell at a higher price. i.e. address the price discrepancy between wet & dry.
15
CASE 8: SUPERMARKET (BCG – Round 1)
Our client is a supermarket in the UK. They’ve been the market leader for 50+ years; however
in the past 3 years they have seen growth stall, and an upstart take their market share. They are
now at 30% market share & flat growth. What’s wrong? How do you help them fix the
problem?

Additional Information Provided After Relevant Questions:

• Costs: COGS = 60%; Maintenance = 9-10%; Labor = 15%; Marketing / Distribution= 2%


• Competitor growing @ 6%/year
• Simplified visual of a store aisle with 30 ft. for Bread, 10 ft. for Hair Gel, 20 ft. for frozen meals
& 20 ft. for Canned Tomatoes, with unit volumes, revenue & profits

Interviewee’s Solution:
Thru simple calculations, if you shifted 10 ft. of shelf-space from Bread to higher-margin Hair Gel,
you might have higher profits of $30/week/aisle.
*$50 average basket size = $1500 * 20 aisles/store = 30,000; * 500 stores in UK = 15,000,000 * P/E
ratio of 10 = 150 million extra profit; and we’re talking real money here, from a simple aisle-
optimization!
Interviewee Notes:
Demand elasticity was a significant factor here. I nailed that; but failed to calculate some #s properly.
Actual Outcome of the Case:
Upstart competitor had figured out optimal shelf-space issue by optimizing:
1. Elasticity of sales to shelf-space
2. Consumers changing demands towards prepared foods
3. Competitive offerings.
*Client supermarket had to figure out the same & put into their own cost/reality model to make this
work.

Commentary/Recommendations:

There are a number of potential issues that the interviewer could be looking for you to analyze here
and this is a situation where it is important that you have a good structure to systematically work
through the options.

I would be concerned that leaping in and analyzing the margins on shelf space doesn’t necessarily
cover off all the issues – it also may not explain why one company is growing and the other is not.

I would approach this question as follows:


• First, I would like to understand the differences between our client’s offering and that of our
competitors to determine the drivers of our competitors’ growth.
• Then, once we understand the factors driving their growth I would like to explore the ways in
which our client could modify its strategy to increase its growth rate.
16
• Looking firstly at the differences between our client and our competitors, I would like to know
the following:
o What is the relative market share of each of our competitor’s and how has this been
growing?
o What is the difference between the products and services we offer and those offered by
our competitors?
ƒ Do they offer different categories of products?
ƒ In which categories have their sales been growing?
o In what offer ways are our competitors differentiated from us?
ƒ Have they been opening more stores?
ƒ Are their stores in areas that have a faster growing population?
ƒ Do they target a demographic segment that has been growing?
ƒ Are they growing sales through alternate channels (e.g. online)

Hopefully from asking these questions you would get some information as to the higher margins that
competitors are earning by stocking different product ranges and an analysis of the profitability
improvements resulting from this change could follow.

If you are giving this case as an interviewer I recommend that you think through prepared answers to
the questions set out above to give the case more life. Exactly what answers you give doesn’t matter
provided they drive the candidate to the conclusion that margin by product category is an important
difference.

17
CASE 12: TELEMATICS (BCG – Round 2) MARKET ENTRY

Your client is interested in telematics (wireless service to vehicles). Should your client enter this
new market? If so, how?

Additional Information Provided After Relevant Questions:

• They are a wireless carrier (like AT&T, Verizon, etc…)


• They have their own stores and also distribute through retailers like Best Buy and Circuit City.

Interviewee’s Solution:
What is the client’s core business?
They are a wireless carrier (like AT&T, Verizon, etc…)
Do they currently have distribution channels set up with retailers?
Yes.
To determine the potential for market entry, I then needed to do the following:
• Define customer segments:
Individual Customers: Commuters, Local, Recreational
Corporate Customers: Sales force, Fleet vehicles (trucks, rental cars, etc…),
Government (police, municipal, etc…)
• Define channels:
Auto Manufacturers, Retailers
• Size the market for each
• List risks and rewards of each

I recommended the retail channel (even though it has not become a big channel) because the market
penetration through auto manufacturers would be small (~1% of drivers per year for first 4 years of
sales) and time consuming (2-3 years from product design to release).

Commentary/Recommendations:
In making a recommendation to the client I would want to answer three main questions:
• First, is this an attractive industry to enter
o What is the market size? What are the growth prospects?
o What are the competitive dynamics?
• Second, is this industry appropriate for our client?
o What are their core competencies?
o Can they exploit their existing resources and capabilities?
• Finally, can they enter the industry profitably?
o What will it cost to enter the market and how quickly can they grow their share?
o What will it cost to acquire and retain customers?
o What revenues will they earn from each customer?

24
CASE 13: MANUFACTURING (BCG – Round 2)
Your client is a private company that manufactures switches for machine tools. The switches
serve one purpose: to stop the machine instantly. Failure to do so may result in damage to the
machine and/or the machine operator. Given the following constraints, how can the company
make more money?
• No changes can be made to the product line
• You only have your current customers
• No international expansion

Additional Information Provided After Relevant Questions:

• Our switches (and our competitors’ switches) have never failed.


• Our customers have dealt exclusively with us for several years, although there is no exclusivity
contract.
• The sales force has strong relationships with the customers, so they just take the orders each year.
• Cost of machine tools that our customer makes: $60,000.

Company Price/switch Mkt Share ($1B market)


Our Company 100 30%
Competitor 1 120 60%
Competitor 2 80 10%

Interviewee’s Solution:
I asked for revenue information for us and our competitors (provided above). When I asked about
perceived quality of our product, I was told that it is comparable to that of both our competitors.
I made an observation that customers are not very price sensitive and suggested raising the price of
our product 20 – 25%. My interviewer asked why that number and not a bigger number, so I
defended my recommendation.
He then changed gears and asked me which company I would acquire if I could acquire one of
my competitors.
I mentioned anti-trust issues associated with acquiring Competitor 1. Then I did the math and
determined that it would be advantageous to acquire Competitor 2 (C2) and raise the price of our
switches to $125. This would provide increased revenue and would price us only 4% over our only
other competitor. Other aspects of acquiring Competitor 2 include the risk of turning off C2’s
customers by a price increase, but they are unlikely to leave, since there is not much difference in
price between $125 (us & C2) and $120 (C1).

Commentary/Recommendations:

This is a common type of question that can be simply and effectively answered by using a standard
framework. I would approach it as follows:

If we are looking at increasing revenue then obviously we need to look at how we could increase
either the quantity sold or the price at which we sell our product.

25
Looking first at our quantity, it appears there are a number of restrictions. You have mentioned that we
have only our current customers and that we cannot expand internationally. Other things we would
need to know are:
• Do our current customers take all their product exclusively from us or do they take some
product from our competitors also? If they also take product from our competitors then we may
be able to increase our sales quantity by some sort of exclusivity arrangement – capturing all of
the demand from our existing customers.
• I would also want to know whether there are other machines at our existing customers’
factories that are suitable for our product but where it has not been installed. If so then
convincing customers to install our product in these other machines would be another way to
increase sales and we may need to look at changing or adapting our sales force and marketing
strategy.

Next, I would look at ways that we can increase price. We are told that we can’t change the product so
we will need to determine if we can persuade our customers to pay more for the existing product. Do
we have any information on the prices and volumes sold by our competitors? (In response to this you
would expect to get the information provided in the case above)

We can see from our competitors’ sales data that the competitor with the highest price also has the
highest market share. This suggests that we may be able to increase our price without adversely
affecting our sales (in fact it may even increase our sales if price is seen by customers as a proxy for
quality)

Before recommending a price increase however I would want to understand if there are any other
factors justifying our competitor’s higher price and high market share. Do they have a bigger sales
force? Do they sell through different channels? Are they entrenched as the industry leader/standard?
Do they have better after sales service? Do we target a segment that is more price sensitive than our
competitor? [The answer to these questions may affect whether recommending a price increase is
appropriate. Given the way the case has been set up, it seems like the focus is going to be on price
elasticity so presumably most of these avenues would be closed down by the interviewer, but asking
these questions would help to complete the case answer in a MECE fashion.]

26
CASE 14: CELL PHONES (BCG – Round 1) OPERATIONS/ SUPPLY CHAIN

Your client is a cell phone carrier in Western Europe in the late 1990s when growth is
exploding. The market as a whole is approximately 60% penetrated. Your client is concerned
because acquisitions costs (the cost to acquire a new customer) have been increasing at a 20%
CAGR over the past few years.
Your client wants to know why costs are increasing and what they should do.

Additional Information Provided After Relevant Questions:

• Acquisition costs largely fall into two categories:


1. Handset subsidies (phones are given to clients for free)
2. Commissions to independent stores who sell the phones
• Total annual acquisition costs are 200 euros per customer.

Interviewee’s Solution:
(The interviewer preferred not to have a framework set up and just have a dialogue).
Q: First I’d like to know more about the handset subsidies. Have costs been constant over time?
Have we always given them out for free? How much of the total spent do they account for?
A: Yes, we have always given them out. Total handset spending is 100 euros but they only
account for 5% of the total CAGR. What does that make the growth in commissions?
My answer: 35%
Q: Thinking about distribution, do we have any other outlets to sell our products. Are the stores
equally distributed from a geographic standpoint?
A: No
Q: How does the commission structure work? Are there different % paid based on different plans?
What has been the distribution of these plans now and in the past?
A: Let’s assume that we have only one plan for simplicity all of the commissions are paid against it.
These plans represent the entire increase in commissions – there are no other marketing expenses
etc. Additionally, prices of the plan have been pretty consistent over time, so that is not accounting
for the increase in total commissions.
Q: I’m assuming that since commissions are increasing, it is becoming increasingly difficult to sell
to customers. As such, I’d like to explore our competitors and their plans? Have more entered the
market? Are their plans unique when compared to ours? How is their distribution channel? How
are their commissions when compared to ours?
A: The competitors have been consistent over time although competition is increasing since
everyone now wants a phone. Market share is consistent across the players and their plans are
non-differentiated. They have similar distribution channels (note – this was the interviewer’s
response word for word. In reality, I later discovered they are the same distribution channels
down to the store). Commissions have been increasing at a similar rate to what we have seen.
Q: Ok, let’s talk about customers. Have we seen any differences in where they go to buy cell
phones? Is their a preference in Europe for certain types of plans or phones and how do we compete
from that perspective?
A: Customers are buying cell phones through the same channels as always and our plans and
phones are competitive?
27
Interviewee’s Solution (cont.):
Q: Well if nothing else in the landscape has changed, it must just be that we have to give competitive
commissions to our sales people to stay in business? Tell me, do they sell phones of competitors as
well?
A: Yes, they do. That’s why we have to incentivize them so well. Now what do you think we could
do about it.

Well, I think there are a number of things:


• We could somehow signal the market that we are not going to continue to give escalating
commissions through a PR program.
• We could try to differentiate our product offerings both in terms of the plan and the phones we
offer.
• We could spend more on marketing (and hopefully, less on commission) in the hopes of helping
the consumer determine what type of phone they want before entering the store.
• We could try to establish different distribution chains to reduce total costs; i.e. the Internet.

Commentary/Recommendations:

A good way to structure this case is as follows:


• First I would want to understand the primary drivers of the increase in acquisition costs. To do
this I would like to find information on:
o each of the components that make up the total acquisition cost elements that make up
acquisition costs and
o how much each of those elements has been growing
• Once we have identified the source of growth in acquisition costs I would then want to explore
the options available to reduce those costs.
o In doing this I would want to focus on the aspects of costs that are the most significant
and that have been growing the fastest
o I would also want to benchmark our costs against those of our competitors to determine
whether they use strategies that are more cost effective.

28
CASE 15: EXPLOSIVES (BCG – Round 2)
Our client is a large manufacturer of explosive products. Recently, their largest customer, a
mining company, has called for a competitive bid for their next 3 year contract. Previously, we
have been their preferred provider and have run uncontested. Our client wants to know how
we should price the bid.

Additional Information Provided After Relevant Questions:

• Our client provides three products:


1. Ammonium nitrate (AN) which is used to create the explosion
2. Ignition systems (IS) used to start the explosion
3. Slot services which are contracting services designed to help clients understand where to
place explosives.
• When asked why the bid was put out for RFP, the answer was that we are not sure but suspect it
could be internal cost pressures.

Interviewee’s Solution:
Q: I’d like to take a look at this by first analyzing the external factors currently impacting our client
(what the needs of the mining firm have been in the past and are now, what the competitive
landscape looks like, and how the industry’s product mix breaks down). Then I’d like to do an
internal analysis of the various options available to our client focusing largely on drivers of
profitability. To start, what do we know about this mining firm and their current contract?
A: The mining firm owns two mines, one in Kentucky and one in Wyoming. Previously, 80% of
the revenue of their contract has been derived from AN and 20% from IS. They don’t purchase
any slot services as they have their own people?
Q: You mentioned the contract from a revenue perspective but do we know what they purchased from
a volume perspective and at what cost?
A: They purchased 40,000 tons of AN at $1000 per ton and IS at $250 per ton of AN (there is a
predetermined ratio of how much IS you need per ton of AN).
Q: Great. Next I’d like to understand a little bit more about the competitive landscape. Do we know
anything about new competitors that have entered the market and how we compare to those firms?
A: Nothing has changed from a competitive standpoint as we have the same competitors as always.
We are generally seen as the market and price leader.
Q: From a product standpoint, how do we compare to those competitors?
A: AN is pretty much a commodity although we do charge about a $10 premium over our
competitors. From an IS standpoint, we are the clear leader and have a quality product for which
people will pay for.
Q: Do all competitors provide both products?
A: Our largest competitor does although they have bad ignition systems. There are other
competitors for ignition system abroad that are closer in quality to ours and AN can be purchased
from many places as it is a commodity.

29
Interviewee’s Solution (cont.):
Q: Great. Now that I know more about the industry landscape, I’d like to look at the current and
future profitability of this contract. We know that the client purchased 40,000 tons of AN at $1,000
per ton so that is $40 million in revenue. They also purchased IS at $250 per ton of AN so that is $10
million in revenue for a total of $50 million. Do we know the margins on these products?
A: Yes, AN gets a 20% margin and IS get an 80% margin.
Q: That means that we are making $8 million of profit off of each component of the contract currently.
What are the specifics of the future contract?
A: It will be for the same components in the same volumes as this contract.
Q: Since, the contract is the same as last time, we need to think about who we are really competing
against to determine price. In this case, it seems that no competitor offers as robust an offering as we
do. However, the customer still has the option to purchase the products separately on their own. Do
we know how much they could save by doing so? Also, what would it cost them in internal costs to
manage the process?
A: They could get the next closest IS system to ours for $100 per ton cheaper by purchasing
overseas. Additionally, let’s assume that they need 1 person @$100k per year to manage the process.
Q: That means they will save $4 million per year on their own although they probably won’t want to
do it.
A: If you think they don’t want to do it, should we charge them more for our integrated service.
Q: I wouldn’t advertise that to the customer even if we do decide to do it because it is poor positioning.
Also, when looking at the number, we could make $16 million in profit so by looking at the scope, I’m
not sure another $100k matters.
A: Agreed. So what should we do?
Q: Well, since AN is a commodity and since we make a much smaller margin on it, I’m not attracted
to trying to reduce price there. However, as we make a much higher margin on IS, and it would be
much harder for them to get these products from abroad, that may be a good place to reduce price if
that is the mine’s number 1 concern.
A: That makes sense. So what should we price it at?
Q: Working backwards, I’m not sure that we necessary need to give an entire $4 million price
concession to our client as they do know that we have a premium product. Instead, let’s reduce our
price by half that so that the maximum they will pay is $8 million for IS systems. That makes the
price $200 per ton. Does that seem reasonable?
A: It does.

30
CASE 16: PHARMACEUTICALS (BCG – Round 2)
Two large branded pharmaceutical companies are merging. The head of development (pre-
clinical and clinical) has a declining budget and wants to re-think his portfolio. He currently
uses a spreadsheet to evaluate his portfolio which contains columns for the chemical name,
estimated launch date, and lead indication. Essentially, your client wants you to add a column
for the current market value of the partially developed drug so that he can evaluate which ones
to push through development. How do you do it?

Additional Information Provided After Relevant Questions:

• There are 60 drugs in development with 1/3 in clinical trials 2/3 in pre-clinical development.
• It takes 10 years to bring a drug from pre-clinical to launch and there is large attrition along the
way.
• Assume that this is a U.S. launch only, that there is no bundling, and that each compound
represents a novel therapy.

Interviewee’s Solution:
Q: To determine the market value of the drug, I would want to do an NPV calculation and so the two
areas I would primarily like to explore is revenue and costs. I’d like to begin with the revenue side of
the equation first and begin discussing the factors that influence volume. Should I begin with a
particular drug or should we talk in generalities?
A: I’d like to focus on the thought process you would use to do the calculation and so I am not
concerned about a specific drug. Generalities are fine.
Q: Ok. The first thing I would want to do is estimate the size of the market and consequently our
market share at the time of launch. To do that I’d like to know the # of people who suffer from a
particular ailment.
A: Where would you find that?
Q: You can go to all kinds of different sources like the American Cancer Society or the American
Lung Association who keep track of this information. In particular, I’d take the growth rate for a
given disease category over the past few years and use that to extrapolate what it would be when the
drug launches to get the total number of people with the illness. Next, I’d want to find out the number
of people who are generally treated for the disease.
A: And where would you find that information?
Q: Similar sources although I suspect I’d might also be able to look at analyst reports depending on
the type of therapy being evaluated.
A: Good
Q: Next, I want to discount that number by the number of people who seek treatment using a drug vs.
another procedure like surgery to get a true sense of the market. Then I would want to take into
account the competitive environment.
A: What in particular would you look for?
Q: Well, I would first want to understand what competitors are on the market now and where we
anticipate them being from a product lifecycle perspective when we expect to launch our therapy. In
particular, I’d want to drill down on comparisons between mechanism of action, indications, and
dosing requirements.
A: What else should you look for?
31
Interviewee’s Solution (cont.):
Q: Knowing that new drugs are always being developed, I’d want to evaluate those that are currently
in the pipeline of our competitors and what we anticipate the impact of them being on the market at
the time of launch. You can find this information in publications like R&D directions as well as other
trade publications.
A: Sounds good. Besides volume what else should you be thinking about?
Q: Pricing. Pricing for drugs has a lot to do with demonstrated efficacy as well as whether you are the
first drug in a given class to be launched. Other things that you might want to consider include
convenience (dosing requirements) and other drugs used to treat the therapy even if the mechanism of
action differs.
A: Great. What about costs?
Q: From a cost perspective, I’d want to consider both fixed and variable costs. This would include
any new facilities that need to be built to manufacture the drug as well as the R&D and sales and
marketing launch costs associated with the product.
A: And what would you do with this information?
Q: I’d determine the free cash flow number for each year and then discount that back to the current
period. Additionally, since the process has high attrition, I’d also want to adjust for the probability of
success at each phase.
A: Great. Let’s say you’ve done all this and it turns out that ½ the compounds in pre-clinical
development have no value. How are you going to plug the hole from a revenue perspective?

Q: Any time you are considering product development there are two options – make or buy. Under
the make category, we could try to expedite the development of existing pipeline drugs to address the
gap but this is fairly unlikely to prove successful given the highly complex development process.
A: What else could address the revenue considerations if you can’t expedite development?
Q: You could work with your existing portfolio of marketed products to increase the amount of
revenue you can get from them. This could include considering shifting them to over the counter
products or evaluating other indications for the drug. However, I suspect we may already be doing
this so it is unlikely to be all of the answer.
A: Good. What else?
Q: We can buy a product or a group of products to plug the hole. To do so, we should go through a
similar compound valuation exercise as we did for our portfolio but also take into account synergies
between our two organizations and the previous track record of the other company in launching
similar projects.
A: Sound good. Thanks.

Commentary/Recommendations:

This case is an interesting spin on the standard market sizing type of question – Rather than evaluate
one market, the candidate has to outline a general process for sizing the market for a number of
different drugs. Layered on top of this the candidate needs to demonstrate a good understand of the
process for deriving an NPV and the relationship between the NPV and the original market sizing
exercise.

The candidate has given a very good answer and there are a number of positive aspects worth noting,
including the following:

32
• Before doing anything else, the candidate sets out a clear road map for tackling the question –
“I am going to do an NPV which will mean looking at revenues and costs”
• The candidate clarifies the expectations of the interviewer – “Should we be looking at
generalities or specifics?” The last thing you want to be doing in a case interview is giving a
great answer that bores the interviewer because it is not the aspect of the question that they
were interested in.
• The candidate has a clear and logical approach to sizing the market for each drug. Not only are
the steps sensible and meaningful but they are also readily verifiable by available data (such as
American Cancer Society data on the number of patients with a particular cancer condition). It
is quite common for interviewers to ask how you would go about getting the data to support a
certain step in you logic so it is important to be choosing divisions where this information is
likely to be available.
• The candidate does not get bogged down when the interviewer changes the question. If you are
asked a new question, like how to fill a hole in the pipeline, then develop a new framework for
approaching this aspect – “there are two options to consider, make or buy”

Similar techniques could be applied to almost any market sizing question.

33
CASE 17: ELECTRONICS RETAILER (BCG – Round 2)
Your client is a major consumer electronics retailer who sells A/V equipment, computers,
content (CDs, DVDs), and software. Recently, the store has seen increased competition from
bricks and mortar stores like Wal-Mart as well as online retailers like Dell. Your client has 3
questions:
1) How is the industry evolving?
2) What is the winning retail model?
3) What should we do?

Additional Information Provided After Relevant Questions:

• None provided

Interviewee’s Solution:
Q: To answer these three questions, I’d like to look in 3 areas. First I’d like to know more about the
current market including understanding customers, the products they are buying, and how these have
changed over time. Next, I’d like to take a look at the different types of competitors that are affecting
our company and our firm’s profitability. Lastly, I’d like to know more about our client in terms of
their business model, including core competencies, strategic direction, and their store model, to better
understand what options are realistic. Is there a particular, place you would like me to start?
A: Before we get started, can you tell me what you think makes a successful retail model?
Q: Sure. I think there are several elements. First, I think that a retailer needs to have a clear
understanding of their customer base. This will drive two things: having the appropriate product mix
to serve them and serving them through the appropriate distribution channel (online vs brick and
mortar). Secondly, while the above two factors are critical, our client needs to ensure that they will
allow for profitability and so careful consideration needs to be paid to revenue and cost considerations.
Lastly, to the extent that is possible, we would want to make sure that our business model is not easily
duplicated so that we can maintain a competitive advantage.
A: Sounds good. Let’s focus on the CD/DVD market. What are the differences between the two
categories?
Q: There are potential differences between consumers as DVDs don’t have as high an adoption rate as
CDs. There are also potential differences between age groups of consumers – I would think younger
people might buy more audio products than older people.
A: Those are reasonable. What about in terms of title fragmentation?
Q: I would think that DVDs are much more consolidated in terms of titles as 10 blockbusters a year
probably account for the majority of sales. I’m not sure this applies to CDs which I would think have
a much wider distribution in terms of titles.
A: How do you see consumer buying patterns change?
Q: More and more, consumers are buying content directly. You can buy songs directly over the
Internet decreasing the need to buy entire CDs. You can also get video on demand removing the need
to buy DVDs.
A: So what are the implications of this?

34
Interviewee’s Solution (cont.):
Q: From the perspective of our client, CDs and DVDs are not going to be a substantial part of their
business going forward because the ability to buy content directly from producers removes the
middleman. Our client will more directly need to rely on other parts of its product mix in order to be
successful. In the interim, it is critical that our client manage the transition carefully in order to
maximize the opportunity associated with this closing window which will likely last for several more
years.
A: Agreed. So, let’s assume that we did some additional analysis and it turns out that our client
needs to double the volume of its other products to counteract this. Fixed costs don’t grow in
proportion to sales – they grow on a 75% scale curve meaning that even though sales double now
FC are only 75% of what they were on a % of revenue basis. Our gross margin is 25% and net
income is 0%. What would our net income be if we doubled volume?
Q: Let’s say that Year 1 revenue was $100 making operating profit 25 and fixed costs 25. In Year 2,
we would make $200 and our operating profit would be $50. Given the scale curve, our FC should be
about $38 giving us net income of $12 or 6%.
A: Exactly.

Commentary/Recommendations:

This is a fantastic sample case because the dialogue provided by the candidate demonstrates exactly
what the case interview experience can end up being like. Most importantly it demonstrates exactly
how that experience can differ wildly from what you were originally expecting, and what the case
interview books tell you to expect.

At the outset of the discussion, the candidate develops a framework for answering the question. All
they have to do now is walk through that framework right?

Wrong.

After the candidate sets out the framework, the interviewer then practically takes over, firing questions
at the candidate, choosing a particular market to focus on, and never really giving them a chance to
thoroughly walk through their carefully laid out framework. The poor candidate is left feeling totally
defensive, thinking on their feet to come up with answers to the interviewer’s interrogation rather than
having the chance to ask the questions and gather information at their own pace.

If this happens to you (and it undoubtedly will), don’t panic!! It is quite common and not necessarily
a bad thing. Despite not getting a chance to do their own analysis this candidate has still nailed the
case because they set out a good framework and gave intelligent answers to the interviewer’s
questions.

This is my assessment of what is going on and why this happens so often:


• Case interviews are about giving the firm a chance to gain an insight into your thought
process – can you think like a consultant.
• The candidate set out a reasonable framework (not necessarily the only one but a
reasonable one) and the interviewer has taken this as a sign that they know what they are
doing. The framework in and of itself is 80% of the answer to the case.

35
• The interviewer then chooses not to let you walk through the framework, either because
they are bored with having asked the same question ten times already that day or because
they think they doing so will give them very little extra insight into your thinking beyond
what they already got from hearing what your framework was going to be.
• Instead they choose to test you more deeply by asking you a series of questions that give
them more insight into your ability to really understand and think about a business
problem or market.
• Far from being a sign of failure, getting grilled with questions in this fashion may
actually be a sign that you are doing well.

Note, there is a fundamental difference between interviewers asking you questions to prompt
you because you don’t seem to know what you are doing and interviewers asking you
questions (as in this case) because they are really testing you.

Don’t assume you can sit back and let them ask you the questions they are interested in. You
cannot. You are always responsible for driving the case and you need to set out a framework
and start working your way logically through it. If however, at some stage during this process,
the interviewer starts firing questions at you then you need to be able to work with it.

Answer their questions as insightfully as you can and let them take you where they want to go.
Think about what things would be like in the real world – How would the CD market be
different to the DVD market? You must have had plenty of experiences with both so apply
some common sense and you are more than half way to a decent answer.

If the questions run out, then tie everything back in to your original framework and keep going.
But more times than not the questions will only run out when they bring you to a case
conclusion.

These interviews can be terrifying because you fell like you have lost control, but they can also
be the most rewarding and, in my view, are the ones most likely to earn you an offer.

36
CASE 20: HOTEL (POST 9-11) (BCG – Round 1)

BCG has been hired after 9/11 to look into quantifying the amount of damage to revenue
streams that has occurred in the Manhattan hotel business. How would you go about
determining / quantifying the impact, and how would you structure a recovery plan?

Additional Information Provided After Relevant Questions:

• Hotels use a metric called “Revpar” which is equal to the average daily room rate * % occupancy.
Revpar was $200 before 9/11, $125 for the remainder of 2001, $150 for 2002, and $175 for 2003.
• I was asked to assume that if 9/11 had not occurred that the Revpar would have been flat for the
entire time period.
• I was told that there were 100,000 hotel rooms in Manhattan. I was then asked to quantify the
revenue impact.

Interviewee’s Solution:

• I structured my discussion by segmenting the market into US business travelers, international


business travelers and leisure travelers. I also tried to make a separation between individual hotels
and chain hotels, but it was clear that the case giver was only interested in looking at things on an
aggregate basis.

• The key, I think, to doing well in this case is to focusing not only on quantifying the damage, but
also to coming up with creative suggestions for a recovery plan.

Commentary/Recommendations:

The quantification question is relatively straightforward multiplication once the candidate has
extracted the relevant data by asking appropriate questions.

The question of a recovery plan is more difficult. I would approach this question using the following
framework:
• What were the primary categories of hotel customers before 9/11? e.g. tourists, business etc
• How much did each category pay? i.e. how much was each category worth?
• How much has each category stopped traveling? e.g. where are the biggest losses?
• Focusing on those categories with the biggest losses what are the main reasons why they have
stopped traveling? e.g. is it a perception of safety in NY for tourists? Is it a perception of
airport delays for business travelers? Etc
• Once the main causes for the most important categories are identified I would try to develop
solutions focused on addressing these causes. This may take the form of marketing and sales
promotions or alternatively lobbying the government to take measures to make travel easier etc
depending on exactly what causes are identified as being most important to the most important
category of customer.

40

Potrebbero piacerti anche